Você está na página 1de 1013

1.

MASTER BUDGET
Hansell Company's management wants to prepare budgets for one of its products,
duraflex, for July 2002. The firm sells the product for $40 per unit and has the following
expected sales units for these months in 2002:

April May June July August September


5,000 5,400 5,500 6,000 7,000 8,000

The production process requires 4 pounds of dura-lOOO and 2 pounds of flexplas.


Q P
dura-lOOO 4 $ 1.25
flexplas. 2 $ 5.00

The firm's policy is to maintain a minimum of 100 units of duraflex on hand at alltimes with no fewer than 10 percent of un
the expected sales for the following month. 100

All materials inventories are to be maintained at 5 ercent of the production needs for the next month,
but not to exceed 1000 pounds 1000

The firm expects all inventories at the end of June to be within the
guidelines. The purchase department expects the materials to cost $1.25 per pound
and $5.00 per pound of dura-lOOO and flexplas, respectively.

The production process requires direct labor at two skill levels.


The rate for labor at the K102 level is $50 per hour and
for the K175 level is $20 per hour.
The K102 level can process one batch of duraflex per hour;
each batch consists of 100 units.

The manufacturing of duraflex also requires one-tenth of an hour of K175 workers' time
for each unit manufactured.

Manufactured overhead is allocated at the rate of $200 per batch and $30 per
direct labor-hour.

Required On the basis of rhe preceding data and projections, prepare the following
budgets for July 2002:
Sales Budget (in dollars).
b. Production budget (in units).
c. Production budget for August (in units).
d. Direct materials purchase budget (in pounds).
e.Direct materials purchase budget (in -dollars).
f.Direct manufacturing labor budget ('in dollars).
1. Master Budget

Sales Budget (in dollars).


HANSEll COMPANY
Sales Budget
For July 2002

Budgeted sales in units 6,000


Budgeted selling price per unit $ 40
Budgeted sales $ 240,000

b. HANSEll COMPANY
Production Budget (in units)
For July 2002

Desired ending inventory (July 31)


(The higher of 100 and 7,000 x 0,1) 700 700
Budgeted sales for July 2002 + 6,000
Total units needed for July 2002 6700
Beginning inventory (July 1) -
(The higher of 100 and 6,000 x 0,1) 600 600
Units to manufacture in July 6100

c. HANSEll COMPANY
Production Budget (in units)
For August 2002

Desired ending inventory (8,000 x 0,1) 800


Budgeted sales + 7,000
Total units needed 7800
Beginning inventory - 700
Units to manufacture in August 7100

d. HANSEll COMPANY
Direct Materials Purchases Budget (in pounds)
For July 2002
Direct Materials
Dura-tOOO Flexplas
(4Ib. each) (2Ib. each)

Materials required for budgeted


production (6,100 units of duraflex) 24,400 12,200
Add: Target inventories (lower of 1,000 or 5 percent of
August production needs) 1,420 710 1000 710
Total materials requirements 25,400 12,910
Less: Expected beginning inventories (lower of 1,000
or 5 percent) 1,220 610 1000 610
Direct materials to be purchased 24,400 12,300
HANSEll COMPANY
e. Direct Materials Purchases Budget (in dollars)
For July 2002
Budgeted Expected
Purchases Purchase
(Pounds) Price per Unit
Dura-lOOO 24,400 $ 1.25
Flexplas 12,300 $ 5.00
Budgeted purchases

HANSEll COMPANY
f. Direct Manufacturing labor Budget
For July 2002

Direct Number
Labor-Hours of Total
per Batch Batches Hours
Kl02 Hours 1.00 61 61
K17SHours 10.00 61 610
Total 671
DLH PER UNIT 0.11

2. CASH BUDGET AND INCOME STATEMENT

Hansell Company expects its trial balance on June 30 to be as follows:

HANSELL COMPANY
Budget Trial Balance
June 30, 2002
Debit

Cash $ 10,000
Accounts receivable $ 80,000
Allowance for bad debts
Inventory $ 25,000
Plants, property, and equipment $ 650,000
Accumulated depreciation
Accounts payable
Wages and salaries payable
Note payable
Stockholders' equity
$ 765,000

Typically, cash sales represent 20 percent of sales and credit sales represent 80 percent.
20% 80%
Sales terms are 2/10, n/30. Hansell bills customers on the first day of each month 2%
Experience has shown that 60 percent of the billings will be' conicted within the discount period.
25 percent by the end of the month after sales,
10 percent by the end of the second month after the sale,
and 5 percent will be uncollectible. The firm
writes off uncollectible accounts after 12 months.

The term of purchses for materials is 2/15, n/60. The firm makes all payments 2%
within the discount period. Experience has shown that 80 percent of the purchases
are paid in the month of the purchase and the remainder are paid in the month immediately following.

In June 2002, the firm budgeted purchases of $25,000 for dura-1000 and $22,000 for flexplas.

dura-1000 $ 25,000
flexplas. $ 22,000

Sixty percent of the factory overhead is variable. 60%


The firm has a monthly fixed factory overhead of $50,000, of which $20,000 is depreciation expense
. The firm pays all manufacturing labor and factory overhead when incurred.

Total budgeted marketing, distribution, customer service, and administrative costs for 2002 are $600,000.
Of this amount, $270,000 is considered fixed and includes depreciation expenses of $150,000
. The remainder varies with sales.
The budgeted total sales for 2002 are $2 million.
All marketing and administrative costs are paid in the month incurred.

management desires to maintain a minimum cash balance of $40,000.


The firm has an agreement with a local bank to borrow its short-term needs in multiples of
$1,000 up to $100,000 at an interest rate of 12 percent. $ 1,000

2. Cash Budget and Budgeted Income Statement

HANSEll COMPANY
Cash Budget
Jul-02
Cash Available
Cash balance, beginning
Add: Cash receipts
July cash sales $240,000 x 20% = $ 48,000 $ 240,000 20% $ 48,000
Collections of receivables
From sales in June
Collection within the discount period
5,500 x $40 x 80% x 60% x 98% $ 103,488
Collection after the discount period
5,500x $40 x 80% x 25% $ 44,000
From sales in May
5,400 x $40 x 80% x 10% $ 17,280

Total cash available in July

Cash Disbursement
Materials purchases

June purchases ($25,000 + $22,000) x 20% x 98% = $ 9,212 $ 9,212


July purchases $92,000 x 80% x 98% = 72,128 $ 72,128

Direct manufacturing labor


Variable factory overhead ($200 x 61 + $30 x 671) x 60%
Fixed factory overhead $50,000-$20,000

Variable marketing, customer services, and administrative expenses


[($600,000 - $270,000) / $2,000,000} x $240,000
Fixed marketing, customer services, and administrative expenses
($270,000 - 150,000) .;-12
Total disbursements

Cash balance before financing

Financing
Amount to borrow $ 12,820 .=13000
Cash balance, July 31,2002

HANSELL COMPANY
Budget Income Statement
Jul-02
Sales
Cost of goods sold* $22.80 x 6,000 =
Gross margin
Selling and administrative expenses
Variable $ 39,600
Fixed $270,000 / 12 = $ 22,500
Net income

*Cost per unit

Direct materials
Dura-lOOO 4 1.25 $ 5.00
Flexplas 2 5 $ 10.00 $ 15.00
Direct labor
Kl021abor 0.01 hour x $50 $ 0.50
K175labor 0.1 hour x $20 $ 2.00 $ 2.50
Factory overhead
Applied based on batch $200 / 100 = $ 2.00
Applied based on direct labor-hour $30 x 0.11 hour = $ 3.30 $ 5.30
Cost per unit $ 22.80
$ 40

wer than 10 percent of units on hand at the end of a period to meet


10%

5%

$ 1.25
$ 5.00

rate per unit


$ 50.00 $ 0.50
$ 20.00
1
100 no. of units in one hour rate per unit

0.10 10.0 $ 2.00

$ 200.00 per batch


$ 30.00 per direct labor-hour.
100

100

(2Ib. each)
Price per Unit Total
$ 30,500
$ 61,500
$ 92,000

Rate
per
Hour
$ 50.00 $ 3,050
$ 20.00 $ 12,200
$ 15,250.00

Credit

$ 3,500

$ 320,000
$ 95,000
$ 24,000
$ 200,000
$ 122,500
$ 765,000

98%
60%
25%
10%
5%

98%
80%
20%

$ 50,000 $ 20,000 $ 30,000

$ 600,000
$ 270,000 $ 150,000

$ 2,000,000

$ 40,000

$ 100,000

$ 10,000
$ 212,768

$ 222,768

$ 81,340

$ 15,250
$ 19,398
$ 30,000

$ 39,600

$ 10,000
$ 195,588

$ 27,180

13000
$ 40,180

$ 240,000
$ 136,800
$ 103,200

$ 62,100
$ 41,100
Farmers' Dairy purchases raw milk from individual farms and processes it
until the splitoff point, when two products - cream and liquid skim - emerge. These two
productsare sold to an independent company, which markets and distributes them to
supermarketsand other retail outlets.
Summary data for May 2004 are

Rawmilk processed, 110,000 gallons; 10,000 gallons are lost in the production
process due to evaporation, spillage, and the like, yielding 25,000 gallons of cream
and 75,000 gallons of liquid skim.
Production sales

Cream 25,000 25% 20,000


Liquidskim 75,000 75% 30,000
100,000
Inventories
Beginning Ending
Inventory Inventory
Raw milk 0 0
Cream 0 5,000
Liquid skim 0 45,000

Cost of purchasing 110,000 gallons of raw milk and processing it until the splitoff
point to yield cream and liquid skim, $400,000.
110,000 $ 400,000

Exhibit16-3 depicts the basic relationships in this example.


How much of the $400,000 joint costs should be allocated to the cost of goods sold
of20,000 gallons of cream and 30,000 gallons of liquid skim, and how much should be
allocatedto the ending inventory of 5,000 gallons of cream and 45,000 gallons of liquid
skim?The joint production costs of $400,000 cannot be traced to either product. That's
because the products are not separated until the splitoff point. Joint-cost-allocation meth·
ods can be used for determining the costs of cream and liquid skim sold and for costing
the inventories of cream and liquid skim.

Sales Value at Splitoff Method


The sales value at splitoff method allocates joint costs to joint roducts on the basisof
the relative total sales value at the splitoff point of the total production of these products
during the accounting period. We illustrate the allocation of joint costs to individual
products in proportion to total sales value at splitoff for Example 1:

Cream
1. Sales value of total production at splitoffpoint $ 200,000

2. Weighting 40%

3. Joint costs allocated $ 160,000

4. Joint production cost per gallon $ 6.40


This method uses the sales value of the entire production of the accounting period. The reasonis
that the joint costs were incurred on all units produced, not just the portion sold during the
current period. Exhibit 16-4 presents the product-line income statement using the sales value
at splitoff method. Both cream and liquid skim have gross-margin percentages of 20%.
You can now see why the sales value at splitoff method follows t e enefits-received
criterion of cost al ocation: Costs are allocated to products in proportion to their expected
revenues. This method i,both straightforward and intuitive. The cost-alloGition base (total
sales value at splitoff) is expressea in terms of a common denominator (the amount of
revenues) that is systematically recorded in the accounting system. To use this method, a
company needs the market selling prices for all products at the splitoff point.

The equality of the grossmargin


percentages for the
individual products always
occurs underthe sales value at
splitoffmethodwhen there are
no beginninginventoriesand all
products are sold at the splitoff
point

Physical-Measure Method
The physical-measure method allocates joint costs to joint products on the basis of the
relative weight, volume, or other physical measure at the splitoff point of the total produc·
tion of these products during the accounting period. In Example 1, the $400,000 joint
costs produced 25,000 gallons of cream and 75,000 gallons ofliquid skim. Using the num·
ber of gallons produced as the physical measure, joint costs are allocated as follows:

Cream
1. Physical measure oftotal production (gallons) $ 25,000

2. Weighting 25%

3. Joint costs allocated $ 100,000

4. Joint production cost per gallon $ 4.00

Exhibit 16-5 presents the product-line income statement using the physical-measure
method. The gross-margin percentages are 50% for cream and 0% for liquid ·skim.
Under the benefits-received criterion, the physical-measure method is less preferred than
the sales value at splitoff method. Why? Because it has no relationship to the revenue·
producing power of the individual products. Consider a gold mine that extracts ore contain·
ing gold, silver, and lead. Use of a common physical measure (tons) would result in almost
all costs being allocated to lead - the product that weighs the most but has the lowest
revenue-producing power. In this case, the method of cost allocation is inconsistent with
the reason for the mine owner incurring mining costs - to find gold and silver, not lead. As
another example, if the joint costs of a hog were assigned to its various products on the basis

Example 2: Assume the same data as in Example 1except that here both cream and Iiq.
uid skim can be processed further:
• Cream ---'>Buttercream: 25,000 gallons of cream are further processed to yield 20,000
gallons of buttercream at additional processing costs of $280,000. Buttercream,
which sells for $25 per gallon, is used in the manufacture of butter-based products.
• Liquid Skim ---'>Condensed Milk: 75,000 gallons of liquid skim are further processed
to yield 50,000 gallons of condensed milk at additional processing costs of $520,000.
Condensed milk sells for $22 per gallon.

Sales during the accounting period were 12,000 gallons of buttercream and 45,000 gal.
Ions of condensed milk. Exhibit 16-6 depicts the basic relationships of how raw milkis
converted into cream and liquid skim in a joint production process, and how the creamis
separately processed into buttercream and the liquid skim is separately processed into
condensed milk. Inventory information follows:

Beginning Ending
Inventory Inventory
Raw milk 0 0
Cream 0 0
liquid skim 0 0
Buttercream 0 8,000
Condensed milk 0 5,000

The net realizable value (NRV) method allocates joint costs to joint products on the
basis of the relative NRV- the final sales value minus the separable costs _ of the total
production of the joint products during the accounting period. The NRV method is typi.
cally used in preference to the sales value at splitoff method only when we don't knowthe
market selling prices for one or more products at splitoff. Joint costs in this example are
allocated as follows: I

Buttercream

1. Final sales value of total production $ 500,000

2. Deduct separable costs to complete and sell $ 280,000

3. Net realizable value at splitoff point $ 220,000

4. Weighting 27.50%

5. Joint costs allocated ( $ 110,000

6. Production cost per gallon $ 19.50

Exhibit 16-7 presents the product-line income statement using the estimated NRV
method. The gross-margin percentages are 22.0% for buttercream and 26.4% for condensed
milk.
The NRV method is often implemented using simplifYing assumptions. For example,
companies that frequently change the number of processing steps beyond the splitoff
point often assume a specific set of such steps. Also, if the selling prices of joint products
vary frequently, a given set of selling prices may be consistently used throughout the
accounting period.3
Because the sales value at splitoff method does not require knowledge of the processing
steps beyond the splitoff point, it is less complex than the NRV method. Howev~r,
using the sales value at splitoff method is not always feasible. That's because there may
not be market prices for at least one of the products at the splitoff point. Market prices
may only be available after processing occurs beyond the splitoff point.

Constant Gross-Margin Percentage NRV Method


The constant gross-margin percentage NRV method allocates joint costs to joint products
in such a way that the overall gross-margin percentage is identical for the individual
products. This method entails three steps. Exhibit 16-8 on the next page shows these
three steps for allocating the $400,000 joint costs between buttercream and condensed
milk in the Farmers' Dairy example. As we describe each step, refer to Exhibit 16-8 for an
illustration of the step.

Step 1: Compute the overall gross-margin percentage for all joint products together.
Note, Exhibit 16-8 uses the final sales value of the total production during the accounting
period, $1,600,000, not the total sales of the period, to calculate the overall gross
margin percentage of 25%.
• Step 2: Multiply the overall gross-margin percentage and the final sales values of each
product to calculate the gross margin for each product. Subtract the gross margin for
each product from the final sales value of each product to obtain the total costs that
each product will bear.
• Step 3: Deduct the separable costs from the total costs that each product will bear to
obtain the joint-cost allocation.

The joint costs allocated to a product can be negative under this method. Some products
may receive negative allocations of joint costs to bring their gross-margin percentages
up to the overall average. Exhibit 16-9 presents the product-line income statement for the
constant gross-margin percentage NRV method.

Joint Costs Allocated


I Using Constant Gross-
I Margin Percentage NRV
Method: Farmers' Dairy
for May 2004

Step 1
Final sales value of total production during the accounting period:
(20,000 gal. x $25/gal.) + (50,000 gal. x $22/gal.) ,'"
Deduct joint and separable costs ($400,000+ $280,000 + $520,000)
Gross margin
Gross-margin percentage ($400,000.;-$1,600,000)

Step 2
Final sales ILalueof total production during Buttercream
the accounting period:
(buttercream, 20,000 gal. x $25/gal.; $ 500,000
condensed milk, 50,000 gal. x $22/gal.)
Deduct gross margin, using overall $ 125,000
gross-margin percentage (25%)
Cost of goods available for sale $ 375,000

Step 3
Deduct separable costs to complete and sell $ 280,000
Joint costs allocated $ 95,000

The constant gross-margin percentage NRV method is different in one fundamental


way from the two other market-based joint-cost-allocation methods described earlier. The
sales value at splitoff method and the NRV method allocate only the joint costs to the joint
products. Neither method takes account of profits earned either before or after the splitoff
point when allocating the joint costs. In contrast, the constant gross-margin percentage
NRV method is both 'a joint-cost method and a profit-allocation method. The total difference
between the sales value of production of all pro ucts and the separable cost of all
products includes both (a) the joint costs al}d (b) the total gross margin. Gross margin is
allocated to the joint products under the constant gross-margin method to determine the
joint-cost allocations so that each product has the same gross-margin percentage.
[232] Gleim #: 5.2.47 -- Source: CMA 1296 3-30
Lankip Company produces two main products and a by-product out of a joint process. The ratio of output quantities to
input quantities of direct material used in the joint process remains consistent from month to month. Lankip has employed
the physical-volume method to allocate joint production costs to the two main products.

The net realizable value of the byproduct is used to reduce the joint production costs before the joint costs are allocated t
regarding Lankip’s operations for the current month are presented in the chart below. During the month, Lankip incurred
joint production costs of $2,520,000. $2,520,000
The main products are not marketable at the split-off point and, thus, have to be processed further

First Second
Main Main
Product Product By-product
Monthly output in pounds 90,000 150,000 60 90,000 150,000 60,000
Selling price per pound $30 $14 $2 $30 $14 $2
Separable process costs $540,000 $660,000 $540,000 $660,000
120,000

The amount of joint production cost that Lankip would allocate to the Second Main Product by using the physical-volume
method to allocate joint production costs would be
2,400,000
1,500,000

Answer (C) is correct. The joint cost to be allocated is $2,400,000 [$2,520,000 total joint cost – (60,000 pounds of
the by-product) × $2]. Accordingly, the joint cost to be allocated to the Second Main Product on a physical-volume
basis is $1,500,000 {[150,000 pounds ÷ (90,000 pounds + 150,000 pounds) × $2,400,000]}.

233] Gleim #: 5.2.48 -- Source: CMA Sample Q 02/2005 2-20


Breegle Company produces three products (B-40, J-60, and H-102) from a single process. Breegle uses the physical
volume method to allocate joint costs of $22,500 per batch to the products. Based on the following information, which
product(s) should Breegle continue to process after the splitoff point in order to maximize profit?

B-40
Physical units produced per batch 1500
Sales value per unit at splitoff $ 10.00
Cost per unit of further processing after splitoff $ 3.05
Sales value per unit after further processing 12.25
Incremental revenue per unit $ 2.25
Less: cost to process further $ (3.05)
Incremental profit per unit $ (0.80)
Sales value EI

$ 8 $ 200,000 40% 5,000


$ 4 $ 300,000 60% 45,000
$ 500,000

400000

Joint Costs Allocated


Using Sales Value at
Splitoff Method: Farmers'
Dairy Product-Line Income
Statement for May 2004

Liquid skim Total


$ 300,000 $ 500,000 Revenues
Costs of goods sold (joint costs)
60% Production costs (
Deduct ending inventory
$ 240,000 $ 400,000 Cost of goods sold (joint costs)
Gross margin
/gal $ 3.20 /gal Gross margin %
Liquid skim Total
$ 75,000 $ 100,000 Revenues
Costs of goods sold (joint costs)
75% Production costs (
Deduct ending inventory
$ 300,000 $ 400,000 Cost of goods sold (joint costs)
Gross margin
/gal $ 4.00 /gal Gross margin %
Cream Buttercream additional processing cos
25,000 20,000 $ 280,000

Liquid Skim Condensed Milk


75,000 50,000 $ 520,000

Joint Costs Allocated


Using NRV Method:
Farmers' Dairy Product-
Line Income Statement for
May-04

Buttercream Condensed Milk Total


Revenue
$ 1,100,000 $ 1,600,000 Costof goods sold
Joint costs
$ 520,000 $ 800,000 Separable costs
Costof goods available for sale
$ 580,000 $ 800,000 Deductending inventory
Cost of goods sold
72.50% Gross margin
Gross margin %
$ 290,000 $ 400,000

/gal $ 16.20 /gal


The constant gross-marginpercentage
NRV method works
backward. For each product,
the gross margin(based on the
overall gross-margin percentagel
and separable costs are
deducted fromfinal sales value
of unitsproduced. Theresulting
dollaramountfor each product
is its allocationofjointcosts.

Underthe constantgross-margin
NRV method,the gross-margin
percentage for each product is
the same, regardlessofitsseparable
costs. This method, in
effect, "subsidizes" products
with relatively high separable
costs by assigning less joint
costs to them. That's why the
product,buttercream,has a 25%
gross marginunderthe constant
gross-margin percentage NRV
method(inExhibit16-9)buthas a
22% gross-margin percentage
underthe NRV method(inExhibit
16-7).
$ 1,600,000

$ 1,200,000 Farmers' Dairy Product-


$ 400,000 I line Income Statement for
25% May 2004: Joint Costs
Allocated Using Constant
Gross-Margin Percentage
Condensed NRV Method
Milk Total Revenues (buttercream, 12,000 gal. x $25/gal
condensed milk, 45,000 gal. x $22/gal.)
$ 1,100,000 $ 1,600,000 Cost of goods sold
Joint costs (from Exhibit 16-8)
$ 275,000 $ 400,000 Separable costs
Cost of goods available for sale
$ 825,000 $ 1,200,000 Deduct ending inventory
(buttercream, 8,000 gal. x $18.75/gal.;"
condensed milk, 5,000 gal. x $16.50/gal.)b
$ 520,000 $ 800,000 Cost of goods sold
$ 305,000 $ 400,000 Gross margin
Gross margin%

a $375,000 -;-20,000 gallons = $18.75/gallon.


b $825,000 -;-50,000 gallons = $16.50/gallon.
. The ratio of output quantities to
month to month. Lankip has employed

s before the joint costs are allocated to the main products. Data
During the month, Lankip incurred

cessed further

roduct by using the physical-volume

nt cost – (60,000 pounds of


roduct on a physical-volume

ess. Breegle uses the physical


the following information, which
mize profit?

J-60 H-102
2000 3200
$ 4.00 $ 7.25
$ 1.00 $ 2.50
5.7 9.75
$ 1.70 $ 2.50
$ (1.00) $ (2.50)
$ 0.70 $ -
nt Costs Allocated
ing Sales Value at
itoff Method: Farmers'
iry Product-Line Income
atement for May 2004

Cream Liquid skim Total


$ 160,000 $ 120,000 $ 280,000
sts of goods sold (joint costs)
oduction costs ( $ 160,000 $ 240,000 $ 400,000
duct ending inventory $ 32,000 $ 144,000 $ 176,000
st of goods sold (joint costs) $ 128,000 $ 96,000 $ 224,000
$ 32,000 $ 24,000 $ 56,000
oss margin % 20% 20% 20%
Cream Liquid skim Total
$ 160,000 $ 120,000 $ 280,000
sts of goods sold (joint costs)
oduction costs ( $ 100,000 $ 300,000 $ 400,000
duct ending inventory $ 20,000 $ 180,000 $ 200,000
st of goods sold (joint costs) $ 80,000 $ 120,000 $ 200,000
$ 80,000 $ - $ 80,000
oss margin % 50% 0% 28.6%
ditional processing cossales value sales EI
$ 25 12000 8,000

$ 22 45000 5,000

nt Costs Allocated
ing NRV Method:
rmers' Dairy Product-
e Income Statement for

Buttercream Condensed Milk Total


$ 300,000 $ 990,000 $ 1,290,000
stof goods sold
$ 110,000 $ 290,000 $ 400,000
parable costs $ 280,000 $ 520,000 $ 800,000
stof goods available for sale $ 390,000 $ 810,000 $ 1,200,000
ductending inventory $ 156,000 $ 81,000 $ 237,000
st of goods sold $ 234,000 $ 729,000 $ 963,000
$ 66,000 $ 261,000 $ 327,000
oss margin % 22.0% 26.4% 25.3%
Condensed
Buttercream Milk Total

ttercream, 12,000 gal. x $25/gal.; $ 300,000 $ 990,000 $ 1,290,000


ilk, 45,000 gal. x $22/gal.)

om Exhibit 16-8) $ 95,000 $ 305,000 $ 400,000


$ 280,000 $ 520,000 $ 800,000
available for sale $ 375,000 $ 825,000 $ 1,200,000

, 8,000 gal. x $18.75/gal.;" $ 150,000 $ 82,500 $ 232,500


ilk, 5,000 gal. x $16.50/gal.)b
$ 225,000 $ 742,500 $ 967,500
$ 75,000 $ 247,500 $ 322,500
25% 25%

-20,000 gallons = $18.75/gallon. $ 18.75 $ 16.50


-50,000 gallons = $16.50/gallon.
ACCOUNTING FOR BYPRODUCTS
Joint production processes may yield not only joint products and main products but
bypro ducts as well. Although byproducts have much lower sales values than the salesvalues
of joint or main products, the presence of bypro ducts in a joint production process
can affect the allocation of joint costs. Let's consider a two-product example consisting of
a main product and a byproduct.

Example 3: The Meatworks Group processes meat from slaughterhouses. One of its
departments cuts lamb shoulders and generates two products:
• Shoulder meat (the main product) -sold for $60 per pack $ 60
• Hock meat (the byproduct) -sold for $4 per pack (net of any selling costs) $ 4
Data under each column indicate the number of packs for this department in July 2004

Beginning
Production Sales Inventory
Shoulder meat 5000 4000 0
Hockmeat 1000 300 0

The joint manufacturing costs of these products in July 2004 were $250,000, comprising
$150,000 for direct materials and $100,000 for conversion costs. Both products are soldat
the splitoff point without further processing, as Exhibit 16-10 shows.
Two byproduct accounting methods are presented. Method A, the production
method, recognizes bypro ducts in the financial statements at the time production is completed.
Method B, the sale method, delays recognition of byproducts until the time of
sale. Recognition of bypro ducts at the time of production is conceptually correct.
Recognition at the time of sales often occurs in practice when the dollar amounts of
byproducts are immateriaJ4 Exhibit 16-11 presents the in ome statement of the
Meatworks Group under both methods.

BY PRODUCT ACCOUNTING METHOD


Income Statement of Method A: Method B:
Meatworks Group For July Recognized Recognized
2004 at Production at Sale

Revenues
Main product: shoulder meat (4,000 packs x $60/pack) $ 240,000 $ 240,000
Byproduct: Hock meat (300 packs x $4/packl 0 $ 1,200
Total revenues $ 240,000 $ 241,200
Cost of goods sold
Total manufacturing costs $ 250,000 $ 250,000
Deduct byproduct revenue (1,000 PCJcksx $4/pack) $ 4,000 0
Net manufacturing costs $ 246,000 $ 250,000
Deduct main-product inventory $ 49,200 $ 50,000
Cost of goods sold $ 196,800 $ 200,000
Gross margin $ 43,200 $ 41,200
Gross-margin percentage 18.0% 17.1%
Inventoriable costs (end of period): $ 49,200 $ 50,000
Main product: Shoulder meat
Byproduct: Hock meat (700 packs x $4/packlc $ 2,800 0
'Recorded at selling prices.

MethodA: Byproducts Recognized at lime Production Is Completed


Thismethod recognizes the byproduct in the financial statements - the 1,000 packs of
hockmeat-in the month it is produced, July 2004. The NRVfrom the byproduct producedis
offset against the costs of the main product (see Concepts in Action, p. 570). The
followingjournal entries illustrate this method:

Dr. Cr.
1. Work in Process $ 150,000
Accounts Payable $ 150,000
To record direct materials purchased and
used in production during July.

2. Work in Process $ 100,000


Various accounts $ 100,000
To record conversion costs in the production process
during July; examples include energy, manufacturing
supplies, all manufacturing labor, and plant depreciation.

3. Byproduct lnventory- Hock Meat (1.000 packs x $4/pack) $ 4,000


Finished Goods-Shoulder Meat ($250,000 - $4,000) $ 246,000
Work in Process ($150,000 + $100,000) $ 250,000
To record cost of goods completed during July.
Distinguishing Joint products
from byproducts is based on
the relative magnitude of their
revenues,butthis can be a gray
area requiringjudgment.

Ending
Inventory
1000
700

$ 150,000
$ 100,000
$ 250,000 Joint cost

NG METHOD
Cost allocation, which is a problem in nearly every organization and nearly every facet of
accounting, provides information needed for both strategic and operating decisions.
For example, how should university costs be allocated among undergraduate programs,
graduate programs, and research? How should the costs of expensive medical equipment,
facilities, and staff be allocated in a hospital? How should manufacturing overhead
be allocated to individual products when deciding the products to emphasize in a
multiple-product company such as Heinz?
Television and newspaper stories about questionable cost-charging practices frequently
involve questions of cost allocation. In one case, a patient in a hospital was
charged $17 for a quart of distilled water - $3.40 of direct costs and $13.60 of cost allocations.
Much of this $13.60 was questionably related to the services provided to the
patient. Cost-allocation issues inevitably arise in disputes over large cost overruns on construction
projects and movies as well. One such well-publicized dispute arose in the case
of the highly successful film "Forrest Gump." A writer, whose royalty depended on the
film's profits, disputed the costs allocated to the film.

Chapters 4 and 5 examined topics related largely to the allocation of indirect coststo
individual products. As we saw then, finding answers to cost-allocation questions is often
difficult. The answers are seldom clearly right or wrong: Nevertheless, in this chapter and
the next, we provide insight into cost allocation and the different issues that arise, evenif
the answers seem elusive. The emphasis in this chapter is on macro issues in cost allocation:
allocation of costs to divisions, plants, and customers. We describe customerprofitability
analysis where the customer rather than the product is the cost object and
revenues and costs are assigned to each customer. We also show how the sales-volume
variance introduced in Chapter 7 can be further analyzed when there are multiple cus·
tomers and multiple products. Chapter 15 describes micro issues in cost allocationallocating
support department costs to operating departments and allocating common
costs to various cost objects - as well as revenue allocations.

There is rarely one "best" way This chapter and Chapter 15


to allocate costs. Cost allocation includematerialon cost allocation.
requiresjudgment,and reasonable This chapter deals with
people may differ in macro issues: allocating costs
their judgments. to divisions, plants, and customers.
Chapter 15 deals with
micro issues: allocatingsupport
costs to operating departments
and allocatingcommoncosts to
variouscost objects.

PURPOSES OF COST ALLOCATION


Indirect costs of a particular cost object are costs that are related to that cost object but cannot
be traced to it in an economically feasible (cost-effective) way. These costs often comprise
a large percentage of the overall costs assigned to such cost objects as products,
customers, and distribution channels. Why do managers allocate indirect costs to these
cost objects? Exhibit 14-1 illustrates four purposes of cost allocation.
The allocation of a particular cost need not simultaneously satisfy all four purposes.
Consider the salary of an aerospace scientist in a central research department of Airbus
Industries. This salary cost
* may be allocated to a product as part of central research costs to satisfy purpose 1 (an
economic decision such as pricing),
* may or may not be allocated to a product to satisfy purpose 2 (motivation, such as
reducing future R&D costs of the product),
*may or may not be allocated to a government contract to satisfy purpose 3 (cost reimbursement,
in which the terms of the contract will guide the allocation decision), and
*cannot be allocated to inventory under generally accepted accounting principles (GAAP)
to satisfypurpose 4 (income and asset measurement for reporting to external parties).

Purpose Illustrations

1.Toprovide informa.tion for To decide whether to add a new airline flight


economic decisions To decide whether to manufacture a component part of
a television set or to purchase it from another
manufacturer
To decide on the selling price for a customized product or
service
To evaluate how much of available capacity is being used
to support different propucts,

2.To motivate managers and To encourage the design of products that are simpler to
employees manufacture or less costly to service
To encourage sales representatives to push high-margin
products or services

3.To justify costs or compute To cost products at a "fair" price, often required by
reimbursement government defense contracts
To compute reimbursement for a consulting firm that is
based on a percentage of the cost savings resulting
from the implerpentation of its recommendations

4.To measure income and assets To cost inventories for financial reporting. (Under generally
for reporting to external parties accepted accounting principles, invento[iable costs
include manufacturing costs but exclude research and
development, marketing, <:IistribHtion, and customerservice
costs.)'
To cost inventories for reporting to tax authorities
Differentcosts are appropriate for different purposes. Consider costs of a product in terms
ofthe business functions in the value chain.
The same set of costs in these six business functions typically will not satisfy each of the
four purposes i Exhibit 1.4-1.
For some decision related to the economic-decision purpose (for example, long-run
product pricing), the costs in all 'six functions should be included.
For the motivatio purpose, ,costs from more than one business function are often
included to emphasize to managers how costs in different functions are related to each
other. For example, product designers in some Japanese companies incorporate costs of
other functions in the value chain - such as production, distribution, and customer service-+
into their product-cost estimates. The aim is to focus attention on how different
product design options affect total costs.
For the cost-reimbursement purpose, the particular contract will often stipulate
whether all six of the business functions or only a subset of them are to be reimbursed.
For instance, cost-reimbursement rules for U.S. government contracts explicitly exclude
marketing costs.
For the purpose of income and asset measurement for reporting to external parties,
inventoriable costs under GAAPinclude only manufacturing costs (and product design
costs in some cases). In the United States, R&D costs in most industries are a period cost
when they are incurred,1 as are marketing, distribution, and customer-service costs

CRITERIA TO GUIDE COST-ALLOCATION DECISIONS

Role of Dominant Criteria

Exhibit 14-2 presents four criteria used to guide cost-allocation decisions. These decisions
affect both the number of indirect-cost pools and the cost-allocation base for each indirect-
cost pool. Managers must first identify the purpose for a particular cost allocation
and then selec the criteria, to aiIocate costs. e emp asize the supe£iority ofthe causeand-
effect and the benefits-received criteria, especially when the purpose of cost allocation
is economic decisions or motivation.2 -

Fairness and ability to bear are less frequently used criteria than cause and effector
benefits received. Fairness is a difficult criterion on which to obtain agreement. What one
party views as fair, another party may view as unfair.3 For example, a university may view
allocating a share of general administrative. costs to government contracts as fair because
general administrative costs are incurred to support all activities of the university. The government
may view the allocation of such costs as unfair because the general administrative
costs would have been incurred by the university whether or not the government contract
had been signed. To get a sense of the ability-to-bear criterion, consider a product that
consumes a large amount of indirect costs but whose selling price is currently below its
direct costs. This product has no ability to bear any of the indirect costs it uses. If the
indirect costs it consumes are allocated to other products, these other products are effectively
subsidizing the product that is losing money.
The cause-and-effect criterion is the primary one used in activity-based costing
(ABC) applications. ABC systems use the concept of a cost hierarchy to identify the
cost drivers that best demonstrate the cause-and-effect relationship between each
activityand the costs in the related cost pool. The cost drivers are then chosen as costallocation
bases.

Criteria for Cost-


Allocation Decisions

1. Cause and Effect. Using this criterion, managers identify the variables that cause resources to be
consumed. For example, managers may use hours of testing as the variable when allocating the costs
of a quality-testing area to products. Cost allocations based on the cause-and-effect criterion are likely
to be the most credible to operating personnel.
2. Benefits Received. Using this criterion, managers identify the beneficiaries of the ~ of the cost
object. The costs of the cost object are allocated among the beneficiaries in proportion to the benefits
each receives. Consider a corporatewide advertising program that promotes the general image ofthe
corporation rather than any individual product. The costs of this program may be allocated on the basis
of division reven~s; the higher the revenues, the higher the division's allocated cost of the advertising
program. The rationale behind this allocation is that divisions with higher revenues apparently
ene 'ted from the advertising more than divisions with lower revenue~ and, therefore, ought to be
allocated more of the advertising costs.
3. Fairness or Equity. This criterion is often cited in government contracts when cost allocations are
the basis for establishing a price satisfactory to the government and its suppliers. Cost allocation here
is viewed as a "reasonable" or "fair" means of establishing a selling price in the minds ofthe
contracting parties. For most allocation decisions, fairness is a difficult-to-achieve objective rather
than an operational criterion.
4. Ability to Bear. This criterion advocates allocating costs in proportion to the cost Object's ability
bear costs allocated to it. An example is the allocation of corporate executive salaries on the basis of
division operating income. The presumption is that the more-profitable divisions have a greater ability
to absorb corporate headquarters' costs.

COST ALLOCATION AND COSTING SYSTEMS


In this section, we focus on the first purpose of cost allocation, to provide information for
economic decisions such as pricing, by measuring the full cosJs of delivering products
based on an ABC system ..

Chapter 5 described how ABC systems define activity-cost pools and use activity-cost
drivers as allocation bases to assign costs of activity-cost pools to products. In this section,
wefocus on how costs are assigned to the activity-cost pools.

We will use Consumer Appliances, Ine. (CAI), to illustrate how costs incurred in different
parts of a company can be assigned, and then reassigned, for costing products, services, customers, or contracts.

Division Activity-Cost Pools and Cost-Allocation Bases, CAI. Inc., for Refrigerator Division (.R)and Clothes Dryer Division (CD)

cost cost
hierarchy allocation
Activity examples of costs amount category base

Design design (.R) $6,000,000 Product Parts


engineering salaries (CD) $4,250,000 sustaining times cubic feet
Setups of Setup labor and (.R) $3,000,000 Batch Setup-
machines equipment cost (CD) $2,400,000 level hours

Manufacturing Plant and (.R) $25,000,000 Output Machine


operations equipment (CD) $18,750,000 unit level hours
energy

Distribution Shipping labor and (.R) $8,000,000 Output Cubic


equipment (CD) $5,500,000 unit level feet

Administration Division executive (.R) $1,000,000 Facility Revenues


salaries (CD) $800,000 sustaining

Facility Annual building and (.R) $4,500,000 All squere


space costs (CD) $3,500,000 feet

CAI has two divisions and each has its own manufacturing plant the
Refrigerator Division with a plant in Minneapolis and the Clothes Dryer Division witha
plant in St. Paul. CAI's headquarters is in a separate location in Minneapolis. In each division,
CAI manufactures and sells multiple products that differ in size and complexity.

CAI collects costs at the following levels:

1. Corporate costs - there are three major categories of corporate costs:

• Treasury costs - interest of $900,000 on debt used to finance the construction of $900,000
new assembly equipment in the two divisions. Cost of new assembly equipment is
$5,200,000 in the Refrigerator Division and $3,800,000 in the Clothes Dryer $5,200,000
Division. $3,800,000
• Human resource management costs-recruitment and ongoing employee training
and development, $1,600,000. $1,600,000
• Corporate administration costs - executive salaries, rent, and general administration
costs, $5,400,000. $5,400,000

2. Division costs - there are two direct-cost categories (direct materials and direct manufacturing
labor) and six indirect-cost categories

Exhibit 14-3 presents the division activitycost


pools and cost-allocation bases. CAI identifies the cost hierarchy category for each
cost pool- output unit-level, batch level, product-sustaining level, and facilitysustaining
level (as described in Chapter 5, pp. 143-144).

Exhibit 14-4 presents an overview diagram of the allocation of corporate and division
overhead costs to products for CAI. Look first at the middle row of the exhibit, where you
see "Division Indirect Costs," and scan the lower half. It is similar to Exhibit 5-3, PanelA
(p. 146), which illustrates ABCsystems using activity-cost pools and cost drivers. The only
additional feature in the lower half of Exhibit 14-4 is that CAI has a cost pool called
Facility Costs (far right, middle row), which accumulates all annual costs of buildings and
furnishings (such as depreciation) incurred in the division. The arrows in the exhibit indicate
that CAI allocates these costs to the other activity-cost pools using the square feet area
required for the different activities (design, setup, manufacturing, distribution, and
administration). The activity-cost pools then include the costs of the building and facilities
needed to perform the various activities.
The activity-cost pools are allocated to products on the basis of cost drivers described
in Exhibit 14-3. These cost drivers are chosen as the cost-allocation bases because there is
a cause-and-effect relationship between the cost drivers and the costs in the activity-cost
pool. A cost rate per unit is calculated for each cost-allocation base. Indirect costs are allocated
to products on the basis of the total quantity of the cost-allocation base for each
activity used by the product.

Next focus on the upper half of Exhibit 14-4: how corporate costs are allocated to divisions
and then to activity-cost pools. Before getting into the details of the allocations, let's
first consider some broader choices that CAIfaces regarding the allocation of corporate costs

Overview Diagram of Allocation of Corporate Overhead and Division Overhead Costs


to Products. CAI. Inc

Corporate Corporate Corporate


ORPORATE { Treasury Human Resource Administration
COSTS Costs Management (CA) Costs
(CHRM) Costs
$900,000 $1,600,000 $5,400,000

CORPORATE { cost of new salary & Division Admin.


COST-ALLOCATION assembly equipment labor cost cost
BASE R $5,200,000 R $44,000,000 $1,000,000
CD $3,800,000 CD $36,000,000 $800,000

CORPORATE{ Refrigerator Clothes Dryer


COSTS Division Division
ALLOCATEDTO
DIVISIONS Treasury $520,000 $380,000

(CHRM) Co $880,000 $720,000

Division Ad $3,000,000 $2,400,000

DIVISION { Allocated Design Machine


INDIRECT Corporate including (CHRM) Costs Setups of
COSTS Treasury including (CHRM) Costs
Costs
DIVISION { Machine-Hours Parts Setup-
COST-ALLOCATION on New X Cubic feet hours
BASE Equipment

COST OBJECT:{
REFRIGERATOR
AND CLOTHES
DRYERPRODUCTS

Allocating Corporate Costs to Divisions and Products


CAI has several choices to make when accumulating and allocating corporate costs to
divisions.
1. Which corporate cost categories should be included in the indirect costs of the divisions?
Should CAI allocate all corporate costs or only a subset of them?

a. Some companies allocate all corporate costs to divisions. They believe that corporate
costs are incurred to supprivision activities. Allocating all corporate costs motivates
division managers to examine how corporate costs are planned and controlled. Als
companies that want to calculate the ~ll cost of products must allocate corporate costs
to activity-cost pools of divisions

b. Other companies do not allocate corporate costs to divisions. They believe that these
costs are not controllable by division managers.

c. Still other companies allocate only those corporate costs, such as corporate human if, resources, that are widely perceive
provide explicit b fi divisions. These companies exc ude corporate costs such as
corporate donations to charitable foundations because division managers often have no
corporate donations to charitable foundations because division managers often have no
sayin making these decisions and because the benefits to the divisions are less evident or
too remote

For some decision purposes, allocating some but not all corporate costs to division
may be the preferred alternative. Consider the performance evaluation of division managers.
The controllability notion (see pp. 192-193) is frequently used to justify excluding
some corporate costs from division reports. For example, the salaries of the top
management at corporate headquarters are often excluded from responsibility accounting
reports of division managers. Although divisions tend to benefit from these corporate
costs, division managers argue they have no say in ("are not responsible for")
how much of these corporate~ces they use or how much they cost. The contrary
argument is that full allocation is justified because the divisions receive benefits from
all corporate costs

2. If CAI allocates corporate costs to divisions, how many cost pools should it use?
One extreme is to aggregate all corporate costs into a single cost pool. The other extreme
is to have numerous individual corporate cost pools. A variety of factors may prompt
managers to consider using multiple cost pools. A major factor is the concept of homogeneous
cost pools.

In a homogeneous cost pool, all of the costs in the cost pool have the same or a similar
cause-and-effect or benefits-received relationship with the cost-allocation base.

Homogeneity of costs is important because creating homogeneous cost pools leads to


more-accurate costs of a given cost object. If a homogeneous cost pool exists, the cost allocations
using that pool will be the same as they would be if costs of each individual activity
in that pool were allocated separately. The greater the degree of homogeneity, the fewer
I the number of cost pools required to explain accurately the differences in how divisions
or products use resources of the company.

For example, when allocating corporate costs to divisions, CAI can combine corporate
administration costs and corporate human resource management costs into a single cost
pool if both cost categories have the same or a similar cause-and-effect relationship with
the same cost-allocation base (say, number of employees in each division). If, however,
each cost category has a cause-and-effect relationship with a different cost-allocation base
(for example, number of employees in each division affects corporate human resource
management costs, whereas revenues of each division affect corporate administration
costs), CAI will prefer to maintain separate cost pools for each of these costs. Determining
homogeneous cost pools requires judgment and should be revisited on a regular basis to
evaluatewhether the cause-and-effect relationship between the cost-allocation base and
the different cost categories in a cost pool has changed.

Another factor in deciding on the number of cost pools is the views of managers. Do
theybelieve that aggregating corporate costs into a single cost pool ignores important
differencesin how divisions use corporate resources? A final factor is the costs of implementinga
multiple cost-pool system. Improvements in information-gathering technologyare
enhancing the capability of companies and reducing the cost of using multiple
costpools.

3. If CAI allocates corporate costs to divisions, which allocation bases should it use?
Generally, the ones that have the best cause-and-effect relationship with costs.

Implementing Corporate Cost Allocations


Weillustrate the allocation of corporate costs to divisions in CAI's ABC system. CAI
choosesto allocate all corporate costs to divisions. It could have chosen the option of not
allocatingsome corporate costs to divisions and, hence, to products. 'Not allocating some
costswould result in total company profitability being less than the sum of individual
divisionand product profitabilities.
The demands for corporate resources by the Refrigerator Division and the Clothes
DryerDivision depend on the demand that each division's products place on these
resources.Exhibit 14-4 graphically represents the allocations.

1. CAI allocates treasury costs to each division on the basis of the cost of new assembly
equipment installed in each division (the cost driver of treasury costs). It allocates the
$900,000 of treasury costs as follows (using information from p. 486):

Each division then creates a separate cost pool consisting of the allocated corporate
treasury costs and reallocates these costs to products on the basis of machinehours
used on the new equipment. Treasury costs are an output unit-level cost because
they represent resources sacrificed on activities performed on each individual unit of
a product.

2. CArs analysis indicates that the demand for corporate human resource management
(CHRM) costs for recruitment and training varies with total salary and labor costs.
Asa result, these costs are allocated to divisions on the basis of the total salary and labor
costs incurred in each division. Suppose salary and labor costs are $44,000,000 in the
Refrigerator Division and $36,000,000 in the Clothes Dryer Division. Then CHRM costs
areallocated to the divisions as follows:
$1,600,000 x $44,000,000 = $880,000
$44,000,000 + $36,000,000
$1,600,000x $36,000,000 = $720,000
$44,000,000 + $36,000,000
Each division reallocates the CHRM costs allocated to it to the indirect-cost pools of
design, machine setup, manufacturing operations, distribution, and division administration
on the basis of the total salary and labor costs incurred for each of these activities.
CHRMcosts are added to the indirect-cost pools in each divisioll and alloc;ated to products
using the cost driver for each cost pool. Thus, CHRM costs are product-sustaining
costs (for the portion of CHRM costs allocated to the design cost pool), batch-level costs
(for the portion of CHRM costs allocated to the machine setup cost pool), output unitlevelcosts
(for the portions of CHRM costs allocated to the manufacturing operations and
distribution cost pools), and facility-sustaining costs (for the portion of CHRM costs allocated
to the division administration cost pool).
3. CAI allocates corporate administration costs to each division on the basis of division
administration costs (see Exhibit 14-3) because corporate administration's main role
is to support division management.

Refrigerator Division:
Clothes Dryer Division:
$5,400,000 x $1,000,000 =$3,000,000
$1,000,000 + $800,000
$5,400,000 x $800,000 = $2,400,000
$1,000,000 + $800,000
Each division adds the allocated corporate administration costs to the division administration
cost pool. The costs in this cost pool are facility-sustaining costs and do not have a
cause-and-effect relationship with individual products pr9duced and sold by each division.
CAl's policy, however, is to allocate all costs to products so that CAl's division managers
become aware of all costs incurred at CAI in their pricing and other decisions. It allocates
the division administration costs (including allocated corporate administration costs) to
products on the basis of product revenues (a benefits-received criterion).
Exhibit 14-4 highlights the different ways CAI allocates corporate overhead costs to
divisions and then to products. The company
• Establishes a separate activity-cost pool at the division level to allocate corporate treasury
costs to products.
• Allocates CHRM costs to divisions on the basis of division salary and labor costs and
then reallocates the CHRM costs allocated to divisions to multiple activity-cost pools
on the basis of the total salary and labor costs in each cost pool. Thus, the activity-cost
pools of each division include an allocation of CHRM costs, as shown in Exhibit 14-4.
• Allocates corporate administration costs to divisions on the basis ~idivision administration
costs and adds these costs to a single activity-cost pool (division administration)
as shown in Exhibit 14-4.
As we described in Chapter 5, focusing on activities and the hierarchy of costs promotes
cost management. The set of activities and the actions necessary to manage costs
are different if a cost is an output unit-level cost, a batch-level cost, a product-sustaining
cost, or a facility-sustaining cost. For example, to manage setup cost, which is a batch-level
cost, CAI must focus on batch-level activities, such as ways to reduce setup-hours and the
cost per setup-hour.
Exhibit 14-4 also reinforces the idea that costing systems have multiple cost objects,
such as corporate headquarters, divisions, departments, and products. An individual cost
item can be simultaneously a direct cost of one cost object and an indirect cost of another
cost object. Consider the salary of the manager of CHRM at CAL Her salary is a direct cost
of the Corporate Human Resource Management group at CAl's corporate office. It is then
allocated as an indirect cost to each of CAl's divisions and reallocated as an indirect cost
of the products in each division.
The issues discussed in this section regarding divisions and products apply nearly
identically to customers, as we shall show next. Instructors and students who, at this point,
want to explore more-detailed issues in cost allocation rather than customer issues can skip ahead
to Chapter 15.

In this problem,
the cost of central corporate support departments are allocated to operating divisions. The corporate
departments provide services to each other as well as to the operating divisions. Also, this problem
illustrates the use of the dual-rate method of allocating support departments' costs. (The dual-rate
method can also be used in manufacturing support department cost allocations).
Computer Horizons budgets the following amounts for its two central corporate support
departments (legal and personnel in supporting each other and the two manufacturing divisions,
the Laptop Division (LTD) and the Work Station Division (WSD):

Budgeted Usage by
To Be Supplied By Legal Personnel LTD WSD Total
Legal (hours) - 250 1,500 750 2,500
(percentages) 0% 10% 60% 30% 100%
Personnel (hours) 2,500 - 22,500 25,000 50,000
(percentages) 5% 0% 45% 50% 100%

Actual Usage by
To Be Supplied By Legal Personnel LTD WSD Total
Legal (hours) - 400 400 1,200 2,000
(percentages) 0% 20% 20% 60% 100%
Personnel (hours) 2,000 - 26,600 11,400 40,000
(percentages) 5% 0% 66.5% 28.5% 100%

Actual costs
Legal Personnel
Fixed $ 360,000 $ 475,000
Variable $ 200,000 $ 600,000

fixed costs are allocated on the basis of budgeted capacity. Variable costs are allocated on the basis
ofactualusage.
LTD WSD
(a) Direct Method
Fixed costs cost X budgeted rate $ 240,000 $ 120,000
$ 225,000 $ 250,000
Total FC allocated $ 465,000 $ 370,000

Variable costs actual


cost x actual rate $ 50,000 $ 150,000
$ 420,000 $ 180,000
Total VC allocated $ 470,000 $ 330,000

B. Step-Down Method CORPORATESUPPORT


DEPARTMENT LTD
(Legal Department First) Legal Personnel
Fixed Costs $ 360,000 $ 475,000
Legal (250 -;-2,500; 1,500 -;-2,500; 750 -;-2,500) $ (360,000) $ 36,000 $ 216,000
Personnel (22,500 -;-47,500; 25,000 -;-47,500) $ - $ (511,000) $ 242,053
$ - $ 458,053
Variable Costs
Legal (400 -;-2,000; 400 -;-2,000; 1,200 -;-2,000) $ 200,000 $ 600,000 $ 40,000
Personnel (26,600 -;-38,000; 11,400 -;-38,000 $ (200,000) $ 40,000 $ 448,000
$ - $ (640,000) $ 488,000
A "central" department (such
as a central research departmentl
provides services inside
the company (for example,
research to the divisions of a
company).

Cost allocations can be used to


motivate managers to consume
less or more of the company's
resources. To discourage use,
the cost of a department's
services could be allocated
according to the amount of services
used. To encourage use
of a department's services (for
example, internal audit). top
management might (1) not
allocate any of the cost of
that department's services or
(2) allocate a fixed amount of
the cost of that department to
other departments regardless
of how much of those services
are used by those other departments
(the other departments
may feel obligated to use the
services to get their "money's
worth
Using the cause-and-effect
criterion to choose a costallocation
base means the base
is a coSt driver of the indirectcost
pool.
The following sequential outline
gives the "big picture" of cost
allocation:
1. Determine the purpose ofthe
allocation, because the purpose
defines what costs will
be allocated.
2. Decide how to allocate the
costs from step 1. To do so,
a. Decide how many indirect-
cost pools to form,
and then
b. Identify an allocation
base (preferably a cost
driver) for each cost pool.

, or contracts.

thes Dryer Division (CD)

Cause-and-Effect Relationship
That Motivates the Choice of

Complex products (more parts and


larger size) require greater design resources
Overhead costs of the setup
activity increase as setup
hours increase.

Manufacturing operations
overhead costs support
machines and, hence, increase with machine usage

Distribution overhead costs


increase with cubic feet of product shipped.

Weak relationship between


division executive salaries and revenues, but justified by CAI on a benefits-received basis

FaciIity costs increase


with square feet of space
$900,000

$1,600,000

$5,400,000

Manufacturing Distribution Administration


operations including (CHRM) Costs including (CHRM) Costs
including (CHRM) Costs
Machine Cubic Revenues
hours feet

that are widely perceived as either, causall related to division activities or


When cost allocations are made using budgeted rates, managers of divisions to which
costs are allocated face no uncertainty about the rates to be used in that budget period.
In contrast, when actual rates are used for cost allocation, managers do not know the
rates to be used until the end of the budget period. If actual rates are used, the efficiency
of the supplier department affects the costs allocated to the user department

WSD Total

$ 108,000
$ 268,947
$ 376,947 $ 835,000
$ 120,000
$ 192,000
$ 312,000 $ 800,000
Facility
including (CHRM) Costs
rs of divisions to which
sed in that budget period.
agers do not know the
tes are used, the efficiency
ser department
103. Gardener Company currently is using its full capacity of 25,000 machine hours to
manufacture product XR-2000.
LJB Corporation placed an order with Gardener for the manufacture of 1,000 units of KT-6500.
LJB would normally manufacture this component.
However, due to a fire at its plant, LJB needs to purchase these units to continue manufacturing other products. This is a o

The following reflects unit cost data, and selling prices.

KT-6500

Material $27.0
Direct labor $12.0
Variable overhead $6.0
Fixed overhead $48.0
Variable selling & administrative $5.0
Fixed selling & administrative $12.0
Normal selling price $125.0
Machine hours required 3.00

What is the minimum unit price that Gardener should charge LJB to manufacture 1,000
units of KT-6500?

CM $75.0
CM per machine hour $25.0

the minimum price must equal at least the VC to produce KT-6500 plus any foregone CM by not producing XR-2000

VC to produce KT-6500 $50.0

Total hours that will be used to prouce KT-6500 instead of producing XR 3,000.00
CM that will be lost from not prducing XR $46,500.0
CM per unit that will be lost from not prducing XR $46.5

The minimum unit price that Gardener should charge LJB to manufacture 1,000 units of KT-6500
$96.50

106. Following are the operating results of the two segments of Parklin Corporation.

Segment A
Sales $10,000.0
Variable costs of goods sold $4,000.0
Fixed costs of goods sold $1,500.0
Gross margin $4,500.0
Variable selling and administrative $2,000.0
Fixed selling and administrative $1,500.0
Operating income (loss) $1,000.0

Variable costs of goods sold are directly related to the operating segments. Fixed costs of
goods sold are allocated to each segment based on the number of employees. Fixed
selling and administrative expenses are allocated equally. If Segment B is eliminated,
$1,500 of fixed costs of goods sold would be eliminated. Assuming Segment B is closed,
the effect on operating income would be

Segment A

Sales $10,000.0
Variable costs of goods sold $4,000.0
Fixed costs of goods sold $1,500.0
Gross margin $4,500.0
Variable selling and administrative $2,000.0
Fixed selling and administrative $1,500.0
Operating income (loss) $1,000.0

Assuming Segment B is closed, the effect on operating income would be

107. Edwards Products has just developed a new product with a manufacturing cost of $30.

The Marketing Director has identified three marketing approaches for this new product.

Approach X Set a selling price of $36 and have the firm’s sales staff sell the
product at a 10% commission with no advertising program.
Estimated annual sales would be 10,000 units.
Approach Y Set a selling price of $38, have the firm’s sales staff sell the
product at a 10% commission, and back them up with a $30,000
advertising program. Estimated annual sales would be 12,000
units.
Approach Z Rely on wholesalers to handle the product. Edwards would sell the
new product to the wholesalers at $32 per unit and incur no selling
expenses. Estimated annual sales would be 14,000 units.
Approach X

selling price $36.0


Q 10,000.00
Rev $360,000.0
manufacturing cost $300,000.0
V selling $36,000.0
Fixed selling
net profit $24,000.0

Rank the three alternatives in order of net profit, from highest net profit to lowest.
107. C Z, X, Y.

108. Parker Manufacturing is analyzing the market potential for its specialty turbines. Parker
developed its pricing and cost structures for their specialty turbines over various relevant
ranges. The pricing and cost data for each relevant range are presented below.

Units produced and sold 1-5


Total fixed costs $200,000.0
Unit variable cost $50,000.0
Unit selling price $100,000.0

Which one of the following production/sales levels would produce the highest operating
income for Parker?

a. 8 units. Rev
b. 10 units. VC
c. 14 units. FC
d. 17 units. OI
8 10 14 17
Rev $800,000.0 $1,000,000.0 $1,400,000.0 $1,700,000.0
VC $400,000.0 $500,000.0 $630,000.0 $765,000.0
FC $400,000.0 $400,000.0 $600,000.0 $800,000.0
OI $0.0 $100,000.0 $170,000.0 $135,000.0
109. Elgers Company produces valves for the plumbing industry. Elgers’ per unit sales price and variable costs are as follow

Sales price $12 $12.0


Variable costs 8 $8.0
Elgers’ practical plant capacity is 40,000 units. 40,000.00
Elgers’ total fixed costs aggregate and. $48,000.0
it has a 40% effective tax rate 40%

The maximum net profit that Elger can earn is

Rev 480000
VC 320000
FC $48,000.0
OI $112,000.0
Tax $44,800.0
Net I $67,200.0

110. Dayton Corporation manufactures pipe elbows for the plumbing industry. Dayton’s per
unit sales price and variable costs are as follows.

Sales price $10 $10.0


Variable costs 7 $7.0
Dayton’s practical plant capacity is 35,000 units. 35,000.00
Dayton’s total fixed costs amount to $42,000, $42,000.0
and the company has a 50% effective tax rate. 50%

If Dayton produced and sold 30,000 units, net income would be 30,000.00

Rev $300,000.0
VC $210,000.0
FC $42,000.0
OI $48,000.0
Tax $24,000.0
Net I $24,000.0

111. Raymund Inc., a bearings manufacturer, has the capacity to produce 7,000 bearings per
month. The company is planning to replace a portion of its labor intensive production
process with a highly automated process, which would increase Raymund’s fixed
manufacturing costs by $30,000 per month and reduce its variable costs by $5 per uni
Raymund’s Income Statement for an average month is as follows.

Sales (5,000 units at $20 per unit) $100,000 5,000.00 $20.0


Variable manufacturing costs $50,000 $10.0 $50,000.0
Variable selling costs 15,000 65,000 $15,000.0
Contribution margin 35,000

Fixed manufacturing costs 16,000 $16,000.0


Fixed selling costs 4,000 20,000 $4,000.0
Operating income $ 15,000

If Raymund installs the automated process, the company’s monthly operating income
would be

Sales (5,000 units at $20 per unit) $100,000 5,000.00 $20.0


Variable manufacturing costs $50,000 $5.0 $25,000.0
Variable selling costs 15,000 65,000 $15,000.0
Contribution margin 35,000

Fixed manufacturing costs 16,000 $46,000.0


Fixed selling costs 4,000 20,000 $4,000.0
Operating income $ 15,000

112. Refrigerator Company manufactures ice-makers for installation in refrigerators. The costs per unit, for 20,000 units of

Direct materials $ 7 $7.0


Direct labor 12 $12.0
Variable overhead 5 $5.0
Fixed overhead 10 $10.0 $200,000.0
Total costs $34 $34.0

Cool Compartments Inc. has offered to sell 20,000 ice-makers to Refrigerator Company for $28 per unit.
If Refrigerator accepts Cool Compartments’ offer the plant would be idled and fixed overhead amounting to $6 per unit co
The total relevant costs associated with the manufacture of ice-makers amount to
relevant
Direct materials $ 7 $7.0
Direct labor 12 $12.0
Variable overhead 5 $5.0
Fixed overhead 10 $6.0
Total costs $34 $30.0 $600,000.0
113. Phillips and Company produces educational software. Its current unit cost, based upon an anticipated volume of 150,0

Selling price $150 $150.0


Variable costs 60 $60.0
Contribution margin 90 $90.0
Fixed costs 60 $60.0 $9,000,000.0
Operating income 30 $30.0

Sales for the coming year are estimated at 175,000 units, which is within the relevant range of Phillip’s cost structure.
Cost management initiatives are expected to yield a 20% reduction in variable costs and a reduction of $750,000 in fixed c
.

Selling price $150 $150.0 $26,250,000.0


Variable costs 60 $48.0 $8,400,000.0
Contribution margin 90 $102.0 $17,850,000.0
Fixed costs 60 $47.1 $8,250,000.0 $8,250,000.0
Operating income 30 $54.9 $9,600,000.0

Phillip’s cost structure for the coming year will include a: variable cost ratio of 32% and operating income of $9,6

114. Sunshine Corporation is considering the purchase of a new machine for $800,000.
The machine is capable of producing 1.6 million units of product over its useful life.
The manufacturer’s engineering specifications state that the machine-related cost of producing each unit of product should
Sunshine’s total anticipated demand over the asset’s useful life is 1.2 million units.
The average cost of materials and labor for each unit is $.40.

In considering whether to buy the new machine, would you recommend that Sunshine use the manufacturer’s engineer
b. No, the machine-related cost of producing each unit is $.67.

manufacturer’s engineering specification


demand

115. Cervine Corporation makes two types of motors for use in various products. Operating
data and unit cost information for its products are presented below.
Product A
Annual unit capacity 10,000 20,000 10,000.00
Annual unit demand 10,000 20,000 10,000.00
Selling price $100 $80 $100.0
Variable manufacturing cost 53 45 $53.0
Fixed manufacturing cost 10 10 $10.0
Variable selling & administrative 10 11 $10.0
Fixed selling & administrative 5 4 $5.0
Fixed other administrative 2 0 $2.0
Unit operating profit $ 20 $10 $20.0
Machine hours per unit 2.0 1.5 2

Cervine has 40,000 productive machine hours available. 40,000.00

The relevant contribution margins, per machine hour for each product, to be utilized in making a decision on product prior

Product A
CM $37.0
CM per machine hours $18.5

116. Two months ago, Hickory Corporation purchased 4,500 pounds of Kaylene at a cost of$15,300.
The market for this product has become very strong, with the price jumping to $4.05 per pound.
Because of the demand, Hickory can buy or sell Kaylene at this price.
Hickory recently received a special order inquiry that would require the use of 4,200 pounds of Kaylene.
In deciding whether to accept the order, management must evaluate a number of decision factors.
Without regard to income taxes, which one of the
following combination of factors correctly depicts relevant and irrelevant decision
factors, respectively?

Relevant Decision Factor


b. Market price of $4.05 per lb.

117. Reynolds Inc. manufactures several different products, including a premium lawn fertilizer and weed killer that is popu
Reynolds is currently operating at less than full capacity because of market saturation for lawn fertilizer.
Sales and cost data for a 40-pound bag of Reynolds lawn fertilizer is as follows.

Selling price
Production cost
Materials and labor $12.3
Variable overhead $3.8
Allocated fixed overhead $4.0
Income (loss) per bag $(1.50)

On the basis of this information, which one of the following alternatives should be
recommended to Reynolds management?

c. Continue to produce and market this product., since it is has a $2.5 CM Reynolds Inc. manufactures several different prod

CM

119. Capital Company has decided to discontinue a product produced on a machine purchased four years ago at a cost of $
The machine has a current book value of $30,000.
Due to technologically improved machinery now available in the marketplace the existing machine has no current salvage v
The company is reviewing the various aspects involved in the production of a new product.
The engineering staff advised that the existing machine can be used to produce the new product.
Other costs involved in the production of the new product will be materials of $20,000 and labor priced at $5,000.
$20,000.00
Ignoring income taxes, the costs relevant to the decision to produce or not to produce the
new product would be $25,000.00

123. Allred Company sells its single product for $30 per unit. $30.00
The contribution margin ratio is 45%, 45%
and fixed costs are $10,000 per month.
Allred has an effective income tax rate of 40%
. If Allred sells 1,000 units in the current month, 1000
Allred’s variable expenses would be

VC ratio 55%
sales revenue $30,000.00
VC $16,500.00

124. Phillips & Company produces educational software. Its unit cost structure, based upon
an anticipated production volume of 150,000 units, is as follows. 150000

Sales price $160 $160


Variable costs 60 $60
Fixed costs 55 $55

The marketing department has estimated sales for the coming year at 175,000 units,which is within the relevant range of P
Phillip’s break-even volume (in units) and anticipated operating income for the coming year would amount to

total FC $8,250,000
CM $100

Rev $28,000,000
VC $10,500,000
CM $17,500,000
FC $8,250,000
OI $9,250,000

Break=FC/CM 82,500.00 units

126. Jeffries Company sells its single product for $30 per unit. $30
The contribution margin ratio is 45%, and. 45%
fixed costs are $10,000 per month $10,000
Sales were 3,000 units in April and 4,000 units in May. 3,000 4,000
How much greater is the May income than the April income?
April May
Rev $90,000 $120,000
VC 55% $16.50 $49,500 $66,000
CM $40,500 $54,000
FC $10,000 $10,000
OI $30,500 $44,000

127. Cervine Corporation makes two types of motors for use in various products. Operating
data and unit cost information for its products are presented below.
Product A

Annual unit capacity 10,000 20,000 10,000.00


Annual unit demand 10,000 20,000 10,000.00
Selling price $100 $80 $100.0
Variable manufacturing cost 53 45 $53.0
Fixed manufacturing cost 10 10 $10.0
Variable selling & administrative 10 11 $10.0
Fixed selling & administrative 5 4 $5.0
Fixed other administrative 2 0 $2.0
Unit operating profit $ 20 $10 $20.0
Machine hours per unit 2.0 1.5 2.0

Cervine has 40,000 productive machine hours available. 40,000.00

. What is the maximum total contribution margin that Cervine can generate in the coming year?

Product A
CM $37.00
CM per machine hours $18.50

first produce as much we can from product A 10,000


Total Machine hours 20,000
Total CM 370,000

the remaining hours to produce product B


total unit that can be produced from prdoct B
Total CM

Total CM

128. Lazar Industries produces two products, Crates and Trunks. Per unit selling prices, costs,
and resource utilization for these products are as follows.

Crates
Selling price $20 $30 20
Direct material costs $ 5 $ 5 5
Direct labor costs 8 10 8
Variable overhead costs 3 5 3
Variable selling costs 1 2 1
Machine hours per unit 2 4 2

Production of Crates and Trunks involves joint processes and use of the same facilities.
The total fixed factory overhead cost is $2,000,000 and total fixed selling and administrative costs are $840,000.
Production and sales are scheduled for 500,000 Crates and 700,000 Trunks.
Lazar has a normal capacity to produce a total of 2,000,000 units in
any combination of Crates and Trunks, and maintains no direct materials, work-inprocess,
or finished goods inventory.

Due to plant renovations Lazar Industries will be limited to 1,000,000 machine hours.
What is the maximum amount of contribution margin Lazar can generate during the
renovation period?
Crates
CM 3.00
CM per machine hours 1.50

first produce as much we can from Trunks


with Total Machine hours
Total CM

129. For the year just ended, Silverstone Company’s sales revenue was $450,000.
Silverstone’s fixed costs were $120,000 and it
s variable costs amounted to $270,000.
For the current year sales are forecasted at $500,000.
If the fixed costs do not change, Silverstone’s profits this year will be

For the current year sales are forecasted at $500,000.


CM will be (1-VC ratio 60%) 200000
OI $80,000

130. Breeze Company has a contribution margin of $4,000 and fixed costs of $1,000. If the
total contribution margin increases by $1,000, operating profit would

131. Wilkinson Company sells its single product for $30 per unit. The contribution margin
ratio is 45% and Wilkinson has fixed costs of $10,000 per month. If 3,000 units are sold
in the current month, Wilkinson’s income would be
133. Starlight Theater stages a number of summer musicals at its theater in northern Ohio.
Preliminary planning has just begun for the upcoming season, and Starlight has
developed the following estimated data.

Average
Number of Attendance per Ticket
Production Performances Performance Price
Mr. Wonderful 12 3500 $18.00
That’s Life 20 3000 $15.00
All That Jazz 12 4000 $20.00

1 Represent payments to production companies and are based on tickets sold.


2 Costs directly associated with the entire run of each production for
costumes, sets, and artist fees.

Starlight will also incur $565,000 of common fixed operating charges (administrative
overhead, facility costs, and advertising) for the entire season, and is subject to a 30%
income tax rate.

If Starlight’s schedule of musicals is held, as planned, how many patrons would have to
attend for Starlight to break even during the summer season?

FC $1,295,000.00
CM $16.20
Breakeven $79,938

134. Carson Inc. manufactures only one product and is preparing its budget for next year based on the following informatio

Selling price per unit $100.00


Variable costs per unit $75.00
Fixed costs $250,000.00
Effective tax rate 35%

If Carson wants to achieve a net income of $1.3 million next year, its sales must be

CM $25.00
EBT $2,000,000.00
Numerator to be used $2,250,000.00
units to achieve 1.3 m 90,000.00
135. MetalCraft produces three inexpensive socket wrench sets that are popular with do-ityourselfers.
Budgeted information for the upcoming year is as follows.
Estimated
Model Selling Price Variable Cost Sales Volume
No. 109 10 5.5 30000
No. 145 15 8 75000
No. 153 20 14 45000
150000
Total fixed costs for the socket wrench product line is $961,000. If the company’s actual
experience remains consistent with the estimated sales volume percentage distribution,
and the firm desires to generate total operating income of $161,200, how many Model
No. 153 socket sets will MetalCraft have to sell?

Model CM CM
No. 109 20% 4.5 0.9
No. 145 50% 7 3.5
No. 153 30% 6 1.8
6.2

Numerator 1122200
units to ac 181000
No. 153 54300

136. Robin Company wants to earn a 6% return on sales after taxes.


The company’s effective income tax rate is 40%, and its contribution margin is 30%.
If Robin has fixed costs of $240,000, the amount of sales required to earn the desired return is

desired return return on sales 6%


tax rate 40%
contribution margin 30%
fixed costs $240,000.00

This problem can be solved by setting up an equation and solving for the required sales amount, which
will be represented by the variable S. S = required sales
Sheet 14 april 2010 p3

169. Fennel Products is using cost-based pricing to determine the selling price for its new
product based on the following information.
Annual volume 25,000 units 25,000.00
Fixed costs $700,000 per year $700,000.00
Variable costs $200 per unit $200.00 $5,000,000.00
Plant investment $3,000,000 $3,000,000.00
Working capital $1,000,000 $1,000,000.00
Effective tax rate 40% 40%

The target price that Fennell needs to set for the new product to achieve a 15% after-tax
return on investment (ROI) would be

(ROI) $600,000.00
investment $4,000,000.00

(ROI) before tax $705,882.35

$ 228.00 $ 238.00 $ 258.00 $ 268.00


Sales Rev 5,700,000.00 5,950,000.00 6,450,000.00 6,700,000.00
VC 5000000 5000000 5000000 5000000
CM 700,000.00 950,000.00 1,450,000.00 1,700,000.00
FC $700,000.00 $700,000.00 $700,000.00 $700,000.00
OI $0.00 $250,000.00 $750,000.00 $1,000,000.00
Tax 0 100000 300000 400000
NI $0.00 $150,000.00 $450,000.00 $600,000.00
0.00% 3.75% 11.25% 15.00%

168. Almelo Manpower Inc. provides contracted bookkeeping services. Almelo has annual
fixed costs of $100,000 and variable costs of $6 per hour. This year the company
budgeted 50,000 hours of bookkeeping services. Almelo prices its services at full cost
and uses a cost-plus pricing approach. The company developed a billing price of $9 per
hour. The company’s mark-up level would be
fixed costs $100,000.00
fixed costs per unit $2.00
VC per hour $6.00
budgeted 50,000 hours 50,000.00
billing price $9.00

Full cost $8.00


mark-up $1.00 12.50%

165. Basic Computer Company (BCC) sells its microcomputers using bid pricing. It develops
its bids on a full cost basis. Full cost includes estimated material, labor, variable
overheads, fixed manufacturing overheads, and reasonable incremental computer
assembly administrative costs, plus a 10% return on full cost. BCC believes bids in
excess of $1,050 per computer are not likely to be considered.

BCC’s current cost structure, based on its normal production levels, is $500 for materials
per computer and $20 per labor hour. Assembly and testing of each computer requires 17
labor hours. BCC expects to incur variable manufacturing overhead of $2 per labor hour,
fixed manufacturing overhead of $3 per labor hour, and incremental administrative costs
of $8 per computer assembled.

BCC has received a request from a school board for 200 computers. Using the full-cost
criteria and desired level of return, which one of the following prices should be
recommended to BCC’s management for bidding purposes?

DM $500.00 per computer


DL $20.00 per labor hour 3400

Assembly and testing of each computer requires 17 hours

VOH $2.00 per labor hour


FOH $3.00 per labor hour
incremental administrative costs $8.00 per computer

Full cost
10% return on full cost.
(BCC) bid pricing.
(BCC) bid pricing per computer
163. Leader Industries is planning to introduce a new product, DMA. It is expected that
10,000 units of DMA will be sold. The full product cost per unit is $300. Invested
capital for this product amounts to $20 million. Leader’s target rate of return on
investment is 20%. The markup percentage for this product, based on operating income
as a percentage of full product cost, will be

Full cost $3,000,000.00


Leader’s target rate of return $4,000,000.00 133%

162. The Robo Division, a decentralized division of GMT Industries, has been approached to
submit a bid for a potential project for the RSP Company.
Robo Division has been informed by RSP that they will not consider bids over $8,000,000.
Robo Division purchases its materials from the Cross Division of GMT Industries.
There would be no additional fixed costs for either the Robo or Cross Divisions.

Information regarding this project is as follows.

Sell to Robo
Cross Division Robo Division
Variable Costs $1,500,000 $4,800,000 $1,500,000.00 $4,800,000.00
Transfer Price 3,700,000 - $3,700,000.00 $0.00

If Robo Division submits a bid for $8,000,000, the amount of contribution margin 8000000
recognized by the Robo Division and GMT Industries, respectively, is
Cross Division Robo Division
Price of the product sold $3,700,000.00 $8,000,000.00
VC $1,500,000.00 $8,500,000.00
CM $2,200,000.00 -$500,000.00

161. Johnson Company manufactures a variety of shoes, and has received a special one-timeonly order directly from a who
Johnson has sufficient idle capacity to accept the special order to manufacture 15,000 pairs of sneakers at a price of $7.50
Johnson’s normal selling price is $11.50 per pair of sneakers.
Variable manufacturing costs are $5.00 per pair and fixed manufacturing costs are $3.00 a pair.
Johnson’s variable selling expense for its normal line of sneakers is $1.00 per pair.
What would the effect on Johnson’s operating income be if the company accepted the special order?

Sales Rev 112500


VC 90000
CM ###

160. The Doll House, a very profitable company, plans to introduce a new type of doll to its
product line. The sales price and costs for the new dolls are as follows.

Selling price per doll $100 $100.00


Variable cost per doll $60 $60.00
Incremental annual fixed costs $456,000 $456,000.00
Income tax rate 30% 30%

If 10,000 new dolls are produced and sold, the effect on Doll House’s profit (loss) would

Selling price per doll $100 $1,000,000.00


Variable cost per doll $60 $600,000.00
Incremental annual fixed costs $456,000 $456,000.00
-$56,000.00
Income tax rate 30% savings -$16,800.00
-$39,200.00

159. Synergy Inc. produces a component that is popular in many refrigeration systems. Data
on three of the five different models of this component are as follows.

Model
A B C
Volume needed (units) 5000 6000 3000
Manufacturing costs
Variable direct costs $10 $24 $20 $ 10.00 $ 24.00 $ 20.00
Variable overhead 5 10 15 $ 5.00 $ 10.00 $ 15.00
Fixed overhead 11 20 17 $ 11.00 $ 20.00 $ 17.00
Total manufacturing costs $26 $54 $52 $ 26.00 $ 54.00 $ 52.00

Cost if purchased $21 $42 $39 $ 21.00 $ 24.00 $ 39.00


Total v manufacturing costs $ 15.00 $ 34.00 $ 35.00
Hours required 2.00 4.00 6.00
14,000.00 7,000.00
Saving (loss) (6.00) 10.00

Synergy applies variable overhead on the basis of machine hours at the rate of $2.50 per hour.
Models A and B are manufactured in the Freezer Department, which has a capacity of 28,000 machine processing hours.
Which one of the following options should be recommended to Synergy's management?

a. Purchase all three products in the quantities required.


b. Manufacture all three products in the quantities required.
c. The Freezer Department's manufacturing plan should include 5,000 units of
Model A and 4,500 units of Model B.
d. The Freezer Department's manufacturing plan should include 2,000 units of
Model A and 6,000 units of Model B.

158. Green Corporation builds custom-designed machinery. A review of selected data and the company’s pricing policies re

• A 10% commission is paid on all sales orders.


• Variable and fixed factory overheads total 40% and 20%, respectively, of direct labor.

• Corporate administrative costs amount to 10% of direct labor.


• When bidding on jobs, Green adds a 25% markup to the total of all factory and administrative costs to cover income taxes
• The firm’s income tax rate is 40%.

The company expects to operate at a maximum of 80% of practical capacity.

Green recently received an invitation to bid on the manufacture of some custom machinery for Kennendale, Inc.
For this project, Green’s production accountants estimate the material and labor costs will be $66,000 and $120,000, respe
$66,000.00 $120,000.00
Accordingly, Green submitted a bid to Kennendale in the amount of $375,000. Feeling Green’s bid was too high, Kennendal

Which one of the following options should be recommended to Green’s management?

• A 10% commission is paid on all sales orders.


• Variable and fixed factory overheads total 40% and 20%, respectively, of direct labor.
• Corporate administrative costs amount to 10% of direct labor.
• When bidding on jobs, Green adds a 25% markup to the total of all factory and administrative costs to cover income taxes
• The firm’s income tax rate is 40%.
DM ###
DL ###
VOH ###
FOH ###
Admin ###
Full cost ###
25% mark ### 25%
### 375000 $37,500.00
• A 10% commission is paid on all sales orders.
### 10%
Accordingly, Green submitted a bid to Kennendale in the amount of $375,000 375000
###

a. Accept the counteroffer because the order will increase operating income.

157. Jones Enterprises manufactures 3 products, A, B, and C. During the month of May
Jones’ production, costs, and sales data were as follows.

Products
A B C
Units of production 30,000.00 20,000.00 70,000.00
Joint production costs to split-off point $480,000
Further processing costs $ - $60,000 $140,000 $ - $ 60,000.00 $ 140,000.00
Further processing cost $ 3.00 $ 2.00
Unit sales price
At split-off 3.75 5.50 10.25 $ 3.75 $ 5.50 $ 10.25
After further processing - 8.00 12.50 $ - $ 8.00 $ 12.50
$ 5.00 $ 10.50
INCREMENTAL PROFIT $ (0.50) $ 0.25
Based on the above information, which one of the following alternatives should be recommended to Jones’ management?

c. Process Product C further but sell Product B at the split-off point.

156. Basic Computer Company (BCC) sells its micro-computers using bid pricing. It develops bids on a full cost basis.
Full cost includes estimated material, labor, variable overheads, fixed manufacturing overheads, and reasonable increment

BCC believes bids in excess of $925 per computer are not likely to be considered. 925
BCC’s current cost structure, based on its normal production levels, is $500 for materials per computer and $20 per labor h
Assembly and testing of each computer requires 12 labor hours.
BCC’s variable manufacturing overhead is $2 per labor hour, fixed manufacturing overhead is $3 per labor hour, and increm

The company has received a request from the School Board for 500 computers. BCC’s
management expects heavy competition in bidding for this job. As this is a very large
order for BCC, and could lead to other educational institution orders, management is
extremely interested in submitting a bid which would win the job, but at a price high
enough so that current net income will not be unfavorably impacted. Management
believes this order can be absorbed within its current manufacturing facility. Which one
of the following bid prices should be recommended to BCC’s management?

DM $500.00 per computer


DL $20.00 per labor hour 6000

Assembly and testing of each computer requires 12 hours

VOH $2.00 per labor hour


FOH $3.00 per labor hour
incremental administrative costs $8.00 per computer

Full cost
10% return on full cost.
(BCC) bid pricing.
(BCC) bid pricing per computer

153. Lazar Industries produces two products, Crates and Boxes. Per unit selling prices, costs,
and resource utilization for these products are as follows.

Crates Boxes
Selling price $20 $30 $20.00 $30.00
Direct material costs $ 5 $ 5 2.5 $5.00 $5.00
Direct labor costs 8 10 $8.00 $10.00
Variable overhead costs 3 5 $ 5.25 $3.00 $5.00 VOH
Variable selling costs 1 2 $1.00 $2.00
Machine hours per unit 2 4 3.50 2.00 4.00

total fixed factory overhead cost $2,000,000.00


total fixed selling and administrative costs $840,000.00
Production and sales 500,000.00 700,000.00

Production of Crates and Boxes involves joint processes and use of the same facilities.
The total fixed factory overhead cost is $2,000,000 and total fixed selling and administrative costs are $840,000.
Production and sales are scheduled for 500,000 units of Crates and 700,000 units of Boxes.
Lazar maintains no direct materials, work-inprocess, or finished goods inventory.

Lazar can reduce direct material costs for Crates by 50% per unit, with no change in direct labor costs.
However, it would increase machine-hour production time by 1-1/2 hours per unit. 1.50
For Crates, variable overhead costs are allocated based on machine hours.

What would be the effect on the total contribution margin if this change was implemented?

Before
CM $3.00 1,500,000.00
CM per constraint. $1.50

After
CM $3.25 1,625,000.00
CM per constraint. $0.93 125,000.00

152. Refrigerator Company manufactures ice-makers for installation in refrigerators. The


costs per unit, for 20,000 units of ice-makers, are as follows. 20,000.00

Make
Direct materials $ 7 $7.00 7
Direct labor 12 $12.00 12
Variable overhead 5 $5.00 5
Fixed overhead 10 $10.00 6
Total costs $34 $34.00 $30.00

$30.00
$600,000.00

Cool Compartments Inc. has offered to sell 20,000 ice-makers to Refrigerator Company for $28 per unit.
If Refrigerator accepts Cool Compartments’ offer, the facilities used to manufacture ice-makers could be used to produce w
Revenues from the sale of water filtration units are estimated at $80,000, with variable costs amounting to 60% of sales.
In addition, $6 per unit of the fixed overhead associated with the manufacture of ice-makers could be eliminated.

For Refrigerator Company to determine the most appropriate action to take in this
situation, the total relevant costs of make vs. buy, respectively, are
c. $600,000 vs. $528,000.
151. Aspen Company plans to sell 12,000 units of product XT and 8,000 units of product RP.
Aspen has a capacity of 12,000 productive machine hours. 12000
The unit cost structure and machine hours required for each product is as follows.
12000 8000
Unit Costs XT RP
Materials $37 $24 $37.00 $24.00
Direct labor 12 13 $12.00 $13.00
Variable overhead 6 3 $6.00 $3.00
Fixed overhead 37 38 $37.00 $38.00
Machine hours required 1.0 1 1.5
VC $55.00 $40.00
Machine hours required 12,000.00 12,000.00

Purchase price $60.00 $45.00


Loss -$5.00 -$5.00
Loss per constraint -$5.00 -$7.50

Aspen can purchase 12,000 units of XT at $60 and/or 8,000 units of RP at $45. Based on the above, which one of the follow

150. The Furniture Company currently has three divisions: Maple, Oak, and Cherry. The oak
furniture line does not seem to be doing well and the president of the company is
considering dropping this line.
If it is dropped, the revenues associated with the Oak
Division will be lost and the related variable costs saved. Also, 50% of the fixed costs
allocated to the oak furniture line would be eliminated. The income statements, by
divisions, are as follows.

Maple
Sales $55,000 $85,000 $100,000 $55,000.000
Variable Costs 40,000 72,000 82,000 $40,000.000
Contribution Margin 15,000 13,000 18,000 $15,000.000
Fixed costs 10,000 14,000 10,200 $10,000.000
Operating profit (loss) $ 5,000 $(1,000) $ 7,800 $5,000.000

Which one of the following options should be recommended to the president of the
company?
Maple
Sales $55,000 $85,000 $100,000 $55,000.000
Variable Costs 40,000 72,000 82,000 $40,000.000
Contribution Margin 15,000 13,000 18,000 $15,000.000
Fixed costs 10,000 14,000 10,200 $10,000.000
Operating profit (loss) $ 5,000 $(1,000) $ 7,800 $5,000.000

b. Continue operating the Oak Division as discontinuance would result in a $6,000


decline in operating profits.

149. Raymund Inc. currently sells its only product to Mall-Stores. Raymund has received a
one-time-only order for 2,000 units from another buyer. Sale of the special order items
will not require any additional selling effort. Raymund has a manufacturing capacity to
produce 7,000 units. Raymund has an effective income tax rate of 40%. Raymund’s
Income Statement, before consideration of the one-time-only order, is as follows.

Sales (5,000 units at $20 per unit) $100,000 5,000.00 $20.0


Variable manufacturing costs $50,000 $10.0 $50,000.0
Variable selling costs 15,000 65,000 $3.0 $15,000.0
Contribution margin 35,000

Fixed manufacturing costs 16,000 $16,000.0


Fixed selling costs 4,000 20,000 $4,000.0
Operating income $ 15,000
Income taxes
Net income

In negotiating a price for the special order, Raymund should set the minimum per unit
selling price at $10.0

148. Lark Industries accepted a contract to provide 30,000 units of Product A and 20,000 units
of Product B. Lark’s staff developed the following information with regard to meeting
this contract.
Product A Product B
30000 20000
Selling Price $75 $125 $75.0 $125.0
Variable costs $30 $48 $30.0 $48.0
Fixed overhead $1,600,000
Machine hours required 3 5 3.00 5.00
Machine hours available 160,000
Cost if outsourced $45 $60 $45.0 $60.0

Lark’s operations manager has identified the following alternatives. Which alternative
should be recommended to Lark’s management?

CM $45.0 $77.0
CM per constraint $ 15.00 $ 15.40

loss if outsourced -$15.0 -$12.0


loss per constraint if outsourced $ -5.00 $ -2.40

147. Aril Industries is a multiproduct company that currently manufactures 30,000 units of
Part 730 each month for use in production. The facilities now being used to produce Part
730 have fixed monthly overhead costs of $150,000, and a theoretical capacity to produce
60,000 units per month. If Aril were to buy Part 730 from an outside supplier, the
facilities would be idle and 40% of fixed costs would continue to be incurred. There are
no alternative uses for the facilities. The variable production costs of Part 730 are $11
per unit. Fixed overhead is allocated based on planned production levels.

If Aril Industries continues to use 30,000 units of Part 730 each month, it would realize a
net benefit by purchasing Part 730 from an outside supplier only if the supplier’s unit
price is less than $ 14.00

146. Current business segment operations for Whitman, a mass retailer, are presented below.

Merchandise Automotive
Sales $ 500,000.00 $ 400,000.00
Variable costs $ 300,000.00 $ 200,000.00
CM $ 200,000.00 $ 200,000.00
Fixed costs $ 100,000.00 $ 100,000.00
Operating income (loss) $ 100,000.00 $ 100,000.00

CM% 40% 50%

Management is contemplating the discontinuance of the Restaurant segment since “it is


losing money.” If this segment is discontinued, $30,000 of its fixed costs will be
eliminated. In addition, Merchandise and Automotive sales will decrease 5% from their
current levels. When considering the decision, Whitman’s controller advised that one of
the financial aspects Whitman should review is contribution margin. Which one of the
following options reflects the current contribution margin ratios for each of Whitman’s
business segments?

Merchandise Automotive
Sales $ 475,000.00 $ 380,000.00
Variable costs $ 285,000.00 $ 190,000.00
CM $ 190,000.00 $ 190,000.00
Fixed costs $ 100,000.00 $ 100,000.00
Operating income (loss) $ 90,000.00 $ 90,000.00

CM% 40% 50%

144. Oakes Inc. manufactured 40,000 gallons of Mononate and 60,000 gallons of Beracyl in a
joint production process, incurring $250,000 of joint costs. Oakes allocates joint costs
based on the physical volume of each product produced. Mononate and Beracyl can each
be sold at the split-off point in a semifinished state or, alternatively, processed further.
Additional data about the two products are as follows.

40,000.00
Mononate
Sales price per gallon at split-off $ 7.00
Sales price per gallon if processed further $ 10.00
Variable production costs if processed further $ 125,000.00

An assistant in the company’s cost accounting department was overheard saying “....that
when both joint and separable costs are considered, the firm has no business processing
either product beyond the split-off point. The extra revenue is simply not worth the
effort.” Which of the following strategies should be recommended for Oakes?

Variable production costs if processed further $ 3.13

Incremental Revenue per unit if processed further $ 3.00


Incremental Cost per unit if processed further $ 3.13
Incremental profit (Loss) $ (0.13)

Incremental Revenue if processed further 120,000.00


Incremental Cost if processed further 125,000.00
Incremental profit (Loss) (5,000.00)
143. Jack Blaze wants to rent store space in a new shopping mall for the three month holiday
shopping season. Blaze believes he has a new product available which has the potential
for good sales. The product can be obtained on consignment at the cost of $20 per unit
and he expects to sell the item for $100 per unit. Due to other business ventures, Blaze’s
risk tolerance is low. He recognizes that, as the product is entirely new, there is an
element of risk. The mall management has offered Blaze three rental options: (1) a fixed
fee of $8,000 per month, (2) a fixed fee of $3,990 per month plus 10% of Blaze’s
revenue, or (3) 30% of Blaze’s revenues. Which one of the following actions would you
recommend to Jack Blaze?

(1) a fixed fee of $8,000 per month, (2) a fixed fee of $3,990 per month pl
8000

142. Eagle Brand Inc. produces two products. Data regarding these products are presented
below.

Product X Product Y
Selling price per unit $100 $130 $ 100.00 $ 130.00
Variable costs per unit $80 $100 $ 80.00 $ 100.00
Raw materials used per unit 4 lbs. 10 lbs. 4.00 10.00

Eagle Brand has 1,000 lbs. of raw materials which can be used to produce Products X and
Y.

Which one of the alternatives below should Eagle Brand accept in order to maximize
contribution margin?

CM 20 30
CM per constraint $ 5.00 $ 3.00
Units that can be produced by the constraint 250.00 100.00

12 20
8 15 Specialty Cakes Inc. produces two typ
RC HS cake. Total fixed costs for the firm are
CM $4.0 $5.0 the two types of cakes are presented
units 10000 15000 25000 2 lbs. 3 lbs.
40% 60% Round Cake Heart-shape Cake
$1.6 $3.0 $4.6 Selling price per unit $12 $20
Variable cost per unit $8 $15
fc $94,000.0 20,434.78 Current sales (units) 10,000 15,000

8,173.91 ### 20,434.78

$12.0 $20.0
$8.0 $15.0 If the product sales mix were to change to three heart-s
RC HS the breakeven volume for each of these products would
CM $4.0 $5.0
units 6250 18750 25000
25% 75%
$1.0 $3.8 $4.8

fc $94,000.0 19,789.47

4,947.37 ### 19,789.47

138. Zipper Company invested $300,000 in a new machine to produce cones for the textile
industry. Zipper’s variable costs are 30% of the selling price, and its fixed costs are
$600,000. Zipper has an effective income tax rate of 40%. The amount of sales required
to earn an 8% after-tax return on its investment would be

Targeted ROI $ 24,000.00


Before Tax Targeted ROI $ 40,000.00
Numerator to be used $ 640,000.00
CM ratio 70%
$ 914,285.71

137. Bargain Press is considering publishing a new textbook. The publisher has developed the
following cost data related to a production run of 6,000, the minimum possible
production run. Bargain Press will sell the textbook for $45 per copy. How many
textbooks must Bargain Press sell in order to generate operating earnings (earnings before
interest and taxes) of 20% on sales? (Round your answer up to the nearest whole
textbook.)
Estimated cost per unit
Development (reviews, class testing, editing) $35,000 $ 35,000.00
Typesetting 18,500 $ 18,500.00
Depreciation on Equipment 9,320 $ 9,320.00
General and Administrative 7,500 $ 7,500.00
Miscellaneous Fixed Costs 4,400 $ 4,400.00
Printing and Binding 30,000 $ 30,000.00 $ 5.00
Sales staff commissions (2% of selling price) 5,400 $ 5,400.00 $ 0.90
Bookstore commissions (25% of selling price) 67,500 $ 67,500.00 $ 11.25
Author’s Royalties (10% of selling price) 27,000 $ 27,000.00 $ 4.50
$ 21.65
Total costs at production of 6,000 copies $ 204,620.00

Feedback: The correct answer is: 5,207 copies. ,


The number of copies required can be found by solving the following equation, where x equals the ,
number of copies: ,
Total sales – total variable costs – fixed costs + typesetting + depreciation expense + general and ,
administrative expenses + miscellaneous fixed costs = (return %)(total sales) ,
Total sales = $45x ,
Total variable cost = (variable cost per unit, or VCU)(x) = VCU(x) ,
Fixed costs = development + typesetting + depreciation + general and administrative + miscellaneous ,
fixed costs ,
Fixed costs = $35,000 + $18,500 + $9,320 + $7,500 + $4,400 = $74,720 ,
VCU = (printing and binding + sales staff commissions + bookstore commissions + royalties) / ,
(number of copies) ,
VCU = ($30,000 + $5,400 + $67,500 + $27,000) / (6,000 copies) = $129,900 / 6,000 ,
VCU = $21.65 per copy ,
$45x – $21.65(x) – $74,720 = (0.2)($45x) ,
$45x – $21.65(x) - $74,720 = $9x ,
$36x – $21.65(x) = $74,720 ,
$14.35x = $74,720 ,
x = 5,206.97, which rounds to 5,207 copies. ,

131. Wilkinson Company sells its single product for $30 per unit. The contribution margin
ratio is 45% and Wilkinson has fixed costs of $10,000 per month. If 3,000 units are sold
in the current month, Wilkinson’s income would be

CM $ 13.50
Total CM $ 40,500.00
FC $ 10,000.00
OI $ 30,500.00
25000 22000

1000

turing other products. This is a one time special order.

XR-2000 XR-2000
Q 6,250.00 $387,500.0
$24.0 Q 5,500.00 $341,000.0
$10.0 $46,500.0
$5.0 $46.50
$40.0
$4.0
$10.0
$105.0
4.00

$62.0
$15.5

y not producing XR-2000

750

106. C c. a decrease of $2,000.

Segment B Total
$15,000.0 $25,000.0
$8,500.0 $12,500.0
$2,500.0 $4,000.0
$4,000.0 $8,500.0
$3,000.0 $5,000.0
$1,500.0 $3,000.0
-$500.0 $500.0

Segment B Total

$10,000.0
$4,000.0
$1,000.0 $2,500.0
-$1,000.0 $3,500.0
$2,000.0
$1,500.0 $3,000.0
-$2,500.0 -$1,500.0

-$2,000.00

107. C Z, X, Y.
Approach Y Approach Z

$38.0 $32.0
12,000.00 14,000.00
$456,000.0 $448,000.0
$360,000.0 $420,000.0
$45,600.0 0
$30,000.0
$20,400.0 $28,000.0

108. C c. 14 units.

6 - 10 11 - 15 16 - 20
$400,000.0 $600,000.0 $800,000.0
$50,000.0 $45,000.0 $45,000.0
$100,000.0 $100,000.0 $100,000.0
e and variable costs are as follows. 109. B b. $67,200.

110. A a. $24,000.

7,000.00 111. B b. $10,000.

$30,000.0
$5.0

$100,000.0

$65,000.0
$35,000.0

$20,000.0
$15,000.0

$100,000.0
increase Raymund’s fixed manufacturing costs
by $30,000 per month and reduce its variable
$40,000.0 costs by $5 per uni
$60,000.0

$50,000.0
$10,000.0

osts per unit, for 20,000 units of ice-makers, are as follows. 112. C
20,000.00

$28 per unit.


ead amounting to $6 per unit could be eliminated. $6.0
n an anticipated volume of 150,000 units, is as follows. 113. C
150,000.00

e of Phillip’s cost structure. 175,000.00


reduction of $750,000 in fixed costs $750,000.0

100%
32%
68%

2% and operating income of $9,600,000.

114. B b. No, the machine-related cost of pro


1,600,000.00
ucing each unit of product should be $.50. $0.5
1,200,000.00
600000

use the manufacturer’s engineering specification of machine related unit production cost?

$800,000.0
$0.67

115. B
Product B
20,000.00
20,000.00
$80.0
$45.0
$10.0
$11.0
$4.0
$0.0
$10.0
1.5

aking a decision on product priorities for the coming year, are

Product B
$24.0
$16.0

116. B
4500 15300 $3.40
$4.05

ds of Kaylene. 4200

Irrelevant Decision Factor


Purchase price of $3.40 per lb.

tilizer and weed killer that is popular in hot, dry climates. 117. C
awn fertilizer.
40

$18.5
$20.0
-$1.50

anufactures several different products, and the loss appear after the deduction of the allocated fixed costs

$2.5

119. A a. $25,000.
ased four years ago at a cost of $70,000.

machine has no current salvage value.

d labor priced at $5,000.


$5,000.00

123. D d. $16,500.

124. B
b. 82,500 units and $9,250,000 of operating income.
h is within the relevant range of Phillip’s cost structure. 175000
ar would amount to

126. B b. $13,500.

1,000

diff
$30,000

$13,500

$13,500

127. B b. $689,992.
689992
Product B

20,000.00
20,000.00
$80.0
$45.0
$10.0
$11.0
$4.0
$0.0
$10.0
1.5

Product B
$24.00
$16.00

370,000

20,000 13333
13,333.33 $319,992.00
320,000 320,000

$689,992 $0.0

128. B b. $2,000,000.

Trunks
30
5
10
5
2
4

e costs are $840,000.

1,000,000
Trunks
8.00
2.00

250,000 Trunks will use all the machine hours


1,000,000
$2,000,000

%
450000 b. $80,000. 129. B
120000
270000 0.6
500000

130. C increase by $1,000.


133. C c. 79,938.

Variable Fixed CM no. of units %


Costs 1 Costs 2
$3.00 $165,000.00 $15.00 42,000.00 28% $4.20
$1.00 $249,000.00 $14.00 60,000.00 40% $5.60
$0.00 $316,000.00 $20.00 48,000.00 32% $6.40
$730,000.00 150,000.00 $16.20

$565,000.00
30%

based on the following information. 134. D d. 90,000 units.

$1,300,000
tyourselfers. 135. B b. 54,300.

sets
sets
sets

961000

161200

136. D d. $1,200,000.

mount, which
169. D d. $268.

a. $228.
b. $238.
c. $258.
d. $268.

15%

168. A a. 12.5%.
100000 6
50000
9
165. D d. $1,026.30.

10%

200

$100,000.00
$68,000.00

$6,800.00
$10,200.00
$1,600.00

$186,600.00
$18,660.00
$205,260.00
$1,026.30
163. C c. 133.3%.
10,000.00 $300.00 $20,000,000.00
20%

162. C c. $(500,000) and $1,700,000

GMT Industries
$11,700,000.00
$10,000,000.00
$1,700,000.00 ###

161. C b. Increase by $22,500.

meonly order directly from a wholesaler.


rs of sneakers at a price of $7.50 per pair. 15000 7.5
11.5
5 3
1
pecial order?

160. C c. $(39,200).

10000

159. C c. The Freezer Department's manufacturing plan should include 5,0


Model A and 4,500 units of Model B.

Should be answered by
elimination
Should be answered by
elimination

$ 2.50
000 machine processing hours. 28,000.00

158. A
the company’s pricing policies revealed the following.

10%
40% 20%

10%
rative costs to cover income taxes and produce a profit. 25%
40%

80%

y for Kennendale, Inc.


be $66,000 and $120,000, respectively.

en’s bid was too high, Kennendale countered with a price of $280,000.

$234,000.00

$ 48,000.00 $ 24,000.00
$12,000.00
rative costs to cover income taxes and produce a profit. $ 84,000.00
157. C c. Process Product C further but sell Product B at the split-off point

TOTAL
120,000.00
$ 480,000.00

mended to Jones’ management?

156. B b. $772.00.
s bids on a full cost basis.
heads, and reasonable incremental computer assembly administrative costs, plus a 10% return on full cost.
er computer and $20 per labor hour.

d is $3 per labor hour, and incremental administrative costs are $8 per computer assembled.

0.1

500

$250,000.00
$120,000.00

$12,000.00
$18,000.00
$4,000.00

$404,000.00 VC $386,000.00
$40,400.00 $0.00
$444,400.00 $386,000.00
$888.80 $772.00

153. A a. $125,000 increase.

For Crates, variable overhead costs are allocated based on


machine hours.
$ 1.50
labor costs.

152. C c. $600,000 vs. $528,000.

Buy
28

$28.00
-$1.60
$26.40
$528,000.00

$28 per unit.


akers could be used to produce water filtration units.
sts amounting to 60% of sales. 0.4 80000 32000 1.6
ers could be eliminated.
151. A a. Produce XT internally and purchase RP.

e above, which one of the following actions should be recommended to Aspen's management?

150. B b. Continue operating the Oak Division as discontinu


decline in operating profits.

50%

Oak Cherry Total


$85,000.000 $100,000.000 ###
$72,000.000 $82,000.000 ###
$13,000.000 $18,000.000 $46,000.000
$14,000.000 $10,200.000 $34,200.000
-$1,000.000 $7,800.000 $11,800.000
Oak Cherry Total
$100,000.000 ###
$82,000.000 ###
$0.000 $18,000.000 $33,000.000
$7,000.000 $10,200.000 $27,200.000
-$7,000.000 $7,800.000 $5,800.000

-$6,000.000

149. A a. $10.
2000

40%

$100,000.0

$65,000.0
$35,000.0

$20,000.0
$15,000.0
$6,000.0
$9,000.0

148. A a. Make 30,000 units of Product A, utilize the remain


B, and outsource the remainder.

Total

$1,600,000.0
160,000.00

147. D d. $14.00.
30,000.00

$ 150,000.00
60,000.00
60% $ 90,000.00
$ 11.00
$ 3.00

146. D d. 40% 50% 30%


Retailing Automotive Restaurant

Restaurant Total
$ 100,000.00 $ 1,000,000.00
$ 70,000.00 $ 570,000.00
$ 30,000.00 $ 430,000.00
$ 50,000.00 $ 250,000.00
$ -20,000.00 $ 180,000.00

30% 43%
95%

Restaurant Total
$ 855,000.00
$ 475,000.00
$ - $ 380,000.00 What will Whitman’s total contribution margin be if
$ 20,000.00 $ 250,000.00 segment is discontinued?
$ -20,000.00 $ 130,000.00 145. D d. $380,000.

#DIV/0! 44%

Mononate Beracyl
144. B b. Sell at split-off Process further.
$ 250,000.00

60,000.00 100,000.00
Beracyl
$ 15.00
$ 18.00
$ 115,000.00

$ 1.92

$ 3.00
$ 1.92
$ 1.08

180,000.00
115,000.00
65,000.00
143. D d. Choose the third option no matter what Blaze exp

20
100

5700

fixed fee of $3,990 per month plus 10% of Blaze’s revenue (3) 30% of Blaze’s revenues.
4560 1710

142. B b. 250 units of product X.

1000

alty Cakes Inc. produces two types of cakes, a 2 lbs. round cake and a 3 lbs. heartshaped
Total fixed costs for the firm are $94,000. Variable costs and sales data for
wo types of cakes are presented below.
d Cake Heart-shape Cake
g price per unit $12 $20
ble cost per unit $8 $15
nt sales (units) 10,000 15,000

x were to change to three heart-shaped cakes for each round cake,


for each of these products would be

138. B b. $914,286.
$ 300,000.00
30%
$ 600,000.00
40%
8%

137. B b. 5,207 copies.

6000
$ 45.00
20%
$ 74,720.00

cellaneous ,

30 131. A a. $30,500
0.45 3000
10000
Section D: Decision Analysis
103. B b. $96.50. 101 169

KT-6500
3,000.00

c. a decrease of $2,000.
fixed manufacturing costs
h and reduce its variable
y $5 per uni

c. $600,000.
achine-related cost of producing each unit is $.67.
ng plan should include 5,000 units of
uct B at the split-off point.
Oak Division as discontinuance would result in a $6,000
oduct A, utilize the remaining capacity to make Product
contribution margin be if the Restaurant

Process further.
no matter what Blaze expects the revenues to be.
150. The Furniture Company currently has three divisions: Maple, Oak, and Cherry. The oak
furniture line does not seem to be doing well and the president of the company is
considering dropping this line.
If it is dropped, the revenues associated with the Oak
Division will be lost and the related variable costs saved. Also, 50% of the fixed costs
allocated to the oak furniture line would be eliminated. The income statements, by
divisions, are as follows.

Maple Oak
Sales $55,000 $85,000 $100,000 $55,000.000 $85,000.000
Variable Costs 40,000 72,000 82,000 $40,000.000 $72,000.000
Contribution Margin 15,000 13,000 18,000 $15,000.000 $13,000.000
Fixed costs 10,000 14,000 10,200 $10,000.000 $14,000.000
Operating profit (loss) $ 5,000 $(1,000) $ 7,800 $5,000.000 -$1,000.000

Which one of the following options should be recommended to the president of the
company?

Maple Oak
Sales $55,000 $85,000 $100,000 $55,000.000
Variable Costs 40,000 72,000 82,000 $40,000.000
Contribution Margin 15,000 13,000 18,000 $15,000.000 $0.000
Fixed costs 10,000 14,000 10,200 $10,000.000 $7,000.000
Operating profit (loss) $ 5,000 $(1,000) $ 7,800 $5,000.000 -$7,000.000

b. Continue operating the Oak Division as discontinuance would result in a $6,000


decline in operating profits.

149. Raymund Inc. currently sells its only product to Mall-Stores. Raymund has received a
one-time-only order for 2,000 units from another buyer. Sale of the special order items
will not require any additional selling effort. Raymund has a manufacturing capacity to
produce 7,000 units. Raymund has an effective income tax rate of 40%. Raymund’s
Income Statement, before consideration of the one-time-only order, is as follows.

Sales (5,000 units at $20 per unit) $100,000 5,000.00 $20.0


Variable manufacturing costs $50,000 $10.0 $50,000.0
Variable selling costs 15,000 65,000 $3.0 $15,000.0
Contribution margin 35,000

Fixed manufacturing costs 16,000 $16,000.0


Fixed selling costs 4,000 20,000 $4,000.0
Operating income $ 15,000
Income taxes
Net income

In negotiating a price for the special order, Raymund should set the minimum per unit
selling price at $10.0

148. Lark Industries accepted a contract to provide 30,000 units of Product A and 20,000 units
of Product B. Lark’s staff developed the following information with regard to meeting
this contract.
Product A Product B
30000 20000
Selling Price $75 $125 $75.0 $125.0
Variable costs $30 $48 $30.0 $48.0
Fixed overhead $1,600,000
Machine hours required 3 5 3.00 5.00
Machine hours available 160,000
Cost if outsourced $45 $60 $45.0 $60.0

Lark’s operations manager has identified the following alternatives. Which alternative
should be recommended to Lark’s management?

CM $45.0 $77.0
CM per constraint $ 15.00 $ 15.40

loss if outsourced -$15.0 -$12.0


loss per constraint if outsourced $ -5.00 $ -2.40

147. Aril Industries is a multiproduct company that currently manufactures 30,000 units of
Part 730 each month for use in production. The facilities now being used to produce Part
730 have fixed monthly overhead costs of $150,000, and a theoretical capacity to produce
60,000 units per month. If Aril were to buy Part 730 from an outside supplier, the
facilities would be idle and 40% of fixed costs would continue to be incurred. There are
no alternative uses for the facilities. The variable production costs of Part 730 are $11
per unit. Fixed overhead is allocated based on planned production levels.

If Aril Industries continues to use 30,000 units of Part 730 each month, it would realize a
net benefit by purchasing Part 730 from an outside supplier only if the supplier’s unit
price is less than $ 14.00

146. Current business segment operations for Whitman, a mass retailer, are presented below.

Merchandise Automotive Restaurant


Sales $ 500,000.00 $ 400,000.00 $ 100,000.00
Variable costs $ 300,000.00 $ 200,000.00 $ 70,000.00
CM $ 200,000.00 $ 200,000.00 $ 30,000.00
Fixed costs $ 100,000.00 $ 100,000.00 $ 50,000.00
Operating income (loss) $ 100,000.00 $ 100,000.00 $ -20,000.00

CM% 40% 50% 30%

Management is contemplating the discontinuance of the Restaurant segment since “it is


losing money.” If this segment is discontinued, $30,000 of its fixed costs will be
eliminated. In addition, Merchandise and Automotive sales will decrease 5% from their
current levels. When considering the decision, Whitman’s controller advised that one of
the financial aspects Whitman should review is contribution margin. Which one of the
following options reflects the current contribution margin ratios for each of Whitman’s
business segments?

Merchandise Automotive Restaurant


Sales $ 475,000.00 $ 380,000.00
Variable costs $ 285,000.00 $ 190,000.00
CM $ 190,000.00 $ 190,000.00 $ -
Fixed costs $ 100,000.00 $ 100,000.00 $ 20,000.00
Operating income (loss) $ 90,000.00 $ 90,000.00 $ -20,000.00

CM% 40% 50% #DIV/0!


150. B b. Continue operating the Oak Division as discontinuance would result in a $6,0
decline in operating profits.

50%

Cherry Total
$100,000.000 $240,000.000
$82,000.000 $194,000.000
$18,000.000 $46,000.000
$10,200.000 $34,200.000
$7,800.000 $11,800.000

Cherry Total
$100,000.000 $155,000.000
$82,000.000 $122,000.000
$18,000.000 $33,000.000
$10,200.000 $27,200.000
$7,800.000 $5,800.000

-$6,000.000

149. A a. $10.
2000

40%

$100,000.0

$65,000.0
$35,000.0
$20,000.0
$15,000.0
$6,000.0
$9,000.0

148. A a. Make 30,000 units of Product A, utilize the remaining capacity to make Prod
B, and outsource the remainder.

Total

$1,600,000.0

160,000.00

147. D d. $14.00.
30,000.00

$ 150,000.00
60,000.00
60% $ 90,000.00
$ 11.00
$ 3.00
146. D d. 40% 50% 30%
Retailing Automotive Restaurant

Total
$ 1,000,000.00
$ 570,000.00
$ 430,000.00
$ 250,000.00
$ 180,000.00

43%

95%

Total
$ 855,000.00
$ 475,000.00
$ 380,000.00 What will Whitman’s total contribution margin be if the Restaurant
$ 250,000.00 segment is discontinued?
$ 130,000.00 145. D d. $380,000.

44%
continuance would result in a $6,000
remaining capacity to make Product
n be if the Restaurant
103. Gardener Company currently is using its full capacity of 25,000 machine hours to
manufacture product XR-2000.
LJB Corporation placed an order with Gardener for the manufacture of 1,000 units of KT-6500.
LJB would normally manufacture this component.
However, due to a fire at its plant, LJB needs to purchase these units to continue manufacturing other products. This is a o

The following reflects unit cost data, and selling prices.

KT-6500

Material $27.0
Direct labor $12.0
Variable overhead $6.0
Fixed overhead $48.0
Variable selling & administrative $5.0
Fixed selling & administrative $12.0
Normal selling price $125.0
Machine hours required 3.00

What is the minimum unit price that Gardener should charge LJB to manufacture 1,000
units of KT-6500?

CM $75.0
CM per machine hour $25.0

the minimum price must equal at least the VC to produce KT-6500 plus any foregone CM by not producing XR-2000

VC to produce KT-6500 $50.0

Total hours that will be used to prouce KT-6500 instead of producing XR 3,000.00
CM that will be lost from not prducing XR $46,500.0
CM per unit that will be lost from not prducing XR $46.5

The minimum unit price that Gardener should charge LJB to manufacture 1,000 units of KT-6500
$96.50

106. Following are the operating results of the two segments of Parklin Corporation.

Segment A
Sales $10,000.0
Variable costs of goods sold $4,000.0
Fixed costs of goods sold $1,500.0
Gross margin $4,500.0
Variable selling and administrative $2,000.0
Fixed selling and administrative $1,500.0
Operating income (loss) $1,000.0

Variable costs of goods sold are directly related to the operating segments. Fixed costs of
goods sold are allocated to each segment based on the number of employees. Fixed
selling and administrative expenses are allocated equally. If Segment B is eliminated,
$1,500 of fixed costs of goods sold would be eliminated. Assuming Segment B is closed,
the effect on operating income would be

Segment A

Sales $10,000.0
Variable costs of goods sold $4,000.0
Fixed costs of goods sold $1,500.0
Gross margin $4,500.0
Variable selling and administrative $2,000.0
Fixed selling and administrative $1,500.0
Operating income (loss) $1,000.0

Assuming Segment B is closed, the effect on operating income would be

107. Edwards Products has just developed a new product with a manufacturing cost of $30.

The Marketing Director has identified three marketing approaches for this new product.

Approach X Set a selling price of $36 and have the firm’s sales staff sell the
product at a 10% commission with no advertising program.
Estimated annual sales would be 10,000 units.
Approach Y Set a selling price of $38, have the firm’s sales staff sell the
product at a 10% commission, and back them up with a $30,000
advertising program. Estimated annual sales would be 12,000
units.
Approach Z Rely on wholesalers to handle the product. Edwards would sell the
new product to the wholesalers at $32 per unit and incur no selling
expenses. Estimated annual sales would be 14,000 units.
Approach X

selling price $36.0


Q 10,000.00
Rev $360,000.0
manufacturing cost $300,000.0
V selling $36,000.0
Fixed selling
net profit $24,000.0

Rank the three alternatives in order of net profit, from highest net profit to lowest.
107. C Z, X, Y.

108. Parker Manufacturing is analyzing the market potential for its specialty turbines. Parker
developed its pricing and cost structures for their specialty turbines over various relevant
ranges. The pricing and cost data for each relevant range are presented below.

Units produced and sold 1-5


Total fixed costs $200,000.0
Unit variable cost $50,000.0
Unit selling price $100,000.0

Which one of the following production/sales levels would produce the highest operating
income for Parker?

a. 8 units. Rev
b. 10 units. VC
c. 14 units. FC
d. 17 units. OI
8 10 14 17
Rev $800,000.0 $1,000,000.0 $1,400,000.0 $1,700,000.0
VC $400,000.0 $500,000.0 $630,000.0 $765,000.0
FC $400,000.0 $400,000.0 $600,000.0 $800,000.0
OI $0.0 $100,000.0 $170,000.0 $135,000.0
109. Elgers Company produces valves for the plumbing industry. Elgers’ per unit sales price and variable costs are as follow

Sales price $12 $12.0


Variable costs 8 $8.0
Elgers’ practical plant capacity is 40,000 units. 40,000.00
Elgers’ total fixed costs aggregate and. $48,000.0
it has a 40% effective tax rate 40%

The maximum net profit that Elger can earn is

Rev 480000
VC 320000
FC $48,000.0
OI $112,000.0
Tax $44,800.0
Net I $67,200.0

110. Dayton Corporation manufactures pipe elbows for the plumbing industry. Dayton’s per
unit sales price and variable costs are as follows.

Sales price $10 $10.0


Variable costs 7 $7.0
Dayton’s practical plant capacity is 35,000 units. 35,000.00
Dayton’s total fixed costs amount to $42,000, $42,000.0
and the company has a 50% effective tax rate. 50%

If Dayton produced and sold 30,000 units, net income would be 30,000.00

Rev $300,000.0
VC $210,000.0
FC $42,000.0
OI $48,000.0
Tax $24,000.0
Net I $24,000.0

111. Raymund Inc., a bearings manufacturer, has the capacity to produce 7,000 bearings per
month. The company is planning to replace a portion of its labor intensive production
process with a highly automated process, which would increase Raymund’s fixed
manufacturing costs by $30,000 per month and reduce its variable costs by $5 per uni
Raymund’s Income Statement for an average month is as follows.

Sales (5,000 units at $20 per unit) $100,000 5,000.00 $20.0


Variable manufacturing costs $50,000 $10.0 $50,000.0
Variable selling costs 15,000 65,000 $15,000.0
Contribution margin 35,000

Fixed manufacturing costs 16,000 $16,000.0


Fixed selling costs 4,000 20,000 $4,000.0
Operating income $ 15,000

If Raymund installs the automated process, the company’s monthly operating income
would be

Sales (5,000 units at $20 per unit) $100,000 5,000.00 $20.0


Variable manufacturing costs $50,000 $5.0 $25,000.0
Variable selling costs 15,000 65,000 $15,000.0
Contribution margin 35,000

Fixed manufacturing costs 16,000 $46,000.0


Fixed selling costs 4,000 20,000 $4,000.0
Operating income $ 15,000

112. Refrigerator Company manufactures ice-makers for installation in refrigerators. The costs per unit, for 20,000 units of

Direct materials $ 7 $7.0


Direct labor 12 $12.0
Variable overhead 5 $5.0
Fixed overhead 10 $10.0 $200,000.0
Total costs $34 $34.0

Cool Compartments Inc. has offered to sell 20,000 ice-makers to Refrigerator Company for $28 per unit.
If Refrigerator accepts Cool Compartments’ offer the plant would be idled and fixed overhead amounting to $6 per unit co
The total relevant costs associated with the manufacture of ice-makers amount to
relevant
Direct materials $ 7 $7.0
Direct labor 12 $12.0
Variable overhead 5 $5.0
Fixed overhead 10 $6.0
Total costs $34 $30.0 $600,000.0
113. Phillips and Company produces educational software. Its current unit cost, based upon an anticipated volume of 150,0

Selling price $150 $150.0


Variable costs 60 $60.0
Contribution margin 90 $90.0
Fixed costs 60 $60.0 $9,000,000.0
Operating income 30 $30.0

Sales for the coming year are estimated at 175,000 units, which is within the relevant range of Phillip’s cost structure.
Cost management initiatives are expected to yield a 20% reduction in variable costs and a reduction of $750,000 in fixed c
.

Selling price $150 $150.0 $26,250,000.0


Variable costs 60 $48.0 $8,400,000.0
Contribution margin 90 $102.0 $17,850,000.0
Fixed costs 60 $47.1 $8,250,000.0 $8,250,000.0
Operating income 30 $54.9 $9,600,000.0

Phillip’s cost structure for the coming year will include a: variable cost ratio of 32% and operating income of $9,6

114. Sunshine Corporation is considering the purchase of a new machine for $800,000.
The machine is capable of producing 1.6 million units of product over its useful life.
The manufacturer’s engineering specifications state that the machine-related cost of producing each unit of product should
Sunshine’s total anticipated demand over the asset’s useful life is 1.2 million units.
The average cost of materials and labor for each unit is $.40.

In considering whether to buy the new machine, would you recommend that Sunshine use the manufacturer’s engineer
b. No, the machine-related cost of producing each unit is $.67.

manufacturer’s engineering specification


demand

115. Cervine Corporation makes two types of motors for use in various products. Operating
data and unit cost information for its products are presented below.
Product A
Annual unit capacity 10,000 20,000 10,000.00
Annual unit demand 10,000 20,000 10,000.00
Selling price $100 $80 $100.0
Variable manufacturing cost 53 45 $53.0
Fixed manufacturing cost 10 10 $10.0
Variable selling & administrative 10 11 $10.0
Fixed selling & administrative 5 4 $5.0
Fixed other administrative 2 0 $2.0
Unit operating profit $ 20 $10 $20.0
Machine hours per unit 2.0 1.5 2

Cervine has 40,000 productive machine hours available. 40,000.00

The relevant contribution margins, per machine hour for each product, to be utilized in making a decision on product prior

Product A
CM $37.0
CM per machine hours $18.5

116. Two months ago, Hickory Corporation purchased 4,500 pounds of Kaylene at a cost of$15,300.
The market for this product has become very strong, with the price jumping to $4.05 per pound.
Because of the demand, Hickory can buy or sell Kaylene at this price.
Hickory recently received a special order inquiry that would require the use of 4,200 pounds of Kaylene.
In deciding whether to accept the order, management must evaluate a number of decision factors.
Without regard to income taxes, which one of the
following combination of factors correctly depicts relevant and irrelevant decision
factors, respectively?

Relevant Decision Factor


b. Market price of $4.05 per lb.

117. Reynolds Inc. manufactures several different products, including a premium lawn fertilizer and weed killer that is popu
Reynolds is currently operating at less than full capacity because of market saturation for lawn fertilizer.
Sales and cost data for a 40-pound bag of Reynolds lawn fertilizer is as follows.

Selling price
Production cost
Materials and labor $12.3
Variable overhead $3.8
Allocated fixed overhead $4.0
Income (loss) per bag $(1.50)

On the basis of this information, which one of the following alternatives should be
recommended to Reynolds management?

c. Continue to produce and market this product., since it is has a $2.5 CM Reynolds Inc. manufactures several different prod

CM

119. Capital Company has decided to discontinue a product produced on a machine purchased four years ago at a cost of $
The machine has a current book value of $30,000.
Due to technologically improved machinery now available in the marketplace the existing machine has no current salvage v
The company is reviewing the various aspects involved in the production of a new product.
The engineering staff advised that the existing machine can be used to produce the new product.
Other costs involved in the production of the new product will be materials of $20,000 and labor priced at $5,000.
$20,000.00
Ignoring income taxes, the costs relevant to the decision to produce or not to produce the
new product would be $25,000.00

123. Allred Company sells its single product for $30 per unit. $30.00
The contribution margin ratio is 45%, 45%
and fixed costs are $10,000 per month.
Allred has an effective income tax rate of 40%
. If Allred sells 1,000 units in the current month, 1000
Allred’s variable expenses would be

VC ratio 55%
sales revenue $30,000.00
VC $16,500.00

124. Phillips & Company produces educational software. Its unit cost structure, based upon
an anticipated production volume of 150,000 units, is as follows. 150000

Sales price $160 $160


Variable costs 60 $60
Fixed costs 55 $55

The marketing department has estimated sales for the coming year at 175,000 units,which is within the relevant range of P
Phillip’s break-even volume (in units) and anticipated operating income for the coming year would amount to

total FC $8,250,000
CM $100

Rev $28,000,000
VC $10,500,000
CM $17,500,000
FC $8,250,000
OI $9,250,000

Break=FC/CM 82,500.00 units

126. Jeffries Company sells its single product for $30 per unit. $30
The contribution margin ratio is 45%, and. 45%
fixed costs are $10,000 per month $10,000
Sales were 3,000 units in April and 4,000 units in May. 3,000 4,000
How much greater is the May income than the April income?
April May
Rev $90,000 $120,000
VC 55% $16.50 $49,500 $66,000
CM $40,500 $54,000
FC $10,000 $10,000
OI $30,500 $44,000

127. Cervine Corporation makes two types of motors for use in various products. Operating
data and unit cost information for its products are presented below.
Product A

Annual unit capacity 10,000 20,000 10,000.00


Annual unit demand 10,000 20,000 10,000.00
Selling price $100 $80 $100.0
Variable manufacturing cost 53 45 $53.0
Fixed manufacturing cost 10 10 $10.0
Variable selling & administrative 10 11 $10.0
Fixed selling & administrative 5 4 $5.0
Fixed other administrative 2 0 $2.0
Unit operating profit $ 20 $10 $20.0
Machine hours per unit 2.0 1.5 2.0

Cervine has 40,000 productive machine hours available. 40,000.00

. What is the maximum total contribution margin that Cervine can generate in the coming year?

Product A
CM $37.00
CM per machine hours $18.50

first produce as much we can from product A 10,000


Total Machine hours 20,000
Total CM 370,000

the remaining hours to produce product B


total unit that can be produced from prdoct B
Total CM

Total CM

128. Lazar Industries produces two products, Crates and Trunks. Per unit selling prices, costs,
and resource utilization for these products are as follows.

Crates
Selling price $20 $30 20
Direct material costs $ 5 $ 5 5
Direct labor costs 8 10 8
Variable overhead costs 3 5 3
Variable selling costs 1 2 1
Machine hours per unit 2 4 2

Production of Crates and Trunks involves joint processes and use of the same facilities.
The total fixed factory overhead cost is $2,000,000 and total fixed selling and administrative costs are $840,000.
Production and sales are scheduled for 500,000 Crates and 700,000 Trunks.
Lazar has a normal capacity to produce a total of 2,000,000 units in
any combination of Crates and Trunks, and maintains no direct materials, work-inprocess,
or finished goods inventory.

Due to plant renovations Lazar Industries will be limited to 1,000,000 machine hours.
What is the maximum amount of contribution margin Lazar can generate during the
renovation period?
Crates
CM 3.00
CM per machine hours 1.50

first produce as much we can from Trunks


with Total Machine hours
Total CM

129. For the year just ended, Silverstone Company’s sales revenue was $450,000.
Silverstone’s fixed costs were $120,000 and it
s variable costs amounted to $270,000.
For the current year sales are forecasted at $500,000.
If the fixed costs do not change, Silverstone’s profits this year will be

For the current year sales are forecasted at $500,000.


CM will be (1-VC ratio 60%) 200000
OI $80,000

130. Breeze Company has a contribution margin of $4,000 and fixed costs of $1,000. If the
total contribution margin increases by $1,000, operating profit would

131. Wilkinson Company sells its single product for $30 per unit. The contribution margin
ratio is 45% and Wilkinson has fixed costs of $10,000 per month. If 3,000 units are sold
in the current month, Wilkinson’s income would be
133. Starlight Theater stages a number of summer musicals at its theater in northern Ohio.
Preliminary planning has just begun for the upcoming season, and Starlight has
developed the following estimated data.

Average
Number of Attendance per Ticket
Production Performances Performance Price
Mr. Wonderful 12 3500 $18.00
That’s Life 20 3000 $15.00
All That Jazz 12 4000 $20.00

1 Represent payments to production companies and are based on tickets sold.


2 Costs directly associated with the entire run of each production for
costumes, sets, and artist fees.

Starlight will also incur $565,000 of common fixed operating charges (administrative
overhead, facility costs, and advertising) for the entire season, and is subject to a 30%
income tax rate.

If Starlight’s schedule of musicals is held, as planned, how many patrons would have to
attend for Starlight to break even during the summer season?

FC $1,295,000.00
CM $16.20
Breakeven $79,938

134. Carson Inc. manufactures only one product and is preparing its budget for next year based on the following informatio

Selling price per unit $100.00


Variable costs per unit $75.00
Fixed costs $250,000.00
Effective tax rate 35%

If Carson wants to achieve a net income of $1.3 million next year, its sales must be

CM $25.00
EBT $2,000,000.00
Numerator to be used $2,250,000.00
units to achieve 1.3 m 90,000.00
135. MetalCraft produces three inexpensive socket wrench sets that are popular with do-ityourselfers.
Budgeted information for the upcoming year is as follows.
Estimated
Model Selling Price Variable Cost Sales Volume
No. 109 10 5.5 30000
No. 145 15 8 75000
No. 153 20 14 45000
150000
Total fixed costs for the socket wrench product line is $961,000. If the company’s actual
experience remains consistent with the estimated sales volume percentage distribution,
and the firm desires to generate total operating income of $161,200, how many Model
No. 153 socket sets will MetalCraft have to sell?

Model CM CM
No. 109 20% 4.5 0.9
No. 145 50% 7 3.5
No. 153 30% 6 1.8
6.2

Numerator 1122200
units to ac 181000
No. 153 54300

136. Robin Company wants to earn a 6% return on sales after taxes.


The company’s effective income tax rate is 40%, and its contribution margin is 30%.
If Robin has fixed costs of $240,000, the amount of sales required to earn the desired return is

desired return return on sales 6%


tax rate 40%
contribution margin 30%
fixed costs $240,000.00

This problem can be solved by setting up an equation and solving for the required sales amount, which
will be represented by the variable S. S = required sales
137. ,"Bargain Press is considering publishing a new textbook. The publisher has developed the ",
,"following cost data related to a production run of 6,000, the minimum possible ",
,"production run. Bargain Press will sell the textbook for $45 per copy. How many ",
,"textbooks must Bargain Press sell in order to generate operating earnings (earnings before ",
,"interest and taxes) of 20% on sales? (Round your answer up to the nearest whole ",
,"textbook.) ",
,"Estimated cost ",
Development (reviews, class testing, editing) ,"$35,000 ",
Typesetting ,"18,500 ",
Depreciation on Equipment ,"9,320 ",
General and Administrative ,"7,500 ",
Miscellaneous Fixed Costs ,"4,400 ",
Printing and Binding ,"30,000 ",
Sales staff commissions (2% of selling price) ,"5,400 ",
Bookstore commissions (25% of selling price) ,"67,500 ",
Author’s Royalties (10% of selling price) ,"27,000 ",
Total costs at production of 6,000 copies ,"$204,620 ",
a. ,"2,076 copies. ",
b. ,"5,207 copies. ",
c. ,"5,412 copies. ",
d. ,"6,199 copies. ",

Feedback: The correct answer is: 5,207 copies. ,


The number of copies required can be found by solving the following equation, where x equals the ,
number of copies: ,
Total sales – total variable costs – fixed costs + typesetting + depreciation expense + general and ,
administrative expenses + miscellaneous fixed costs = (return %)(total sales) ,
Total sales = $45x ,
Total variable cost = (variable cost per unit, or VCU)(x) = VCU(x) ,
Fixed costs = development + typesetting + depreciation + general and administrative + miscellaneous ,
fixed costs ,
Fixed costs = $35,000 + $18,500 + $9,320 + $7,500 + $4,400 = $74,720 ,
VCU = (printing and binding + sales staff commissions + bookstore commissions + royalties) / ,
(number of copies) ,
VCU = ($30,000 + $5,400 + $67,500 + $27,000) / (6,000 copies) = $129,900 / 6,000 ,
VCU = $21.65 per copy ,
$45x – $21.65(x) – $74,720 = (0.2)($45x) ,
$45x – $21.65(x) - $74,720 = $9x ,
$36x – $21.65(x) = $74,720 ,
$14.35x = $74,720 ,
x = 5,206.97, which rounds to 5,207 copies. ,
25000 22000

1000

turing other products. This is a one time special order.

XR-2000 XR-2000
Q 6,250.00 $387,500.0
$24.0 Q 5,500.00 $341,000.0
$10.0 $46,500.0
$5.0 $46.50
$40.0
$4.0
$10.0
$105.0
4.00

$62.0
$15.5

y not producing XR-2000

750

106. C c. a decrease of $2,000.

Segment B Total
$15,000.0 $25,000.0
$8,500.0 $12,500.0
$2,500.0 $4,000.0
$4,000.0 $8,500.0
$3,000.0 $5,000.0
$1,500.0 $3,000.0
-$500.0 $500.0

Segment B Total

$10,000.0
$4,000.0
$1,000.0 $2,500.0
-$1,000.0 $3,500.0
$2,000.0
$1,500.0 $3,000.0
-$2,500.0 -$1,500.0

-$2,000.00

107. C Z, X, Y.
Approach Y Approach Z

$38.0 $32.0
12,000.00 14,000.00
$456,000.0 $448,000.0
$360,000.0 $420,000.0
$45,600.0 0
$30,000.0
$20,400.0 $28,000.0

108. C c. 14 units.

6 - 10 11 - 15 16 - 20
$400,000.0 $600,000.0 $800,000.0
$50,000.0 $45,000.0 $45,000.0
$100,000.0 $100,000.0 $100,000.0
e and variable costs are as follows. 109. B b. $67,200.

110. A a. $24,000.

7,000.00 111. B b. $10,000.

$30,000.0
$5.0

$100,000.0

$65,000.0
$35,000.0

$20,000.0
$15,000.0

$100,000.0
increase Raymund’s fixed manufacturing costs
by $30,000 per month and reduce its variable
$40,000.0 costs by $5 per uni
$60,000.0

$50,000.0
$10,000.0

osts per unit, for 20,000 units of ice-makers, are as follows. 112. C
20,000.00

$28 per unit.


ead amounting to $6 per unit could be eliminated. $6.0
n an anticipated volume of 150,000 units, is as follows. 113. C
150,000.00

e of Phillip’s cost structure. 175,000.00


reduction of $750,000 in fixed costs $750,000.0

100%
32%
68%

2% and operating income of $9,600,000.

114. B b. No, the machine-related cost of pro


1,600,000.00
ucing each unit of product should be $.50. $0.5
1,200,000.00
600000

use the manufacturer’s engineering specification of machine related unit production cost?

$800,000.0
$0.67

115. B
Product B
20,000.00
20,000.00
$80.0
$45.0
$10.0
$11.0
$4.0
$0.0
$10.0
1.5

aking a decision on product priorities for the coming year, are

Product B
$24.0
$16.0

116. B
4500 15300 $3.40
$4.05

ds of Kaylene. 4200

Irrelevant Decision Factor


Purchase price of $3.40 per lb.

tilizer and weed killer that is popular in hot, dry climates. 117. C
awn fertilizer.
40

$18.5
$20.0
-$1.50

anufactures several different products, and the loss appear after the deduction of the allocated fixed costs

$2.5

119. A a. $25,000.
ased four years ago at a cost of $70,000.

machine has no current salvage value.

d labor priced at $5,000.


$5,000.00

123. D d. $16,500.

124. B
b. 82,500 units and $9,250,000 of operating income.
h is within the relevant range of Phillip’s cost structure. 175000
ar would amount to

126. B b. $13,500.

1,000

diff
$30,000

$13,500

$13,500

127. B b. $689,992.
689992
Product B

20,000.00
20,000.00
$80.0
$45.0
$10.0
$11.0
$4.0
$0.0
$10.0
1.5

Product B
$24.00
$16.00

370,000

20,000 13333
13,333.33 $319,992.00
320,000 320,000

$689,992 $0.0

128. B b. $2,000,000.

Trunks
30
5
10
5
2
4

e costs are $840,000.

1,000,000
Trunks
8.00
2.00

250,000 Trunks will use all the machine hours


1,000,000
$2,000,000

%
450000 b. $80,000. 129. B
120000
270000 0.6
500000

130. C increase by $1,000.


133. C c. 79,938.

Variable Fixed CM no. of units %


Costs 1 Costs 2
$3.00 $165,000.00 $15.00 42,000.00 28% $4.20
$1.00 $249,000.00 $14.00 60,000.00 40% $5.60
$0.00 $316,000.00 $20.00 48,000.00 32% $6.40
$730,000.00 150,000.00 $16.20

$565,000.00
30%

based on the following information. 134. D d. 90,000 units.

$1,300,000
tyourselfers. 135. B b. 54,300.

sets
sets
sets

961000

161200

136. D d. $1,200,000.

mount, which
cellaneous ,
103. B b. $96.50.

KT-6500
3,000.00

c. a decrease of $2,000.
fixed manufacturing costs
h and reduce its variable
y $5 per uni

c. $600,000.
achine-related cost of producing each unit is $.67.
1. Account for all units (physical flow of quantities).

Beginning WIP Total Units to Account for =


Started Units this period Finished or Transferred-out Goods
Total Units to Account for - Ending WIP
Spoilage (lost)

Step 2: compute Equivalent Units of Production


EUP Computation under FIFO (If materials are added as work in process (continually):
Material Conversion

Total units Completed


- Beginning WIP (regardless of % of completion)
Units Started and Completed this Period - -
+ Amount needed to complete Beginning WIP
+ Amount Completed on Ending WIP
EUP under FIFO - -

If materials are added at the beginning of a process :


Total Units Completed
+ Amount of materials Needed to Complete BWIP zero
.+ Amount of materials Added to Date on EWIP 100%
EUP for Materials -

If materials are added at the end of a process


Total Units Completed
+ Amount of materials Needed to Complete BWIP
+ Amount of materials Added to Date on EWIP zero
EUP for Materials -

EUP : Weighted Average


Ü  EUP under weighted average costing may be computed as follows:
Total Units Completed this period
Work to date on Ending WIP
EUP under weighted average -
-
d-out Goods
Consider Bushells Company, which produces 12-ounce bottles of iced tea at ils
Sydney bottling plant. The annual fixed manufacturing costs of the bottling plant are
Bushells currently uses absorption costing with a standard-costing systemfor . $5,400,000
external reporting purposes, and it calculates its budgeted fixed manufacturing rate ana
per case basis (one case is twenty-four 12-ounce bottles of iced tea). We will now examine
four different capacity levels used as the denominator to compute the budgeted fixed
~'manufacturing overhead cost rate: theoretical capacity, practical capacity, normal capaci
.utilization, and master-budget capacity utilization

In business and accounting, capacity ordinarily means "constraint," an "upper limit.'


Theoretical capacity is the level of capacity based on producing at full efficiency all the time. Bushells can produce 10,000
operating at maximum speed. Assuming 360 days per year, the theoretical annual capacity for three 8-hour shifts per da

Theoretical capacity is theoretical in the sense


10,000 cases per shift 10,000.00 interruptions because of bottle breakage on th
3 shifts per day 3.00 represents an ideal goal of capacity usage. The
360 days 360.00
= 10,800,000 cases 10,800,000.00

Practical capacity is the level of capacity that reduces theoretical capacity by unavoidable operating interruptions, such a
so on. Assume that the pra ical capacity is the practical production rate of 8,000 cases per shift for three shifts per day for

8,000 cases per shift 8000


3 shifts per day 3
300 days 300
= 7,200,000 cases 7,200,000.00

Both theoretical capacity and practical capacity measure capacity levels in terms of what a plant can supply- available cap
budget capacity utilization measure capacity levels in terms of demand for the output of the plant - the amount of the av
demand for its products. In many cases, budgeted demand is well below the production capacity available.

Normal capacity utilization is the level of capacity utilization that satisfies average customer demand over a period (say,
Master-budget capacity utilization is expected level of capacity utilization for the current budget period, typically one ye
when an industry has cyclical periods of high and low demand or
when management believes that the budgeted production for the coming period is not representativeof long-run deman

ConsiderBushells' master budget for 2004, based on production of 4,000,000 cases of tea per year.Despite using this mas
2004,top management believes that over the next three years the normal (average) annual production level will be 5,000
4,000,000 cases to be "abnormally" low. Why? Because a major competitor (Tea- Mania)has been sharply reducing its sel
expects that the competitor's lower price and advertising blitz will not be a long-run phenomenon and that, in 2005, Bus
ConsiderBushells' master budget for 2004, based on production of 4,000,000 cases of tea per year.Despite using this mas
2004,top management believes that over the next three years the normal (average) annual production level will be 5,000
4,000,000 cases to be "abnormally" low. Why? Because a major competitor (Tea- Mania)has been sharply reducing its sel
expects that the competitor's lower price and advertising blitz will not be a long-run phenomenon and that, in 2005, Bus

Master budget for 2004 VS.


8,000 cases per shift 8000
one shift working 300 days 300
2,400,000.00

8,000 cases per shift 8000


another shift working 200 days 200
1,600,000.00

master budget for 2004 4,000,000.00

Effect on Budgeted Fixed Manufacturing Overhead Cost Rate


Wenow illustrate how each of these four denominator levels affect the budgeted fixed manufacturing overhead cost rate.
5,400,000 for 2004. This lump-sum amount is incurred to provide the capacity to bottle iced tea.This lump sum, includes,
the compensation of the plant manager. The budgeted fixed manufacturing overhead cost rates for 2004 for each of the f

fixed manufacturing costs of 5,400,000 for 2004 $5,400,000.00

Budgeted Fixed Manufacturing Overhead Cost per Case

Theoreticalcapacity $0.50
Practicalcapacity $0.75
Normalcapacity utilization $1.08
Master-budgetcapacity $1.35 170%
utilization

Now assume that the Standard variable manufacturing cost is $5.20 per case. The total standard manufacturing cost per c
concepts is

Total
Standard variable manufacturing cost is $5.20 Manufacturing
Cost per Case

Theoreticalcapacity $5.70
Practicalcapacity $5.95
Normalcapacity utilization $6.28
Master-budgetcapacity $6.55
utilization
Effect on Financial Statements
The magnitude of the favorable/unfavorable production-volume variance under absorption costing will be affected by the
fixed manufacturing cost per case. Assume Bushells' actual production in 2004 is 4,400,000 cases of iced tea. Actual sales
inventory for 2004 and no price variances, spending variances, or efficiency variances in manufacturing costs. Those assum
and actual fixed manufacturing overhead costs are both $5,400,000. The average selling price per case of iced tea is $8.00

actual production in 2004 is 4,400,000.00


Actual sales for 2004 are 4,200,000.00 price
budgeted fixed manufacturing overhead costs and actual fix $5,400,000.00

The production-volume variance :

FOH allocated

Theoreticalcapacity $2,200,000.00
Practicalcapacity $3,300,000.00
Normalcapacity utilization $4,752,000.00
Master-budgetcapacity $5,940,000.00
utilization

How Bushells handles its end-of-period variances will determine the effect these p
will have on the company's operating income. We now discuss the three alternati
use to handle the production-volume varianc

1. Adjusted allocation-rate approach. This approach restates all amounts in the general and su
budgeted cost rates. Given that actual fIXed manufacturing overhead costs are $5,400,000 and
recalculated fixed manufacturing overhead cost is $1.23 per case ($5,400, 000 /4,4OO,OOO ca
allocation-rate approach results in the choice of the capacity level used to calculate the budge
having no effect on end-of-period financial statements.
In effect,an actual costing system is adopted at the en
Actual Rate $1.23

2. Proration approach. The underallocated or overallocated overhead is spread among (a)endin


(c) cost of goods sold. The proration restates the ending balances of (a), (b), and (c) to what the
used rather than budgeted cost rates. The proration approach also results in the choice of the c
fixedmanufacturing overhead cost per case having no effect on end-of-period financial stateme

3. Write-off variances to cost of goods sold approach. Exhibit 9-7 shows how use of this appro
Recall, Bushells had no beginning inventory,production of 4,400,000 cases, and sales of 4,200,0
31, 2004, is 200,000 cases. Using master-budget capacity utilizationas the denominator results
manufacturing overheadcost per case to the 200,000 cases in ending inventory. Accordingly, op
capacity utilization concept. The differences in operating income for the four denominator-level
amounts of fixed manufacturing overhead being inventoried at the end of2004:

Fixed Manufacturing Overhead in Dec. 31, 2004,Inventory 200,000.00

Theoreticalcapacity $100,000.00
Practicalcapacity $150,000.00
Normalcapacity utilization $216,000.00
Master-budgetcapacity $270,000.00
utilization

Income Statement Effects of Using Alternative Capacity-level Concepts Bushells Company for 2004

Theoretical Practical
10,800,000.00 7,200,000.00
sales rev $33,600,000.00 $33,600,000.00
Cost of Goods Sold
Beginning inventory $0.00 $0.00
Variable manufacturing costs $22,880,000.00 $22,880,000.00
Fixed manufacturing costs $2,200,000.00 $3,300,000.00
Cost for goods available for sale $25,080,000.00 $26,180,000.00
deduct ending inventory $1,140,000.00 $1,190,000.00
Total COGS(at standard costs) $23,940,000.00 $24,990,000.00
Adjustment for manufacturing variances $3,200,000.00 $2,100,000.00
Total COGS $27,140,000.00 $27,090,000.00
Gross Margin $6,460,000.00 $6,510,000.00
Operating cost $2,810,000.00 $2,810,000.00
Operating income $3,650,000.00 $3,700,000.00
the $54,000 difference ($3,820,000 - $3,766,000) in operating income betweenthe master-budget
capacity and the normal capacity utilization concepts is due to thedifference in fixed manufacturing
overhead inventoried ($270,000- $216,000)

Suppose Bushells Company is computing its operating income for 2006. That year's results are identical to the results for 2
utilization for 2006 is 6,000,000 cases instead of 4,000,000 cases. Production in 2006 is 4,400,000 cases. There is no begin
other than the production-volume variance.Bushells writes off this variance to cost of goods sold. Sales in 2006 are 4,200

master-budget capacity utilization for 2006 is 6,000,000.00


Production in 2006 is 4,400,000.00
Sales in 2006 are 4,200,000.00
Ending Inventory 200,000.00

Theonly change in Exhibit 9-7 results would be for the master-budget capacity utilization level. The
budgeted fIXed manufacturing overhead cost rate for 2006 is

fixed manufacturing costs of 5,400,000 for 2004

Budgeted Fixed Manufacturing Overhead Cost per Case

Master-budgetcapacity $0.90
allocated fixed cost $3,960,000.00
PVV $1,440,000.00 $1,440,000.00

Themanufacturing cost per case is $6.10 ($5.20 + $0.90). So, the production-volume variance for
2006is
$6.10

Income Statement 2006

sales rev 33,600,000.00


Cost of Goods Sold
Beginning inventory 0
Variable manufacturing costs $22,880,000.00
Fixed manufacturing costs $3,960,000.00
Cost for goods available for sale $26,840,000.00
deduct ending inventory $1,220,000.00
Total COGS(at standard costs) $25,620,000.00
Adjustment for manufacturing variances $1,440,000.00
Total COGS $27,060,000.00
Gross Margin $6,540,000.00
Operating cost $2,810,000.00
Operating income $3,730,000.00 -$90,000.00

The higher denominator level used to calculate budgeted fixed manufacturing cost per case inthe 2006 master budget
that fewer fixed manufacturing overhead costs are inventoried in 2006 than in 2004, given identical sales and producti
and assuming the production-volumevariance is written off to cost of goods sold.
e
00,000
ana
xamine
ed
capaci

.'
the time. Bushells can produce 10,000 cases of iced tea per shift when the bottling lines are
capacity for three 8-hour shifts per day is

tical capacity is theoretical in the sense that it does not allow for any plant mainte. nance,
ptions because of bottle breakage on the filling lines, or any other factor. Theoretical capacity
nts an ideal goal of capacity usage. Theoretical capacity is unattainable in the real world.

oidable operating interruptions, such as scheduled maintenance time, shutdowns for holidays, and
ses per shift for three shifts per day for 300 days a year. The practical annual capacity

f what a plant can supply- available capacity. In contrast, normal capacity utilization and master.
put of the plant - the amount of the available capacity that the plant expects to use based on the
uction capacity available.

customer demand over a period (say, 2 to 3 years) that includes seasonal, cyclical, and trendfactors.
current budget period, typically one year. These two capacity utilization levels can differ-for example,

s not representativeof long-run demand.

s of tea per year.Despite using this master-budget capacity utilization level of 4,000,000 cases for
e) annual production level will be 5,000,000 cases. They view 2004's budgeted production level of
Mania)has been sharply reducing its selling price and spending large amounts on advertising. Bushells
un phenomenon and that, in 2005, Bushells' production and sales will be higher.
s of tea per year.Despite using this master-budget capacity utilization level of 4,000,000 cases for
e) annual production level will be 5,000,000 cases. They view 2004's budgeted production level of
Mania)has been sharply reducing its selling price and spending large amounts on advertising. Bushells
un phenomenon and that, in 2005, Bushells' production and sales will be higher.

normal (average) annual production level will be 5,000,000 cases.


5,000,000.00

head Cost Rate


xed manufacturing overhead cost rate. Bushells has budgeted fixed manufacturing costs of
ottle iced tea.This lump sum, includes, among other costs, lease costs for bottling equipment ,and
ad cost rates for 2004 for each of the four capacity-level concepts are

Thebudgeted fixed manufacturing overhead cost rate based on master-budget capacity


utilization, $1.35 per case, is 170% higher than the cost rate based on theoretical capacity,
$0.50per case. This big difference in cost rates is because theoretical capacity is much
largerthan the master-budget capacity utilization.

otal standard manufacturing cost per case with alternative capacity-level


sorption costing will be affected by the choice of the denominator used to calculate the budgeted
400,000 cases of iced tea. Actual sales for 2004 are 4,200,000 cases. Also assume no beginning
es in manufacturing costs. Those assumptions mean budgeted fixed manufacturing overhead costs
elling price per case of iced tea is $8.00. Operating costs equal $2,810,000.

$8.00

PVV The higher the denominator level, (1) the lower the budgeted FMOH cost rate,
and the lower the amount of FMOH allocated to output produced (because
the budgeted FMOH cost rate is lower), and (3) the higher the unfavorable PVV
$3,200,000.00 U (because the higher the denominator level, the more likely actual output will
$2,100,000.00 U fall short of that level).
$648,000.00 U
-$540,000.00 F

etermine the effect these production-volume variances


discuss the three alternative approaches Bushells can

amounts in the general and subsidiary ledgers by using actual rather than
head costs are $5,400,000 and actual production is 4,400,000 cases,the
e ($5,400, 000 /4,4OO,OOO cases, rounded up to the nearest cent). The adjusted
el used to calculate the budgeted fixed manufacturingoverhead cost per case

adopted at the end of the period.


head is spread among (a)ending work in process, (b) ending finished goods, and
s of (a), (b), and (c) to what they would have been if actual cost rates had been
so results in the choice of the capacity level used to calculate the budgeted
nd-of-period financial statements .

7 shows how use of this approch affects Bushells' operating income for 2004.
000 cases, and sales of 4,200,000 cases. Hence, the ending inventory on December
tionas the denominator results in assigning the highest amount of fixed
ding inventory. Accordingly, operating income is highest using the master-budget
for the four denominator-level concepts in Exhibit 9-7 are due to different
he end of2004:

pany for 2004

Normal Master-budget
5,000,000.00 4,000,000.00
$33,600,000.00 $33,600,000.00

$0.00 $0.00
$22,880,000.00 $22,880,000.00
$4,752,000.00 $5,940,000.00
$27,632,000.00 $28,820,000.00
$1,256,000.00 $1,310,000.00
$26,376,000.00 $27,510,000.00
$648,000.00 -$540,000.00
$27,024,000.00 $26,970,000.00
$6,576,000.00 $6,630,000.00
$2,810,000.00 $2,810,000.00
$3,766,000.00 $3,820,000.00
$54,000.00
$54,000.00
master-budget
d manufacturing

results are identical to the results for 2004, shown in Exhibit 9-7, except that master-budget capacity
06 is 4,400,000 cases. There is no beginning inventory on January 1, 2006, and there are no variances
of goods sold. Sales in 2006 are 4,200,000 cases.

tion level. The

variance for
ost per case inthe 2006 master budget means
004, given identical sales and production levels
d.
budgeted FMOH cost rate,
tput produced (because
higher the unfavorable PVV
re likely actual output will
Price $200.00 100% $ 1.00
VC $120.00 60% $ 0.60
CM $80.00 40% $ 0.40
FC $2,000.00
CM% = CM/Price 40%
Q.Break=FC/CM 25
Units
0 1 5
Revenues $0.00 $200.00 $1,000.00
VC $0.00 $120.00 $600.00
CM $0.00 $80.00 $400.00
FC $2,000.00 $2,000.00 $2,000.00
OI ($2,000.00) ($1,920.00) ($1,600.00)

CM% = CM/Price #DIV/0! 40% 40%


VC%=VC/Price #DIV/0! 60% 60%
Break $=FC/CM%

Q Margin of safety =current Q- BE.Q


$ Margin of safety =current Rev-BE.Rev

Target OI $ 1,200.00 TOI=Rev-VC-FC


TOI Q=FC+TOI/$CM 40 How many units must be sold to generate enough CM to
.=since each $ of revenue earn 40 cent of CM What is the revenue in$ required to generate $1200 TOI?
thus to earn $3200 CM rev must =
3200/.4 $ 8,000.00 .=FC+TOI/CM%

Target NI= TOI-(TOI x%T)


Target NI= TOI x (1-T%)
Target OI= TNI / (1-T%)
Since, TOI=Rev-VC-FC
thus, Rev-VC-FC= TNI / (1-T%)
TNI $ 960.00 How many units must be sold to generate enough CM to
TNI Q=FC+(TNI/1-T%)/$CM 45.00
TNI Revenue=FC+(TNI/1-T%)/%CM $ 9,000.00

Consider a decision about which features to add to an existing product.


Different choices can affect selling prices, variable cost per unit, fixed costs,
units sold, and operating income.

Strategic decisions invariably entail risk. CVP analysis evaluates how operating income will be
affected if the original predicted data are not achieved - say, if sales are 10%lower than
estimated. Evaluating this risk affects other strategic decisions a company might make. For
example, if the probability of a decline in sales seems high, a manager may take actions to
shift the cost structure to have more variable costs and fewer fixed ,) costs.
Strategic decisions invariably entail risk. CVP analysis evaluates how operating income will be
affected if the original predicted data are not achieved - say, if sales are 10%lower than
estimated. Evaluating this risk affects other strategic decisions a company might make. For
example, if the probability of a decline in sales seems high, a manager may take actions to
shift the cost structure to have more variable costs and fewer fixed ,) costs.

Decision to Advertise
New Q will be 44 instead of 40
New FC will be $500 higher than old FC due to Adv.
Selling Price will be the same

Units without Adv. Units with Adv.


40 44
CM $3,200.00 $3,520.00
FC $ (2,000.00) $ (2,500.00)
OI $1,200.00 $1,020.00

Operating income decreases by $180, so Mary should not advertise.

Decision to Reduce Selling Price from $200 to $175


New VC will be $115 instead of $120
New CM will be $175-$115=$60, instead of $80 60
FC will be the same

Units without reducing. Units with reducing.


40 50
CM $3,200.00 $3,000.00
FC $ (2,000.00) $ (2,000.00)
OI $1,200.00 $1,000.00

Contribution margin from lowering price to $175: ($175 - $115) per unit x 50 units
Contribution margin from maintaining price at $200: ($200 - $120) per unit x 40 units
Change in contribution margin from lowering price

Mary can immediately see the revenues that need to be generated to reach particular operating-income levels, given a
variable cost per unit. For example, revenues of $6,40.0 ($2Q{) per unit x 32 units) are required to earn an operating inco
and variable cost per unit is $100. Mary can also use Exhibit 3-4 to assess what revenues she needs to break even (earn o
the booth rental at the Chicago convention is raised to $2,800 (increasing fIxed costs to $2,800) or if the software supplier
variable cost to $150 per unit).
Rev
must
be
200
FC VC CM CM%
1 2000 $ 100.00 $ 100.00 50%
2 2000 $ 120.00 $ 80.00 40%
3 2000 $ 150.00 $ 50.00 25%
4 2400 $ 100.00 $ 100.00 50%
5 2400 $ 120.00 $ 80.00 40%
6 2400 $ 150.00 $ 50.00 25%
7 2800 $ 100.00 $ 100.00 50%
8 2800 $ 120.00 $ 80.00 40%
9 2800 $ 150.00 $ 50.00 25%

COST PLANNING AND CVP


Alternative fixed-Cost/Variable-Cost Structures

Compare line 2 (fIxed costs, $2,000; variable cost per unit, $120)-and line 7 (fIxed
costs, $2,800; variable cost per unit, $100) See how the revenues required to break
even are higher for line 7 ($5,600 versus $5,000 in line 2) ..whereas the revenues
required to earn $2,000 of operating income are lower in line T ($9,600 versus
$10,000 in line 2). Line 7, with higher fIxed costs, has more risk of loss (has a higher
breakeven point) but offers a greater return (more profits) as revenues increase.

CVP analysis can help managers evaluate various fIXed-costjvariable-cost structures. To


consider these choices in more detail, let's return to Do-All Software. Mary is paying a
$2,000 booth-rental fee. Suppose Computer
Conventions offers Mary three rental alternatives:
Option 1: $2,000 fIXed fee
Option 2: $800 fIXed fee plus 15% of convention revenues
Option 3: 25% of convention revenues with no fIxed fee

If Mary sells 40 units, she should be indiff


various options. Each option results in ope
$1,200.

Quantity sold 40
Price $ 200.00
VC $ 120.00 VC
incremental VC in option 2 (15% from rev) $ 30.00 CM per unit
incremental VC in option 3 (25% from rev) $ 50.00 Quantity sold
CM
CM%
FC
OI

The CVP analysis, however, highlights the different risks of loss and different return
differ from 40 units. The higher risk of a loss in option 1 is because of its higher fixe
higher breakeven point (25 units) and a lower margin of safety (40 - 25 = 15 units) r
representing option 1 intersects the
horizontal axis farther to the right than the lines representing options 2 and 3.

Quantity sold 25
Price $ 200.00
VC $ 120.00 VC
incremental VC in option 2 (15% from rev) $ 30.00 CM per unit
incremental VC in option 3 (25% from rev) $ 50.00 Quantity sold
CM
CM%
FC
OI
Q Break

Consider operating income under each option if the number of units sold dr
20. Exhibit 3-5 shows that option 1 leads to an operating loss, whereas optio
continue to generate operating incomes. (A vertical line from X = 20 units so
option 1 line below the horizontal axis in the mauve area and cuts the optio
lines above the horizontal axis in the blue-green area.) The higher risk ofloss
however, must be evaluated against its potential benefits. Option 1 has the
margin per unit because of its low variable costs.

Quantity sold 20
Price $ 200.00
VC $ 120.00 VC
incremental VC in option 2 (15% from rev) $ 30.00 CM per unit
incremental VC in option 3 (25% from rev) $ 50.00 Quantity sold
CM
CM%
FC
OI
Q Break

Once fixed costs are recovered at sales of 25 units, each additional unit sold adds $80 of contribution margin and, therefo
$80 of operating income per unit. For example, at sales of 60 units, option 1 shows an operating income of $2,800, greate
under options 2 and 3. By moving from option 1 toward option 3, Mary faces less risk of loss when demand is low, both b
contribution margin per unit. She must, however, accept less operating income when
demand is high because of the higher variable costs of option 3 compared with options 1 and 2. The choice among option
the level of demand for the software package and her willingness to risk losses if demand
is low.
Once fixed costs are recovered at sales of 25 units, each additional unit sold adds $80 of contribution margin and, therefo
$80 of operating income per unit. For example, at sales of 60 units, option 1 shows an operating income of $2,800, greate
under options 2 and 3. By moving from option 1 toward option 3, Mary faces less risk of loss when demand is low, both b
contribution margin per unit. She must, however, accept less operating income when
demand is high because of the higher variable costs of option 3 compared with options 1 and 2. The choice among option
the level of demand for the software package and her willingness to risk losses if demand
is low.

Quantity sold 60
Price $ 200.00
VC $ 120.00 VC
incremental VC in option 2 (15% from rev) $ 30.00 CM per unit
incremental VC in option 3 (25% from rev) $ 50.00 Quantity sold
CM
CM%
FC
OI
Q Break

The risk-return tradeoff across alternative cost structures can be measured as operating leverage. Operating le
on changes in operating income as changes occur in units sold and, hence, in contribution margin.
Organizations with a high proportion of fixed costs in their cost structures, as is the case under option 1, have h
option 1 in Exhibit 3-5 is the steepest of the three lines. Small increases in sales lead to large increases in operati
incomes. Small decreases in sales result in relatively large decreases in operating incomes, leading to a greater r
the degree of operating leverage equals contribution margin divided by operating income.

If Mary sells 40 units, she should be indiff


various options. Each option results in ope
$1,200.

Quantity sold 40
Price $ 200.00
VC $ 120.00 VC
incremental VC in option 2 (15% from rev) $ 30.00 CM per unit
incremental VC in option 3 (25% from rev) $ 50.00 Quantity sold
CM
CM%
FC
OI

Operating Leverage=CM/OI

These numbers indicate that, when sales are 40 units, a percentage change in sales and contribution margin will result in
for option 1, but the same percentage change in operating income for option 3.
Consider, for example, a sales increase of 50% from 40 to 60 units. Contribution margin will increase by 50% under each
2.67 x 50% = 133% from $1,200 to $2,800 in option 1, but it will increase only by 1.00 x 50% = 50% from $1,200 to $1,800
These numbers indicate that, when sales are 40 units, a percentage change in sales and contribution margin will result in
for option 1, but the same percentage change in operating income for option 3.
Consider, for example, a sales increase of 50% from 40 to 60 units. Contribution margin will increase by 50% under each
2.67 x 50% = 133% from $1,200 to $2,800 in option 1, but it will increase only by 1.00 x 50% = 50% from $1,200 to $1,800
leverage at a given level of sales helps managers calculate the effect of fluctuations in sales
on operating incomes.

Quantity sold 60
Price $ 200.00
VC $ 120.00 VC
incremental VC in option 2 (15% from rev) $ 30.00 CM per unit
incremental VC in option 3 (25% from rev) $ 50.00 Quantity sold
CM
CM%
FC
OI
50 % in units sold will raise the CM with 50% in each option

50 % in units sold will raise the OI with diff. %


it is the same like%increase in CM x O.leverage

Operating Leverage=CM/OI or CM/CM-FC

These numbers indicate that, when sales are 40 units, a percentage change in sales and contribution margin will result in
for option 1, but the same percentage change in operating income for option 3.
Consider, for example, a sales increase of 50% from 40 to 60 units. Contribution margin will increase by 50% under each o
2.67 x 50% = 133% from $1,200 to $2,800 in option 1, but it will increase only by 1.00 x 50% = 50% from $1,200 to $1,800
leverage at a given level of sales helps managers calculate the effect of fluctuations in sales
on operating incomes.

old sales
(1) % change in EBIT 133% $ 8,000.00
(2) % change in output (sales) 50% 50%
DOL (1)/ (2) 2.67

(1) EBIT + FC it the same as CM $ 3,200.00


(2) EBIT $ 1,200.00
DOL (1)/ (2) 2.67

What does "DOLs 40 units = "2.67" really mean?


It means that a 1 percent change in SALES from the 40-unit sales position causes a 2.67 percen
change in SALES from the 40-unit position causes a percentage change in EBIT that is 2.67 tim
What does "DOLs 40 units = "2.67" really mean?
It means that a 1 percent change in SALES from the 40-unit sales position causes a 2.67 percen
change in SALES from the 40-unit position causes a percentage change in EBIT that is 2.67 tim
decrease in SALES causes a 8.01 percent decrease in EBIT, while a 4 percent increase in SALES c

How would knowledge of a firm's DOL be of use to a financial manager?


The manager would know in advance what impact a potential change in sales would have on
this advance knowledge, the firm may decide to make some changes in its sales policy and/ or
not like to operate under conditions of a high degree of operating leverage. Since, in that situa
operating loss.

The degree of operating leverage should thus be viewed as a measure of "potential risk" whic
sales and production cost variability.The degree of operating leverage itself is not the source o

12 20
8 15
RC HS
CM $4.0 $5.0
units 10000 15000 25000
40% 60%
$1.6 $3.0 $4.6

fc $94,000.0 20,434.78

8,173.91 12,260.87 20,434.78


$12.0 $20.0
$8.0 $15.0
RC HS
CM $4.0 $5.0
units 6250 18750 25000
25% 75%
$1.0 $3.8 $4.8

fc $94,000.0 19,789.47

4,947.37 14,842.11 19,789.47


Units
25 40 45
$5,000.00 $8,000.00 $9,000.00
$3,000.00 $4,800.00 $5,400.00
$2,000.00 $3,200.00 $3,600.00
$2,000.00 $2,000.00 $2,000.00
$0.00 $1,200.00 $1,600.00
25
40% 40% 40%
60% 60% 60%
$ 5,000.00

15 20
3,000 4,000 4800

ld to generate enough CM to cover FC and TOI???


uired to generate $1200 TOI??

ld to generate enough CM to cover FC and TNI???

1,600.00 2000 3,600.00


45.00
Diff
4
$ 320.00
$ (500.00)
$ (180.00)

se.

Diff
10
$ (200.00)
0
$ (200.00)

$ 3,000.00
$ 3,200.00
$ (200.00)

rating-income levels, given alternative levels of fIxed costs and


red to earn an operating income of $1,200 if fIxed/costs are $2,000
needs to break even (earn operating income of $0) if, for example,
00) or if the software supplier raises its price to $150 (increasing
Rev
must
be
To earn OI
zero $ 1,200.00 $ 1,600.00 $ 2,000.00
rev must be $ 4,000.00 $ 6,400.00 $ 7,200.00 $ 8,000.00
rev must be $ 5,000.00 $ 8,000.00 $ 9,000.00 $ 10,000.00
rev must be $ 8,000.00 $ 12,800.00 $ 14,400.00 $ 16,000.00
rev must be $ 4,800.00 $ 7,200.00 $ 8,000.00 $ 8,800.00
rev must be $ 6,000.00 $ 9,000.00 $ 10,000.00 $ 11,000.00
rev must be $ 9,600.00 $ 14,400.00 $ 16,000.00 $ 17,600.00
rev must be $ 5,600.00 $ 8,000.00 $ 8,800.00 $ 9,600.00
rev must be $ 7,000.00 $ 10,000.00 $ 11,000.00 $ 12,000.00
rev must be $ 11,200.00 $ 16,000.00 $ 17,600.00 $ 19,200.00

40 units, she should be indifferent across the


ns. Each option results in operating income of

Option 1 Option 2 Option 3


$ 200.00 $ 200.00 $ 200.00
120 $ 150.00 $ 170.00
$ 80.00 $ 50.00 $ 30.00
40 40 40
$ 3,200.00 $ 2,000.00 $ 1,200.00
40% 25% 15%
2000 800 0
$ 1,200.00 $ 1,200.00 $ 1,200.00

ks of loss and different returns associated with each option if sales


1 is because of its higher fixed costs ($2,000), which result in a
of safety (40 - 25 = 15 units) relative to the other options. The line

senting options 2 and 3.

Option 1 Option 2 Option 3


$ 200.00 $ 200.00 $ 200.00
120 $ 150.00 $ 170.00
$ 80.00 $ 50.00 $ 30.00
25 25 25
$ 2,000.00 $ 1,250.00 $ 750.00
40% 25% 15%
2000 800 0
$ - $ 450.00 $ 750.00
25.00 16.00 -

f the number of units sold drops to


operating loss, whereas options 2 and 3
rtical line from X = 20 units sold cuts the
auve area and cuts the options 2 and 3
n area.) The higher risk ofloss in option 1,
al benefits. Option 1 has the highest contribution
ts.

Option 1 Option 2 Option 3


$ 200.00 $ 200.00 $ 200.00
120 $ 150.00 $ 170.00
$ 80.00 $ 50.00 $ 30.00
20 20 20
$ 1,600.00 $ 1,000.00 $ 600.00
40% 25% 15%
2000 800 0
$ (400.00) $ 200.00 $ 600.00
25.00 16.00 -

tribution margin and, therefore,


ting income of $2,800, greater than the operating incomes for sales of 60 units
s when demand is low, both because of lower fixed costs and because she loses less

d 2. The choice among options 1, 2, and 3 will be:;influenced by her confidence in


tribution margin and, therefore,
ting income of $2,800, greater than the operating incomes for sales of 60 units
s when demand is low, both because of lower fixed costs and because she loses less

d 2. The choice among options 1, 2, and 3 will be:;influenced by her confidence in

Option 1 Option 2 Option 3


$ 200.00 $ 200.00 $ 200.00
120 $ 150.00 $ 170.00
$ 80.00 $ 50.00 $ 30.00
60 60 60
$ 4,800.00 $ 3,000.00 $ 1,800.00
40% 25% 15%
2000 800 0
$ 2,800.00 $ 2,200.00 $ 1,800.00
25.00 16.00 -

ting leverage. Operating leverage describes the effects that fixed costs have
ution margin.
ase under option 1, have high operating leverage. The line representing
to large increases in operating
omes, leading to a greater risk of operating losses. At any given level of sales,
ome.

40 units, she should be indifferent across the


ns. Each option results in operating income of

Option 1 Option 2 Option 3


$ 200.00 $ 200.00 $ 200.00
120 $ 150.00 $ 170.00
$ 80.00 $ 50.00 $ 30.00
40 40 40
$ 3,200.00 $ 2,000.00 $ 1,200.00
40% 25% 15%
2000 800 0
$ 1,200.00 $ 1,200.00 $ 1,200.00

$ 2.67 $ 1.67 $ 1.00

tribution margin will result in 2.67 times that percentage change in operating income

increase by 50% under each option. Operating income, however, will increase by
= 50% from $1,200 to $1,800 in option 3 (see Exhibit 3-5). The degree of operating
tribution margin will result in 2.67 times that percentage change in operating income

increase by 50% under each option. Operating income, however, will increase by
= 50% from $1,200 to $1,800 in option 3 (see Exhibit 3-5). The degree of operating

Option 1 Option 2 Option 3


$ 200.00 $ 200.00 $ 200.00
120 $ 150.00 $ 170.00
$ 80.00 $ 50.00 $ 30.00
60 60 60
$ 4,800.00 $ 3,000.00 $ 1,800.00 50%
40% 25% 15%
2000 800 0
$ 2,800.00 $ 2,200.00 $ 1,800.00
50% 50% 50%

133% 83% 50%


133% 83% 50%

$ 1.71 $ 1.36 $ 1.00

ribution margin will result in 2.67 times that percentage change in operating income

ncrease by 50% under each option. Operating income, however, will increase by
= 50% from $1,200 to $1,800 in option 3 (see Exhibit 3-5). The degree of operating

new sales
$ 12,000.00

on causes a 2.67 percent change in EBIT. In fact, any percentage


in EBIT that is 2.67 times as large. For example, a 3 percent
on causes a 2.67 percent change in EBIT. In fact, any percentage
in EBIT that is 2.67 times as large. For example, a 3 percent
cent increase in SALES causes a 10.68 percent increase in EBIT.

er?
n sales would have on operating profit. Sometimes, in response to
its sales policy and/ or cost structure. As a general rule, firms do
age. Since, in that situation, a small drop in sales may lead to an

of "potential risk" which becomes "active" only in the presence of


tself is not the source of the variability.

Specialty Cakes Inc. produces two types of cakes, a 2 lbs. round cake and a 3 lbs. heartshaped
cake. Total fixed costs for the firm are $94,000. Variable costs and sales data for
the two types of cakes are presented below.
2 lbs. 3 lbs.
Round Cake Heart-shape Cake
Selling price per unit $12 $20
Variable cost per unit $8 $15
Current sales (units) 10,000 15,000
If the product sales mix were to change to three heart-shaped cakes for each round cake,
the breakeven volume for each of these products would be
lbs. heartshaped
Budgeted production in units
Actual production in units
Budgeted Allocation base (ex.hours)
Actual allocation base (ex.hours)
the standard allocation base per unit
the Actual allocation base per unit
Total Actual OH
Total Actual VOH
Total Actual FOH
Total Budgeted OH
Total Budgeted VOH
Total Budgeted FOH
Total Applied OH
Applied VOH
Applied FOH
Budgeted VOH rate
Budgeted FOH rate

1)
Total Budgeted VOH = Budgeted Rate x Budgeted Allocation Base

Budgeted VOH Rate Budgeted Allocation Base

Total Actual VOH = Actual Rate x Actual Allocation Base

2)
Allocated VOH( the same in the flexible budget)
Budgeted Input Allowed for actual output X Budgeted VOH Rate

Budgeted Input Allowed for actual output = Budgeted input (allocation base) per unit X Actual units

3) Actual Inputs X budgeted Rate


$ -

4) Actual Inputs X Actual Rate .=Actual costs incurred


0

1)
Total Budgeted FOH = Budgeted Rate x Budgeted Allocation Base
BudgetedFOH Rate Budgeted Allocation Base

Total Actual FOH = Actual Rate x Actual Allocation Base

2)
Allocated FOH
Budgeted Input Allowed for actual output X Budgeted FOH Rate

Budgeted Input Allowed for actual output = Budgeted input (allocation base) per unit X Actual units

flexible budget same as static budget


0

4) Actual Inputs X Actual Rate .=Actual FOH costs incurred


0

Budgeted cost rate variable manufacturing overhead


Budgeted cost rate fixed manufacturing overhead
The budgeted input per unit is …..hours.
Budgeted production and sales for May 2004
Budgeted fixed manufacturing overhead costs

Actual results for May 2004


wings parts produced and sold
Actual allocation base (actual inputs)
Total VMOH
Total FMOH

Actual VOH Rate

VMOH

Actual costs Incurred

Actual Allocation Base X Actual Rate


- #DIV/0!
Actual costs Incurred
#DIV/0!

Total VOH variance (over/under apllied) #DIV/0!


FBV #DIV/0!
Efficiency V $ -
Spending V #DIV/0!

FMOH

Actual costs Incurred

Actual Allocation Base X Actual Rate


- $ -
Actual costs Incurred

$ -
$ -

Total FOH variance (over/under apllied) $ -


PVV $ -
FBV $ -
Spending V $ -

Budgeted production in units 200,000.00


Actual production in units 190,000.00
Budgeted Allocation base (ex.hours)
Actual allocation base (ex.hours)
the standard allocation base per unit
the Actual allocation base per unit
Total Actual OH
Total Actual VOH
Total Actual FOH $ 595,000.00
Total Budgeted OH
Total Budgeted VOH
Total Budgeted FOH $ 600,000.00
Total Applied OH
Applied VOH
as the PVV = the dif. between F
Applied FOH $ 570,000.00
Budgeted VOH rate
Budgeted FOH rate $ 3.00

FMOH

Actual costs Incurred

Actual Allocation Base X Actual Rate


- $ -
Actual costs Incurred

$ -
$ -

Total FOH variance (over/under apllied) $ 570,000.00


PVV $ (30,000.00)
FBV
Spending V $ -

Budgeted production in units 5,000.00


Actual production in units 4500
Budgeted Allocation base (ex.hours) 10,000.00
Actual allocation base (ex.hours) 9,600.00
the standard allocation base per unit
the Actual allocation base per unit
Total Actual OH
Total Actual VOH $ 39,360.00
Total Actual FOH
Total Budgeted OH $ 150,000.00
Total Budgeted VOH
Total Budgeted FOH
Total Applied OH
Applied VOH
Applied FOH
Budgeted VOH rate
Budgeted FOH rate
Budgeted OH rate $ 15.00

Budgeted cost rate variable manufacturing overhead


Budgeted cost rate fixed manufacturing overhead
The budgeted input per unit is …..hours.
Budgeted production and sales for May 2004
Budgeted fixed manufacturing overhead costs

Actual results for May 2004


wings parts produced and sold
Actual allocation base (actual inputs)
Total VMOH
Total FMOH

Actual VOH Rate

VMOH

Actual costs Incurred

Actual Allocation Base X Actual Rate


9,600.00 $ -
Actual costs Incurred

$ -

Total VOH variance (over/under apllied) $ 36,000.00


FBV $ 36,000.00
Efficiency V $ (2,400.00) 2400
Spending V $ 38,400.00
Budgeted production in units
Actual production in units
Budgeted Allocation base (ex.hours)
Actual allocation base (ex.hours) 430,000.00
the standard allocation base per unit
the Actual allocation base per unit
Total Actual OH $ 1,600,000.00
Total Actual VOH
Total Actual FOH
Total Budgeted OH
Total Budgeted VOH
Total Budgeted FOH $ 1,500,000.00
Total Applied OH $ 1,400,000.00
Applied VOH $ 200,000.00
Applied FOH $ 1,200,000.00
Budgeted VOH rate $ 0.50
Budgeted FOH rate $ 3.00

1)
Total Budgeted VOH = Budgeted Rate x Budgeted Allocation Base

Budgeted VOH Rate Budgeted Allocation Base

Total Actual VOH = Actual Rate x Actual Allocation Base

2)
Allocated VOH( the same in the flexible budget)
Budgeted Input Allowed for actual output X Budgeted VOH Rate
0.5
Budgeted Input Allowed for actual output = Budgeted input (allocation base) per unit X Actual units

3) Actual Inputs X budgeted Rate


430000 0.5 $ 215,000.00

4) Actual Inputs X Actual Rate .=Actual costs incurred


0
Budgeted cost rate variable manufacturing overhead
Budgeted cost rate fixed manufacturing overhead
The budgeted input per unit is …..hours.
Budgeted production and sales for May 2004
Budgeted fixed manufacturing overhead costs
Budgeted V manufacturing overhead costs
VOH per unit
Actual results for May 2004
wings parts produced and sold
Actual allocation base (actual inputs)
Total VMOH
Total FMOH

Actual VOH Rate

VMOH

Actual costs Incurred

Actual Allocation Base X Actual Rate


10,300.00 $ 57.86
Actual costs Incurred

$ 596,000.00

Total VOH variance (over/under apllied) $ (2,000.00)


FBV $ (2,000.00)
Efficiency V $ (24,000.00)
Spending V $ 22,000.00

FMOH
Actual costs Incurred

Actual Allocation Base X Actual Rate


- $ -
Actual costs Incurred

$ -
$ 410,000.00

Total FOH variance (over/under apllied) $ (410,000.00)


PVV $ (400,000.00)
FBV $ (10,000.00)
Spending V $ (10,000.00)

653] Gleim #: 9.5.137 -- Source: CMA 687 4-17


Selo Imports uses flexible budgeting for the control of costs. The company’s annual master budget includes $324,000 for
fixed production supervisory salaries at a volume of 180,000 units. Supervisory salaries are expected to be incurred
uniformly through the year. During September, 15,750 units were produced and production supervisory salaries incurred
were $28,000. A performance report for September should reflect a budget variance of

Budgeted production in units


Actual production in units
Budgeted Allocation base (ex.hours)
Actual allocation base (ex.hours)
the standard allocation base per unit
the Actual allocation base per unit
Total Actual OH
Total Actual VOH
Total Actual FOH
Total Budgeted OH
Total Budgeted VOH
Total Budgeted FOH
Total Applied OH
Applied VOH
Applied FOH
Budgeted VOH rate
Budgeted FOH rate

1)
Total Budgeted VOH = Budgeted Rate x Budgeted Allocation Base

Budgeted VOH Rate Budgeted Allocation Base

Total Actual VOH = Actual Rate x Actual Allocation Base

2)
Allocated VOH( the same in the flexible budget)
Budgeted Input Allowed for actual output X Budgeted VOH Rate

Budgeted Input Allowed for actual output = Budgeted input (allocation base) per unit X Actual units

3) Actual Inputs X budgeted Rate


$ -

4) Actual Inputs X Actual Rate .=Actual costs incurred


0

1)
Total Budgeted FOH = Budgeted Rate x Budgeted Allocation Base

BudgetedFOH Rate Budgeted Allocation Base

Total Actual FOH = Actual Rate x Actual Allocation Base

2)
Allocated FOH
Budgeted Input Allowed for actual output X Budgeted FOH Rate

Budgeted Input Allowed for actual output = Budgeted input (allocation base) per unit X Actual units

flexible budget same as static budget


0
4) Actual Inputs X Actual Rate .=Actual FOH costs incurred
0

Budgeted cost rate variable manufacturing overhead


Budgeted cost rate fixed manufacturing overhead
The budgeted input per unit is …..hours.
Budgeted production and sales for May 2004
Budgeted fixed manufacturing overhead costs

Actual results for May 2004


wings parts produced and sold
Actual allocation base (actual inputs)
Total VMOH
Total FMOH

Actual VOH Rate

VMOH

Actual costs Incurred

Actual Allocation Base X Actual Rate


440,000.00 $ 0.80
Actual costs Incurred

$ 352,000.00

Total VOH variance (over/under apllied) $ (55,000.00)


FBV $ (55,000.00)
Efficiency V $ (33,000.00)
Spending V $ (22,000.00)

FMOH
Actual costs Incurred

Actual Allocation Base X Actual Rate


- $ -
Actual costs Incurred

$ -
$ 575,000.00

Total FOH variance (over/under apllied) $ 19,000.00


PVV $ (6,000.00)
FBV $ 25,000.00
Spending V $ 25,000.00

Budgeted production in units 200,000.00


Actual production in units 190,000.00
Budgeted Allocation base (ex.hours)
Actual allocation base (ex.hours)
the standard allocation base per unit
the Actual allocation base per unit
Total Actual OH
Total Actual VOH
Total Actual FOH $ 595,000.00
Total Budgeted OH
Total Budgeted VOH
Total Budgeted FOH $ 600,000.00
Total Applied OH
Applied VOH
as the PVV = the dif. between F
Applied FOH $ 570,000.00
Budgeted VOH rate
Budgeted FOH rate $ 3.00

FMOH

Actual costs Incurred


Actual Allocation Base X Actual Rate
- $ -
Actual costs Incurred

$ -
$ -

Total FOH variance (over/under apllied) $ 570,000.00


PVV $ (30,000.00)
FBV
Spending V $ -

242. C
242. A company had a total labor variance of $15,000 favorable and a labor efficiency
variance of $18,000 unfavorable. The labor price variance was
a. $3,000 favorable.
b. $3,000 unfavorable.
c. $33,000 favorable.
d. $33,000 unfavorable

238. B
238. At the beginning of the year, Douglas Company prepared the following monthly budget
for direct materials.
Units produced and sold 10,000 15,000 10000 15000
Direct material $15,000 $22,500 $ 15,000 $ 22,500
1.5 1.5
12000
At the end of the month, the company's records showed that 12,000 units were produced
and sold and $20,000 was spent for direct materials. The variance for direct materials is
a. $2,000 favorable. 18000 20000
b. $2,000 unfavorable.
c. $5,000 favorable.
d. $5,000 unfavorable.

237. D
237. Lee Manufacturing uses a standard cost system with overhead applied based on direct
labor hours. The manufacturing budget for the production of 5,000 units for the month of
May included the following information.
Direct labor (10,000 hours at $15 per hour) $150,000 10000 15
Variable overhead 30,000
Fixed overhead 80,000
During May, 6,000 units were produced and the direct labor efficiency variance was
$1,500 unfavorable. Based on this information, the actual number of direct labor hours
used in May was
a. 9,900 hours.
b. 10,100 hours.
c. 11,900 hours. R DA
d. 12,100 hours E DS
SQA

236. C
236. A company isolates its raw material price variance in order to provide the earliest
possible information to the manager responsible for the variance. The budgeted amount
of material usage for the year was computed as follows.
150,000 units of finished goods x 3 pounds/unit x $2.00/pound = $900,000.
150000 3 2 $ 900,000
Actual results for the year were the following.
Finished goods produced 160,000 units 160,000
Raw materials purchased 500,000 pounds 500,000
Raw materials used 490,000 pounds 490,000
Cost per pound $2.02 $ 2.02
The raw material price variance for the year was
a. $9,600 unfavorable.
b. $9,800 unfavorable.
c. $10,000 unfavorable.
d. $20,000 unfavorable

(10,000.00)

235. B
235. Christopher Akers is the chief executive officer of SBL Inc., a masonry contractor. The
financial statements have just arrived showing a $3,000 loss on the new stadium job that
was budgeted to show a $6,000 profit. Actual and budget information relating to the
materials for the job are as follows.
Actual Budget
Bricks - number of bundles 3,000 2,850 3000 2850
Bricks - cost per bundle $7.90 $8.00 $ 7.90 $ 8.00
Which one of the following is a correct statement regarding the stadium job for SBL?
a. The price variance was favorable by $285.
b. The price variance was favorable by $300.
c. The efficiency variance was unfavorable by $1,185.
d. The flexible budget variance was unfavorable by $900.

234. D
234. Frisco Company recently purchased 108,000 units of raw material for $583,200. Three
units of raw materials are budgeted for use in each finished good manufactured, with the
raw material standard set at $16.50 for each completed product. Frisco manufactured
32,700 finished units during the period just ended and used 99,200 units of raw material.
If management is concerned about the timely reporting of variances in an effort to
improve cost control and bottom-line performance, the materials purchase price variance
should be reported as
a. $6,050 unfavorable.
b. $9,920 favorable.
c. $10,800 unfavorable.
d. $10,800 favorable

233. D
233. MinnOil performs oil changes and other minor maintenance services (e.g., tire pressure
checks) for cars. The company advertises that all services are completed within 15
minutes for each service. On a recent Saturday, 160 cars were serviced resulting in the
following labor variances: rate, $19 unfavorable; efficiency, $14 favorable. If MinnOil’s
standard labor rate is $7 per hour, determine the actual wage rate per hour and the actual
hours worked.
Wage Rate Hours Worked
a. $6.55 42.00.
b. $6.67 42.71.
c. $7.45 42.00.
d. $7.50 38.00.
.=Total Budgeted VOH
0

.= Total actual VOH


0

0
.=Total Budgeted FOH
0

.= Total actual FOH


0

wing parts

units

#DIV/0!

Flexible Budget :
Budgeted Input Allowed
Actual Inputs X for Actual Output X
Budgeted Rate Budgeted Rate
Actual Inputs X Budgeted Rate Budgeted Input Allowed for Actual Outp
- $ - -
-
Budgeted Rate
$ - $ -
$ -

Flexible Budget :
Same Budgeted Same Budgeted
Lump Sum Lump Sum
As in Static Budget regardless of As in Static Budget regardless of
Output Level Output Level
Actual Inputs X Budgeted Rate Budgeted Input Allowed for Actual Outp
- $ - -
-
Budgeted Rate
$ - $ -
$ - $ -
$ -

244. D d. $30,000 unfavorable.


244. Cordell Company uses a standard cost system. On January 1 of the current year, Cordell
budgeted fixed manufacturing overhead cost of $600,000 and production at 200,000
units. During the year, the firm produced 190,000 units and incurred fixed manufacturing
overhead of $595,000. The production volume variance for the year was
s the PVV = the dif. between FB(static) and it is give, and the allocated FOH that = budgeted rate x
actual units

Flexible Budget :
Same Budgeted Same Budgeted
Lump Sum Lump Sum
As in Static Budget regardless of As in Static Budget regardless of
Output Level Output Level
Actual Inputs X Budgeted Rate Budgeted Input Allowed for Actual Outp
- $ - -
-
Budgeted Rate
$ - $ -
$ -
$ 600,000.00

243. B b. $4.00.

243. Lee manufacturing uses a standard cost system with overhead applied based on direct
labor hours. The manufacturing budget for the production of 5,000 units for the month of
June included 10,000 hours of direct labor at $15 per hour, $150,000. During June, 4,500
units were produced, using 9,600 direct labor hours, incurring $39,360 of variable
overhead, and showing a variable overhead efficiency variance of $2,400 unfavorable.
The standard variable overhead rate per direct labor hour was
$ 4.00

2.00
5,000.00 wing parts

4,500.00 units
9,600.00

$ -

Since the eff.Var.= the diff between , and it is equal (2,400), the Q will be solved through a formula.

Flexible Budget :
Budgeted Input Allowed
Actual Inputs X for Actual Output X
Budgeted Rate Budgeted Rate
Actual Inputs X Budgeted Rate Budgeted Input Allowed for Actual Outp
9,600.00 $ 4.00 2.00
9,000.00
Budgeted Rate
$ 38,400.00 $ 4.00
$ 36,000.00

9000x -9600x=-2,400 (x is the budgeted rate)

U
x= (4.00)
246. B b. $115,000 favorable.
Sp
Actual VOH Actua inpxbudgeted rate
215000
sp
Actual FOH static
$ 1,500,000.00

$ 1,600,000.00 $ 1,715,000.00 $ 115,000.00

.=Total Budgeted VOH


0

.= Total actual VOH


0

0
$ 60.00 60 247. D
$ 40.00 40
247. The JoyT Company manufactures Maxi Dolls for sale i
wing parts year, JoyT estimated variable factory overhead of $600,000
$ 400,000.00 $400,000. JoyT uses a standard costing system, and factory
600,000.00 produced on the basis of standard direct labor hours. The
60.00 budgeted for this year was 10,000 direct labor hours, and J
labor hours.
units Based on the output accomplished during this year, 9,900
10,300.00 should have been used. Actual variable factory overhead w
$ 596,000.00 factory overhead was $410,000 for the year. Based on this
$ 410,000.00 overhead spending variance for JoyT for this year was

$ 57.86

Flexible Budget :
Budgeted Input Allowed
Actual Inputs X for Actual Output X
Budgeted Rate Budgeted Rate
Actual Inputs X Budgeted Rate Budgeted Input Allowed for Actual Outp
10,300.00 $ 60.00 -
-
Budgeted Rate
$ 618,000.00 $ 60.00
$ 594,000.00

Flexible Budget :
Same Budgeted Same Budgeted
Lump Sum Lump Sum
As in Static Budget regardless of As in Static Budget regardless of
Output Level Output Level
Actual Inputs X Budgeted Rate Budgeted Input Allowed for Actual Outp
- $ - -
-
Budgeted Rate
$ - $ -
$ 400,000.00 $ -
$ 400,000.00

C. $1,000 Unfavorable.
Answer (C) is correct. The budget (spending) variance for fi
udget includes $324,000 for $324,000 cost of supervisory salaries is fixed and is incurre
xpected to be incurred difference between actual costs of $28,000 and the budge
upervisory salaries incurred
.=Total Budgeted VOH
0

.= Total actual VOH


0

.=Total Budgeted FOH


0

.= Total actual FOH


0

0
$ 0.75 [Fact Pattern #73]
$ 3.00 Franklin Glass Works’ production budget for the year ende
2.00 standard hours of labor for completion. Total overhead wa
200,000.00 units estimated to be $3.00 per unit. Both fixed and variable ove
$ 600,000.00 The actual data for the year ended November 30 are prese
Actual production in units 198,000
Actual direct labor hours 440,000
Actual variable overhead $352,000
198,000.00 units Actual fixed overhead $575,000
440,000.00
$ 352,000.00 voh 300000
$ 575,000.00 unit x allo 2
unit x allo 400,000.00
$ 0.80 0.75

Flexible Budget :
Budgeted Input Allowed
Actual Inputs X for Actual Output X
Budgeted Rate Budgeted Rate
Actual Inputs X Budgeted Rate Budgeted Input Allowed for Actual Outp
440,000.00 $ 0.75 2.00
396,000.00
Budgeted Rate
$ 330,000.00 $ 0.75
$ 297,000.00

Flexible Budget :
Same Budgeted Same Budgeted
Lump Sum Lump Sum
As in Static Budget regardless of As in Static Budget regardless of
Output Level Output Level
Actual Inputs X Budgeted Rate Budgeted Input Allowed for Actual Outp
- $ - -
-
Budgeted Rate
$ - $ -
$ 600,000.00 $ -
$ 600,000.00

244. D d. $30,000 unfavorable.


244. Cordell Company uses a standard cost system. On January 1 of the current year, Cordell
budgeted fixed manufacturing overhead cost of $600,000 and production at 200,000
units. During the year, the firm produced 190,000 units and incurred fixed manufacturing
overhead of $595,000. The production volume variance for the year was

s the PVV = the dif. between FB(static) and it is give, and the allocated FOH that = budgeted rate x
actual units

Flexible Budget :
Same Budgeted Same Budgeted
Lump Sum Lump Sum
As in Static Budget regardless of As in Static Budget regardless of
Output Level Output Level
Actual Inputs X Budgeted Rate Budgeted Input Allowed for Actual Outp
- $ - -
-
Budgeted Rate
$ - $ -
$ -
$ 600,000.00

173. D
$ 15,000.00 $ (18,000.00) $ 33,000.00 174. A
175. C
176. C
177. B
178. A
179. D
180. C
181. B
182. D
183. A
184. B
185. C
186. D
187. A
188. B

189. D
190. C
$ (2,000) 191. B
192. A
193. D
194. D
195. A
196. C
197. A
198. B
199. C
200. D
5000 201. C
202. D
$ 150,000 2 203. B
30000 204. D
80000 205. C
6000 206. D
1500 207. C
208. B
209. D
210. B
211. B
1500 12000 $ 180,000 212. A
$ 181,500 213. D
12,100 214. D
215. D
216. C
217. C
218. C
219. B
220. B
221. C
222. D
223. B

224. B
225. A
226. D
227. B
228. C
229. C
230. B
231. A
232. A
233. D

239. A
240. A
241. B
3000 242. C
243. B
244. D
245. B
246. B
$ (300.00) 247. D
248. D
249. D
250. D
251. D
252. D
253. D
254. D
255. B
256. B
257. B
108000 583200 5.4 258. B
3 259. B
16.5 5.5 10,800.00 260. B
32700 99200 261. C
262. C
263. D
264. C
265. C
266. A
267. D
268. D
269. B
270. D
271. D
272. B
273. B
274. A
15 275. A
160 276. D
277. A
278. B
279. C
280. C
281. B
282. B
283. D
284. A
285. D
286. B
287. B
288. C
289. B
290. C
291. C
292. B
293. B
294. B
295. C
296. A
297. A
298. C
299. B
300. A
301. D
302. C
303. B
304. B
305. A
306. C
307. A
308. C
309. B
310. D
311. A
312. B
313. D
314. A
315. B
316. D
317. B
318. B
319. A
320. D
321. D
322. C
323. A
324. B
325. C
326. A
327. A
328. A
329. A
330. A
331. B
332. C
333. C
334. B
335. C
336. B
337. C
338. C
339. C
340. C
341. D
342. A
343. B
344. A
345. C
346. A
347. D
348. D
349. D
350. C
351. B
352. D
353. A
354. A
355. C
356. C
Allocated:
Budgeted Input Allowed
for Actual Output X
Budgeted Rate
Allowed for Actual Output Budgeted Input Allowed for Actual Output
- - -
-
Budgeted Rate
$ -
$ -

Allocated:
Budgeted Input Allowed
for Actual Output X
Budgeted Rate
Allowed for Actual Output Budgeted Input Allowed for Actual Output
- - -
-
Budgeted Rate
$ -
$ -
Allocated:
Budgeted Input Allowed
for Actual Output X
Budgeted Rate
Allowed for Actual Output Budgeted Input Allowed for Actual Output
- - -
-
Budgeted Rate
$ -
$ 570,000.00
ved through a formula...

Allocated:
Budgeted Input Allowed
for Actual Output X
Budgeted Rate
Allowed for Actual Output Budgeted Input Allowed for Actual Output
4,500.00 2.00 4,500.00
9,000.00
Budgeted Rate
$ 4.00
$ 36,000.00
246. Harper Company’s performance report indicated the following information for the past
month.
Actual total overhead $1,600,000
Budgeted fixed overhead 1,500,000
5,000 favorable. Applied fixed overhead at $3 per labor hour 1,200,000
Applied variable overhead at $.50 per labor hour 200,000
Actual labor hours 430,000
Harper’s total overhead spending variance for the month was
d. $22,000 favorable.

res Maxi Dolls for sale in toy stores. In planning for this
y overhead of $600,000 and fixed factory overhead of
ting system, and factory overhead is allocated to units
direct labor hours. The denominator level of activity
direct labor hours, and JoyT used 10,300 actual direct 10000

during this year, 9,900 standard direct labor hours 9900


able factory overhead was $596,000, and actual fixed
the year. Based on this information, the variable
yT for this year was

Allocated:
Budgeted Input Allowed
for Actual Output X
Budgeted Rate
Allowed for Actual Output Budgeted Input Allowed for Actual Output
- - -
-
Budgeted Rate
$ 60.00
$ 594,000.00

Allocated:
Budgeted Input Allowed
for Actual Output X
Budgeted Rate
Allowed for Actual Output Budgeted Input Allowed for Actual Output
- - -
-
Budgeted Rate
$ 40.00
$ -

spending) variance for fixed O/H equals actual minus budgeted fixed O/H. The
es is fixed and is incurred at $27,000 per month. Thus, the variance is the
$28,000 and the budgeted costs of $27,000, or $1,000 unfavorable.
udget for the year ended November 30 was based on 200,000 units. Each unit requires two
tion. Total overhead was budgeted at $900,000 for the year, and the fixed overhead rate was
h fixed and variable overhead are assigned to the product on the basis of direct labor hours.
November 30 are presented as follows.

Allocated:
Budgeted Input Allowed
for Actual Output X
Budgeted Rate
Allowed for Actual Output Budgeted Input Allowed for Actual Output
198,000.00 2.00 198,000.00
396,000.00
Budgeted Rate
$ 0.75
$ 297,000.00

Allocated:
Budgeted Input Allowed
for Actual Output X
Budgeted Rate
Allowed for Actual Output Budgeted Input Allowed for Actual Output
- 3.00 198,000.00
594,000.00
Budgeted Rate
$ 3.00
$ 594,000.00

Allocated:
Budgeted Input Allowed
for Actual Output X
Budgeted Rate
Allowed for Actual Output Budgeted Input Allowed for Actual Output
- - -
-
Budgeted Rate
$ -
$ 570,000.00
mation for the past
Maria Lopez is the newly appointed president of Laser Products. She is examining the May
2004 results for the Aerospace Products Division. This division manufactures wing parts for
satellites. Lopez's current concern is with manufacturing overhead costs at the Aerospace
Products Division. Both variable manufacturing overhead costs and fixed manufacturing
overhead costs are allocated to the wing parts on the basis of laser-cutting-hours. The
Budgeted cost rates are variable manufacturing overhead of $200 per hour and fixed
manufacturing overhead of $240 per hour.

The budgeted laser-cutting time per wing part is 1.5 hours.

Budgeted production and sales for May 2004 is 5,000 wing parts. Budgeted fixed
manufacturing overhead costs for May 2004 is $1,800,000.

The Budgeted cost rates are variable manufacturing overhead of $200 per hour and fixed manufact

Budgeted cost rate variable manufacturing overhead $ 200.00


Budgeted cost rate fixed manufacturing overhead $ 240.00
The budgeted laser-cutting time per wing part is 1.5 hours. 1.50
Budgeted production and sales for May 2004 5,000.00
Budgeted fixed manufacturing overhead costs $ 1,800,000.00

Actual results for May 2004


wings parts produced and sold 4,800.00
Laser cutting hours used 8,400.00
VMOH $ 1,478,400.00
FMOH $ 1,832,200.00

Actual VOH Rate $ 176.00

VMOH

Actual costs Incurred Actual Inputs X


Budgeted Rate
Actual Allocation Base X Actual Rate Actual Inputs X Budgeted Rate
8,400.00 $ 176.00 8,400.00
Actual costs Incurred
$ 1,478,400.00 $ 1,680,000.00

Total VOH variance (over/under apllied) $ (38,400.00)


FBV $ (38,400.00)
Efficiency V $ (240,000.00)
Spending V $ 201,600.00

FMOH Same Budgeted


Lump Sum
Actual costs Incurred As in Static Budget regardless of
Output Level
Actual Allocation Base X Actual Rate Actual Inputs X Budgeted Rate
- $ - -
Actual costs Incurred

$ - $ -
$ 1,832,200.00 $ 1,800,000.00

Total FOH variance (over/under apllied) $ (104,200.00)


PVV $ (72,000.00)
FBV $ (32,200.00)
Spending V $ (32,200.00)
mining the May
s wing parts for
the Aerospace
anufacturing
hours. The
and fixed

fixed

our and fixed manufacturing overhead of $240 per hour.

wing parts

units

Flexible Budget : Allocated:


Budgeted Input Allowed Budgeted Input Allowed
for Actual Output X for Actual Output X
Budgeted Rate Budgeted Rate
dgeted Rate Budgeted Input Allowed for Actual Output Budgeted Input Allowed
$ 200.00 1.50 4,800.00 1.50
7,200.00
Budgeted Rate
$ 200.00
$ 1,440,000.00

Flexible Budget :
Same Budgeted Allocated:
Lump Sum Budgeted Input Allowed
regardless of As in Static Budget regardless of for Actual Output X
Output Level Budgeted Rate
dgeted Rate Budgeted Input Allowed for Actual Output Budgeted Input Allowed
$ - - - 1.50
-
Budgeted Rate
$ -
$ -
$ 1,800,000.00
Input Allowed

Budgeted Input Allowed for Actual Output


4,800.00
7,200.00
Budgeted Rate
$ 200.00
$ 1,440,000.00

Input Allowed

Budgeted Input Allowed for Actual Output


4,800.00
7,200.00
Budgeted Rate
$ 240.00
$ 1,728,000.00
150. The Furniture Company currently has three divisions: Maple, Oak, and Cherry. The oak
furniture line does not seem to be doing well and the president of the company is
considering dropping this line.
If it is dropped, the revenues associated with the Oak
Division will be lost and the related variable costs saved. Also, 50% of the fixed costs
allocated to the oak furniture line would be eliminated. The income statements, by
divisions, are as follows.

Maple
Sales $55,000 $85,000 $100,000 $55,000.000
Variable Costs 40,000 72,000 82,000 $40,000.000
Contribution Margin 15,000 13,000 18,000 $15,000.000
Fixed costs 10,000 14,000 10,200 $10,000.000
Operating profit (loss) $ 5,000 $(1,000) $ 7,800 $5,000.000

Which one of the following options should be recommended to the president of the
company?

Maple
Sales $55,000 $85,000 $100,000 $55,000.000
Variable Costs 40,000 72,000 82,000 $40,000.000
Contribution Margin 15,000 13,000 18,000 $15,000.000
Fixed costs 10,000 14,000 10,200 $10,000.000
Operating profit (loss) $ 5,000 $(1,000) $ 7,800 $5,000.000

b. Continue operating the Oak Division as discontinuance would result in a $6,000


decline in operating profits.

149. Raymund Inc. currently sells its only product to Mall-Stores. Raymund has received a
one-time-only order for 2,000 units from another buyer. Sale of the special order items
will not require any additional selling effort. Raymund has a manufacturing capacity to
produce 7,000 units. Raymund has an effective income tax rate of 40%. Raymund’s
Income Statement, before consideration of the one-time-only order, is as follows.

Sales (5,000 units at $20 per unit) $100,000 5,000.00 $20.0


Variable manufacturing costs $50,000 $10.0 $50,000.0
Variable selling costs 15,000 65,000 $3.0 $15,000.0
Contribution margin 35,000

Fixed manufacturing costs 16,000 $16,000.0


Fixed selling costs 4,000 20,000 $4,000.0
Operating income $ 15,000
Income taxes
Net income

In negotiating a price for the special order, Raymund should set the minimum per unit
selling price at $10.0

148. Lark Industries accepted a contract to provide 30,000 units of Product A and 20,000 units
of Product B. Lark’s staff developed the following information with regard to meeting
this contract.
Product A Product B
30000 20000
Selling Price $75 $125 $75.0 $125.0
Variable costs $30 $48 $30.0 $48.0
Fixed overhead $1,600,000
Machine hours required 3 5 3.00 5.00
Machine hours available 160,000
Cost if outsourced $45 $60 $45.0 $60.0

Lark’s operations manager has identified the following alternatives. Which alternative
should be recommended to Lark’s management?

CM $45.0 $77.0
CM per constraint $ 15.00 $ 15.40

loss if outsourced -$15.0 -$12.0


loss per constraint if outsourced $ -5.00 $ -2.40

147. Aril Industries is a multiproduct company that currently manufactures 30,000 units of
Part 730 each month for use in production. The facilities now being used to produce Part
730 have fixed monthly overhead costs of $150,000, and a theoretical capacity to produce
60,000 units per month. If Aril were to buy Part 730 from an outside supplier, the
facilities would be idle and 40% of fixed costs would continue to be incurred. There are
no alternative uses for the facilities. The variable production costs of Part 730 are $11
per unit. Fixed overhead is allocated based on planned production levels.

If Aril Industries continues to use 30,000 units of Part 730 each month, it would realize a
net benefit by purchasing Part 730 from an outside supplier only if the supplier’s unit
price is less than $ 14.00

146. Current business segment operations for Whitman, a mass retailer, are presented below.

Merchandise Automotive
Sales $ 500,000.00 $ 400,000.00
Variable costs $ 300,000.00 $ 200,000.00
CM $ 200,000.00 $ 200,000.00
Fixed costs $ 100,000.00 $ 100,000.00
Operating income (loss) $ 100,000.00 $ 100,000.00

CM% 40% 50%

Management is contemplating the discontinuance of the Restaurant segment since “it is


losing money.” If this segment is discontinued, $30,000 of its fixed costs will be
eliminated. In addition, Merchandise and Automotive sales will decrease 5% from their
current levels. When considering the decision, Whitman’s controller advised that one of
the financial aspects Whitman should review is contribution margin. Which one of the
following options reflects the current contribution margin ratios for each of Whitman’s
business segments?

Merchandise Automotive
Sales $ 475,000.00 $ 380,000.00
Variable costs $ 285,000.00 $ 190,000.00
CM $ 190,000.00 $ 190,000.00
Fixed costs $ 100,000.00 $ 100,000.00
Operating income (loss) $ 90,000.00 $ 90,000.00

CM% 40% 50%

144. Oakes Inc. manufactured 40,000 gallons of Mononate and 60,000 gallons of Beracyl in a
joint production process, incurring $250,000 of joint costs. Oakes allocates joint costs
based on the physical volume of each product produced. Mononate and Beracyl can each
be sold at the split-off point in a semifinished state or, alternatively, processed further.
Additional data about the two products are as follows.

40,000.00
Mononate
Sales price per gallon at split-off $ 7.00
Sales price per gallon if processed further $ 10.00
Variable production costs if processed further $ 125,000.00

An assistant in the company’s cost accounting department was overheard saying “....that
when both joint and separable costs are considered, the firm has no business processing
either product beyond the split-off point. The extra revenue is simply not worth the
effort.” Which of the following strategies should be recommended for Oakes?

Variable production costs if processed further $ 3.13

Incremental Revenue per unit if processed further $ 3.00


Incremental Cost per unit if processed further $ 3.13
Incremental profit (Loss) $ (0.13)

Incremental Revenue if processed further 120,000.00


Incremental Cost if processed further 125,000.00
Incremental profit (Loss) (5,000.00)

143. Jack Blaze wants to rent store space in a new shopping mall for the three month holiday
shopping season. Blaze believes he has a new product available which has the potential
for good sales. The product can be obtained on consignment at the cost of $20 per unit
and he expects to sell the item for $100 per unit. Due to other business ventures, Blaze’s
risk tolerance is low. He recognizes that, as the product is entirely new, there is an
element of risk. The mall management has offered Blaze three rental options: (1) a fixed
fee of $8,000 per month, (2) a fixed fee of $3,990 per month plus 10% of Blaze’s
revenue, or (3) 30% of Blaze’s revenues. Which one of the following actions would you
recommend to Jack Blaze?

(1) a fixed fee of $8,000 per month, (2) a fixed fee of $3,990 per mon
8000

142. Eagle Brand Inc. produces two products. Data regarding these products are presented
below.

Product X Product Y
Selling price per unit $100 $130 $ 100.00 $ 130.00
Variable costs per unit $80 $100 $ 80.00 $ 100.00
Raw materials used per unit 4 lbs. 10 lbs. 4.00 10.00

Eagle Brand has 1,000 lbs. of raw materials which can be used to produce Products X and
Y.

Which one of the alternatives below should Eagle Brand accept in order to maximize
contribution margin?

CM 20 30
CM per constraint $ 5.00 $ 3.00
Units that can be produced by the constraint 250.00 100.00

12 20
8 15 Specialty Cakes Inc. produces two
RC HS cake. Total fixed costs for the firm
CM $4.0 $5.0 the two types of cakes are presen
units 10000 15000 25000 2 lbs. 3 lbs.
40% 60% Round Cake Heart-shape Cake
$1.6 $3.0 $4.6 Selling price per unit $12 $20
Variable cost per unit $8 $15
fc $94,000.0 20,434.78 Current sales (units) 10,000 15,00

8,173.91 12,260.87 20,434.78

$12.0 $20.0
$8.0 $15.0 If the product sales mix were to change to three he
RC HS the breakeven volume for each of these products w
CM $4.0 $5.0
units 6250 18750 25000
25% 75%
$1.0 $3.8 $4.8

fc $94,000.0 19,789.47

4,947.37 14,842.11 19,789.47

138. Zipper Company invested $300,000 in a new machine to produce cones for the textile
industry. Zipper’s variable costs are 30% of the selling price, and its fixed costs are
$600,000. Zipper has an effective income tax rate of 40%. The amount of sales required
to earn an 8% after-tax return on its investment would be

Targeted ROI $ 24,000.00


Before Tax Targeted ROI $ 40,000.00
Numerator to be used $ 640,000.00
CM ratio 70%
$ 914,285.71

137. Bargain Press is considering publishing a new textbook. The publisher has developed the
following cost data related to a production run of 6,000, the minimum possible
production run. Bargain Press will sell the textbook for $45 per copy. How many
textbooks must Bargain Press sell in order to generate operating earnings (earnings before
interest and taxes) of 20% on sales? (Round your answer up to the nearest whole
textbook.)
Estimated cost per unit
Development (reviews, class testing, editing) $35,000 $ 35,000.00
Typesetting 18,500 $ 18,500.00
Depreciation on Equipment 9,320 $ 9,320.00
General and Administrative 7,500 $ 7,500.00
Miscellaneous Fixed Costs 4,400 $ 4,400.00
Printing and Binding 30,000 $ 30,000.00 $ 5.00
Sales staff commissions (2% of selling price) 5,400 $ 5,400.00 $ 0.90
Bookstore commissions (25% of selling price) 67,500 $ 67,500.00 $ 11.25
Author’s Royalties (10% of selling price) 27,000 $ 27,000.00 $ 4.50
$ 21.65
Total costs at production of 6,000 copies $ 204,620.00

Feedback: The correct answer is: 5,207 copies. ,


The number of copies required can be found by solving the following equation, where x equals the ,
number of copies: ,
Total sales – total variable costs – fixed costs + typesetting + depreciation expense + general and ,
administrative expenses + miscellaneous fixed costs = (return %)(total sales) ,
Total sales = $45x ,
Total variable cost = (variable cost per unit, or VCU)(x) = VCU(x) ,
Fixed costs = development + typesetting + depreciation + general and administrative + miscellaneous ,
fixed costs ,
Fixed costs = $35,000 + $18,500 + $9,320 + $7,500 + $4,400 = $74,720 ,
VCU = (printing and binding + sales staff commissions + bookstore commissions + royalties) / ,
(number of copies) ,
VCU = ($30,000 + $5,400 + $67,500 + $27,000) / (6,000 copies) = $129,900 / 6,000 ,
VCU = $21.65 per copy ,
$45x – $21.65(x) – $74,720 = (0.2)($45x) ,
$45x – $21.65(x) - $74,720 = $9x ,
$36x – $21.65(x) = $74,720 ,
$14.35x = $74,720 ,
x = 5,206.97, which rounds to 5,207 copies. ,

131. Wilkinson Company sells its single product for $30 per unit. The contribution margin
ratio is 45% and Wilkinson has fixed costs of $10,000 per month. If 3,000 units are sold
in the current month, Wilkinson’s income would be

CM $ 13.50
Total CM $ 40,500.00
FC $ 10,000.00
OI $ 30,500.00
150. B b. Continue operating the Oak Division as discontinuanc
decline in operating profits.

50%

Oak Cherry Total


$85,000.000 $100,000.000 $240,000.000
$72,000.000 $82,000.000 $194,000.000
$13,000.000 $18,000.000 $46,000.000
$14,000.000 $10,200.000 $34,200.000
-$1,000.000 $7,800.000 $11,800.000

Oak Cherry Total


$100,000.000 $155,000.000
$82,000.000 $122,000.000
$0.000 $18,000.000 $33,000.000
$7,000.000 $10,200.000 $27,200.000
-$7,000.000 $7,800.000 $5,800.000

-$6,000.000

149. A a. $10.
2000

40%

$100,000.0

$65,000.0
$35,000.0
$20,000.0
$15,000.0
$6,000.0
$9,000.0

148. A a. Make 30,000 units of Product A, utilize the remaining


B, and outsource the remainder.

Product B Total

$1,600,000.0

160,000.00

147. D d. $14.00.
30,000.00

$ 150,000.00
60,000.00
60% $ 90,000.00
$ 11.00
$ 3.00
146. D d. 40% 50% 30%
Retailing Automotive Restaurant

Restaurant Total
$ 100,000.00 $ 1,000,000.00
$ 70,000.00 $ 570,000.00
$ 30,000.00 $ 430,000.00
$ 50,000.00 $ 250,000.00
$ -20,000.00 $ 180,000.00

30% 43%

95%

Restaurant Total
$ 855,000.00
$ 475,000.00
$ - $ 380,000.00 What will Whitman’s total contribution margin be if the
$ 20,000.00 $ 250,000.00 segment is discontinued?
$ -20,000.00 $ 130,000.00 145. D d. $380,000.

#DIV/0! 44%

Mononate Beracyl
144. B b. Sell at split-off Process further.
$ 250,000.00

60,000.00 100,000.00
Beracyl
$ 15.00
$ 18.00
$ 115,000.00

$ 1.92

$ 3.00
$ 1.92
$ 1.08

180,000.00
115,000.00
65,000.00

143. D d. Choose the third option no matter what Blaze expects

20
100

5700

(2) a fixed fee of $3,990 per month plus 10% of Blaze’s revenue (3) 30% of Blaze’s revenues.
4560 1710

142. B b. 250 units of product X.


1000

Specialty Cakes Inc. produces two types of cakes, a 2 lbs. round cake and a 3 lbs. heartshaped
cake. Total fixed costs for the firm are $94,000. Variable costs and sales data for
the two types of cakes are presented below.
2 lbs. 3 lbs.
Round Cake Heart-shape Cake
Selling price per unit $12 $20
Variable cost per unit $8 $15
Current sales (units) 10,000 15,000

les mix were to change to three heart-shaped cakes for each round cake,
olume for each of these products would be

138. B b. $914,286.
$ 300,000.00
30%
$ 600,000.00
40%
8%

137. B b. 5,207 copies.

6000
$ 45.00
20%

$ 74,720.00

equals the ,

miscellaneous ,
30 131. A a. $30,500
0.45 3000
10000
e Oak Division as discontinuance would result in a $6,000
Product A, utilize the remaining capacity to make Product
al contribution margin be if the Restaurant

Process further.
on no matter what Blaze expects the revenues to be.
169. Fennel Products is using cost-based pricing to determine the selling price for its new
product based on the following information.
Annual volume 25,000 units 25,000.00
Fixed costs $700,000 per year $700,000.00
Variable costs $200 per unit $200.00 $5,000,000.00
Plant investment $3,000,000 $3,000,000.00
Working capital $1,000,000 $1,000,000.00
Effective tax rate 40% 40%

The target price that Fennell needs to set for the new product to achieve a 15% after-tax
return on investment (ROI) would be

(ROI) $600,000.00
investment $4,000,000.00

(ROI) before tax $705,882.35

$ 228.00 $ 238.00 $ 258.00 $ 268.00


Sales Rev 5,700,000.00 5,950,000.00 6,450,000.00 6,700,000.00
VC 5000000 5000000 5000000 5000000
CM 700,000.00 950,000.00 1,450,000.00 1,700,000.00
FC $700,000.00 $700,000.00 $700,000.00 $700,000.00
OI $0.00 $250,000.00 $750,000.00 $1,000,000.00
Tax 0 100000 300000 400000
NI $0.00 $150,000.00 $450,000.00 $600,000.00
0.00% 3.75% 11.25% 15.00%

168. Almelo Manpower Inc. provides contracted bookkeeping services. Almelo has annual
fixed costs of $100,000 and variable costs of $6 per hour. This year the company
budgeted 50,000 hours of bookkeeping services. Almelo prices its services at full cost
and uses a cost-plus pricing approach. The company developed a billing price of $9 per
hour. The company’s mark-up level would be

fixed costs $100,000.00


fixed costs per unit $2.00
VC per hour $6.00
budgeted 50,000 hours 50,000.00
billing price $9.00

Full cost $8.00


mark-up $1.00 12.50%
165. Basic Computer Company (BCC) sells its microcomputers using bid pricing. It develops
its bids on a full cost basis. Full cost includes estimated material, labor, variable
overheads, fixed manufacturing overheads, and reasonable incremental computer
assembly administrative costs, plus a 10% return on full cost. BCC believes bids in
excess of $1,050 per computer are not likely to be considered.

BCC’s current cost structure, based on its normal production levels, is $500 for materials
per computer and $20 per labor hour. Assembly and testing of each computer requires 17
labor hours. BCC expects to incur variable manufacturing overhead of $2 per labor hour,
fixed manufacturing overhead of $3 per labor hour, and incremental administrative costs
of $8 per computer assembled.

BCC has received a request from a school board for 200 computers. Using the full-cost
criteria and desired level of return, which one of the following prices should be
recommended to BCC’s management for bidding purposes?

DM $500.00 per computer


DL $20.00 per labor hour 3400

Assembly and testing of each computer requires 17 hours

VOH $2.00 per labor hour


FOH $3.00 per labor hour
incremental administrative costs $8.00 per computer

Full cost
10% return on full cost.
(BCC) bid pricing.
(BCC) bid pricing per computer

163. Leader Industries is planning to introduce a new product, DMA. It is expected that
10,000 units of DMA will be sold. The full product cost per unit is $300. Invested
capital for this product amounts to $20 million. Leader’s target rate of return on
investment is 20%. The markup percentage for this product, based on operating income
as a percentage of full product cost, will be

Full cost $3,000,000.00


Leader’s target rate of return $4,000,000.00 133%

162. The Robo Division, a decentralized division of GMT Industries, has been approached to
submit a bid for a potential project for the RSP Company.
Robo Division has been informed by RSP that they will not consider bids over $8,000,000.
Robo Division purchases its materials from the Cross Division of GMT Industries.
There would be no additional fixed costs for either the Robo or Cross Divisions.

Information regarding this project is as follows.

Sell to Robo
Cross Division Robo Division
Variable Costs $1,500,000 $4,800,000 $1,500,000.00 $4,800,000.00
Transfer Price 3,700,000 - $3,700,000.00 $0.00

If Robo Division submits a bid for $8,000,000, the amount of contribution margin 8000000
recognized by the Robo Division and GMT Industries, respectively, is
Cross Division Robo Division
Price of the product sold $3,700,000.00 $8,000,000.00
VC $1,500,000.00 $8,500,000.00
CM $2,200,000.00 -$500,000.00

161. Johnson Company manufactures a variety of shoes, and has received a special one-timeonly order directly from a who
Johnson has sufficient idle capacity to accept the special order to manufacture 15,000 pairs of sneakers at a price of $7.50
Johnson’s normal selling price is $11.50 per pair of sneakers.
Variable manufacturing costs are $5.00 per pair and fixed manufacturing costs are $3.00 a pair.
Johnson’s variable selling expense for its normal line of sneakers is $1.00 per pair.

What would the effect on Johnson’s operating income be if the company accepted the special order?

Sales Rev 112500


VC 90000
CM $22,500.00
160. The Doll House, a very profitable company, plans to introduce a new type of doll to its
product line. The sales price and costs for the new dolls are as follows.

Selling price per doll $100 $100.00


Variable cost per doll $60 $60.00
Incremental annual fixed costs $456,000 $456,000.00
Income tax rate 30% 30%

If 10,000 new dolls are produced and sold, the effect on Doll House’s profit (loss) would

Selling price per doll $100 $1,000,000.00


Variable cost per doll $60 $600,000.00
Incremental annual fixed costs $456,000 $456,000.00
-$56,000.00
Income tax rate 30% savings -$16,800.00
-$39,200.00

159. Synergy Inc. produces a component that is popular in many refrigeration systems. Data
on three of the five different models of this component are as follows.

Model
A B C
Volume needed (units) 5000 6000 3000
Manufacturing costs
Variable direct costs $10 $24 $20 $ 10.00 $ 24.00 $ 20.00
Variable overhead 5 10 15 $ 5.00 $ 10.00 $ 15.00
Fixed overhead 11 20 17 $ 11.00 $ 20.00 $ 17.00
Total manufacturing costs $26 $54 $52 $ 26.00 $ 54.00 $ 52.00

Cost if purchased $21 $42 $39 $ 21.00 $ 24.00 $ 39.00

Total v manufacturing costs $ 15.00 $ 34.00 $ 35.00


Hours required 2.00 4.00 6.00
14,000.00 7,000.00
Saving (loss) (6.00) 10.00

Synergy applies variable overhead on the basis of machine hours at the rate of $2.50 per hour.
Models A and B are manufactured in the Freezer Department, which has a capacity of 28,000 machine processing hours.
Which one of the following options should be recommended to Synergy's management?
a. Purchase all three products in the quantities required.
b. Manufacture all three products in the quantities required.
c. The Freezer Department's manufacturing plan should include 5,000 units of
Model A and 4,500 units of Model B.
d. The Freezer Department's manufacturing plan should include 2,000 units of
Model A and 6,000 units of Model B.

158. Green Corporation builds custom-designed machinery. A review of selected data and the company’s pricing policies re

• A 10% commission is paid on all sales orders.


• Variable and fixed factory overheads total 40% and 20%, respectively, of direct labor.

• Corporate administrative costs amount to 10% of direct labor.


• When bidding on jobs, Green adds a 25% markup to the total of all factory and administrative costs to cover income taxes
• The firm’s income tax rate is 40%.

The company expects to operate at a maximum of 80% of practical capacity.

Green recently received an invitation to bid on the manufacture of some custom machinery for Kennendale, Inc.
For this project, Green’s production accountants estimate the material and labor costs will be $66,000 and $120,000, respe
$66,000.00 $120,000.00
Accordingly, Green submitted a bid to Kennendale in the amount of $375,000. Feeling Green’s bid was too high, Kennendal

Which one of the following options should be recommended to Green’s management?

• A 10% commission is paid on all sales orders.


• Variable and fixed factory overheads total 40% and 20%, respectively, of direct labor.
• Corporate administrative costs amount to 10% of direct labor.
• When bidding on jobs, Green adds a 25% markup to the total of all factory and administrative costs to cover income taxes
• The firm’s income tax rate is 40%.

DM $66,000.00
DL $120,000.00
VOH $ 48,000.00
FOH $ 24,000.00
Admin $12,000.00
Full cost $270,000.00
25% mark $67,500.00 25%
$337,500.00 375000 $37,500.00
• A 10% commission is paid on all sales orders.
$37,500.00 10%
Accordingly, Green submitted a bid to Kennendale in the amount of $375,000 375000
$375,000.00

a. Accept the counteroffer because the order will increase operating income.

157. Jones Enterprises manufactures 3 products, A, B, and C. During the month of May
Jones’ production, costs, and sales data were as follows.

Products
A B C
Units of production 30,000.00 20,000.00 70,000.00
Joint production costs to split-off point $480,000
Further processing costs $ - $60,000 $140,000 $ - $ 60,000.00 $ 140,000.00
Further processing cost $ 3.00 $ 2.00
Unit sales price
At split-off 3.75 5.50 10.25 $ 3.75 $ 5.50 $ 10.25
After further processing - 8.00 12.50 $ - $ 8.00 $ 12.50
$ 5.00 $ 10.50
INCREMENTAL PROFIT $ (0.50) $ 0.25
Based on the above information, which one of the following alternatives should be recommended to Jones’ management?

c. Process Product C further but sell Product B at the split-off point.

156. Basic Computer Company (BCC) sells its micro-computers using bid pricing. It develops bids on a full cost basis.
Full cost includes estimated material, labor, variable overheads, fixed manufacturing overheads, and reasonable increment

BCC believes bids in excess of $925 per computer are not likely to be considered. 925

BCC’s current cost structure, based on its normal production levels, is $500 for materials per computer and $20 per labor h
Assembly and testing of each computer requires 12 labor hours.
BCC’s variable manufacturing overhead is $2 per labor hour, fixed manufacturing overhead is $3 per labor hour, and increm

The company has received a request from the School Board for 500 computers. BCC’s
management expects heavy competition in bidding for this job. As this is a very large
order for BCC, and could lead to other educational institution orders, management is
extremely interested in submitting a bid which would win the job, but at a price high
enough so that current net income will not be unfavorably impacted. Management
believes this order can be absorbed within its current manufacturing facility. Which one
of the following bid prices should be recommended to BCC’s management?

DM $500.00 per computer


DL $20.00 per labor hour 6000

Assembly and testing of each computer requires 12 hours

VOH $2.00 per labor hour


FOH $3.00 per labor hour
incremental administrative costs $8.00 per computer

Full cost
10% return on full cost.
(BCC) bid pricing.
(BCC) bid pricing per computer

153. Lazar Industries produces two products, Crates and Boxes. Per unit selling prices, costs,
and resource utilization for these products are as follows.

Crates Boxes
Selling price $20 $30 $20.00 $30.00
Direct material costs $ 5 $ 5 2.5 $5.00 $5.00
Direct labor costs 8 10 $8.00 $10.00
Variable overhead costs 3 5 $ 5.25 $3.00 $5.00 VOH
Variable selling costs 1 2 $1.00 $2.00
Machine hours per unit 2 4 3.50 2.00 4.00

total fixed factory overhead cost $2,000,000.00


total fixed selling and administrative costs $840,000.00
Production and sales 500,000.00 700,000.00

Production of Crates and Boxes involves joint processes and use of the same facilities.
The total fixed factory overhead cost is $2,000,000 and total fixed selling and administrative costs are $840,000.
Production and sales are scheduled for 500,000 units of Crates and 700,000 units of Boxes.
Lazar maintains no direct materials, work-inprocess, or finished goods inventory.

Lazar can reduce direct material costs for Crates by 50% per unit, with no change in direct labor costs.
However, it would increase machine-hour production time by 1-1/2 hours per unit. 1.50
For Crates, variable overhead costs are allocated based on machine hours.

What would be the effect on the total contribution margin if this change was implemented?

Before
CM $3.00 1,500,000.00
CM per constraint. $1.50

After
CM $3.25 1,625,000.00
CM per constraint. $0.93 125,000.00

152. Refrigerator Company manufactures ice-makers for installation in refrigerators. The


costs per unit, for 20,000 units of ice-makers, are as follows. 20,000.00

Make
Direct materials $ 7 $7.00 7
Direct labor 12 $12.00 12
Variable overhead 5 $5.00 5
Fixed overhead 10 $10.00 6
Total costs $34 $34.00 $30.00

$30.00
$600,000.00

Cool Compartments Inc. has offered to sell 20,000 ice-makers to Refrigerator Company for $28 per unit.
If Refrigerator accepts Cool Compartments’ offer, the facilities used to manufacture ice-makers could be used to produce w
Revenues from the sale of water filtration units are estimated at $80,000, with variable costs amounting to 60% of sales.
In addition, $6 per unit of the fixed overhead associated with the manufacture of ice-makers could be eliminated.

For Refrigerator Company to determine the most appropriate action to take in this
situation, the total relevant costs of make vs. buy, respectively, are
c. $600,000 vs. $528,000.

151. Aspen Company plans to sell 12,000 units of product XT and 8,000 units of product RP.
Aspen has a capacity of 12,000 productive machine hours. 12000
The unit cost structure and machine hours required for each product is as follows.
12000 8000
Unit Costs XT RP
Materials $37 $24 $37.00 $24.00
Direct labor 12 13 $12.00 $13.00
Variable overhead 6 3 $6.00 $3.00
Fixed overhead 37 38 $37.00 $38.00
Machine hours required 1.0 1.5 1 1.5
VC $55.00 $40.00
Machine hours required 12,000.00 12,000.00

Purchase price $60.00 $45.00


Loss -$5.00 -$5.00
Loss per constraint -$5.00 -$7.50

Aspen can purchase 12,000 units of XT at $60 and/or 8,000 units of RP at $45. Based on the above, which one of the follow
169. D d. $268.

a. $228.
b. $238.
c. $258.
d. $268.

15%

168. A a. 12.5%.
100000 6
50000
9
165. D d. $1,026.30.

10%

200

$100,000.00
$68,000.00

$6,800.00
$10,200.00
$1,600.00

$186,600.00
$18,660.00
$205,260.00
$1,026.30

163. C c. 133.3%.
10,000.00 $300.00 $20,000,000.00
20%
162. C c. $(500,000) and $1,700,000

GMT Industries
$11,700,000.00
$10,000,000.00
$1,700,000.00 $1,700,000.00

161. C b. Increase by $22,500.

y order directly from a wholesaler.


neakers at a price of $7.50 per pair. 15000 7.5
11.5
5 3
1
160. C c. $(39,200).

10000

159. C c. The Freezer Department's manufacturing plan should include 5,000 u


Model A and 4,500 units of Model B.

Should be answered by
elimination

$ 2.50
achine processing hours. 28,000.00
158. A
mpany’s pricing policies revealed the following.

10%
40% 20%

10%
costs to cover income taxes and produce a profit. 25%
40%

80%

Kennendale, Inc.
6,000 and $120,000, respectively.

d was too high, Kennendale countered with a price of $280,000.

$234,000.00

$ 48,000.00 $ 24,000.00
$12,000.00
costs to cover income taxes and produce a profit. $ 84,000.00
157. C c. Process Product C further but sell Product B at the split-off point.

TOTAL
120,000.00
$ 480,000.00

ed to Jones’ management?

156. B b. $772.00.
on a full cost basis.
and reasonable incremental computer assembly administrative costs, plus a 10% return on full cost.

mputer and $20 per labor hour.

per labor hour, and incremental administrative costs are $8 per computer assembled.

0.1
500

$250,000.00
$120,000.00

$12,000.00
$18,000.00
$4,000.00

$404,000.00 VC $386,000.00
$40,400.00 $0.00
$444,400.00 $386,000.00
$888.80 $772.00

153. A a. $125,000 increase.

For Crates, variable overhead costs are allocated based


on machine hours.
$ 1.50
152. C c. $600,000 vs. $528,000.

Buy
28

$28.00
-$1.60
$26.40
$528,000.00

ould be used to produce water filtration units.


ounting to 60% of sales. 0.4 80000 32000 1.6
uld be eliminated.

151. A a. Produce XT internally and purchase RP.


ve, which one of the following actions should be recommended to Aspen's management?
g plan should include 5,000 units of
ct B at the split-off point.
13(Q10)
A review of the year-end accounting records of Elk Industries discloses the following
information.
Raw materials $ 80,000 80000
Work-in-process 128,000 128000
Finished goods 272, 000 272000
Cost of goods sold 1,120,000 1120000 74%
1520000

The company's underapplied overhead equals $133,000 . On the basis of this information, Elk's cost of goods sold is most a

a. $987,000.
b. $1,213,100. 98000
c. $1,218,000. $ 1,218,000.00
d. $1,253,000.

SANS COM Corporation utilizes an activity - based costing system for applying costs to its two products, P an
the assembly department, material handling costs vary directly with the number of parts inserted into the p
Machinery is recalibrated and oiled each weekend regardless of the number of parts inserted during the pre
week. Both material handling and machinery maintenance costs are charged to the product on the basis of t
number of parts inserted. Due to reengineering of the production process for Product P, the number of inse
parts per finished unit has been reduced. How will the redesign of the production process for Product P affe
activity-based cost of Product Q?

Material Handling costs $ 85.00 per part


machinery maintenance costs (fixed cost) $ 30,000.00 in total 150

Before redesign the production process

Assembly department
P
number of parts inserted into the product 100
Allocated Material Handling costs (No. of parts) $ 8,500.00
Allocated machinery maintenance costs (fixed cost) $ 15,000.00
$ 23,500.00
Material Handling costs per unit $ 85.00
machinery maintenance costs per unit $ 150.00
Due to reengineering of the production process for Product P
the number of insertion parts per finished unit has been reduced.
Material Handling costs (No. of parts) $ 85.00 per part
machinery maintenance costs (fixed cost) $ 30,000.00 in total $ 171.43
Assembly department
P
number of parts inserted into the product 75
Allocated Material Handling costs (No. of parts) $ 6,375.00
Allocated machinery maintenance costs (fixed cost) $ 12,857.14
$ 19,232.14
Material Handling costs per unit $ 85.00
machinery maintenance costs per unit $ 171.43

c. Material handling cost per Q unit will remain unchanged, and machinery maintenance cost per Q
133000
on, Elk's cost of goods sold is most appropriately reported as

ng costs to its two products, P and Q. In


mber of parts inserted into the product.
r of parts inserted during the previous
d to the product on the basis of the
or Product P, the number of insertion
uction process for Product P affect the

Cost Driver
(No. of parts)
(No. of parts)

mbly department
Q Total number of parts inserted into the product
100 200
$ 8,500.00
$ 15,000.00
$ 23,500.00
$ 85.00
$ 150.00
mbly department
Q
100 175
$ 8,500.00
$ 17,142.86
$ 25,642.86
$ 85.00
$ 171.43

nery maintenance cost per Q unit will increase.


166. Adam Corporation manufactures computer tables and has the following budgeted indirect
manufacturing cost information for next year.

Support Departments Operating Departments


Maintenance Systems Machining
Budgeted overhead $360,000 $95,000 $200,000
Support work furnished
From Maintenance 10% 50%
From Systems 5% 45%

If Adam uses the direct method to allocate support department costs to production
departments, the total overhead (rounded to the nearest dollar) for the Machining
Department to allocate to its products would be

Allocated service dpt. Costs


from maint. $200,000
from sys. $45,000
Machining OH $200,000
Total Machining OH $445,000

165. Logo Inc. has two data services departments (the Systems Department and the Facilities
Department) that provide support to the company’s three production departments
(Machining Department, Assembly Department, and Finishing Department). The
overhead costs of the Systems Department are allocated to other departments on the basis
of computer usage hours. The overhead costs of the Facilities Department are allocated
based on square feet occupied (in thousands). Other information pertaining to Logo is as
follows.

Computer
Department Overhead Usage Hours
Systems $200,000 300
Facilities $100,000 900
Machining $400,000 3600
Assembly $550,000 1800
Finishing $620,000 2700
9300

Logo employs the step-down method of allocating service department costs and begins
with the Systems Department. Which one of the following correctly denotes the amount
of the Systems Department’s overhead that would be allocated to the Facilities
Department and the Facilities Department’s overhead charges that would be allocated to
the Machining Department?
Systems to Facilities
the % ofsys costs that should be allocated to facil 10.0% $20,000

allocated costs from sys to Facilities


Facilities
New total facil
% that should be used to allocate facil to mach

163. Adam Corporation manufactures computer tables and has the following budgeted indirect
manufacturing cost information for next year.

Support Departments Operating Departments


Maintenance Systems Machining
Budgeted overhead $360,000 $95,000 $200,000
Support work furnished
From Maintenance 10% 50%
From Systems 5% 45%

If Adam uses the step-down method, beginning with the Maintenance Department, to
allocate support department costs to production departments, the total overhead (rounded
to the nearest dollar) for the Machining Department to allocate to its products would be

Allocated service dpt. Costs


allocated cost from maint. To macin $180,000
allocated costs from maint. To sys. $36,000
new costs for sys. $131,000
allocated cost from sys to machin 47.4% $62,053
Machining OH $200,000
Total Machining OH $442,053

162. Logo Inc. has two data services departments (the Systems Department and the Facilities
Department) that provide support to the company’s three production departments
(Machining Department, Assembly Department, and Finishing Department). The
overhead costs of the Systems Department are allocated to other departments on the basis
of computer usage hours. The overhead costs of the Facilities Department are allocated
based on square feet occupied (in thousands). Other information pertaining to Logo is as
follows.
Computer
Department Overhead Usage Hours
Systems $200,000 300
Facilities $100,000 900
Machining $400,000 3600
Assembly $550,000 1800
Finishing $620,000 2700
9300

If Logo employs the direct method of allocating service department costs, the overhead of
the Systems Department would be allocated by dividing the overhead amount by

8100

156. Atmel Inc. manufactures and sells two products. Data with regard to these products are
given below.
Product A

Units produced and sold 30,000 12,000 30000


Machine hours required per unit 2 3 2
Receiving orders per product line 50 150 50
Production orders per product line 12 18 12
Production runs 8 12 20
Inspections 20 30

Total budgeted machine hours are 100,000. The budgeted overhead costs are shown below.

Receiving costs $450,000 $450,000


Engineering costs 300,000 $300,000
Machine setup costs 25,000 $25,000
Inspection costs 200,000 $200,000
Total budgeted overhead $975,000 $975,000

The cost driver for engineering costs is the number of production orders per product
line. Using activity-based costing, the engineering cost per unit for Product B would be
152. Patterson Corporation expects to incur $70,000 of factory overhead and $60,000 of
general and administrative costs next year. Direct labor costs at $5 per hour are expected
to total $50,000. If factory overhead is to be applied per direct labor hour, how much
overhead will be applied to a job incurring 20 hours of direct labor?

OH $70,000
Hours 10,000
rate $7
Applied $140

151. Cynthia Rogers, the cost accountant for Sanford Manufacturing, is preparing a
management report which must include an allocation of overhead. The budgeted
overhead for each department and the data for one job are shown below.

Department
Tooling
Supplies $ 690 $ 80 $690
Supervisor's salaries 1,400 1,800 $1,400
Indirect labor 1,000 4,000 $1,000
Depreciation 1,200 5,200 $1,200
Repairs 4,400 3,000 $4,400
Total budgeted overhead $8,690 $14,080 $8,690
Total direct labor hours 440 640 440
Direct labor hours on Job #231 10 2 10

Using the departmental overhead application rates, and allocating overhead on the basis
of direct labor hours, overhead applied to Job #231 in the Tooling Department would be

OH $8,690
Hours 440
rate $19.75
Applied $197.50

150. John Sheng, cost accountant at Starlet Company, is developing departmental


factory overhead application rates for the company's tooling and fabricating
departments. The budgeted overhead for each department and the data for one job
are shown below.

Departments
Tooling
Supplies $ 850 $ 200 $850
Supervisors' salaries 1,500 2,000 $1,500
Indirect labor 1,200 4,880 $1,200
Depreciation 1,000 5,500 $1,000
Repairs 4,075 3,540 $4,075
Total budgeted overhead $8,625 $16,120 $8,625
Total direct labor hours 460 620 460
Direct labor hours on Job #231 12 3 12

Using the departmental overhead application rates, total overhead applied to Job #231
in the Tooling and Fabricating Departments will be

Tooling
rate $19
Applied $225

Fabricating
rate $26
Applied $78

total overhead applied


$303

149. Baldwin Printing Company uses a job order costing system and applies overhead based
on machine hours. A total of 150,000 machine hours have been budgeted for the year.
During the year, an order for 1,000 units was completed and incurred the following

Direct material costs $1,000 1000


Direct labor costs 1,500 1500
Actual overhead 1,980 1980
Machine hours 450 450

The accountant calculated the inventory cost of this order to be $4.30 per unit. The
annual budgeted overhead in dollars was

Numerator
denmenator $150,000

total actual cost 4480


total normal cost 4300
Applied OH 1800

Applied OH = Bud.Rate x 450


Applied OH $1800 = Bud.Rate x 450
thus rate = 4.0
as the denmenator =$150000
annual budgeted overhead in dollars was $600,000

143. Atmel Inc. manufactures and sells two products. Data with regard to these products are
given below.
Product A

Units produced and sold 30,000 12,000 30000


Machine hours required per unit 2 3 2
Receiving orders per product line 50 150 50
Production orders per product line 12 18 12
Production runs 8 12 20
Inspections 20 30

Total budgeted machine hours are 100,000. The budgeted overhead costs are shown below.

Receiving costs $450,000 $450,000


Engineering costs 300,000 $300,000
Machine setup costs 25,000 $25,000
Inspection costs 200,000 $200,000
Total budgeted overhead $975,000 $975,000

Using activity-based costing, the per unit overhead cost allocation of receiving costs
for product A is $112,500
$3.75

142. The Chocolate Baker specializes in chocolate baked goods. The firm has long assessed
the profitability of a product line by comparing revenues to the cost of goods sold.
However, Barry White, the firm’s new accountant, wants to use an activity-based costing
system that takes into consideration the cost of the delivery person. Listed below are
activity and cost information relating to two of Chocolate Baker’s major products.

Muffins
Revenue $53,000 $46,000 $53,000
Cost of goods sold 26,000 21,000 $26,000

Delivery Activity
Number of deliveries 150 85 150
Average length of delivery 10 Minutes 15 Minutes 10
Cost per hour for delivery $20.00 $20.00 $20
Cost per Minute for delivery 0.333333333

Average length of deliveries 1500


Delivery Activity cost 500
Using activity-based costing, which one of the following statements is correct?

$26,500
50%

141. Pelder Products Company manufactures two types of engineering diagnostic equipment
used in construction. The two products are based upon different technologies, x-ray and
ultra-sound, but are manufactured in the same factory. Pelder has computed the
manufacturing cost of the x-ray and ultra-sound products by adding together direct
materials, direct labor, and overhead cost applied based on the number of direct labor
hours. The factory has three overhead departments that support the single production line
that makes both products. Budgeted overhead spending for the departments is as follows.

Department
Engineering design Material handling
$6,000 $5,000

Pelder’s budgeted manufacturing activities and costs for the period are as follows

Activity X-Ray
Units produced and sold 50 100 50
Direct materials used $5,000 $8,000 $5,000
Direct labor hours used 100 300 $100
Direct labor cost $4,000 $12,000 $4,000
Number of parts used 400 600 400
Number of engineering changes 2 1 2
Number of product setups 8 7 8

The budgeted cost to manufacture one ultra-sound machine using the activity-based
costing method is
ABC Rates ultra-sound
Engineering design $2,000 2,000
Material handling $5 3,000
Setup $200 1,400
total cost $26,400
per unit $264

139. Waller Co. uses a weighted-average process-costing system.


Material B is added at two different points in the production of shirms, 40% is added when the units are 20% completed, a
At the end of the quarter, there are 22,000 shirms in process, all of which are 50% completed.

With respect to Material B, the ending shirms in process represent how many equivalent units?

EUP : Weighted Average


Ü  EUP under weighted average costing may be computed as follows:
Total Units Completed this period
Work to date on Ending WIP 8,800.00
EUP under weighted average 8,800.00

138. Jones Corporation uses a first-in, first-out (FIFO) process costing system. Jones has the
following unit information for the month of August.

Beginning work-in-process inventory, 100% complete


for materials, 75% complete for conversion cost
100% 75%

Units complete and transferred out

Ending work-in-process inventory, 100% complete


for materials, 60% complete for conversion costs
100% 60%

The number of equivalent units of production for conversion costs for the month of
August is
Step 2: compute Equivalent Units of Production
EUP Computation under FIFO (If materials are added as work in process (continually):
Material Conversion

Total units Completed 90000


- Beginning WIP (regardless of % of completion) 10000
Units Started and Completed this Period - 80,000.00
+ Amount needed to complete Beginning WIP 2500
+ Amount Completed on Ending WIP 4800
EUP under FIFO - 87,300.00

135. San Jose Inc. uses a weighted-average process costing system. All materials are
introduced at the start of manufacturing, and conversion cost is incurred evenly
throughout production. The company started 70,000 units during May and had the
following work-in-process inventories at the beginning and end of the month.

May-01 30000 units 40% complete


May-31 24000 units 25% complete

Assuming no spoilage or defective units, the total equivalent units used to assign costs for
May are

1. Account for all units (physical flow of quantities).

Beginning WIP 30,000.00 Total Units to Account for =


Started Units this period 70,000.00 Finished or Transferred-out Good
Total Units to Account for 100,000.00 Ending WIP
Spoilage (lost)

EUP : Weighted Average


Ü  EUP under weighted average costing may be computed as follows:
Materials Conversion Cost
Total Units Completed this period 76,000.00 76,000.00
Work to date on Ending WIP 24,000.00 6000
EUP under weighted average 100,000.00 82,000.00
134. Oster Manufacturing uses a weighted-average process costing system and has the
following costs and activity during October.

Materials $40,000 $40,000


Conversion cost 32,500 $32,500
Total beginning work-in-process inventory $72,500 $72,500

Materials $ 700,000 $700,000


Conversion cost 617,500 $617,500
Total production costs - October $1,317,500 $1,317,500

Production completed 60,000 units 60,000


Work-in-process, October 31 20,000 units 31,200 25%

All materials are introduced at the start of the manufacturing process, and conversion cost
is incurred uniformly throughout production. Conversations with plant personnel reveal
that, on average, month-end in-process inventory is 25% complete. Assuming no
spoilage, how should Oster’s October manufacturing cost be assigned?

Production Completed Work-in-Process


75% $555,000 $185,000
Work-in-process, October 31 92% $600,000 $50,000
20000 $1,155,000 $235,000
M 20000
C 5000

Completed
M 60000
C 60000

Colt Company uses a weighted-average process cost system to account for the cost of
producing a chemical compound. As part of production, Material B is added when the
goods are 80% complete.
Beginning work-in-process inventory for the current month was 20,000 units, 90% complete.
During the month, 70,000 units were started in process,
and 65,000 of these units were completed. There were no lost or spoiled units. If the
ending inventory was 60% complete, the total equivalent units for Material B for the
month was

1. Account for all units (physical flow of quantities).


Beginning WIP 20,000.00 Total Units to Account for =
Started Units this period 70,000.00 Finished or Transferred-out Good
Total Units to Account for 90,000.00 Ending WIP
Spoilage (lost)

EUP : Weighted Average


Ü  EUP under weighted average costing may be computed as follows:
M C
Total Units Completed this period 65,000.00
Work to date on Ending WIP - zero as the material added when %80
EUP under weighted average 65,000.00

132. Southwood Industries uses a process costing system and inspects its goods at the end of
manufacturing. The inspection as of June 30 revealed the following information for the
month of June.

Good units completed 16,000 16000


Normal spoilage (units) 300 300
Abnormal spoilage (units) 100 100

Southwood Industries uses a process costing system and inspects its goods at the end of
manufacturing. The inspection as of June 30 revealed the following information for the
month of June.
Good units completed 16,000 16000
Normal spoilage (units) 300
Abnormal spoilage (units) 100
Unit costs were: materials, $3.50; and conversion costs, $6.00. The number of units that
Southwood would transfer to its finished goods inventory and the related cost of these

M CONV
Beginning WIP XX
Started Units this period XX $3.5 $6.0
$3.5 $6.0
Total Units to Account for 16000 0
1. Finished or Transferred-out Good 16000 $152,000.0
2. Ending WIP
3. NormalSpoilage (lost) 300 $2,850.0 product cost
4. Abnormal Spoilage (lost) 100 $950 period cost

The number of units that Southwood would transfer to its finished goods inventory and the related cost of these
16000

128. Fashion Inc. manufactures women’s dresses using cotton and polyester. Since the same
style dresses are made out of both fabrics, Fashions uses operation costing.

During June,1,000 cotton dresses were completely produced.


Also during June, 1,500 polyester dresses were started by adding all materials at the beginning of the process.
Of these 1,500 dresses, 700 were completely finished and the remainder were 25 percent complete by the end of the mon
1500
There was no work-in-process inventory at the beginning of June.

Costs incurred during June were as follows.


Cotton $10,000 $10,000.0
Polyester 22,500 $22,500.0
Conversion costs 13,300 $13,300.0

The cost per unit to manufacture one polyester dress during June was

M C
$22,500 $13,300.0
units 1500 1900
per unit $15 $7 $22

Mack Inc. uses a weighted-average process costing system. Direct materials and
conversion costs are incurred evenly during the production process. During the month of
October, the following costs were incurred.
Direct materials $39,700
Conversion costs 70,000
The work-in-process inventory as of October 1 consisted of 5,000 units, valued at $4,300,
that were 20% complete. During October, 27,000 units were transferred out. Inventory
as of October 31 consisted of 3,000 units that were 50% complete. The weighted-average
inventory cost per unit completed in October was

Beginning WIP XX 5000 $4,300 20%


Started Units this period X 25000

Total Units to Account for 30000 30000

1. Finished or Transferred-out Good 27000


2. Ending WIP 3000 50%

During December, Krause Chemical Company had the following selected data
concerning the manufacture of Xyzine, an industrial cleaner.
Production Flow Physical Units
Completed and transferred to the next department 100
Add: Ending work-in-process inventory 10 (40% complete as
to conversion)
Total units to account for 110
Less: Beginning work-in-process inventory 20 (60% complete as
to conversion)
Units started during December 90
All material is added at the beginning of processing in this department, and conversion
costs are added uniformly during the process. The beginning work-in-process inventory
had $120 of raw material and $180 of conversion costs incurred. Material added during
December was $540 and conversion costs of $1,484 were incurred. Krause uses the
weighted-average process-costing method. The total raw material costs in the ending
work-in-process inventory for December is

M CONV
Beginning WIP XX 20 60% conv $120 $180
Started Units this period X 90 $540 $1,484
$660 $1,664
Total Units to Account for 110 110

1. Finished or Transferred-out Good 100


2. Ending WIP 10 40% conv

M conv
Ü  EUP under weighted average costing may be computed as follows:
Total Units Completed this period XX 100 100
Work to date on Ending WIP XX 10 4
_____
EUP under weighted average XXXX 110 104

6.00 16.00

The total raw material costs in the ending work-in-process inventory for Dece 60.00

Southwood Industries uses a process costing system and inspects its goods at the end of
manufacturing. The inspection as of June 30 revealed the following information for the
month of June.
Good units completed 16,000 16000
Normal spoilage (units) 300
Abnormal spoilage (units) 100
Unit costs were: materials, $3.50; and conversion costs, $6.00. The number of units that
Southwood would transfer to its finished goods inventory and the related cost of these

M CONV
Beginning WIP XX
Started Units this period XX $3.5 $6.0
$3.5 $6.0
Total Units to Account for 16000 0

1. Finished or Transferred-out Good 16000 $152,000.0


2. Ending WIP
3. NormalSpoilage (lost) 300 $2,850.0 product cost
4. Abnormal Spoilage (lost) 100 $950.0 period cost

The number of units that Southwood would transfer to its finished goods inventory and the related cost of these
16000
Colt Company uses a weighted-average process cost system to account for the cost of
producing a chemical compound. As part of production, Material B is added when the
goods are 80% complete. Beginning work-in-process inventory for the current month
was 20,000 units, 90% complete. During the month, 70,000 units were started in process,
and 65,000 of these units were completed. There were no lost or spoiled units. If the
ending inventory was 60% complete, the total equivalent units for Material B for the
month was

M CONV
Beginning WIP XX 20000 90%
Started Units this period X 70000
$0.0 $0.0
Total Units to Account for 90000 90000

1. Finished or Transferred-out Good 65000


2. Ending WIP 25000 0

M conv
Ü  EUP under weighted average costing may be computed as follows:
Total Units Completed this period XX 65000 65000
Work to date on Ending WIP XX 0 15000
_____
EUP under weighted average XXXX 65000 80000

123. Tempo Company produces three products from a joint process. The three products are
sold after further processing as there is no market for any of the products at the split-off
point. Joint costs per batch are $315,000. Other product information is shown below.

Product A Product B
Units produced per batch 20000 30000
Further processing and marketing cost per unit $0.70 $3.00
Final sales value per unit $5.00 $6.00
If Tempo uses the net realizable value method of allocating joint costs, how much of the
joint costs will be allocated to each unit of Product C?

NRV $86,000.00 $90,000.00


19.55% 20.45%
$61,568.18 $64,431.82
20000 30000
$3.08 $2.15

112. Consider the following situation for Donaldson Company for the prior year.
• The company produced 1,000 units and sold 900 units, both as budgeted. 1000
• There were no beginning or ending work-in-process inventories and no
beginning finished goods inventory.
• Budgeted and actual fixed costs were equal, all variable manufacturing costs
are affected by volume of production only, and all variable selling costs are
affected by sales volume only.
• Budgeted per unit revenues and costs were as follows.
Per Unit
Sales price $100 $100.00
Direct materials 30 $30.00
Direct labor 20 $20.00
Variable manufacturing costs 10 $10.00
Fixed manufacturing costs 5 $5.00
Variable selling costs 12 $12.00
Fixed selling costs ($3,600 total) 4 $4.00
Fixed administrative costs ($1,800 total) 2 $2.00

Assuming that Donaldson uses variable costing, the operating income for the prior year
was

sales rev $90,000.00


Cost of Goods Sold
Beginning inventory
Variable manufacturing costs $60,000.00
Fixed manufacturing costs $5,000.00
Cost for goods available for sale $65,000.00
deduct ending inventory 6000
Total COGS(at standard costs) $59,000.00
Adjustment for manufacturing variances
Total COGS $59,000.00
Gross Margin $31,000.00
Operating cost $16,200.00
Operating income $14,800.00

109. Merlene Company uses a standard cost accounting system. Data for the last fiscal year
are as follows
Units

Beginning inventory of finished goods 100 100


Production during the year 700 700
Sales 750 750
Ending inventory of finished goods 50 50

Per Unit
Product selling price $200 $200.00
Standard variable manufacturing cost 90 $90.00
Standard fixed manufacturing cost 20* $20.00

Budgeted selling and administrative costs (all fixed) $45,000 $45,000.00


*Denominator level of activity is 750 units for the year. 750.00

There were no price, efficiency, or spending variances for the year, and actual selling and
administrative expenses equaled the budget amount. Any volume variance is written off
to cost of goods sold in the year incurred. There are no work-in-process inventories.

The amount of operating income earned by Merlene for the last fiscal year using variable
costing was

sales rev 150000


Cost of Goods Sold
Beginning inventory 9000
Variable manufacturing costs 63000
Fixed manufacturing costs 15000
Cost for goods available for sale 87000
deduct ending inventory 4500
Total COGS(at standard costs) 82500
Adjustment for manufacturing variances
Total COGS 82500
Gross Margin 67500
Operating cost $45,000.00
Operating income $22,500.00
107. Chassen Company, a cracker and cookie manufacturer, has the following unit costs for
the month of June

Variable Variable
manufacturing cost marketing cost
$5.00 $3.50

A total of 100,000 units were manufactured during June of which 10,000 remain in
ending inventory. Chassen uses the first-in, first-out (FIFO) inventory method, and the
10,000 units are the only finished goods inventory at month-end. Using the full
absorption costing method, Chassen's finished goods inventory value would be $70,000.00

87. Fowler Co. provides the following summary of its total budgeted production costs at
three production levels.
Volume in Units
1000 1500 2000
Cost A $1,420.00 $1.42 $2,130.00 $1.42 $2,840.00
Cost b $1,550.00 $1.55 $2,200.00 $1.47 $2,900.00
Cost c $1,000.00 $1.00 $1,000.00 $0.67 $1,000.00
Cost d $1,630.00 $1.63 $2,445.00 $1.63 $3,260.00

The cost behavior of each of the Costs A through D, respectively, is

81. A review of Plunkett Corporation’s accounting records for last year disclosed the
following selected information

Variable costs
Direct materials used $ 56,000 $56,000.00
Direct labor 179,100 $179,100.00
Manufacturing overhead 154,000 $154,000.00
Selling costs 108,400 $108,400.00

Fixed costs
Manufacturing overhead 267,000 $267,000.00
Selling costs 121,000 $121,000.00
Administrative costs 235,900 $235,900.00

In addition, the company suffered a $27,700 uninsured factory fire loss during the year.
What were Plunkett’s product costs and period costs for last year?

Product Period
$656,100.00 $493,000.00

80. Kimber Company has the following unit cost for the current year

Raw material $20.00 $20.00


Direct labor 25.00 $25.00
Variable manufacturing overhead 10.00 $10.00
Fixed manufacturing overhead 15.00 $15.00 $120,000.00
Total unit cost $70.00 $70.00

Fixed manufacturing cost is based on an annual activity level of 8,000 units. Based on
these data, the total manufacturing cost expected to be incurred to manufacture 9,000
units in the current year is
$615,000.00
166. D d. $445,000

perating Departments Total


Fabrication
$300,000 $955,000

40% 100%
50% 100%

165. D
d. $20,000 $24,000.

Square Feet
Occupied
1000
600
2000
3000
5000
11600
Facilities to Machining

$20,000
$100,000
$120,000
20% $24,000

163. C c. $442,053.

perating Departments Total


Fabrication
$300,000 $955,000

40% 100%
50% 100%

162. B b. 8,100 hours


Square Feet
Occupied
1000
600
2000
3000
5000
11600

156. C c. $15.00.

Product B total

12000 42000
3 5
150 200
18 30
30 50

100000

30 $10,000

$180,000
$15
$70,000 $60,000 152. C c. $140.
$5
$50,000
20.00

151. B b. $197.50

Fabricating
$80
$1,800
$4,000
$5,200
$3,000
$14,080
640
2

150. B b. $303.
Fabricating
$200
$2,000
$4,880
$5,500
$3,540
$16,120
620
3

149. B b. $600,000
$150,000
1000

4.3
a. $3.75 143. A

Product B total

12000 42000
3 5
150 200
18 30
30 50

100,000

200 $2,250
30 $10,000

142. C

Cheesecake
$46,000
$21,000

85
15
$20
0.33333333

1275
425

$24,575 $1,925 c. The muffins are $1,925 more profitable.


53%

141. B b. $264.

Setup Total
$3,000 $14,000

Product
Ultra-Sound
100
$8,000
$300
$12,000
600 1,000
1 3
7 15
139. B b. 8,800 units.

e units are 20% completed, and the remaining 60% of Material B is added when the units are 80% completed.
22000

138. A a. 87,300.

Units

10000

90000

8000
135. D d. 100,000 82,000

70000

ccount for = 24,000.00 Total Units Completed this period


r Transferred-out Goods 76,000.00
24,000.00

onversion Cost
134. C c. $1,155,000 $235,000

M C
$740,000 $650,000

the total cost of the material should be allocated between the completed units and WIP based on the % of un
so, we have to calculate the no. of units in WIP

133. A a. 65,000 units.


ccount for = 65,000.00
r Transferred-out Goods 65,000.00
25,000.00

rial added when %80

$9.5
elated cost of these
$154,850.0

128. C c. $22.00.

1000
ng of the process.
mplete by the end of the month. 900
1900

126. D d. $4.00.
Total Units Completed this period XX
Current cost Work to date on Ending WIP XX
DM $39,700 _____
Conv $70,000 EUP under weighted average XXXX
$109,700

$114,000

127. C c. $60.
132. B b. 16,000 $154,850
Units Transferred Cost

$9.5

elated cost of these


$154,850.0
133. A a. 65,000 units.

123. D d $3.78.

$315,000

Product C
50000
$1.72
$7.00
$264,000.00 $440,000.00
60.00%
$189,000.00 $315,000.00
50000
$3.78

112. C c. $14,800
900 100

Budgeted FOH Allocated PVV

$5,000.00

$3,600.00
$1,800.00

Simple
solution

sales rev $90,000.00


VM 54000
V SGA 10800
CM $25,200.00
FM $5,000.00
FSGA $5,400.00
OI $14,800.00
109. D d. $31,000.

$15,000.00 $14,000.00 $1,000.00

dremmon

sales rev 150000


VM 67500
V SGA
CM 82500
FM 15,000.00
FSGA $45,000.00
OI 22,500.00
107. B b. $70,000.

Fixed Fixed
manufacturing cost marketing cost
$2.00 $4.00

100000

10000

87. D d. variable, semi-variable, fixed, and variable.

$1.42 VC
$1.45 SV Fixed cost fixed in total &
$0.50 FC variable cost fixed per unit
$1.63 VC

81. C c. $656,100 $493,000

$27,700.00
80. C c. $615,000.

8000
9000
c. $442,053.
nits and WIP based on the % of units
ed this period XX 27000
ding WIP XX 1500
_____
d average XXXX 28500

$4.0
U
Collection Pattern for Credit Sales
Month of sale 30%
One month following sale 40%
Second month following sale 25%
Karmee’s cost of goods sold averages 40% of the sales value. Karmee’s objective is to maintain a target inventory equal to 30
in units. Purchases of merchandise for resale are paid for in the month following the sale.

Jan feb march april may

Sales 600,000 650,000 700,000 625,000 720,000


cash sales 120,000 130,000 140,000 125,000 144,000
credit sales 480,000 520,000 560,000 500,000 576,000
336,000 364,000 392,000 350,000 403,200
Accounts payable (COGS) 240,000 260,000 280,000 250,000 288,000
Payroll
Other disbursements
BI 78,000 84,000 75,000 86,400
Purchase 318,000 266,000 271,000 261,400 297,600
EI 78,000.0 84,000.0 75,000.0 86,400.0 96,000.0
(78,000.0) (84,000.0) (75,000.0) (86,400.0)

vc 60,000.0 65,000.0 70,000.0 62,500.0 72,000.0


vc cash (60,000.0) (65,000.0) (70,000.0) (62,500.0)

fc -60000 -60000 -60000 -60000 -60000


fc
fc -90000 -90000 -90000 -90000 -90000
fc -45000 -45000
fc -120000
total fc -195000 -150000 -150000 -315000 -150000

Budget Information
Total Cash receipt cash 120,000.0 130,000.0 140,000.0 125,000.0 144,000.0
30% cr 144,000.0 156,000.0 168,000.0 150,000.0 172,800.0
40%cr 192,000.00 208,000.00 224,000.00 200,000.00
25%cr 120,000.00 130,000.00 140,000.00

Total Cash disb

Total Cash receipt 264,000.00 478,000.00 636,000.00 629,000.00 656,800.00


Total Cash disb - - 1,000.00
Net cash flow 264,000 636,000 655,800
Cumulative excess cash 736,000
Minimum cash balance -100000
Cumulative finance surplus (deficit) requirment 536,000
interest 1000
excess of the loan 736,000

loan of the month 636,000 1,391,800


minimum loan 100000 20000
aintain a target inventory equal to 30% of the next month’s sales
e.

june

800,000
160,000
640,000
448,000
320,000

96,000
224,000
-
(96,000.0)

80,000.0
(72,000.0)

-60000

-90000

-150000
160,000.0
192,000.0
230,400.00
125,000.00

707,400.00
The Raymar Company example

The firm has established a $200,000 line of credit with its bank at a 12% annual rate of interest on which borrowin
deficits must be made in $10,000 increments. There is no outstanding balance on the line of credit loan on April 1
repayments are to be made in any month in which there is a surplus of cash.
Interest is to be paid monthly. If there are no outstanding balances on the loans, Raymar will invest any cash in ex
desired end-of-month cash balance in U.S. Treasury bills. Raymar intends to maintain a minimum balance of $100
of each month by either borrowing for deficits below the minimum balance or investing any excess cash. Expected
collection and disbursement patterns are shown in the column to the right.

• Collections. 50% of the current month’s sales budget and 50% of the previous month’s sales budget.
• Accounts payable disbursements. 75% of the current month’s accounts payable budget and 25% of the
previous month’s accounts payable budget.
• All other disbursements occur in the month in which they are budgeted.

March April May


Sales 40,000 50,000 100,000
Accounts payable 30,000 40,000 40,000
Payroll 60,000 70,000 50,000
Other disbursements 25,000 30,000 10,000

Budget Information

Total Cash receipt

20,000.00 25,000.00 50,000.00


20,000.00 25,000.00
Total Cash disb
22,500.00 30,000.00 30,000.00
7,500.00 10,000.00

Total Cash receipt 20,000.00 45,000.00 75,000.00


Total Cash disb 107,500 137,500.00 101,000.00
Net cash flow (87,500) (92,500) (26,000)
Cumulative excess cash 7,500
Minimum cash balance -100000
Cumulative finance surplus (deficit) requirment (192,500)
interest 1000
excess of the loan 7,500

loan of the month (92,500) (18,500)


minimum loan 100000 20000
e of interest on which borrowings for cash
he line of credit loan on April 1. Principal

ymar will invest any cash in excess of its


in a minimum balance of $100,000 at the end
sting any excess cash. Expected monthly

sales budget.
t and 25% of the
1. Account for all units (physical flow of quantities).

Beginning WIP Total Units to Account for =


Started Units this period Finished or Transferred-out Goods
Total Units to Account for - Ending WIP
Spoilage (lost)

Step 2: compute Equivalent Units of Production


EUP Computation under FIFO (If materials are added as work in process (continually):
Material Conversion

Total units Completed


- Beginning WIP (regardless of % of completion)
Units Started and Completed this Period - -
+ Amount needed to complete Beginning WIP
+ Amount Completed on Ending WIP
EUP under FIFO - -

If materials are added at the beginning of a process :


Total Units Completed
+ Amount of materials Needed to Complete BWIP zero
.+ Amount of materials Added to Date on EWIP 100%
EUP for Materials -

If materials are added at the end of a process


Total Units Completed
+ Amount of materials Needed to Complete BWIP
+ Amount of materials Added to Date on EWIP zero
EUP for Materials -

EUP : Weighted Average


Ü  EUP under weighted average costing may be computed as follows:
Total Units Completed this period
Work to date on Ending WIP
EUP under weighted average -
-
ed-out Goods
[Fact Pattern #10]
Kimbeth Manufacturing uses a process cost system to The beginning inventory was 60%
manufacture Dust Density Sensors for the mining industry. 20% 20% complete for conversion costs. Th
The following information pertains to operations for the 90% 90% complete for materials and 40% c
month of May. costs.
Costs pertaining to the month of May
Units • Beginning inventory costs are mater
Beginning work-in-process inventory, May 1 16,000 16,000.00 labor, $20,320; and overhead, $15,240
Started in production during May 100,000 100,000.00
Completed production during May 92,000 92,000.00
Ending work-in-process inventory, May 31 24,0 24,000.00 • Costs incurred during May are mater
direct labor, $182,880; and overhead,

1. Account for all units (physical flow of quantities).

Beginning WIP 16,000.00 Total Units to Account for =


Started Units this period 100,000.00 Finished or Transferred-out Goods
Total Units to Account for 116,000.00 Ending WIP
Spoilage (lost)

Step 2: compute Equivalent Units of Production


EUP Computation under FIFO (If materials are added as work in process (continually):
Material Conversion

Total units Completed 92,000.00 92,000.00


- Beginning WIP (regardless of % of completion) 16,000.00 16,000.00
Units Started and Completed this Period 76,000.00 76,000.00
+ Amount needed to complete Beginning WIP 6,400.00 12800
+ Amount Completed on Ending WIP 21,600.00 9600
EUP under FIFO 104,000.00 98,400.00
$ 4.50 $ 5.83
5.83

EUP : Weighted Average


Ü  EUP under weighted average costing may be computed as follows:
Total Units Completed this period 92,000.00 92,000.00
Work to date on Ending WIP 21,600.00 9600
EUP under weighted average 113,600.00 101,600.00
ng inventory was 60% 60% complete for materials and
ete for conversion costs. The ending inventory was
ete for materials and 40% complete for conversion 40%

ning to the month of May are as follows:


inventory costs are materials, $54,560; direct $ 54,560
20; and overhead, $15,240. $ 20,320
$ 15,240

rred during May are materials used, $468,000; $ 468,000


$182,880; and overhead, $391,160 $ 182,880
$ 391,160

or = 116,000.00
ferred-out Goods 92,000.00
24,000.00

[145] Gleim #: 4.2.33 -- Source: CMA 695 3-1


(Refers to Fact Pattern #10) B. 104,000 units. 104,000.00
Using the first-in, first-out (FIFO) method, Kimbeth’s equivalent units of production (EUP) for materials are

[146] Gleim #: 4.2.34 -- Source: CMA 695 3-2


(Refers to Fact Pattern #10) D. 98,400 units 98,400.00
Using the FIFO method, Kimbeth’s equivalent units of production for conversion costs a

[147] Gleim #: 4.2.35 -- Source: CMA 695 3-3


(Refers to Fact Pattern #10) B. $4.50 $ 4.50
Using the FIFO method, Kimbeth’s equivalent unit cost of materials for May is

[148] Gleim #: 4.2.36 -- Source: CMA 695 3-4


(Refers to Fact Pattern #10) B. $5.83 $ 5.83
Using the FIFO method, Kimbeth’s equivalent unit conversion cost for May is

[149] Gleim #: 4.2.37 -- Source: CMA 695 3-5 21600 9600


(Refers to Fact Pattern #10) A. $153,168 97,200.00 55,968.00
Using the FIFO method, Kimbeth’s the total cost of units in the ending work-in-process inventory at May 31 is
[150] Gleim #: 4.2.38 -- Source: CMA 695 3-6
(Refers to Fact Pattern #10)
Using the weighted-average method, Kimbeth’s equivalent unit cost of materials for May is
$ 4.60
[151] Gleim #: 4.2.39 -- Source: CMA 695 3-7
(Refers to Fact Pattern #10)
Using the weighted-average method, Kimbeth’s equivalent unit conversion cost for May is
$ 6.00
uction (EUP) for materials are

153,168.00
n-process inventory at May 31 is
als for May is

st for May is
Gregg Industries manufactures molded chairs. The three models of molded chairs, which are all variations of the same des
are Standard (can be stacked), Deluxe (with arms), and Executive (with arms and padding). The company uses batch
manufacturing and has an operation costing system.
Gregg has an extrusion operation and subsequent operations to form, trim, and finish the chairs.
Plastic sheets are produced by the extrusion operation, some of which are sold directly to other manufacturers.
During the forming operation, the remaining plastic sheets are molded into chair seats and the legs are added; the standar
During the trim operation, the arms are added to the deluxe and executive models and the chair edges are smoothed.
Only the executive model enters the finish operation where the padding is added.
All of the units produced are subject to the same steps within each
operation, and no units are in process at the end of the period. The units of production and direct materials costs were as f

Units Extrusion Form


Produced Materials Materials
Plastic sheets 5,000.00 $60,000
Standard mode 6,000.00 $72,000 $24,000
Deluxe model 3,000.00 $36,000 $12,000
Executive model 2,000.00 $24,000 $8,000
16,000.00 $192,000.00 $44,000.00

Units attributable to each stage 16,000.00 11,000.00


Unit material cost $12.00 $4.00

Plastic sheets are produced by the During the forming operation, the remaining
extrusion operation, some of which plastic sheets are molded into chair seats and
are sold directly to other the legs are added; the standard model is sold
manufacturers. after this operation.

Manufacturing costs applied during the month were:


Extrusion Form
Operation Operation
Direct labor $152,000 $60,000
Overhead $240,000 $72,000
$392,000 $132,000

Units attributable to each stage 16,000.00 11,000.00


Unit conversion cost $24.50 $12.00

(Refers to Fact Pattern #33)


Gregg Industries’ unit cost of a standard model is $52.50

A standard model passes through the extrusion and form operations. Thus, its unit cost includes the
materials and conversion costs for both operations. The unit materials and conversion costs for the extrusion
operation are $12.00 ($192,000 ÷ 16,000 units) and $24.50 [($152,000 + $240,000) ÷ 16,000 units], respectively. The
unit materials and conversion costs for the form operation are $4.00 [$44,000 ÷ (16,000 – 5,000) units] and $12.00
[($60,000 + $72,000) ÷ (16,000 – 5,000) units], respectively. Accordingly, the unit cost of a standard model is $52.50
($12.00 + $24.50 + $4.00 + $12.00).
A standard model passes through the extrusion and form operations. Thus, its unit cost includes the
materials and conversion costs for both operations. The unit materials and conversion costs for the extrusion
operation are $12.00 ($192,000 ÷ 16,000 units) and $24.50 [($152,000 + $240,000) ÷ 16,000 units], respectively. The
unit materials and conversion costs for the form operation are $4.00 [$44,000 ÷ (16,000 – 5,000) units] and $12.00
[($60,000 + $72,000) ÷ (16,000 – 5,000) units], respectively. Accordingly, the unit cost of a standard model is $52.50
($12.00 + $24.50 + $4.00 + $12.00).

(Refers to Fact Pattern #33)


Gregg Industries’ unit cost of a deluxe model is $69.30

(Refers to Fact Pattern #33)


Gregg Industries’ total product cost of the executive model is

Total DM 50,000.00
Total Conv. $142,600.00
Total cost $192,600

An executive model passes through all four operations. Thus, its unit cost equals that of the deluxe model plus the
unit costs incurred in the finish operation. The unit cost of the deluxe model is $69.30. The unit materials and
conversion costs for the finish operation are $6.00 [$12,000 ÷ (16,000 – 14,000) units] and $21.00 [($18,000 + $24,000)
÷ (16,000 – 14,000) units], respectively. Consequently, the unit cost of the executive model is $96.30 ($69.30 + $6.00 +
$21.00), and the total product cost is $192,600 (2,000 units × $96.30).

Refers to Fact Pattern #33)


Assume that 1,000 units of Gregg Industries’ deluxe model remained in work-in-process at the end of the period and that
these units were 100% complete as to materials and 60% complete as to trim operation conversion. What is the balance of
work-in-process?
No. of units 1000 Total DM $19,000.00
Total Conv. $36,500.00
Total TRIM Conv. $9,000.00
Total cost $64,500.00
EUP Conv
Completed Deluxe mo 2000
partial Deluxe model 600
completed Executive 2000
EUP 4600
l variations of the same design,
company uses batch

r manufacturers.
legs are added; the standard model is sold after this operation.
ir edges are smoothed.

ect materials costs were as follows

Trim Finish
Materials Materials

$9,000
$6,000 $12,000
$15,000.00 $12,000.00

5,000.00 2,000.00
$3.00 $6.00

ation, the remaining During the trim operation, the arms are
into chair seats and added to the deluxe and executive Only the executive model enters
andard model is sold models and the chair edges are the finish operation where the
eration. smoothed. padding is added.

Trim Finish
Operation Operation
$30,000 $18,000
$39,000 $24,000
$69,000 $42,000

5,000.00 2,000.00
$13.80 $21.00
EUP 4600
cost 15.00

ost includes the


osts for the extrusion
nits], respectively. The
00) units] and $12.00
ndard model is $52.50
ost includes the
osts for the extrusion
nits], respectively. The
00) units] and $12.00
ndard model is $52.50

deluxe model plus the


nit materials and
1.00 [($18,000 + $24,000)
$96.30 ($69.30 + $6.00 +

end of the period and that


sion. What is the balance of
Butteco has the following cost components for 100,000 units of product for the year:
Direct materials $200,000
Direct labor 100,000
Manufacturing overhead 200,000
Selling and administrative expense 150,000
All costs are variable except for $100,000 of manufacturing overhead and $100,000 of selling and administrative
expenses. The total costs to produce and sell 110,000 units for the year are

per unit
units 100,000
Direct materials $200,000 $2.00
Direct labor $100,000 $1.00
V.Manufacturing overhead $100,000 $1.00
F.Manufacturing overhead $100,000
Selling and administrative expense $50,000 $0.50
F.Selling and administrative expense $100,000

V.Manuf. $4.00
F.manf $100,000
V.SGA $0.50
F.SGA $100,000

The total costs to produce and sell 110,000 units for the year are
110000 $695,000.00

Madtack Company’s beginning and ending inventories for the


month of November are
November. 1 November .30
Direct materials $67,000 $62,000
Work-in-process $145,000 $171,000
Finished goods $85,000 $78,000

Production data for the month of November follows:


Direct labor $200,000
Actual overhead $132,000
Direct materials purchased $163,000
Transportation in $4,000
Purchase returns and allowances $2,000

Madtack uses one overhead control account and charges


overhead to production at 70% of direct labor cost. The
company does not formally recognize over/underapplied
overhead until year-end.

$370,000

CoGs Total
Beg.RM 67,000
Add: Purchase 163,000
Less : Returns and discounts 2,000
Net Purchase 161,000
Add: Freight-in 4,000
RM Available for use 232,000
Less : End.RM 62,000
DM used in Production 170,000
DL 200,000 370,000
MOH 140,000
Total manufacturing costs for the period 510,000
Add: Beg. WIP 145,000
Less: End. WIP 171,000
Costs of goods Manufacturing 484,000
Add: Beg.Finished 85,000
Goods Available for sale 569,000
Less: End. Finished 78,000
Costs of goods Sold 491,000

Lucy Sportswear manufactures a specialty line of T-shirts using a job-order costing system. During March, the following
costs were incurred in completing job ICU2: direct materials, $13,700; direct labor, $4,800; administrative, $1,400; and
selling, $5,600. Overhead was applied at the rate of $25 per machine hour, and job ICU2 required 800 machine hours. If
job ICU2 resulted in 7,000 good shirts, the cost of goods sold per unit would be

CoGs Total
Beg.RM 13,700
Add: Purchase
Less : Returns and discounts
Net Purchase -
Add: Freight-in
RM Available for use 13,700
Less : End.RM
DM used in Production 13,700
DL 4,800
MOH 20,000
Total manufacturing costs for the period 38,500
Add: Beg. WIP -
Less: End. WIP -
Costs of goods Manufacturing 38,500
Add: Beg.Finished
Goods Available for sale 38,500
Less: End. Finished
Costs of goods Sold 38,500 7000

Zeta Company is preparing its annual profit plan. As part of


its analysis of the profitability of individual products, the
controller estimates the amount of overhead that should be
allocated to the individual product lines from the
information given as follows:

Wall Mirrors Specialty Windows


Units produced 25 25 25 25
Material moves per product line 5 15 5 15
Direct labor hours per unit 200 200 200 200
Budgeted materials handling costs $50,000 $50,000

Under a costing system that allocates overhead on the basis of direct labor hours, Zeta Company’s materials handling cos
allocated to one unit of wall mirrors would be
allocated to 25 25000
allocated to one unit $1,000

Under activity-based costing (ABC), Zeta’s materials handling costs allocated to one unit of wall mirrors would be

allocated to 25 12500
allocated to one unit $500

New-Rage Cosmetics has used a traditional cost accounting system to apply quality control costs uniformly to all products
at a rate of 14.5% of direct labor cost. Monthly direct labor cost for Satin Sheen makeup is $27,500. In an attempt to
distribute quality control costs more equitably, New-Rage is considering activity-based costing. The monthly data shown
in the chart below have been gathered for Satin Sheen makeup.
administrative
Madtack Company’s prime cost for November is

Madtack Company’s total manufacturing cost for November is

Madtack Company’s cost of goods transferred to finished goods inventory for November is

Madtack Company’s cost of goods sold for November is

March, the following


trative, $1,400; and
800 machine hours. If
5.5

Specialty Windows total


50
20
400

s materials handling costs

mirrors would be

niformly to all products


. In an attempt to
monthly data shown
1. Account for all units (physical flow of quantities).

Beginning WIP Total Units to Account for =


Started Units this period Finished or Transferred-out Goods
Total Units to Account for - Ending WIP
Spoilage (lost)

Step 2: compute Equivalent Units of Production


EUP Computation under FIFO (If materials are added as work in process (continually):
Material Conversion

Total units Completed


- Beginning WIP (regardless of % of completion)
Units Started and Completed this Period - -
+ Amount needed to complete Beginning WIP
+ Amount Completed on Ending WIP
EUP under FIFO - -

If materials are added at the beginning of a process :


Total Units Completed
+ Amount of materials Needed to Complete BWIP zero
.+ Amount of materials Added to Date on EWIP 100%
EUP for Materials -

If materials are added at the end of a process


Total Units Completed
+ Amount of materials Needed to Complete BWIP
+ Amount of materials Added to Date on EWIP zero
EUP for Materials -

EUP : Weighted Average


Ü  EUP under weighted average costing may be computed as follows:
Total Units Completed this period
Work to date on Ending WIP
EUP under weighted average -
-
d-out Goods
Dremmon Corporation uses a standard cost accounting system. Data for the last fiscal
year are as follows.
Units
Beginning inventory of finished goods 100
Production during the year 700
Sales 750
Ending inventory of finished goods 50

Per Unit
Product selling price $200 $200
Standard variable manufacturing cost 90 $90
Standard fixed manufacturing cost 20* $20
$110
Budgeted selling and administrative costs (all fixed) $45,000 45,000
*Denominator level of activity is 750 units for the year. 750

Absorb.
sales rev $150,000
Cost of Goods Sold
Beginning inventory $11,000
Variable manufacturing costs $63,000
Fixed manufacturing costs $14,000
Cost for goods available for sale $88,000
deduct ending inventory $5,500
Total COGS(at standard costs) $82,500
Adjustment for manufacturing variances $1,000
Total COGS $83,500
Gross Margin $66,500
Operating cost $45,000
Operating income $21,500

There were no price, efficiency, or spending variances for the year, and actual selling and
administrative expenses equaled the budget amount. Any volume variance is written off
to cost of goods sold in the year incurred. There are no work-in-process inventories.
Assuming that Dremmon used absorption costing, the amount of operating income earned
in the last fiscal year was $21,500

Bethany Company has just completed the first month of producing a new product but has
not yet shipped any of this product. The product incurred variable manufacturing costs of
$5,000,000, fixed manufacturing costs of $2,000,000, variable marketing costs of
$1,000,000, and fixed marketing costs of $3,000,000.

variable manufacturing costs $5,000,000


fixed manufacturing costs $2,000,000
variable marketing costs $1,000,000
fixed marketing costs $3,000,000

If Bethany uses the variable cost method to value inventory, the inventory value of the
new product would be

$5,000,000

Consider the following situation for Weisman Corporation for the prior year.
• The company produced 1,000 units and sold 900 units, both as budgeted.
• There were no beginning or ending work-in-process inventories and no beginning
finished goods inventory.
• Budgeted and actual fixed costs were equal, all variable manufacturing costs are
affected by volume of production only, and all variable selling costs are affected
by sales volume only.
• Budgeted per unit revenues and costs were as follows.

Per Unit Total


Sales price $100 $100
Direct materials 30 $30
Direct labor 20 $20
Variable manufacturing costs 10 $10 $60
Fixed manufacturing costs 5 $5
Variable selling costs 12 $12
Fixed selling costs ($3,600 total) 4 $4 $3,600
Fixed administrative costs ($1,800 total) 2 $2 $1,800

The operating income for Weisman for the prior year using absorption costing was

Absorb.
sales rev $90,000
Cost of Goods Sold
Beginning inventory
Variable manufacturing costs $60,000
Fixed manufacturing costs $5,000
Cost for goods available for sale $65,000
deduct ending inventory $6,500
Total COGS(at standard costs) $58,500
Adjustment for manufacturing variances $0
Total COGS $58,500
Gross Margin $31,500
Operating cost $16,200
Operating income $15,300

Mill Corporation had the following unit costs for the recently concluded calendar year.

Variable Fixed
Manufacturing $8 $3
Nonmanufacturing $2 $6

Inventory for Mill’s sole product totaled 6,000 units on January 1 and 5,200 units on
December 31. When compared to variable costing income, Mill’s absorption costing
income is

$2,400 Lower

During the month of May, Robinson Corporation sold 1,000 units. The cost per unit for
May was as follows.

Cost Per Unit


Direct materials $ 5.50 $5.5
Direct labor 3.00 $3.0
Variable manufacturing overhead 1.00 $1.0
Fixed manufacturing overhead 1.50 $1.5
Variable administrative costs .50 $0.5
Fixed administrative costs 3.50 $3.5

May’s income using absorption costing was $9,500. The income for May, if variable
costing had been used, would have been $9,125. The number of units Robinson
produced during May was 1,250
[Fact Pattern #36] The planned per-unit cost figures sh
Valyn Corporation employs an absorption costing system for
internal reporting purposes; however, the company is
considering using variable costing. Data regarding Valyn’s Direct materials
planned and actual operations for the calendar year are Direct labor
presented below. Variable manufacturing
overhead
Fixed manufacturing
overhead
Variable selling expenses
Fixed selling expenses
Planned Actual Variable administrative
Activity Activity expenses
Beginning finished goods Fixed administrative
inventory in units 35,000 35,000 expenses
Sales in units 140,000 125,000 Total
Production in units 140,000 130,000
endning finished goods 40,000
inventory in units

Valyn uses a predetermined manufacturing overhead rate for applying manufacturing overhead to its product; thus, a com
purposes. Any over- or underapplied manufacturing overhead is closed to the cost of goods sold account at the end of the

The beginning finished goods inventory for absorption costing purposes was valued at the previous year’s planned unit
planned unit manufacturing cost. There are no work-in-process inventories at either the beginning or the end of the year. T
planned and actual unit selling price for the current year was $ 70.00 per unit.

(Refers to Fact Pattern #36)


The value of Valyn Corporation’s actual ending finished goods inventory on the absorption costing basis was

(Refers to Fact Pattern #36)


Valyn Corporation’s total fixed costs expensed this year on the absorption costing basis were

Fixed selling & Admin

Beginning finished goods

Sales in units
Production in units

endning finished goods

Budgeted FMOH
Allocated MOH

Given that 125,000 of those units were sold, cost of


goods sold was debited for $625,000 of fixed overhead (125,000 units × $5)

At year-end, the underapplied overhead was also added to cost of goods sold.
Because production was expected to be 140,000 units, the overhead application rate for
the $700,000 of planned fixed manufacturing overhead was $5 per unit.

Only 130,000 units were manufactured. Hence,


$650,000 (130,000 units × $5) of overhead was applied to units in process.

Because inventory increased from 35,000 to


40,000 units (35,000 BI + 130,000 produced – 125,000 sold), $25,000 (5,000-unit increase × $5) of the applied fixed
manufacturing overhead for the period was inventoried, not expensed

Actual overhead was $715,000, so the


underapplied overhead was $65,000 ($715,000 – $650,000).

This amount was charged to cost of goods sold at year-end.


The total of the fixed costs expensed was therefore $2,095,000 ($980,000 selling expenses + $425,000 administrative
expenses + $625,000 standard manufacturing overhead costs of units sold + $65,000 underapplied overhead).

sales rev
Cost of Goods Sold
Beginning inventory
Variable manufacturing costs
Fixed manufacturing costs
FMOH=sold*5
Cost for goods available for sale
deduct ending inventory
Total COGS(at standard costs)
FMOH=sold*5

Adjustment for manufacturing variances


Total COGS
Gross Margin
Operating cost
Operating income
(Refers to Fact Pattern #36)
The value of Valyn Corporation’s actual ending finished goods inventory on the variable costing basis was

Using variable costing, the unit cost of ending inventory is $25 ($12 direct materials + $9
direct labor + $4 variable overhead). Given beginning inventory of 35,000 units, the ending inventory equals
40,000 units (35,000 BI + 130,000 produced – 125,000 sold). Thus, ending inventory was $1,000,000 (40,000
units × $25).

(Refers to Fact Pattern #36)


Valyn Corporation’s absorption costing operating income was

(Refers to Fact Pattern #36)


Valyn Corporation’s actual manufacturing contribution margin calculated on the variable costing basis was
$ 8,750,000.00
VM 3,125,000

8,750,000.00

(Refers to Fact Pattern #36)


The total variable cost currently expensed by Valyn Corporation on the variable costing basis was
4,375,000

(Refers to Fact Pattern #36)


The difference between Valyn Corporation’s operating income calculated on the absorption costing basis and calculated
on the variable costing basis was 25,000

Fact Pattern #37]


Osawa, Inc. planned and actually manufactured 200,000 units of its single product in its first year of operations. Variable
manufacturing costs were $30 per unit of product. Planned and actual fixed manufacturing costs were $600,000, and sellin
administrative costs totaled $400,000. Osawa sold 120,000 units of product at a selling price of $40 per unit.

(Refers to Fact Pattern #37)


Osawa’s operating income using absorption (full) costing is

sales rev 4,800,000


Cost of Goods Sold
Beginning inventory
Variable manufacturing costs 6,000,000
Fixed manufacturing costs 600,000
Cost for goods available for sale 6,600,000
deduct ending inventory 2,640,000
Total COGS(at standard costs) 3,960,000
Adjustment for manufacturing variances
Total COGS 3,960,000
Gross Margin 840,000
Operating cost 400,000
Operating income 440,000
Budgeted FOH
$15,000

FOH allocated PVV


VC $14,000 $1,000 U
$150,000

$9,000
$63,000
$15,000
$87,000 We did not use the appropriate format
in solving the VC method
$4,500
$82,500

$67,500
$45,000
$22,500

Assuming that Dremmon used V costing, the amount of operating income earned
$22,500
production 1000
sales 900
end.inv. 100

Budgeted FOH Allocated PVV


$5,000 $5,000 $0

VC
$90,000

$60,000
$5,000
$65,000
We did not use the appropriate format in
solving the VC method
We did not use the appropriate format in
6000 solving the VC method
$59,000
$0
$59,000
$31,000
$16,200
$14,800

BI 6000
E.Inv 5200
Diff 800

1000

$9,500.0
$9,125.0
Diff $375.0 250
lanned per-unit cost figures shown in the next schedule were based on the estimated production and sale of 140,000 units for the year
Planned Costs Incurred Costs
Per Unit Total
12 1,680,000 1,560,000 12
9 1,260,000 1,170,000 9
ble manufacturing 4 560,000 4 520,000

manufacturing
5 700,000 715,000
ble selling expenses 8 1,120,000 1,000,000
selling expenses 7 980,000 980,000
ble administrative
2 280,000 250,000
administrative
3 420,000 425,000
$ 50 $ 7,000,000 $ 6,620,000

ead to its product; thus, a combined manufacturing overhead rate of $ 9.00 per unit was employed for absorp
sold account at the end of the reporting year.

evious year’s planned unit


nning or the end of the year. The

costing basis was 1,200,000.00

1,405,000
The beginning finished goods inventory included 35,000 units, each of
which had absorbed $5 of fixed manufacturing overhead.
175,000

Given that 125,000 of those units were sold, cost of goods sold was
625,000 debited for $625,000 of fixed overhead (125,000 units × $5).
Given that 125,000 of those units were sold, cost of goods sold was
debited for $625,000 of fixed overhead (125,000 units × $5).

650,000 Each unit produced during the year also absorbed $5 of fixed
manufacturing overhead

200,000

700,000
650,000
715,000 50,000 u pvv
15,000 u FBV
625,000

700,000

650,000 Applied OH = Actual production X Budgeted Rate

$5) of the applied fixed

25,000

715,000 65,000

$425,000 administrative
pplied overhead).
2,095,000

FMOH=sold*5+underapplied
FMOH=sold*5+underapplied

ng basis was 1,000,000

nventory equals
000,000 (40,000

sting basis was

costing basis and calculated

year of operations. Variable 200,000


osts were $600,000, and selling and $ 30.00 600,000 3
of $40 per unit. 400,000 120,000 40
80,000

(Refers to Fact Pattern #37)


Osawa’s operating income for the year using variable costing is
VC
4,800,000

6,000,000
600,000
6,600,000
2,400,000
4,200,000

4,200,000
600,000
400,000
200,000
40,000 units for the year.

was employed for absorption costing


Budgeted production in units
Actual production in units
Budgeted Allocation base (ex.hours)
Actual allocation base (ex.hours) 430,000.00
the standard allocation base per unit
the Actual allocation base per unit
Total Actual OH $ 1,600,000.00
Total Actual VOH
Total Actual FOH
Total Budgeted OH
Total Budgeted VOH
Total Budgeted FOH $ 1,500,000.00
Total Applied OH $ 1,400,000.00
Applied VOH $ 200,000.00
Applied FOH $ 1,200,000.00
Budgeted VOH rate $ 0.50
Budgeted FOH rate $ 3.00

1)
Total Budgeted VOH = Budgeted Rate x Budgeted Allocation Base

Budgeted VOH Rate Budgeted Allocation Base .=Total Budgeted VOH


0

Total Actual VOH = Actual Rate x Actual Allocation Base .= Total actual VOH
0

2)
Allocated VOH( the same in the flexible budget)
Budgeted Input Allowed for actual output X Budgeted VOH Rate
0.5 0
Budgeted Input Allowed for actual output = Budgeted input (allocation base) per unit X Actual units

3) Actual Inputs X budgeted Rate


430000 0.5 $ 215,000.00

4) Actual Inputs X Actual Rate .=Actual costs incurred


0
1)
Total Budgeted FOH = Budgeted Rate x Budgeted Allocation Base

BudgetedFOH Rate Budgeted Allocation Base .=Total Budgeted FOH


0

Total Actual FOH = Actual Rate x Actual Allocation Base .= Total actual FOH
0

2)
Allocated FOH
Budgeted Input Allowed for actual output X Budgeted FOH Rate
0
Budgeted Input Allowed for actual output = Budgeted input (allocation base) per unit X Actual units

flexible budget same as static budget


0

4) Actual Inputs X Actual Rate .=Actual FOH costs incurred


0
246. Harper Company’s perform
month.
Actual total overhead $1,600,00
Budgeted fixed overhead 1,500,
246. B b. $115,000 favorable. Applied fixed overhead at $3 pe
Sp Applied variable overhead at $.5
Actual VOH Actua inpxbudgeted rate Actual labor hours 430,000
215000 Harper’s total overhead spendin
sp
Actual FOH static
$ 1,500,000.00

$ 1,600,000.00 $ 1,715,000.00 $ 115,000.00

.=Total Budgeted VOH

.= Total actual VOH

X Actual units
0
.=Total Budgeted FOH

.= Total actual FOH

X Actual units
0
246. Harper Company’s performance report indicated the following information for the past

Actual total overhead $1,600,000


Budgeted fixed overhead 1,500,000
Applied fixed overhead at $3 per labor hour 1,200,000
Applied variable overhead at $.50 per labor hour 200,000
Actual labor hours 430,000
Harper’s total overhead spending variance for the month was
Budgeted production in units
Actual production in units
Budgeted Allocation base (ex.hours)
Actual allocation base (ex.hours)
the standard allocation base per unit
the Actual allocation base per unit
Total Actual OH
Total Actual VOH
Total Actual FOH

1)
Total Budgeted VOH = Budgeted Rate x Budgeted Allocation Base

Budgeted VOH Rate Budgeted Allocation Base .=Total Budgeted VOH


0

Total Actual VOH = Actual Rate x Actual Allocation Base .= Total actual VOH
0

2)
Allocated VOH( the same in the flexible budget)
Budgeted Input Allowed for actual output X Budgeted VOH Rate
0
Budgeted Input Allowed for actual output = Budgeted input (allocation base) per unit X Actual units

3) Actual Inputs X budgeted Rate


0

4) Actual Inputs X Actual Rate .=Actual costs incurred


0

1)
Total Budgeted FOH = Budgeted Rate x Budgeted Allocation Base

BudgetedFOH Rate Budgeted Allocation Base .=Total Budgeted FOH


0

Total Actual FOH = Actual Rate x Actual Allocation Base .= Total actual FOH
0
2)
Allocated FOH
Budgeted Input Allowed for actual output X Budgeted FOH Rate
0
Budgeted Input Allowed for actual output = Budgeted input (allocation base) per unit X Actual units

flexible budget same as static budget


0

4) Actual Inputs X Actual Rate .=Actual FOH costs incurred


0
0
0
Job 298
Actual DM 4606 2003 Actual DL hours 27,000.00
Actual DL 1579 Actual MOH 1,215,000.00
6185 Actual rate $ 45.00
Actual DL hours for Job 298 88
Actual Allocated OH 88 X $45 $ 3,960.00 2003 Budgeted DL hours 28,000.00
Budgeted MOH 1,120,000.00
Total Manf. Cost $ 10,145.00 Budgeted rate $ 40.00
Actual Price $ 15,000.00
Gross Margin $ 4,855.00
Gross Margin % 32%
Applied MOH 1,080,000.00
Net (Dr) Balance 135,000.00 12.05%
Job 298Applied MOH 88X$40 $ 3,520.00
Total Manf. Cost(normal costing) $ 9,705.00 Dr CoGS 135,000.00
DR Applied 1,080,000.00
Diff $ 440.00 CR MOH control 1,215,000.00

DR Cogs 2,025.00 2%
DR WIP 3,915.00 3%
DR FG 129,060.00 96%
Dr Applied OH 1,080,000.00 135,000.00
CR MOH control 1,215,000.00
Comparative Balance Sheet (A & B) 31-12-2000
Sin SAD
Cash
Marketable sec.10%
Acc.Rec
Inventory
Total CA 85,000 85,000
Net P,P&E 150,000 150,000
Total Assets 235,000 235,000

S.T Liability 20,000 20,000


Acc.Pay 20,000 50,000
CL 40,000 70,000
L.T.loans 45,000 15,000
Equity 150,000 150,000
235,000 235,000

Requirment of inventory finance 8 Mnth 30,000 30,000

L.T. loan 10% (7 Years) 30,000


S.T.Loan 8% (8 Mnth) 30,000
Interest on Loans 3,000 1,600

Net Operating Income & net Income


Net Operating Income 53,000 53,000
Interest on loans (3,000) (1,600)
50,000 51,400
Tax 50% (25,000) (25,700)
Net Income 25,000 25,700

ROI 10.6% 10.9%

Measure of riskeness
Measure of Liquidity risk
Current Ratio 2.1 1.2
WC 45,000 15,000
Comparative Balance Sheet (A & B) 31-12-2000
A B
Cash 10,000 10,000
Marketable sec.10% - 15,000
Acc.Rec 20,000 20,000
Inventory 30,000 30,000
Total CA 60,000 75,000
Net P,P&E 125,000 125,000
Total Assets 185,000 200,000

CL 45,000 45,000
L.T.loans 40,000 40,000
Equity 100,000 115,000
185,000 200,000

Measure of riskeness
Measure of Liquidity risk
Current Ratio 1.3 1.7
Quick Ratio 0.67 1.0
WC 15,000 30,000

Net Income 20,000 20,750


% tax 50% 50%
Interst on M.S 10% - 1,500
Interst on M.S 10%(net of Tax) - 750

ROI 10.8% 10.4%

Company (A )exposed to liquidity risk with


high % than (B)
INV 19000

Rev 30000 RI=NI - Inv*.15


RI 2000
Imputed charge 15%

Inv*.15 2850

2000=NI -2850
NI=4850 4850
NI = Rev- Cost
4850=30000-cost
cost=30000-4850 25150
Tucariz Company processes Duo into two joint products, Big and Mini. Duo is
purchased in 1,000 gallon drums for $2,000. Processing costs are $3,000 to process the
1,000 gallons of Duo into 800 gallons of Big and 200 gallons of Mini. The selling price
is $9 per gallon for Big and $4 per gallon for Mini. Big can be processed further into 600
gallons of Giant if $1,000 of additional processing costs are incurred. Giant can be sold
for $17 per gallon. If the net-realizable-value method were used to allocate costs to the
joint products, the total cost of producing Giant would be

Duo is purchased in 1,000 gallon drums for $2,000.


1000 $2,000
Processing costs are $3,000 to process the 1,000 gallons of Duo into 800 gallons of Big and 200 gallons of Mini.
$3,000
Joint costs $5,000
split price split % further gal
Big 800 $9 $7,200 90% 600
Mini 200 $4 $800 10%
$8,000

If the net-realizable-value method were used to allocate costs to the joint products, the total cost of producing Giant would

1)The first step we have to allocate the portion of the joint costs with NRV % $4,600
2) then adding the $1000 additional costs concerning Giant $1,000
$5,600

Fitzpatrick Corporation uses a joint manufacturing process in the production of two


products, Gummo and Xylo. Each batch in the joint manufacturing process yields 5,000
pounds of an intermediate material, Valdene, at a cost of $20,000. Each batch of Gummo
uses 60% of the Valdene and incurs $10,000 of separate costs. The resulting 3,000
pounds of Gummo sells for $10 per pound. The remaining Valdene is used in the
production of Xylo which incurs $12,000 of separable costs per batch. Each batch of
Xylo yields 2,000 pounds and sells for $12 per pound. Fitzpatrick uses the net realizable
value method to allocate the joint material costs. The company is debating whether or
not to process Xylo further into a new product, Zinten, which would incur an additional
$4,000 in costs and sell for $15 per pound. If Zinten is produced, income would increase
by
a. $2,000. 124. A
b. $5,760.
c. $14,000.
d. $26,000.
Quant. Price Rev
if we have a product that can be sold with 2000 $12 $24,000

but we can process it further to bring a higher revenue


Quant. Price Rev
2000 $15 $30,000

with additional cost $4,000

so we have incremental revenue = $6,000

thus the increase in income would be $2,000

28. Tucariz Company processes Duo into two joints products, Big and Mini. Duo is purchased in 1,000-gallon drums for $2,0
Processing costs are $3,000 to process the 1,000 gallons of Duo into 800 gallons of Big and 200 gallons of Mini.
The selling price is $9 per gallon for Big and $4 per gallon for Mini. If the physical measure method is used to allocate joint
llons of Mini.

price rev costs NRV NRV %


$17 $10,200 $1,000 $9,200 92%
$800 8%
$10,000 100%
121. A $5,600.
of producing Giant would be
28. Key = b
000-gallon drums for $2,000. ($3,000 + $2,000) x 200/1,000 = $1,000
llons of Mini.
d is used to allocate joint costs to the final products, the total cost assigned to produce Mini is
103. Gardener Company currently is using its full capacity of 25,000 machine hours to
manufacture product XR-2000.
LJB Corporation placed an order with Gardener for the manufacture of 1,000 units of KT-6500.
LJB would normally manufacture this component.
However, due to a fire at its plant, LJB needs to purchase these units to continue manufacturing other products. This is a o

The following reflects unit cost data, and selling prices.

KT-6500

Material $27.0
Direct labor $12.0
Variable overhead $6.0
Fixed overhead $48.0
Variable selling & administrative $5.0
Fixed selling & administrative $12.0
Normal selling price $125.0
Machine hours required 3.00

What is the minimum unit price that Gardener should charge LJB to manufacture 1,000
units of KT-6500?

CM $75.0
CM per machine hour $25.0

the minimum price must equal at least the VC to produce KT-6500 plus any foregone CM by not producing XR-2000

VC to produce KT-6500 $50.0

Total hours that will be used to prouce KT-6500 instead of producing XR 3,000.00
CM that will be lost from not prducing XR $46,500.0
CM per unit that will be lost from not prducing XR $46.5

The minimum unit price that Gardener should charge LJB to manufacture 1,000 units of KT-6500
$96.50

106. Following are the operating results of the two segments of Parklin Corporation.

Segment A
Sales $10,000.0
Variable costs of goods sold $4,000.0
Fixed costs of goods sold $1,500.0
Gross margin $4,500.0
Variable selling and administrative $2,000.0
Fixed selling and administrative $1,500.0
Operating income (loss) $1,000.0

Variable costs of goods sold are directly related to the operating segments. Fixed costs of
goods sold are allocated to each segment based on the number of employees. Fixed
selling and administrative expenses are allocated equally. If Segment B is eliminated,
$1,500 of fixed costs of goods sold would be eliminated. Assuming Segment B is closed,
the effect on operating income would be

Segment A

Sales $10,000.0
Variable costs of goods sold $4,000.0
Fixed costs of goods sold $1,500.0
Gross margin $4,500.0
Variable selling and administrative $2,000.0
Fixed selling and administrative $1,500.0
Operating income (loss) $1,000.0

Assuming Segment B is closed, the effect on operating income would be

107. Edwards Products has just developed a new product with a manufacturing cost of $30.

The Marketing Director has identified three marketing approaches for this new product.

Approach X Set a selling price of $36 and have the firm’s sales staff sell the
product at a 10% commission with no advertising program.
Estimated annual sales would be 10,000 units.
Approach Y Set a selling price of $38, have the firm’s sales staff sell the
product at a 10% commission, and back them up with a $30,000
advertising program. Estimated annual sales would be 12,000
units.
Approach Z Rely on wholesalers to handle the product. Edwards would sell the
new product to the wholesalers at $32 per unit and incur no selling
expenses. Estimated annual sales would be 14,000 units.
Approach X

selling price $36.0


Q 10,000.00
Rev $360,000.0
manufacturing cost $300,000.0
V selling $36,000.0
Fixed selling
net profit $24,000.0

Rank the three alternatives in order of net profit, from highest net profit to lowest.
107. C Z, X, Y.

108. Parker Manufacturing is analyzing the market potential for its specialty turbines. Parker
developed its pricing and cost structures for their specialty turbines over various relevant
ranges. The pricing and cost data for each relevant range are presented below.

Units produced and sold 1-5


Total fixed costs $200,000.0
Unit variable cost $50,000.0
Unit selling price $100,000.0

Which one of the following production/sales levels would produce the highest operating
income for Parker?

a. 8 units. Rev
b. 10 units. VC
c. 14 units. FC
d. 17 units. OI
8 10 14 17
Rev $800,000.0 $1,000,000.0 $1,400,000.0 $1,700,000.0
VC $400,000.0 $500,000.0 $630,000.0 $765,000.0
FC $400,000.0 $400,000.0 $600,000.0 $800,000.0
OI $0.0 $100,000.0 $170,000.0 $135,000.0
109. Elgers Company produces valves for the plumbing industry. Elgers’ per unit sales price and variable costs are as follow

Sales price $12 $12.0


Variable costs 8 $8.0
Elgers’ practical plant capacity is 40,000 units. 40,000.00
Elgers’ total fixed costs aggregate and. $48,000.0
it has a 40% effective tax rate 40%

The maximum net profit that Elger can earn is

Rev 480000
VC 320000
FC $48,000.0
OI $112,000.0
Tax $44,800.0
Net I $67,200.0

110. Dayton Corporation manufactures pipe elbows for the plumbing industry. Dayton’s per
unit sales price and variable costs are as follows.

Sales price $10 $10.0


Variable costs 7 $7.0
Dayton’s practical plant capacity is 35,000 units. 35,000.00
Dayton’s total fixed costs amount to $42,000, $42,000.0
and the company has a 50% effective tax rate. 50%

If Dayton produced and sold 30,000 units, net income would be 30,000.00

Rev $300,000.0
VC $210,000.0
FC $42,000.0
OI $48,000.0
Tax $24,000.0
Net I $24,000.0

111. Raymund Inc., a bearings manufacturer, has the capacity to produce 7,000 bearings per
month. The company is planning to replace a portion of its labor intensive production
process with a highly automated process, which would increase Raymund’s fixed
manufacturing costs by $30,000 per month and reduce its variable costs by $5 per uni
Raymund’s Income Statement for an average month is as follows.

Sales (5,000 units at $20 per unit) $100,000 5,000.00 $20.0


Variable manufacturing costs $50,000 $10.0 $50,000.0
Variable selling costs 15,000 65,000 $15,000.0
Contribution margin 35,000

Fixed manufacturing costs 16,000 $16,000.0


Fixed selling costs 4,000 20,000 $4,000.0
Operating income $ 15,000

If Raymund installs the automated process, the company’s monthly operating income
would be

Sales (5,000 units at $20 per unit) $100,000 5,000.00 $20.0


Variable manufacturing costs $50,000 $5.0 $25,000.0
Variable selling costs 15,000 65,000 $15,000.0
Contribution margin 35,000

Fixed manufacturing costs 16,000 $46,000.0


Fixed selling costs 4,000 20,000 $4,000.0
Operating income $ 15,000

112. Refrigerator Company manufactures ice-makers for installation in refrigerators. The costs per unit, for 20,000 units of

Direct materials $ 7 $7.0


Direct labor 12 $12.0
Variable overhead 5 $5.0
Fixed overhead 10 $10.0 $200,000.0
Total costs $34 $34.0

Cool Compartments Inc. has offered to sell 20,000 ice-makers to Refrigerator Company for $28 per unit.
If Refrigerator accepts Cool Compartments’ offer the plant would be idled and fixed overhead amounting to $6 per unit co
The total relevant costs associated with the manufacture of ice-makers amount to
relevant
Direct materials $ 7 $7.0
Direct labor 12 $12.0
Variable overhead 5 $5.0
Fixed overhead 10 $6.0
Total costs $34 $30.0 $600,000.0
113. Phillips and Company produces educational software. Its current unit cost, based upon an anticipated volume of 150,0

Selling price $150 $150.0


Variable costs 60 $60.0
Contribution margin 90 $90.0
Fixed costs 60 $60.0 $9,000,000.0
Operating income 30 $30.0

Sales for the coming year are estimated at 175,000 units, which is within the relevant range of Phillip’s cost structure.
Cost management initiatives are expected to yield a 20% reduction in variable costs and a reduction of $750,000 in fixed c
.

Selling price $150 $150.0 $26,250,000.0


Variable costs 60 $48.0 $8,400,000.0
Contribution margin 90 $102.0 $17,850,000.0
Fixed costs 60 $47.1 $8,250,000.0 $8,250,000.0
Operating income 30 $54.9 $9,600,000.0

Phillip’s cost structure for the coming year will include a: variable cost ratio of 32% and operating income of $9,6

114. Sunshine Corporation is considering the purchase of a new machine for $800,000.
The machine is capable of producing 1.6 million units of product over its useful life.
The manufacturer’s engineering specifications state that the machine-related cost of producing each unit of product should
Sunshine’s total anticipated demand over the asset’s useful life is 1.2 million units.
The average cost of materials and labor for each unit is $.40.

In considering whether to buy the new machine, would you recommend that Sunshine use the manufacturer’s engineer
b. No, the machine-related cost of producing each unit is $.67.

manufacturer’s engineering specification


demand

115. Cervine Corporation makes two types of motors for use in various products. Operating
data and unit cost information for its products are presented below.
Product A
Annual unit capacity 10,000 20,000 10,000.00
Annual unit demand 10,000 20,000 10,000.00
Selling price $100 $80 $100.0
Variable manufacturing cost 53 45 $53.0
Fixed manufacturing cost 10 10 $10.0
Variable selling & administrative 10 11 $10.0
Fixed selling & administrative 5 4 $5.0
Fixed other administrative 2 0 $2.0
Unit operating profit $ 20 $10 $20.0
Machine hours per unit 2.0 1.5 2

Cervine has 40,000 productive machine hours available. 40,000.00

The relevant contribution margins, per machine hour for each product, to be utilized in making a decision on product prior

Product A
CM $37.0
CM per machine hours $18.5

116. Two months ago, Hickory Corporation purchased 4,500 pounds of Kaylene at a cost of$15,300.
The market for this product has become very strong, with the price jumping to $4.05 per pound.
Because of the demand, Hickory can buy or sell Kaylene at this price.
Hickory recently received a special order inquiry that would require the use of 4,200 pounds of Kaylene.
In deciding whether to accept the order, management must evaluate a number of decision factors.
Without regard to income taxes, which one of the
following combination of factors correctly depicts relevant and irrelevant decision
factors, respectively?

Relevant Decision Factor


b. Market price of $4.05 per lb.

117. Reynolds Inc. manufactures several different products, including a premium lawn fertilizer and weed killer that is popu
Reynolds is currently operating at less than full capacity because of market saturation for lawn fertilizer.
Sales and cost data for a 40-pound bag of Reynolds lawn fertilizer is as follows.

Selling price
Production cost
Materials and labor $12.3
Variable overhead $3.8
Allocated fixed overhead $4.0
Income (loss) per bag $(1.50)

On the basis of this information, which one of the following alternatives should be
recommended to Reynolds management?

c. Continue to produce and market this product., since it is has a $2.5 CM Reynolds Inc. manufactures several different prod

CM

119. Capital Company has decided to discontinue a product produced on a machine purchased four years ago at a cost of $
The machine has a current book value of $30,000.
Due to technologically improved machinery now available in the marketplace the existing machine has no current salvage v
The company is reviewing the various aspects involved in the production of a new product.
The engineering staff advised that the existing machine can be used to produce the new product.
Other costs involved in the production of the new product will be materials of $20,000 and labor priced at $5,000.
$20,000.00
Ignoring income taxes, the costs relevant to the decision to produce or not to produce the
new product would be $25,000.00

123. Allred Company sells its single product for $30 per unit. $30.00
The contribution margin ratio is 45%, 45%
and fixed costs are $10,000 per month.
Allred has an effective income tax rate of 40%
. If Allred sells 1,000 units in the current month, 1000
Allred’s variable expenses would be

VC ratio 55%
sales revenue $30,000.00
VC $16,500.00

124. Phillips & Company produces educational software. Its unit cost structure, based upon
an anticipated production volume of 150,000 units, is as follows. 150000

Sales price $160 $160


Variable costs 60 $60
Fixed costs 55 $55

The marketing department has estimated sales for the coming year at 175,000 units,which is within the relevant range of P
Phillip’s break-even volume (in units) and anticipated operating income for the coming year would amount to

total FC $8,250,000
CM $100

Rev $28,000,000
VC $10,500,000
CM $17,500,000
FC $8,250,000
OI $9,250,000

Break=FC/CM 82,500.00 units

126. Jeffries Company sells its single product for $30 per unit. $30
The contribution margin ratio is 45%, and. 45%
fixed costs are $10,000 per month $10,000
Sales were 3,000 units in April and 4,000 units in May. 3,000 4,000
How much greater is the May income than the April income?
April May
Rev $90,000 $120,000
VC 55% $16.50 $49,500 $66,000
CM $40,500 $54,000
FC $10,000 $10,000
OI $30,500 $44,000

127. Cervine Corporation makes two types of motors for use in various products. Operating
data and unit cost information for its products are presented below.
Product A

Annual unit capacity 10,000 20,000 10,000.00


Annual unit demand 10,000 20,000 10,000.00
Selling price $100 $80 $100.0
Variable manufacturing cost 53 45 $53.0
Fixed manufacturing cost 10 10 $10.0
Variable selling & administrative 10 11 $10.0
Fixed selling & administrative 5 4 $5.0
Fixed other administrative 2 0 $2.0
Unit operating profit $ 20 $10 $20.0
Machine hours per unit 2.0 1.5 2.0

Cervine has 40,000 productive machine hours available. 40,000.00

. What is the maximum total contribution margin that Cervine can generate in the coming year?

Product A
CM $37.00
CM per machine hours $18.50

first produce as much we can from product A 10,000


Total Machine hours 20,000
Total CM 370,000

the remaining hours to produce product B


total unit that can be produced from prdoct B
Total CM

Total CM

128. Lazar Industries produces two products, Crates and Trunks. Per unit selling prices, costs,
and resource utilization for these products are as follows.

Crates
Selling price $20 $30 20
Direct material costs $ 5 $ 5 5
Direct labor costs 8 10 8
Variable overhead costs 3 5 3
Variable selling costs 1 2 1
Machine hours per unit 2 4 2

Production of Crates and Trunks involves joint processes and use of the same facilities.
The total fixed factory overhead cost is $2,000,000 and total fixed selling and administrative costs are $840,000.
Production and sales are scheduled for 500,000 Crates and 700,000 Trunks.
Lazar has a normal capacity to produce a total of 2,000,000 units in
any combination of Crates and Trunks, and maintains no direct materials, work-inprocess,
or finished goods inventory.

Due to plant renovations Lazar Industries will be limited to 1,000,000 machine hours.
What is the maximum amount of contribution margin Lazar can generate during the
renovation period?
Crates
CM 3.00
CM per machine hours 1.50

first produce as much we can from Trunks


with Total Machine hours
Total CM

129. For the year just ended, Silverstone Company’s sales revenue was $450,000.
Silverstone’s fixed costs were $120,000 and it
s variable costs amounted to $270,000.
For the current year sales are forecasted at $500,000.
If the fixed costs do not change, Silverstone’s profits this year will be

For the current year sales are forecasted at $500,000.


CM will be (1-VC ratio 60%) 200000
OI $80,000

130. Breeze Company has a contribution margin of $4,000 and fixed costs of $1,000. If the
total contribution margin increases by $1,000, operating profit would

131. Wilkinson Company sells its single product for $30 per unit. The contribution margin
ratio is 45% and Wilkinson has fixed costs of $10,000 per month. If 3,000 units are sold
in the current month, Wilkinson’s income would be
133. Starlight Theater stages a number of summer musicals at its theater in northern Ohio.
Preliminary planning has just begun for the upcoming season, and Starlight has
developed the following estimated data.

Average
Number of Attendance per Ticket
Production Performances Performance Price
Mr. Wonderful 12 3500 $18.00
That’s Life 20 3000 $15.00
All That Jazz 12 4000 $20.00

1 Represent payments to production companies and are based on tickets sold.


2 Costs directly associated with the entire run of each production for
costumes, sets, and artist fees.

Starlight will also incur $565,000 of common fixed operating charges (administrative
overhead, facility costs, and advertising) for the entire season, and is subject to a 30%
income tax rate.

If Starlight’s schedule of musicals is held, as planned, how many patrons would have to
attend for Starlight to break even during the summer season?

FC $1,295,000.00
CM $16.20
Breakeven $79,938

134. Carson Inc. manufactures only one product and is preparing its budget for next year based on the following informatio

Selling price per unit $100.00


Variable costs per unit $75.00
Fixed costs $250,000.00
Effective tax rate 35%

If Carson wants to achieve a net income of $1.3 million next year, its sales must be

CM $25.00
EBT $2,000,000.00
Numerator to be used $2,250,000.00
units to achieve 1.3 m 90,000.00
135. MetalCraft produces three inexpensive socket wrench sets that are popular with do-ityourselfers.
Budgeted information for the upcoming year is as follows.
Estimated
Model Selling Price Variable Cost Sales Volume
No. 109 10 5.5 30000
No. 145 15 8 75000
No. 153 20 14 45000
150000
Total fixed costs for the socket wrench product line is $961,000. If the company’s actual
experience remains consistent with the estimated sales volume percentage distribution,
and the firm desires to generate total operating income of $161,200, how many Model
No. 153 socket sets will MetalCraft have to sell?

Model CM CM
No. 109 20% 4.5 0.9
No. 145 50% 7 3.5
No. 153 30% 6 1.8
6.2

Numerator 1122200
units to ac 181000
No. 153 54300

136. Robin Company wants to earn a 6% return on sales after taxes.


The company’s effective income tax rate is 40%, and its contribution margin is 30%.
If Robin has fixed costs of $240,000, the amount of sales required to earn the desired return is

desired return return on sales 6%


tax rate 40%
contribution margin 30%
fixed costs $240,000.00

This problem can be solved by setting up an equation and solving for the required sales amount, which
will be represented by the variable S. S = required sales
Sheet 14 april 2010 p3

169. Fennel Products is using cost-based pricing to determine the selling price for its new
product based on the following information.
Annual volume 25,000 units 25,000.00
Fixed costs $700,000 per year $700,000.00
Variable costs $200 per unit $200.00 $5,000,000.00
Plant investment $3,000,000 $3,000,000.00
Working capital $1,000,000 $1,000,000.00
Effective tax rate 40% 40%

The target price that Fennell needs to set for the new product to achieve a 15% after-tax
return on investment (ROI) would be

(ROI) $600,000.00
investment $4,000,000.00

(ROI) before tax $705,882.35

$ 228.00 $ 238.00 $ 258.00 $ 268.00


Sales Rev 5,700,000.00 5,950,000.00 6,450,000.00 6,700,000.00
VC 5000000 5000000 5000000 5000000
CM 700,000.00 950,000.00 1,450,000.00 1,700,000.00
FC $700,000.00 $700,000.00 $700,000.00 $700,000.00
OI $0.00 $250,000.00 $750,000.00 $1,000,000.00
Tax 0 100000 300000 400000
NI $0.00 $150,000.00 $450,000.00 $600,000.00
0.00% 3.75% 11.25% 15.00%

168. Almelo Manpower Inc. provides contracted bookkeeping services. Almelo has annual
fixed costs of $100,000 and variable costs of $6 per hour. This year the company
budgeted 50,000 hours of bookkeeping services. Almelo prices its services at full cost
and uses a cost-plus pricing approach. The company developed a billing price of $9 per
hour. The company’s mark-up level would be
fixed costs $100,000.00
fixed costs per unit $2.00
VC per hour $6.00
budgeted 50,000 hours 50,000.00
billing price $9.00

Full cost $8.00


mark-up $1.00 12.50%

165. Basic Computer Company (BCC) sells its microcomputers using bid pricing. It develops
its bids on a full cost basis. Full cost includes estimated material, labor, variable
overheads, fixed manufacturing overheads, and reasonable incremental computer
assembly administrative costs, plus a 10% return on full cost. BCC believes bids in
excess of $1,050 per computer are not likely to be considered.

BCC’s current cost structure, based on its normal production levels, is $500 for materials
per computer and $20 per labor hour. Assembly and testing of each computer requires 17
labor hours. BCC expects to incur variable manufacturing overhead of $2 per labor hour,
fixed manufacturing overhead of $3 per labor hour, and incremental administrative costs
of $8 per computer assembled.

BCC has received a request from a school board for 200 computers. Using the full-cost
criteria and desired level of return, which one of the following prices should be
recommended to BCC’s management for bidding purposes?

DM $500.00 per computer


DL $20.00 per labor hour 3400

Assembly and testing of each computer requires 17 hours

VOH $2.00 per labor hour


FOH $3.00 per labor hour
incremental administrative costs $8.00 per computer

Full cost
10% return on full cost.
(BCC) bid pricing.
(BCC) bid pricing per computer
163. Leader Industries is planning to introduce a new product, DMA. It is expected that
10,000 units of DMA will be sold. The full product cost per unit is $300. Invested
capital for this product amounts to $20 million. Leader’s target rate of return on
investment is 20%. The markup percentage for this product, based on operating income
as a percentage of full product cost, will be

Full cost $3,000,000.00


Leader’s target rate of return $4,000,000.00 133%

162. The Robo Division, a decentralized division of GMT Industries, has been approached to
submit a bid for a potential project for the RSP Company.
Robo Division has been informed by RSP that they will not consider bids over $8,000,000.
Robo Division purchases its materials from the Cross Division of GMT Industries.
There would be no additional fixed costs for either the Robo or Cross Divisions.

Information regarding this project is as follows.

Sell to Robo
Cross Division Robo Division
Variable Costs $1,500,000 $4,800,000 $1,500,000.00 $4,800,000.00
Transfer Price 3,700,000 - $3,700,000.00 $0.00

If Robo Division submits a bid for $8,000,000, the amount of contribution margin 8000000
recognized by the Robo Division and GMT Industries, respectively, is
Cross Division Robo Division
Price of the product sold $3,700,000.00 $8,000,000.00
VC $1,500,000.00 $8,500,000.00
CM $2,200,000.00 -$500,000.00

161. Johnson Company manufactures a variety of shoes, and has received a special one-timeonly order directly from a who
Johnson has sufficient idle capacity to accept the special order to manufacture 15,000 pairs of sneakers at a price of $7.50
Johnson’s normal selling price is $11.50 per pair of sneakers.
Variable manufacturing costs are $5.00 per pair and fixed manufacturing costs are $3.00 a pair.
Johnson’s variable selling expense for its normal line of sneakers is $1.00 per pair.
What would the effect on Johnson’s operating income be if the company accepted the special order?

Sales Rev 112500


VC 90000
CM ###

160. The Doll House, a very profitable company, plans to introduce a new type of doll to its
product line. The sales price and costs for the new dolls are as follows.

Selling price per doll $100 $100.00


Variable cost per doll $60 $60.00
Incremental annual fixed costs $456,000 $456,000.00
Income tax rate 30% 30%

If 10,000 new dolls are produced and sold, the effect on Doll House’s profit (loss) would

Selling price per doll $100 $1,000,000.00


Variable cost per doll $60 $600,000.00
Incremental annual fixed costs $456,000 $456,000.00
-$56,000.00
Income tax rate 30% savings -$16,800.00
-$39,200.00

159. Synergy Inc. produces a component that is popular in many refrigeration systems. Data
on three of the five different models of this component are as follows.

Model
A B C
Volume needed (units) 5000 6000 3000
Manufacturing costs
Variable direct costs $10 $24 $20 $ 10.00 $ 24.00 $ 20.00
Variable overhead 5 10 15 $ 5.00 $ 10.00 $ 15.00
Fixed overhead 11 20 17 $ 11.00 $ 20.00 $ 17.00
Total manufacturing costs $26 $54 $52 $ 26.00 $ 54.00 $ 52.00

Cost if purchased $21 $42 $39 $ 21.00 $ 24.00 $ 39.00


Total v manufacturing costs $ 15.00 $ 34.00 $ 35.00
Hours required 2.00 4.00 6.00
14,000.00 7,000.00
Saving (loss) (6.00) 10.00

Synergy applies variable overhead on the basis of machine hours at the rate of $2.50 per hour.
Models A and B are manufactured in the Freezer Department, which has a capacity of 28,000 machine processing hours.
Which one of the following options should be recommended to Synergy's management?

a. Purchase all three products in the quantities required.


b. Manufacture all three products in the quantities required.
c. The Freezer Department's manufacturing plan should include 5,000 units of
Model A and 4,500 units of Model B.
d. The Freezer Department's manufacturing plan should include 2,000 units of
Model A and 6,000 units of Model B.

158. Green Corporation builds custom-designed machinery. A review of selected data and the company’s pricing policies re

• A 10% commission is paid on all sales orders.


• Variable and fixed factory overheads total 40% and 20%, respectively, of direct labor.

• Corporate administrative costs amount to 10% of direct labor.


• When bidding on jobs, Green adds a 25% markup to the total of all factory and administrative costs to cover income taxes
• The firm’s income tax rate is 40%.

The company expects to operate at a maximum of 80% of practical capacity.

Green recently received an invitation to bid on the manufacture of some custom machinery for Kennendale, Inc.
For this project, Green’s production accountants estimate the material and labor costs will be $66,000 and $120,000, respe
$66,000.00 $120,000.00
Accordingly, Green submitted a bid to Kennendale in the amount of $375,000. Feeling Green’s bid was too high, Kennendal

Which one of the following options should be recommended to Green’s management?

• A 10% commission is paid on all sales orders.


• Variable and fixed factory overheads total 40% and 20%, respectively, of direct labor.
• Corporate administrative costs amount to 10% of direct labor.
• When bidding on jobs, Green adds a 25% markup to the total of all factory and administrative costs to cover income taxes
• The firm’s income tax rate is 40%.
DM ###
DL ###
VOH ###
FOH ###
Admin ###
Full cost ###
25% mark ### 25%
### 375000 $37,500.00
• A 10% commission is paid on all sales orders.
### 10%
Accordingly, Green submitted a bid to Kennendale in the amount of $375,000 375000
###

a. Accept the counteroffer because the order will increase operating income.

157. Jones Enterprises manufactures 3 products, A, B, and C. During the month of May
Jones’ production, costs, and sales data were as follows.

Products
A B C
Units of production 30,000.00 20,000.00 70,000.00
Joint production costs to split-off point $480,000
Further processing costs $ - $60,000 $140,000 $ - $ 60,000.00 $ 140,000.00
Further processing cost $ 3.00 $ 2.00
Unit sales price
At split-off 3.75 5.50 10.25 $ 3.75 $ 5.50 $ 10.25
After further processing - 8.00 12.50 $ - $ 8.00 $ 12.50
$ 5.00 $ 10.50
INCREMENTAL PROFIT $ (0.50) $ 0.25
Based on the above information, which one of the following alternatives should be recommended to Jones’ management?

c. Process Product C further but sell Product B at the split-off point.

156. Basic Computer Company (BCC) sells its micro-computers using bid pricing. It develops bids on a full cost basis.
Full cost includes estimated material, labor, variable overheads, fixed manufacturing overheads, and reasonable increment

BCC believes bids in excess of $925 per computer are not likely to be considered. 925
BCC’s current cost structure, based on its normal production levels, is $500 for materials per computer and $20 per labor h
Assembly and testing of each computer requires 12 labor hours.
BCC’s variable manufacturing overhead is $2 per labor hour, fixed manufacturing overhead is $3 per labor hour, and increm

The company has received a request from the School Board for 500 computers. BCC’s
management expects heavy competition in bidding for this job. As this is a very large
order for BCC, and could lead to other educational institution orders, management is
extremely interested in submitting a bid which would win the job, but at a price high
enough so that current net income will not be unfavorably impacted. Management
believes this order can be absorbed within its current manufacturing facility. Which one
of the following bid prices should be recommended to BCC’s management?

DM $500.00 per computer


DL $20.00 per labor hour 6000

Assembly and testing of each computer requires 12 hours

VOH $2.00 per labor hour


FOH $3.00 per labor hour
incremental administrative costs $8.00 per computer

Full cost
10% return on full cost.
(BCC) bid pricing.
(BCC) bid pricing per computer

153. Lazar Industries produces two products, Crates and Boxes. Per unit selling prices, costs,
and resource utilization for these products are as follows.

Crates Boxes
Selling price $20 $30 $20.00 $30.00
Direct material costs $ 5 $ 5 2.5 $5.00 $5.00
Direct labor costs 8 10 $8.00 $10.00
Variable overhead costs 3 5 $ 5.25 $3.00 $5.00 VOH
Variable selling costs 1 2 $1.00 $2.00
Machine hours per unit 2 4 3.50 2.00 4.00

total fixed factory overhead cost $2,000,000.00


total fixed selling and administrative costs $840,000.00
Production and sales 500,000.00 700,000.00

Production of Crates and Boxes involves joint processes and use of the same facilities.
The total fixed factory overhead cost is $2,000,000 and total fixed selling and administrative costs are $840,000.
Production and sales are scheduled for 500,000 units of Crates and 700,000 units of Boxes.
Lazar maintains no direct materials, work-inprocess, or finished goods inventory.

Lazar can reduce direct material costs for Crates by 50% per unit, with no change in direct labor costs.
However, it would increase machine-hour production time by 1-1/2 hours per unit. 1.50
For Crates, variable overhead costs are allocated based on machine hours.

What would be the effect on the total contribution margin if this change was implemented?

Before
CM $3.00 1,500,000.00
CM per constraint. $1.50

After
CM $3.25 1,625,000.00
CM per constraint. $0.93 125,000.00

152. Refrigerator Company manufactures ice-makers for installation in refrigerators. The


costs per unit, for 20,000 units of ice-makers, are as follows. 20,000.00

Make
Direct materials $ 7 $7.00 7
Direct labor 12 $12.00 12
Variable overhead 5 $5.00 5
Fixed overhead 10 $10.00 6
Total costs $34 $34.00 $30.00

$30.00
$600,000.00

Cool Compartments Inc. has offered to sell 20,000 ice-makers to Refrigerator Company for $28 per unit.
If Refrigerator accepts Cool Compartments’ offer, the facilities used to manufacture ice-makers could be used to produce w
Revenues from the sale of water filtration units are estimated at $80,000, with variable costs amounting to 60% of sales.
In addition, $6 per unit of the fixed overhead associated with the manufacture of ice-makers could be eliminated.

For Refrigerator Company to determine the most appropriate action to take in this
situation, the total relevant costs of make vs. buy, respectively, are
c. $600,000 vs. $528,000.
151. Aspen Company plans to sell 12,000 units of product XT and 8,000 units of product RP.
Aspen has a capacity of 12,000 productive machine hours. 12000
The unit cost structure and machine hours required for each product is as follows.
12000 8000
Unit Costs XT RP
Materials $37 $24 $37.00 $24.00
Direct labor 12 13 $12.00 $13.00
Variable overhead 6 3 $6.00 $3.00
Fixed overhead 37 38 $37.00 $38.00
Machine hours required 1.0 1 1.5
VC $55.00 $40.00
Machine hours required 12,000.00 12,000.00

Purchase price $60.00 $45.00


Loss -$5.00 -$5.00
Loss per constraint -$5.00 -$7.50

Aspen can purchase 12,000 units of XT at $60 and/or 8,000 units of RP at $45. Based on the above, which one of the follow

150. The Furniture Company currently has three divisions: Maple, Oak, and Cherry. The oak
furniture line does not seem to be doing well and the president of the company is
considering dropping this line.
If it is dropped, the revenues associated with the Oak
Division will be lost and the related variable costs saved. Also, 50% of the fixed costs
allocated to the oak furniture line would be eliminated. The income statements, by
divisions, are as follows.

Maple
Sales $55,000 $85,000 $100,000 $55,000.000
Variable Costs 40,000 72,000 82,000 $40,000.000
Contribution Margin 15,000 13,000 18,000 $15,000.000
Fixed costs 10,000 14,000 10,200 $10,000.000
Operating profit (loss) $ 5,000 $(1,000) $ 7,800 $5,000.000

Which one of the following options should be recommended to the president of the
company?
Maple
Sales $55,000 $85,000 $100,000 $55,000.000
Variable Costs 40,000 72,000 82,000 $40,000.000
Contribution Margin 15,000 13,000 18,000 $15,000.000
Fixed costs 10,000 14,000 10,200 $10,000.000
Operating profit (loss) $ 5,000 $(1,000) $ 7,800 $5,000.000

b. Continue operating the Oak Division as discontinuance would result in a $6,000


decline in operating profits.

149. Raymund Inc. currently sells its only product to Mall-Stores. Raymund has received a
one-time-only order for 2,000 units from another buyer. Sale of the special order items
will not require any additional selling effort. Raymund has a manufacturing capacity to
produce 7,000 units. Raymund has an effective income tax rate of 40%. Raymund’s
Income Statement, before consideration of the one-time-only order, is as follows.

Sales (5,000 units at $20 per unit) $100,000 5,000.00 $20.0


Variable manufacturing costs $50,000 $10.0 $50,000.0
Variable selling costs 15,000 65,000 $3.0 $15,000.0
Contribution margin 35,000

Fixed manufacturing costs 16,000 $16,000.0


Fixed selling costs 4,000 20,000 $4,000.0
Operating income $ 15,000
Income taxes
Net income

In negotiating a price for the special order, Raymund should set the minimum per unit
selling price at $10.0

148. Lark Industries accepted a contract to provide 30,000 units of Product A and 20,000 units
of Product B. Lark’s staff developed the following information with regard to meeting
this contract.
Product A Product B
30000 20000
Selling Price $75 $125 $75.0 $125.0
Variable costs $30 $48 $30.0 $48.0
Fixed overhead $1,600,000
Machine hours required 3 5 3.00 5.00
Machine hours available 160,000
Cost if outsourced $45 $60 $45.0 $60.0

Lark’s operations manager has identified the following alternatives. Which alternative
should be recommended to Lark’s management?

CM $45.0 $77.0
CM per constraint $ 15.00 $ 15.40

loss if outsourced -$15.0 -$12.0


loss per constraint if outsourced $ -5.00 $ -2.40

147. Aril Industries is a multiproduct company that currently manufactures 30,000 units of
Part 730 each month for use in production. The facilities now being used to produce Part
730 have fixed monthly overhead costs of $150,000, and a theoretical capacity to produce
60,000 units per month. If Aril were to buy Part 730 from an outside supplier, the
facilities would be idle and 40% of fixed costs would continue to be incurred. There are
no alternative uses for the facilities. The variable production costs of Part 730 are $11
per unit. Fixed overhead is allocated based on planned production levels.

If Aril Industries continues to use 30,000 units of Part 730 each month, it would realize a
net benefit by purchasing Part 730 from an outside supplier only if the supplier’s unit
price is less than $ 14.00

146. Current business segment operations for Whitman, a mass retailer, are presented below.

Merchandise Automotive
Sales $ 500,000.00 $ 400,000.00
Variable costs $ 300,000.00 $ 200,000.00
CM $ 200,000.00 $ 200,000.00
Fixed costs $ 100,000.00 $ 100,000.00
Operating income (loss) $ 100,000.00 $ 100,000.00

CM% 40% 50%

Management is contemplating the discontinuance of the Restaurant segment since “it is


losing money.” If this segment is discontinued, $30,000 of its fixed costs will be
eliminated. In addition, Merchandise and Automotive sales will decrease 5% from their
current levels. When considering the decision, Whitman’s controller advised that one of
the financial aspects Whitman should review is contribution margin. Which one of the
following options reflects the current contribution margin ratios for each of Whitman’s
business segments?

Merchandise Automotive
Sales $ 475,000.00 $ 380,000.00
Variable costs $ 285,000.00 $ 190,000.00
CM $ 190,000.00 $ 190,000.00
Fixed costs $ 100,000.00 $ 100,000.00
Operating income (loss) $ 90,000.00 $ 90,000.00

CM% 40% 50%

144. Oakes Inc. manufactured 40,000 gallons of Mononate and 60,000 gallons of Beracyl in a
joint production process, incurring $250,000 of joint costs. Oakes allocates joint costs
based on the physical volume of each product produced. Mononate and Beracyl can each
be sold at the split-off point in a semifinished state or, alternatively, processed further.
Additional data about the two products are as follows.

40,000.00
Mononate
Sales price per gallon at split-off $ 7.00
Sales price per gallon if processed further $ 10.00
Variable production costs if processed further $ 125,000.00

An assistant in the company’s cost accounting department was overheard saying “....that
when both joint and separable costs are considered, the firm has no business processing
either product beyond the split-off point. The extra revenue is simply not worth the
effort.” Which of the following strategies should be recommended for Oakes?

Variable production costs if processed further $ 3.13

Incremental Revenue per unit if processed further $ 3.00


Incremental Cost per unit if processed further $ 3.13
Incremental profit (Loss) $ (0.13)

Incremental Revenue if processed further 120,000.00


Incremental Cost if processed further 125,000.00
Incremental profit (Loss) (5,000.00)
143. Jack Blaze wants to rent store space in a new shopping mall for the three month holiday
shopping season. Blaze believes he has a new product available which has the potential
for good sales. The product can be obtained on consignment at the cost of $20 per unit
and he expects to sell the item for $100 per unit. Due to other business ventures, Blaze’s
risk tolerance is low. He recognizes that, as the product is entirely new, there is an
element of risk. The mall management has offered Blaze three rental options: (1) a fixed
fee of $8,000 per month, (2) a fixed fee of $3,990 per month plus 10% of Blaze’s
revenue, or (3) 30% of Blaze’s revenues. Which one of the following actions would you
recommend to Jack Blaze?

(1) a fixed fee of $8,000 per month, (2) a fixed fee of $3,990 per month pl
8000

142. Eagle Brand Inc. produces two products. Data regarding these products are presented
below.

Product X Product Y
Selling price per unit $100 $130 $ 100.00 $ 130.00
Variable costs per unit $80 $100 $ 80.00 $ 100.00
Raw materials used per unit 4 lbs. 10 lbs. 4.00 10.00

Eagle Brand has 1,000 lbs. of raw materials which can be used to produce Products X and
Y.

Which one of the alternatives below should Eagle Brand accept in order to maximize
contribution margin?

CM 20 30
CM per constraint $ 5.00 $ 3.00
Units that can be produced by the constraint 250.00 100.00

12 20
8 15 Specialty Cakes Inc. produces two typ
RC HS cake. Total fixed costs for the firm are
CM $4.0 $5.0 the two types of cakes are presented
units 10000 15000 25000 2 lbs. 3 lbs.
40% 60% Round Cake Heart-shape Cake
$1.6 $3.0 $4.6 Selling price per unit $12 $20
Variable cost per unit $8 $15
fc $94,000.0 20,434.78 Current sales (units) 10,000 15,000

8,173.91 ### 20,434.78

$12.0 $20.0
$8.0 $15.0 If the product sales mix were to change to three heart-s
RC HS the breakeven volume for each of these products would
CM $4.0 $5.0
units 6250 18750 25000
25% 75%
$1.0 $3.8 $4.8

fc $94,000.0 19,789.47

4,947.37 ### 19,789.47

138. Zipper Company invested $300,000 in a new machine to produce cones for the textile
industry. Zipper’s variable costs are 30% of the selling price, and its fixed costs are
$600,000. Zipper has an effective income tax rate of 40%. The amount of sales required
to earn an 8% after-tax return on its investment would be

Targeted ROI $ 24,000.00


Before Tax Targeted ROI $ 40,000.00
Numerator to be used $ 640,000.00
CM ratio 70%
$ 914,285.71

137. Bargain Press is considering publishing a new textbook. The publisher has developed the
following cost data related to a production run of 6,000, the minimum possible
production run. Bargain Press will sell the textbook for $45 per copy. How many
textbooks must Bargain Press sell in order to generate operating earnings (earnings before
interest and taxes) of 20% on sales? (Round your answer up to the nearest whole
textbook.)
Estimated cost per unit
Development (reviews, class testing, editing) $35,000 $ 35,000.00
Typesetting 18,500 $ 18,500.00
Depreciation on Equipment 9,320 $ 9,320.00
General and Administrative 7,500 $ 7,500.00
Miscellaneous Fixed Costs 4,400 $ 4,400.00
Printing and Binding 30,000 $ 30,000.00 $ 5.00
Sales staff commissions (2% of selling price) 5,400 $ 5,400.00 $ 0.90
Bookstore commissions (25% of selling price) 67,500 $ 67,500.00 $ 11.25
Author’s Royalties (10% of selling price) 27,000 $ 27,000.00 $ 4.50
$ 21.65
Total costs at production of 6,000 copies $ 204,620.00

Feedback: The correct answer is: 5,207 copies. ,


The number of copies required can be found by solving the following equation, where x equals the ,
number of copies: ,
Total sales – total variable costs – fixed costs + typesetting + depreciation expense + general and ,
administrative expenses + miscellaneous fixed costs = (return %)(total sales) ,
Total sales = $45x ,
Total variable cost = (variable cost per unit, or VCU)(x) = VCU(x) ,
Fixed costs = development + typesetting + depreciation + general and administrative + miscellaneous ,
fixed costs ,
Fixed costs = $35,000 + $18,500 + $9,320 + $7,500 + $4,400 = $74,720 ,
VCU = (printing and binding + sales staff commissions + bookstore commissions + royalties) / ,
(number of copies) ,
VCU = ($30,000 + $5,400 + $67,500 + $27,000) / (6,000 copies) = $129,900 / 6,000 ,
VCU = $21.65 per copy ,
$45x – $21.65(x) – $74,720 = (0.2)($45x) ,
$45x – $21.65(x) - $74,720 = $9x ,
$36x – $21.65(x) = $74,720 ,
$14.35x = $74,720 ,
x = 5,206.97, which rounds to 5,207 copies. ,

131. Wilkinson Company sells its single product for $30 per unit. The contribution margin
ratio is 45% and Wilkinson has fixed costs of $10,000 per month. If 3,000 units are sold
in the current month, Wilkinson’s income would be

CM $ 13.50
Total CM $ 40,500.00
FC $ 10,000.00
OI $ 30,500.00
25000 22000

1000

turing other products. This is a one time special order.

XR-2000 XR-2000
Q 6,250.00 $387,500.0
$24.0 Q 5,500.00 $341,000.0
$10.0 $46,500.0
$5.0 $46.50
$40.0
$4.0
$10.0
$105.0
4.00

$62.0
$15.5

y not producing XR-2000

750

106. C c. a decrease of $2,000.

Segment B Total
$15,000.0 $25,000.0
$8,500.0 $12,500.0
$2,500.0 $4,000.0
$4,000.0 $8,500.0
$3,000.0 $5,000.0
$1,500.0 $3,000.0
-$500.0 $500.0

Segment B Total

$10,000.0
$4,000.0
$1,000.0 $2,500.0
-$1,000.0 $3,500.0
$2,000.0
$1,500.0 $3,000.0
-$2,500.0 -$1,500.0

-$2,000.00

107. C Z, X, Y.
Approach Y Approach Z

$38.0 $32.0
12,000.00 14,000.00
$456,000.0 $448,000.0
$360,000.0 $420,000.0
$45,600.0 0
$30,000.0
$20,400.0 $28,000.0

108. C c. 14 units.

6 - 10 11 - 15 16 - 20
$400,000.0 $600,000.0 $800,000.0
$50,000.0 $45,000.0 $45,000.0
$100,000.0 $100,000.0 $100,000.0
e and variable costs are as follows. 109. B b. $67,200.

110. A a. $24,000.

7,000.00 111. B b. $10,000.

$30,000.0
$5.0

$100,000.0

$65,000.0
$35,000.0

$20,000.0
$15,000.0

$100,000.0
increase Raymund’s fixed manufacturing costs
by $30,000 per month and reduce its variable
$40,000.0 costs by $5 per uni
$60,000.0

$50,000.0
$10,000.0

osts per unit, for 20,000 units of ice-makers, are as follows. 112. C
20,000.00

$28 per unit.


ead amounting to $6 per unit could be eliminated. $6.0
n an anticipated volume of 150,000 units, is as follows. 113. C
150,000.00

e of Phillip’s cost structure. 175,000.00


reduction of $750,000 in fixed costs $750,000.0

100%
32%
68%

2% and operating income of $9,600,000.

114. B b. No, the machine-related cost of pro


1,600,000.00
ucing each unit of product should be $.50. $0.5
1,200,000.00
600000

use the manufacturer’s engineering specification of machine related unit production cost?

$800,000.0
$0.67

115. B
Product B
20,000.00
20,000.00
$80.0
$45.0
$10.0
$11.0
$4.0
$0.0
$10.0
1.5

aking a decision on product priorities for the coming year, are

Product B
$24.0
$16.0

116. B
4500 15300 $3.40
$4.05

ds of Kaylene. 4200

Irrelevant Decision Factor


Purchase price of $3.40 per lb.

tilizer and weed killer that is popular in hot, dry climates. 117. C
awn fertilizer.
40

$18.5
$20.0
-$1.50

anufactures several different products, and the loss appear after the deduction of the allocated fixed costs

$2.5

119. A a. $25,000.
ased four years ago at a cost of $70,000.

machine has no current salvage value.

d labor priced at $5,000.


$5,000.00

123. D d. $16,500.

124. B
b. 82,500 units and $9,250,000 of operating income.
h is within the relevant range of Phillip’s cost structure. 175000
ar would amount to

126. B b. $13,500.

1,000

diff
$30,000

$13,500

$13,500

127. B b. $689,992.
689992
Product B

20,000.00
20,000.00
$80.0
$45.0
$10.0
$11.0
$4.0
$0.0
$10.0
1.5

Product B
$24.00
$16.00

370,000

20,000 13333
13,333.33 $319,992.00
320,000 320,000

$689,992 $0.0

128. B b. $2,000,000.

Trunks
30
5
10
5
2
4

e costs are $840,000.

1,000,000
Trunks
8.00
2.00

250,000 Trunks will use all the machine hours


1,000,000
$2,000,000

%
450000 b. $80,000. 129. B
120000
270000 0.6
500000

130. C increase by $1,000.


133. C c. 79,938.

Variable Fixed CM no. of units %


Costs 1 Costs 2
$3.00 $165,000.00 $15.00 42,000.00 28% $4.20
$1.00 $249,000.00 $14.00 60,000.00 40% $5.60
$0.00 $316,000.00 $20.00 48,000.00 32% $6.40
$730,000.00 150,000.00 $16.20

$565,000.00
30%

based on the following information. 134. D d. 90,000 units.

$1,300,000
tyourselfers. 135. B b. 54,300.

sets
sets
sets

961000

161200

136. D d. $1,200,000.

mount, which
169. D d. $268.

a. $228.
b. $238.
c. $258.
d. $268.

15%

168. A a. 12.5%.
100000 6
50000
9
165. D d. $1,026.30.

10%

200

$100,000.00
$68,000.00

$6,800.00
$10,200.00
$1,600.00

$186,600.00
$18,660.00
$205,260.00
$1,026.30
163. C c. 133.3%.
10,000.00 $300.00 $20,000,000.00
20%

162. C c. $(500,000) and $1,700,000

GMT Industries
$11,700,000.00
$10,000,000.00
$1,700,000.00 ###

161. C b. Increase by $22,500.

meonly order directly from a wholesaler.


rs of sneakers at a price of $7.50 per pair. 15000 7.5
11.5
5 3
1
pecial order?

160. C c. $(39,200).

10000

159. C c. The Freezer Department's manufacturing plan should include 5,0


Model A and 4,500 units of Model B.

Should be answered by
elimination
Should be answered by
elimination

$ 2.50
000 machine processing hours. 28,000.00

158. A
the company’s pricing policies revealed the following.

10%
40% 20%

10%
rative costs to cover income taxes and produce a profit. 25%
40%

80%

y for Kennendale, Inc.


be $66,000 and $120,000, respectively.

en’s bid was too high, Kennendale countered with a price of $280,000.

$234,000.00

$ 48,000.00 $ 24,000.00
$12,000.00
rative costs to cover income taxes and produce a profit. $ 84,000.00
157. C c. Process Product C further but sell Product B at the split-off point

TOTAL
120,000.00
$ 480,000.00

mended to Jones’ management?

156. B b. $772.00.
s bids on a full cost basis.
heads, and reasonable incremental computer assembly administrative costs, plus a 10% return on full cost.
er computer and $20 per labor hour.

d is $3 per labor hour, and incremental administrative costs are $8 per computer assembled.

0.1

500

$250,000.00
$120,000.00

$12,000.00
$18,000.00
$4,000.00

$404,000.00 VC $386,000.00
$40,400.00 $0.00
$444,400.00 $386,000.00
$888.80 $772.00

153. A a. $125,000 increase.

For Crates, variable overhead costs are allocated based on


machine hours.
$ 1.50
labor costs.

152. C c. $600,000 vs. $528,000.

Buy
28

$28.00
-$1.60
$26.40
$528,000.00

$28 per unit.


akers could be used to produce water filtration units.
sts amounting to 60% of sales. 0.4 80000 32000 1.6
ers could be eliminated.
151. A a. Produce XT internally and purchase RP.

e above, which one of the following actions should be recommended to Aspen's management?

150. B b. Continue operating the Oak Division as discontinu


decline in operating profits.

50%

Oak Cherry Total


$85,000.000 $100,000.000 ###
$72,000.000 $82,000.000 ###
$13,000.000 $18,000.000 $46,000.000
$14,000.000 $10,200.000 $34,200.000
-$1,000.000 $7,800.000 $11,800.000
Oak Cherry Total
$100,000.000 ###
$82,000.000 ###
$0.000 $18,000.000 $33,000.000
$7,000.000 $10,200.000 $27,200.000
-$7,000.000 $7,800.000 $5,800.000

-$6,000.000

149. A a. $10.
2000

40%

$100,000.0

$65,000.0
$35,000.0

$20,000.0
$15,000.0
$6,000.0
$9,000.0

148. A a. Make 30,000 units of Product A, utilize the remain


B, and outsource the remainder.

Total

$1,600,000.0
160,000.00

147. D d. $14.00.
30,000.00

$ 150,000.00
60,000.00
60% $ 90,000.00
$ 11.00
$ 3.00

146. D d. 40% 50% 30%


Retailing Automotive Restaurant

Restaurant Total
$ 100,000.00 $ 1,000,000.00
$ 70,000.00 $ 570,000.00
$ 30,000.00 $ 430,000.00
$ 50,000.00 $ 250,000.00
$ -20,000.00 $ 180,000.00

30% 43%
95%

Restaurant Total
$ 855,000.00
$ 475,000.00
$ - $ 380,000.00 What will Whitman’s total contribution margin be if
$ 20,000.00 $ 250,000.00 segment is discontinued?
$ -20,000.00 $ 130,000.00 145. D d. $380,000.

#DIV/0! 44%

Mononate Beracyl
144. B b. Sell at split-off Process further.
$ 250,000.00

60,000.00 100,000.00
Beracyl
$ 15.00
$ 18.00
$ 115,000.00

$ 1.92

$ 3.00
$ 1.92
$ 1.08

180,000.00
115,000.00
65,000.00
143. D d. Choose the third option no matter what Blaze exp

20
100

5700

fixed fee of $3,990 per month plus 10% of Blaze’s revenue (3) 30% of Blaze’s revenues.
4560 1710

142. B b. 250 units of product X.

1000

alty Cakes Inc. produces two types of cakes, a 2 lbs. round cake and a 3 lbs. heartshaped
Total fixed costs for the firm are $94,000. Variable costs and sales data for
wo types of cakes are presented below.
d Cake Heart-shape Cake
g price per unit $12 $20
ble cost per unit $8 $15
nt sales (units) 10,000 15,000

x were to change to three heart-shaped cakes for each round cake,


for each of these products would be

138. B b. $914,286.
$ 300,000.00
30%
$ 600,000.00
40%
8%

137. B b. 5,207 copies.

6000
$ 45.00
20%
$ 74,720.00

cellaneous ,

30 131. A a. $30,500
0.45 3000
10000
Section D: Decision Analysis
103. B b. $96.50. 101 169

KT-6500
3,000.00

c. a decrease of $2,000.
fixed manufacturing costs
h and reduce its variable
y $5 per uni

c. $600,000.
achine-related cost of producing each unit is $.67.
ng plan should include 5,000 units of
uct B at the split-off point.
Oak Division as discontinuance would result in a $6,000
oduct A, utilize the remaining capacity to make Product
contribution margin be if the Restaurant

Process further.
no matter what Blaze expects the revenues to be.
Gregg Industries manufactures molded chairs. The three models of molded chairs, which are all variations of the same des
are Standard (can be stacked), Deluxe (with arms), and Executive (with arms and padding). The company uses batch
manufacturing and has an operation costing system.
Gregg has an extrusion operation and subsequent operations to form, trim, and finish the chairs.
Plastic sheets are produced by the extrusion operation, some of which are sold directly to other manufacturers.
During the forming operation, the remaining plastic sheets are molded into chair seats and the legs are added; the standar
During the trim operation, the arms are added to the deluxe and executive models and the chair edges are smoothed.
Only the executive model enters the finish operation where the padding is added.
All of the units produced are subject to the same steps within each
operation, and no units are in process at the end of the period. The units of production and direct materials costs were as f

Units Extrusion Form


Produced Materials Materials
Plastic sheets 5,000.00 $60,000
Standard mode 6,000.00 $72,000 $24,000
Deluxe model 3,000.00 $36,000 $12,000
Executive model 2,000.00 $24,000 $8,000
16,000.00 $192,000.00 $44,000.00

Units attributable to each stage 16,000.00 11,000.00


Unit material cost $12.00 $4.00

Plastic sheets are produced by the During the forming operation, the remaining
extrusion operation, some of which plastic sheets are molded into chair seats and
are sold directly to other the legs are added; the standard model is sold
manufacturers. after this operation.

Manufacturing costs applied during the month were:


Extrusion Form
Operation Operation
Direct labor $152,000 $60,000
Overhead $240,000 $72,000
$392,000 $132,000

Units attributable to each stage 16,000.00 11,000.00


Unit conversion cost $24.50 $12.00

(Refers to Fact Pattern #33)


Gregg Industries’ unit cost of a standard model is $52.50

A standard model passes through the extrusion and form operations. Thus, its unit cost includes the
materials and conversion costs for both operations. The unit materials and conversion costs for the extrusion
operation are $12.00 ($192,000 ÷ 16,000 units) and $24.50 [($152,000 + $240,000) ÷ 16,000 units], respectively. The
unit materials and conversion costs for the form operation are $4.00 [$44,000 ÷ (16,000 – 5,000) units] and $12.00
[($60,000 + $72,000) ÷ (16,000 – 5,000) units], respectively. Accordingly, the unit cost of a standard model is $52.50
($12.00 + $24.50 + $4.00 + $12.00).
A standard model passes through the extrusion and form operations. Thus, its unit cost includes the
materials and conversion costs for both operations. The unit materials and conversion costs for the extrusion
operation are $12.00 ($192,000 ÷ 16,000 units) and $24.50 [($152,000 + $240,000) ÷ 16,000 units], respectively. The
unit materials and conversion costs for the form operation are $4.00 [$44,000 ÷ (16,000 – 5,000) units] and $12.00
[($60,000 + $72,000) ÷ (16,000 – 5,000) units], respectively. Accordingly, the unit cost of a standard model is $52.50
($12.00 + $24.50 + $4.00 + $12.00).

(Refers to Fact Pattern #33)


Gregg Industries’ unit cost of a deluxe model is $69.30

(Refers to Fact Pattern #33)


Gregg Industries’ total product cost of the executive model is

Total DM 50,000.00
Total Conv. $142,600.00
Total cost $192,600

An executive model passes through all four operations. Thus, its unit cost equals that of the deluxe model plus the
unit costs incurred in the finish operation. The unit cost of the deluxe model is $69.30. The unit materials and
conversion costs for the finish operation are $6.00 [$12,000 ÷ (16,000 – 14,000) units] and $21.00 [($18,000 + $24,000)
÷ (16,000 – 14,000) units], respectively. Consequently, the unit cost of the executive model is $96.30 ($69.30 + $6.00 +
$21.00), and the total product cost is $192,600 (2,000 units × $96.30).

Refers to Fact Pattern #33)


Assume that 1,000 units of Gregg Industries’ deluxe model remained in work-in-process at the end of the period and that
these units were 100% complete as to materials and 60% complete as to trim operation conversion. What is the balance of
work-in-process?
No. of units 1000 Total DM $19,000.00
Total Conv. $36,500.00
Total TRIM Conv. $9,000.00
Total cost $64,500.00
EUP Conv
Completed Deluxe mo 2000
partial Deluxe model 600
completed Executive 2000
EUP 4600
l variations of the same design,
company uses batch

r manufacturers.
legs are added; the standard model is sold after this operation.
ir edges are smoothed.

ect materials costs were as follows

Trim Finish
Materials Materials

$9,000
$6,000 $12,000
$15,000.00 $12,000.00

5,000.00 2,000.00
$3.00 $6.00

ation, the remaining During the trim operation, the arms are
into chair seats and added to the deluxe and executive Only the executive model enters
andard model is sold models and the chair edges are the finish operation where the
eration. smoothed. padding is added.

Trim Finish
Operation Operation
$30,000 $18,000
$39,000 $24,000
$69,000 $42,000

5,000.00 2,000.00
$13.80 $21.00
EUP 4600
cost 15.00

ost includes the


osts for the extrusion
nits], respectively. The
00) units] and $12.00
ndard model is $52.50
ost includes the
osts for the extrusion
nits], respectively. The
00) units] and $12.00
ndard model is $52.50

deluxe model plus the


nit materials and
1.00 [($18,000 + $24,000)
$96.30 ($69.30 + $6.00 +

end of the period and that


sion. What is the balance of
Butteco has the following cost components for 100,000 units of product for the year:
Direct materials $200,000
Direct labor 100,000
Manufacturing overhead 200,000
Selling and administrative expense 150,000
All costs are variable except for $100,000 of manufacturing overhead and $100,000 of selling and administrative
expenses. The total costs to produce and sell 110,000 units for the year are

per unit
units 100,000
Direct materials $200,000 $2.00
Direct labor $100,000 $1.00
V.Manufacturing overhead $100,000 $1.00
F.Manufacturing overhead $100,000
Selling and administrative expense $50,000 $0.50
F.Selling and administrative expense $100,000

V.Manuf. $4.00
F.manf $100,000
V.SGA $0.50
F.SGA $100,000

The total costs to produce and sell 110,000 units for the year are
110000 $695,000.00

Madtack Company’s beginning and ending inventories for the


month of November are
November. 1 November .30
Direct materials $67,000 $62,000
Work-in-process $145,000 $171,000
Finished goods $85,000 $78,000

Production data for the month of November follows:


Direct labor $200,000
Actual overhead $132,000
Direct materials purchased $163,000
Transportation in $4,000
Purchase returns and allowances $2,000

Madtack uses one overhead control account and charges


overhead to production at 70% of direct labor cost. The
company does not formally recognize over/underapplied
overhead until year-end.

$370,000

CoGs Total
Beg.RM 67,000
Add: Purchase 163,000
Less : Returns and discounts 2,000
Net Purchase 161,000
Add: Freight-in 4,000
RM Available for use 232,000
Less : End.RM 62,000
DM used in Production 170,000
DL 200,000
MOH 140,000
Total manufacturing costs for the period 510,000
Add: Beg. WIP 145,000
Less: End. WIP 171,000
Costs of goods Manufacturing 484,000
Add: Beg.Finished 85,000
Goods Available for sale 569,000
Less: End. Finished 78,000
Costs of goods Sold 491,000

Lucy Sportswear manufactures a specialty line of T-shirts using a job-order costing system. During March, the following
costs were incurred in completing job ICU2: direct materials, $13,700; direct labor, $4,800; administrative, $1,400; and
selling, $5,600. Overhead was applied at the rate of $25 per machine hour, and job ICU2 required 800 machine hours. If
job ICU2 resulted in 7,000 good shirts, the cost of goods sold per unit would be

CoGs Total
Beg.RM 13,700
Add: Purchase
Less : Returns and discounts
Net Purchase -
Add: Freight-in
RM Available for use 13,700
Less : End.RM
DM used in Production 13,700
DL 4,800
MOH 20,000
Total manufacturing costs for the period 38,500
Add: Beg. WIP -
Less: End. WIP -
Costs of goods Manufacturing 38,500
Add: Beg.Finished
Goods Available for sale 38,500
Less: End. Finished
Costs of goods Sold 38,500

Zeta Company is preparing its annual profit plan. As part of


its analysis of the profitability of individual products, the
controller estimates the amount of overhead that should be
allocated to the individual product lines from the
information given as follows:

Wall Mirrors
Units produced 25 25 25
Material moves per product line 5 15 5
Direct labor hours per unit 200 200 200
Budgeted materials handling costs $50,000 $50,000

Under a costing system that allocates overhead on the basis of direct labor hours, Zeta Company’s materials handling cos
allocated to one unit of wall mirrors would be
allocated to 25 25000
allocated to one unit $1,000

Under activity-based costing (ABC), Zeta’s materials handling costs allocated to one unit of wall mirrors would be

allocated to 25 12500
allocated to one unit $500

New-Rage Cosmetics has used a traditional cost accounting system to apply quality control costs uniformly to all products
at a rate of 14.5% of direct labor cost. Monthly direct labor cost for Satin Sheen makeup is $27,500. In an attempt to
distribute quality control costs more equitably, New-Rage is considering activity-based costing. The monthly data shown
in the chart below have been gathered for Satin Sheen makeup.
85. A
Taylor Corporation is determining the cost behavior of several items in order to budget
for the upcoming year. Past trends have indicated the following dollars were spent at
three different levels of output.

In establishing a budget for 14,000 units, Taylor should treat Costs A, B, and C,
respectively, as

Unit Levels Unit Levels


10000 per unit 12000 per unit 15000 per unit
A $25,000 #DIV/0! $29,000 $2.42 $35,000 $2.33 semi
B $10,000 #DIV/0! $15,000 $1.25 $15,000 $1.00 fixed
C $15,000 #DIV/0! $18,000 $1.50 $22,500 $1.50 VC per unit fixed

97. A company employs a just-in-time (JIT) production system and utilizes backflush
accounting. All acquisitions of raw materials are recorded in a raw materials control
account when purchased. All conversion costs are recorded in a control account as
incurred, while the assignment of conversion costs are from an allocated conversion cost
account. Company practice is to record the cost of goods manufactured at the time the
units are completed using the estimated budgeted cost of the goods manufactured.
The budgeted cost per unit for one of the company's products is as follows

Direct Materials $15.00 15


Conversion costs 35.00 35
Total budgeted unit cost $50.00 50

During the current accounting period, 80,000 units of product were completed, and
75,000 units were sold. The entry to record the cost of the completed units for the period
would be
b. Finished Goods - Control 4,000,000 4000000
Raw Material - Control 1,200,000 1200000
Conversion Cost Allocated 2,800,000. 2800000

99. From the following budgeted data, calculate the budgeted indirect cost rate that would be
used in a normal costing system

Total direct labor hours 250,000 250,000


Direct costs $10,000,000 $ 10,000,000
Total indirect labor hours 50,000 50,000
Total indirect-labor-related costs $ 5,000,000 $ 5,000,000
Total indirect non-labor related costs $ 7,000,000 $ 7,000,000

Total indirect costs $ 12,000,000


Total allocation base 250,000
$ 48

104. B b. $8,500.
The marketing manager of Ames Company has learned the following about a new
product that is being introduced by Ames. Sales of this product are planned at $100,000 100000
for the first year. Sales commission expense is budgeted at 8% of sales plus the 8.0%
marketing manager's incentive budgeted at an additional ½%. The preparation of a 0.5%
product brochure will require 20 hours of marketing salaried staff time at an average rate
of $100 per hour, and 10 hours, at $150 per hour, for an outside illustrator's effort. The 100
variable marketing cost for this new product will be

Sales commission expense 8000


500
$ 8,500 variable marketing cost for this new product will be

110. A
110. Bethany Company has just completed the first month of producing a new product but has
not yet shipped any of this product. The product incurred variable manufacturing costs of
$5,000,000, fixed manufacturing costs of $2,000,000, variable marketing costs of
$1,000,000, and fixed marketing costs of $3,000,000. $1,000,000

If Bethany uses the variable cost method to value inventory, the inventory value of the new product would be
administrative
370,000 Madtack Company’s prime cost for November is

Madtack Company’s total manufacturing cost for November is

Madtack Company’s cost of goods transferred to finished goods inventory for November is

Madtack Company’s cost of goods sold for November is

March, the following


istrative, $1,400; and
800 machine hours. If
7000 5.5

Specialty Windows total


25 50
15 20
200 400

y’s materials handling costs

mirrors would be

uniformly to all products


0. In an attempt to
he monthly data shown
Fixed cost fixed in total &
variable cost fixed per unit
er unit fixed

97. B

80000
99. D d. $48.

20

$5,000,000
$2,000,000
$3,000,000

uct would be $ 5,000,000


or November is
Dremmon Corporation uses a standard cost accounting system. Data for the last fiscal
year are as follows.
Units
Beginning inventory of finished goods 100
Production during the year 700
Sales 750
Ending inventory of finished goods 50

Per Unit
Product selling price $200 $200
Standard variable manufacturing cost 90 $90
Standard fixed manufacturing cost 20* $20
$110
Budgeted selling and administrative costs (all fixed) $45,000 45,000
*Denominator level of activity is 750 units for the year. 750

Absorb.
sales rev $150,000
Cost of Goods Sold
Beginning inventory $11,000
Variable manufacturing costs $63,000
Fixed manufacturing costs $14,000
Cost for goods available for sale $88,000
deduct ending inventory $5,500
Total COGS(at standard costs) $82,500
Adjustment for manufacturing variances $1,000
Total COGS $83,500
Gross Margin $66,500
Operating cost $45,000
Operating income $21,500

There were no price, efficiency, or spending variances for the year, and actual selling and
administrative expenses equaled the budget amount. Any volume variance is written off
to cost of goods sold in the year incurred. There are no work-in-process inventories.
Assuming that Dremmon used absorption costing, the amount of operating income earned
in the last fiscal year was $21,500

Bethany Company has just completed the first month of producing a new product but has
not yet shipped any of this product. The product incurred variable manufacturing costs of
$5,000,000, fixed manufacturing costs of $2,000,000, variable marketing costs of
$1,000,000, and fixed marketing costs of $3,000,000.

variable manufacturing costs $5,000,000


fixed manufacturing costs $2,000,000
variable marketing costs $1,000,000
fixed marketing costs $3,000,000

If Bethany uses the variable cost method to value inventory, the inventory value of the
new product would be

$5,000,000

Consider the following situation for Weisman Corporation for the prior year.
• The company produced 1,000 units and sold 900 units, both as budgeted.
• There were no beginning or ending work-in-process inventories and no beginning
finished goods inventory.
• Budgeted and actual fixed costs were equal, all variable manufacturing costs are
affected by volume of production only, and all variable selling costs are affected
by sales volume only.
• Budgeted per unit revenues and costs were as follows.

Per Unit Total


Sales price $100 $100
Direct materials 30 $30
Direct labor 20 $20
Variable manufacturing costs 10 $10 $60
Fixed manufacturing costs 5 $5
Variable selling costs 12 $12
Fixed selling costs ($3,600 total) 4 $4 $3,600
Fixed administrative costs ($1,800 total) 2 $2 $1,800

The operating income for Weisman for the prior year using absorption costing was

Absorb.
sales rev $90,000
Cost of Goods Sold
Beginning inventory
Variable manufacturing costs $60,000
Fixed manufacturing costs $5,000
Cost for goods available for sale $65,000
deduct ending inventory $6,500
Total COGS(at standard costs) $58,500
Adjustment for manufacturing variances $0
Total COGS $58,500
Gross Margin $31,500
Operating cost $16,200
Operating income $15,300

Mill Corporation had the following unit costs for the recently concluded calendar year.

Variable Fixed
Manufacturing $8 $3
Nonmanufacturing $2 $6

Inventory for Mill’s sole product totaled 6,000 units on January 1 and 5,200 units on
December 31. When compared to variable costing income, Mill’s absorption costing
income is

$2,400 Lower

During the month of May, Robinson Corporation sold 1,000 units. The cost per unit for
May was as follows.

Cost Per Unit


Direct materials $ 5.50 $5.5
Direct labor 3.00 $3.0
Variable manufacturing overhead 1.00 $1.0
Fixed manufacturing overhead 1.50 $1.5
Variable administrative costs .50 $0.5
Fixed administrative costs 3.50 $3.5

May’s income using absorption costing was $9,500. The income for May, if variable
costing had been used, would have been $9,125. The number of units Robinson
produced during May was 1,250
[Fact Pattern #36] The planned per-unit cost figure
Valyn Corporation employs an absorption costing system for
internal reporting purposes; however, the company is
considering using variable costing. Data regarding Valyn’s Direct materials
planned and actual operations for the calendar year are Direct labor
presented below. Variable manufacturing
overhead
Fixed manufacturing
overhead
Variable selling expenses
Fixed selling expenses
Planned Actual Variable administrative
Activity Activity expenses
Beginning finished goods Fixed administrative
inventory in units 35,000 35,000 expenses
Sales in units 140,000 125,000 Total
Production in units 140,000 130,000
endning finished goods 40,000
inventory in units

Valyn uses a predetermined manufacturing overhead rate for applying manufacturing overhead to its product; thus, a com
purposes. Any over- or underapplied manufacturing overhead is closed to the cost of goods sold account at the end of the

The beginning finished goods inventory for absorption costing purposes was valued at the previous year’s planned unit
planned unit manufacturing cost. There are no work-in-process inventories at either the beginning or the end of the year. T
planned and actual unit selling price for the current year was $ 70.00 per unit.

(Refers to Fact Pattern #36)


The value of Valyn Corporation’s actual ending finished goods inventory on the absorption costing basis was

(Refers to Fact Pattern #36)


Valyn Corporation’s total fixed costs expensed this year on the absorption costing basis were

Fixed selling & Admin

Beginning finished goods

Sales in units
Production in units

endning finished goods

Budgeted FMOH
Allocated MOH

Given that 125,000 of those units were sold, cost of


goods sold was debited for $625,000 of fixed overhead (125,000 units × $5)

At year-end, the underapplied overhead was also added to cost of goods sold.
Because production was expected to be 140,000 units, the overhead application rate for
the $700,000 of planned fixed manufacturing overhead was $5 per unit.

Only 130,000 units were manufactured. Hence,


$650,000 (130,000 units × $5) of overhead was applied to units in process.

Because inventory increased from 35,000 to


40,000 units (35,000 BI + 130,000 produced – 125,000 sold), $25,000 (5,000-unit increase × $5) of the applied fixed
manufacturing overhead for the period was inventoried, not expensed

Actual overhead was $715,000, so the


underapplied overhead was $65,000 ($715,000 – $650,000).

This amount was charged to cost of goods sold at year-end.


The total of the fixed costs expensed was therefore $2,095,000 ($980,000 selling expenses + $425,000 administrative
expenses + $625,000 standard manufacturing overhead costs of units sold + $65,000 underapplied overhead).

sales rev
Cost of Goods Sold
Beginning inventory
Variable manufacturing costs
Fixed manufacturing costs
FMOH=so
Cost for goods available for sale
deduct ending inventory
Total COGS(at standard costs)
FMOH=so

Adjustment for manufacturing variances


Total COGS
Gross Margin
Operating cost
Operating income
(Refers to Fact Pattern #36)
The value of Valyn Corporation’s actual ending finished goods inventory on the variable costing basis was

Using variable costing, the unit cost of ending inventory is $25 ($12 direct materials + $9
direct labor + $4 variable overhead). Given beginning inventory of 35,000 units, the ending inventory equals
40,000 units (35,000 BI + 130,000 produced – 125,000 sold). Thus, ending inventory was $1,000,000 (40,000
units × $25).

(Refers to Fact Pattern #36)


Valyn Corporation’s absorption costing operating income was

(Refers to Fact Pattern #36)


Valyn Corporation’s actual manufacturing contribution margin calculated on the variable costing basis was
$ 8,750,000.00
VM 3,125,000

8,750,000.00

(Refers to Fact Pattern #36)


The total variable cost currently expensed by Valyn Corporation on the variable costing basis was
4,375,000

(Refers to Fact Pattern #36)


The difference between Valyn Corporation’s operating income calculated on the absorption costing basis and calculated
on the variable costing basis was 25,000

Fact Pattern #37]


Osawa, Inc. planned and actually manufactured 200,000 units of its single product in its first year of operations. Variable
manufacturing costs were $30 per unit of product. Planned and actual fixed manufacturing costs were $600,000, and sellin
administrative costs totaled $400,000. Osawa sold 120,000 units of product at a selling price of $40 per unit.

(Refers to Fact Pattern #37)


Osawa’s operating income using absorption (full) costing is

sales rev 4,800,000


Cost of Goods Sold
Beginning inventory
Variable manufacturing costs 6,000,000
Fixed manufacturing costs 600,000
Cost for goods available for sale 6,600,000
deduct ending inventory 2,640,000
Total COGS(at standard costs) 3,960,000
Adjustment for manufacturing variances
Total COGS 3,960,000
Gross Margin 840,000
Operating cost 400,000
Operating income 440,000

85. A
Taylor Corporation is determining the cost behavior of several items in order to budget
for the upcoming year. Past trends have indicated the following dollars were spent at
three different levels of output.

In establishing a budget for 14,000 units, Taylor should treat Costs A, B, and C,
respectively, as

Unit Levels Unit Levels


10000 per unit 12000 per unit 15000 per unit
A $25,000 $2.50 $29,000 $2.42 $35,000 $2.33 semi
B $10,000 $1.00 $15,000 $1.25 $15,000 $1.00 fixed
C $15,000 $1.50 $18,000 $1.50 $22,500 $1.50 VC per unit fixed

105. Which of the following correctly shows the treatment of (1) factory insurance, (2) direct
labor, and (3) finished goods shipping costs under absorption costing and variable
costing?
Absorption Costing Variable Costing
Product Cost Period Cost Product Cost Period Cost
a. 1, 2 3 2 1, 3.
b. 2 1, 3 1, 2 3.
c. 1, 2 3 1 2, 3.
d. 1 2, 3 2, 3 1.

Product Cost under absorption costing Product Cost under variable costing
All Manufacturing Costs such as: All variable Manufacturing Costs such as:
DM DM
DL DL
MOH VMOH

Product Cost under absorption costing Product Cost under variable costing
All Manufacturing Costs such as: All variable Manufacturing Costs such as:
DM DM
DL DL
VMOH VMOH
FMOH factory insurance

Both methods treat SGA as aperiod cost, finished goods shipping costs

(1) factory insurance, FMOH


(2) direct labor, VMOH
and (3) finished goods shipping costs SGA
Budgeted FOH
$15,000

FOH allocated PVV


VC $14,000 $1,000 U
$150,000

$9,000
$63,000
$15,000
$87,000 We did not use the appropriate format
in solving the VC method
$4,500
$82,500

$67,500
$45,000
$22,500

Assuming that Dremmon used V costing, the amount of operating income earned
$22,500
production 1000
sales 900
end.inv. 100

Budgeted FOH Allocated PVV


$5,000 $5,000 $0

VC
$90,000

$60,000
$5,000
$65,000
We did not use the appropriate format in
solving the VC method
We did not use the appropriate format in
6000 solving the VC method
$59,000
$0
$59,000
$31,000
$16,200
$14,800

BI 6000
E.Inv 5200
Diff 800

1000

$9,500.0
$9,125.0
Diff $375.0 250
lanned per-unit cost figures shown in the next schedule were based on the estimated production and sale of 140,000 units for the year
Planned Costs Incurred Costs
Per Unit Total
12 1,680,000 1,560,000 12
9 1,260,000 1,170,000 9
ble manufacturing 4 560,000 4 520,000

manufacturing
5 700,000 715,000
ble selling expenses 8 1,120,000 1,000,000
selling expenses 7 980,000 980,000
ble administrative
2 280,000 250,000
administrative
3 420,000 425,000
$ 50 $ 7,000,000 $ 6,620,000

to its product; thus, a combined manufacturing overhead rate of $ 9.00 per unit was employed for absorp
account at the end of the reporting year.

ous year’s planned unit


ng or the end of the year. The

ting basis was 1,200,000.00

1,405,000
The beginning finished goods inventory included 35,000 units, each of
which had absorbed $5 of fixed manufacturing overhead.
175,000

Given that 125,000 of those units were sold, cost of goods sold was
625,000 debited for $625,000 of fixed overhead (125,000 units × $5).
Given that 125,000 of those units were sold, cost of goods sold was
debited for $625,000 of fixed overhead (125,000 units × $5).

650,000 Each unit produced during the year also absorbed $5 of fixed
manufacturing overhead

200,000

700,000
650,000
715,000 50,000 u pvv
15,000 u FBV
625,000

700,000

650,000 Applied OH = Actual production X Budgeted Rate

of the applied fixed

25,000

715,000 65,000

25,000 administrative
ed overhead).
2,095,000

FMOH=sold*5+underapplied
FMOH=sold*5+underapplied

1,000,000

ntory equals
000 (40,000

ng basis and calculated

r of operations. Variable 200,000


s were $600,000, and selling and $ 30.00 600,000 3
$40 per unit. 400,000 120,000 40
80,000

(Refers to Fact Pattern #37)


Osawa’s operating income for the year using variable costing is
VC
4,800,000

6,000,000
600,000
6,600,000
2,400,000
4,200,000

4,200,000
600,000
400,000
200,000

Fixed cost fixed in total & variable


cost fixed per unit

105. A
variable costing
cturing Costs such as:
40,000 units for the year.

was employed for absorption costing


RC HS
CM 4 5
units 10000 15000 25000
0.4 0.6
1.6 3 4.6

fc 94000 20434.78261

8173.913
Nash Glassworks Company has budgeted fixed manufacturing overhead of $100,000 per month. The
company uses absorption costing for both external and internal financial reporting purposes. Budgeted overhead
rates for cost allocations for the month of April using alternative unit output denominator levels are shown in the
next column.

budgeted fixed manufacturing overhead $100,000.00


Actual production 800,000.00
sales 700,000.00
end inv. 100,000.00
Budgeted Fixed Manufacturing Overhead Cost per unit allocated
Theoreticalcapacity 1,500,000.00 $0.067 $53,333.33
Practicalcapacity 1,250,000.00 $0.080 $64,000.00
Normalcapacity utilization 775,000.00 $0.129 $103,225.81
Master-budgetcapacity 800,000.00 $0.125 $100,000.00
utilization

The choice of a production volume level as a denominator in the computation of fixed overhead r
significantly affect reported net income. Which one of the following statements is true for Nash G
Company if its beginning inventory is zero, production exceeded sales, and variances are adjustm
goods sold? The choice of Practical capacity as the denominator level will result in a lower net inc
if master-budget capacity is chosen.

Answer (A) is correct. The choice of practical rather than master budget capacity as the denominator level will result in a lo
costing net income. Practical capacity is the maximum level at which output is produced efficiently, with an allowance for un
interruptions, for example, for holidays and scheduled maintenance. Because this level will be higher than master-budget (ex
use will usually result in the underapplication of fixed overhead. For example, given costs of $100,000 and master-budget cap
800,000 units, $.125 per unit is the application rate. If practical capacity is 1,250,000 units, the application rate is $.08 per un
is 800,000 units, fixed overhead will not be over- or underapplied given the use of master-budget capacity. However, there w
units × $.08) of underapplied fixed overhead if practical capacity is the denominator level. Consequently, given that the begin
and that production exceeded sales, less fixed overhead will be inventoried at the lower practical capacity rate than at the m
Thus, master-budget net income will be greater.
onth. The
s. Budgeted overhead
evels are shown in the

PVV ending inv will absorb


$46,666.67 U $6,666.67
$36,000.00 U $8,000.00
-$3,225.81 F $12,903.23
$0.00 $12,500.00

utation of fixed overhead rates can


atements is true for Nash Glassworks
and variances are adjustments to cost of
will result in a lower net income amount than

enominator level will result in a lower absorption


fficiently, with an allowance for unavoidable
l be higher than master-budget (expected) capacity, its
of $100,000 and master-budget capacity of
the application rate is $.08 per unit. If actual production
budget capacity. However, there will be $36,000 (450,000
Consequently, given that the beginning inventory is zero
actical capacity rate than at the master-budget rate.
Dremmon Corporation uses a standard cost accounting system. Data for the last fiscal
year are as follows.
Units
Beginning inventory of finished goods 100
Production during the year 700
Sales 750
Ending inventory of finished goods 50

Per Unit
Product selling price $200 $200
Standard variable manufacturing cost 90 $90
Standard fixed manufacturing cost 20* $20
$110
Budgeted selling and administrative costs (all fixed) $45,000 45,000
*Denominator level of activity is 750 units for the year. 750

Absorb.
sales rev $150,000
Cost of Goods Sold
Beginning inventory $11,000
Variable manufacturing costs $63,000
Fixed manufacturing costs $14,000
Cost for goods available for sale $88,000
deduct ending inventory $5,500
Total COGS(at standard costs) $82,500
Adjustment for manufacturing variances $1,000
Total COGS $83,500
Gross Margin $66,500
Operating cost $45,000
Operating income $21,500

There were no price, efficiency, or spending variances for the year, and actual selling and
administrative expenses equaled the budget amount. Any volume variance is written off
to cost of goods sold in the year incurred. There are no work-in-process inventories.
Assuming that Dremmon used absorption costing, the amount of operating income earned
in the last fiscal year was $21,500

Bethany Company has just completed the first month of producing a new product but has
not yet shipped any of this product. The product incurred variable manufacturing costs of
$5,000,000, fixed manufacturing costs of $2,000,000, variable marketing costs of
$1,000,000, and fixed marketing costs of $3,000,000.

variable manufacturing costs $5,000,000


fixed manufacturing costs $2,000,000
variable marketing costs $1,000,000
fixed marketing costs $3,000,000

If Bethany uses the variable cost method to value inventory, the inventory value of the
new product would be

$5,000,000

Consider the following situation for Weisman Corporation for the prior year.
• The company produced 1,000 units and sold 900 units, both as budgeted.
• There were no beginning or ending work-in-process inventories and no beginning
finished goods inventory.
• Budgeted and actual fixed costs were equal, all variable manufacturing costs are
affected by volume of production only, and all variable selling costs are affected
by sales volume only.
• Budgeted per unit revenues and costs were as follows.

Per Unit Total


Sales price $100 $100
Direct materials 30 $30
Direct labor 20 $20
Variable manufacturing costs 10 $10 $60
Fixed manufacturing costs 5 $5
Variable selling costs 12 $12
Fixed selling costs ($3,600 total) 4 $4 $3,600
Fixed administrative costs ($1,800 total) 2 $2 $1,800

The operating income for Weisman for the prior year using absorption costing was

Absorb.
sales rev $90,000
Cost of Goods Sold
Beginning inventory
Variable manufacturing costs $60,000
Fixed manufacturing costs $5,000
Cost for goods available for sale $65,000
deduct ending inventory $6,500
Total COGS(at standard costs) $58,500
Adjustment for manufacturing variances $0
Total COGS $58,500
Gross Margin $31,500
Operating cost $16,200
Operating income $15,300

Mill Corporation had the following unit costs for the recently concluded calendar year.

Variable Fixed
Manufacturing $8 $3
Nonmanufacturing $2 $6

Inventory for Mill’s sole product totaled 6,000 units on January 1 and 5,200 units on
December 31. When compared to variable costing income, Mill’s absorption costing
income is

$2,400 Lower

During the month of May, Robinson Corporation sold 1,000 units. The cost per unit for
May was as follows.

Cost Per Unit


Direct materials $ 5.50 $5.5
Direct labor 3.00 $3.0
Variable manufacturing overhead 1.00 $1.0
Fixed manufacturing overhead 1.50 $1.5
Variable administrative costs .50 $0.5
Fixed administrative costs 3.50 $3.5

May’s income using absorption costing was $9,500. The income for May, if variable
costing had been used, would have been $9,125. The number of units Robinson
produced during May was 1,250
[Fact Pattern #36] The planned per-unit cost figure
Valyn Corporation employs an absorption costing system for
internal reporting purposes; however, the company is
considering using variable costing. Data regarding Valyn’s Direct materials
planned and actual operations for the calendar year are Direct labor
presented below. Variable manufacturing
overhead
Fixed manufacturing
overhead
Variable selling expenses
Fixed selling expenses
Planned Actual Variable administrative
Activity Activity expenses
Beginning finished goods Fixed administrative
inventory in units 35,000 35,000 expenses
Sales in units 140,000 125,000 Total
Production in units 140,000 130,000
endning finished goods 40,000
inventory in units

Valyn uses a predetermined manufacturing overhead rate for applying manufacturing overhead to its product; thus, a com
purposes. Any over- or underapplied manufacturing overhead is closed to the cost of goods sold account at the end of the

The beginning finished goods inventory for absorption costing purposes was valued at the previous year’s planned unit
planned unit manufacturing cost. There are no work-in-process inventories at either the beginning or the end of the year. T
planned and actual unit selling price for the current year was $ 70.00 per unit.

(Refers to Fact Pattern #36)


The value of Valyn Corporation’s actual ending finished goods inventory on the absorption costing basis was

(Refers to Fact Pattern #36)


Valyn Corporation’s total fixed costs expensed this year on the absorption costing basis were

Fixed selling & Admin

Beginning finished goods

Sales in units
Production in units

endning finished goods

Budgeted FMOH
Allocated MOH

Given that 125,000 of those units were sold, cost of


goods sold was debited for $625,000 of fixed overhead (125,000 units × $5)

At year-end, the underapplied overhead was also added to cost of goods sold.
Because production was expected to be 140,000 units, the overhead application rate for
the $700,000 of planned fixed manufacturing overhead was $5 per unit.

Only 130,000 units were manufactured. Hence,


$650,000 (130,000 units × $5) of overhead was applied to units in process.

Because inventory increased from 35,000 to


40,000 units (35,000 BI + 130,000 produced – 125,000 sold), $25,000 (5,000-unit increase × $5) of the applied fixed
manufacturing overhead for the period was inventoried, not expensed

Actual overhead was $715,000, so the


underapplied overhead was $65,000 ($715,000 – $650,000).

This amount was charged to cost of goods sold at year-end.


The total of the fixed costs expensed was therefore $2,095,000 ($980,000 selling expenses + $425,000 administrative
expenses + $625,000 standard manufacturing overhead costs of units sold + $65,000 underapplied overhead).

sales rev
Cost of Goods Sold
Beginning inventory
Variable manufacturing costs
Fixed manufacturing costs
FMOH=so
Cost for goods available for sale
deduct ending inventory
Total COGS(at standard costs)
FMOH=so

Adjustment for manufacturing variances


Total COGS
Gross Margin
Operating cost
Operating income
(Refers to Fact Pattern #36)
The value of Valyn Corporation’s actual ending finished goods inventory on the variable costing basis was

Using variable costing, the unit cost of ending inventory is $25 ($12 direct materials + $9
direct labor + $4 variable overhead). Given beginning inventory of 35,000 units, the ending inventory equals
40,000 units (35,000 BI + 130,000 produced – 125,000 sold). Thus, ending inventory was $1,000,000 (40,000
units × $25).

(Refers to Fact Pattern #36)


Valyn Corporation’s absorption costing operating income was

(Refers to Fact Pattern #36)


Valyn Corporation’s actual manufacturing contribution margin calculated on the variable costing basis was
$ 8,750,000.00
VM 3,125,000

8,750,000.00

(Refers to Fact Pattern #36)


The total variable cost currently expensed by Valyn Corporation on the variable costing basis was
4,375,000

(Refers to Fact Pattern #36)


The difference between Valyn Corporation’s operating income calculated on the absorption costing basis and calculated
on the variable costing basis was 25,000

Fact Pattern #37]


Osawa, Inc. planned and actually manufactured 200,000 units of its single product in its first year of operations. Variable
manufacturing costs were $30 per unit of product. Planned and actual fixed manufacturing costs were $600,000, and sellin
administrative costs totaled $400,000. Osawa sold 120,000 units of product at a selling price of $40 per unit.

(Refers to Fact Pattern #37)


Osawa’s operating income using absorption (full) costing is

sales rev 4,800,000


Cost of Goods Sold
Beginning inventory
Variable manufacturing costs 6,000,000
Fixed manufacturing costs 600,000
Cost for goods available for sale 6,600,000
deduct ending inventory 2,640,000
Total COGS(at standard costs) 3,960,000
Adjustment for manufacturing variances
Total COGS 3,960,000
Gross Margin 840,000
Operating cost 400,000
Operating income 440,000

85. A
Taylor Corporation is determining the cost behavior of several items in order to budget
for the upcoming year. Past trends have indicated the following dollars were spent at
three different levels of output.

In establishing a budget for 14,000 units, Taylor should treat Costs A, B, and C,
respectively, as

Unit Levels Unit Levels


10000 per unit 12000 per unit 15000 per unit
A $25,000 $2.50 $29,000 $2.42 $35,000 $2.33 semi
B $10,000 $1.00 $15,000 $1.25 $15,000 $1.00 fixed
C $15,000 $1.50 $18,000 $1.50 $22,500 $1.50 VC per unit fixed
Budgeted FOH
$15,000

FOH allocated PVV


VC $14,000 $1,000 U
$150,000

$9,000
$63,000
$15,000
$87,000 We did not use the appropriate format
in solving the VC method
$4,500
$82,500

$67,500
$45,000
$22,500

Assuming that Dremmon used V costing, the amount of operating income earned
$22,500
production 1000
sales 900
end.inv. 100

Budgeted FOH Allocated PVV


$5,000 $5,000 $0

VC
$90,000

$60,000
$5,000
$65,000
We did not use the appropriate format in
solving the VC method
We did not use the appropriate format in
6000 solving the VC method
$59,000
$0
$59,000
$31,000
$16,200
$14,800

BI 6000
E.Inv 5200
Diff 800

1000

$9,500.0
$9,125.0
Diff $375.0 250
lanned per-unit cost figures shown in the next schedule were based on the estimated production and sale of 140,000 units for the year
Planned Costs Incurred Costs
Per Unit Total
12 1,680,000 1,560,000 12
9 1,260,000 1,170,000 9
ble manufacturing 4 560,000 4 520,000

manufacturing
5 700,000 715,000
ble selling expenses 8 1,120,000 1,000,000
selling expenses 7 980,000 980,000
ble administrative
2 280,000 250,000
administrative
3 420,000 425,000
$ 50 $ 7,000,000 $ 6,620,000

to its product; thus, a combined manufacturing overhead rate of $ 9.00 per unit was employed for absorp
account at the end of the reporting year.

ous year’s planned unit


ng or the end of the year. The

ting basis was 1,200,000.00

1,405,000
The beginning finished goods inventory included 35,000 units, each of
which had absorbed $5 of fixed manufacturing overhead.
175,000

Given that 125,000 of those units were sold, cost of goods sold was
625,000 debited for $625,000 of fixed overhead (125,000 units × $5).
Given that 125,000 of those units were sold, cost of goods sold was
debited for $625,000 of fixed overhead (125,000 units × $5).

650,000 Each unit produced during the year also absorbed $5 of fixed
manufacturing overhead

200,000
107 a - Under absorption costing the fixed sel
fixed costs. Additionally, $5 per unit of fixed m
is $625,000. In total, this is now $2,030,000 in
applied overhead is closed to cost of goods so
over-applied) or added to (in the case of unde
700,000 calculated above. Overhead was applied at $5
650,000 overhead was applied. The actual incurred wa
(a form of expensing). This brings the total to
715,000 50,000 u pvv
15,000 u FBV
625,000

700,000

650,000 Applied OH = Actual production X Budgeted Rate

of the applied fixed

25,000

715,000 65,000

25,000 administrative
ed overhead).
2,095,000

FMOH=sold*5+underapplied
FMOH=sold*5+underapplied

1,000,000

ntory equals
000 (40,000

ng basis and calculated

r of operations. Variable 200,000


s were $600,000, and selling and $ 30.00 600,000 3
$40 per unit. 400,000 120,000 40
80,000

(Refers to Fact Pattern #37)


Osawa’s operating income for the year using variable costing is
VC
4,800,000

6,000,000
600,000
6,600,000
2,400,000
4,200,000

4,200,000
600,000
400,000
200,000

Fixed cost fixed in total & variable


cost fixed per unit
40,000 units for the year.

was employed for absorption costing


on costing the fixed selling ($980,000) and administrative ($425,000) costs are expensed in full. This is $1,405,000 of
y, $5 per unit of fixed manufacturing costs will be expensed for each unit sold. There were 125,000 units sold, so this
his is now $2,030,000 in expensed fixed costs. However, we are not done. The question states that any over- or under-
osed to cost of goods sold. This means that any over- or under-applied overhead will be deducted from (in the case of
d to (in the case of under-applied) the amount we
rhead was applied at $5 per unit produced. 130,000 units were produced, so this means that $650,000 of fixed
The actual incurred was $715,000. This $65,000 of under-applied overhead needs to be added to cost of goods sold
This brings the total to $2,095,000 of fixed costs that were expensed.
,405,000 of
sold, so this
ver- or under-
n the case of

f fixed
f goods sold
A firm has budgeted sales of 10,000 units of its sole product at $17 per
unit. Variable costs are expected to be $10 per unit and fixed costs are budgeted at
$50,000. A comparison of budgeted to actual results is as follows:

budgeted sales 10,000 $170,000 Actual sales 11000


Price $17 Price $16
VC $10 $100,000 $10
FC $50,000 $50,000
CM $7 $6
operating income $20,000

Although sales were greater than predicted, the contribution margin is less than
expected.
a) The discrepancy can be analyzed in terms of the sales price variance and
the sales volume variance.
Selling Price = (Actual sel
Variance pric
sales price variance -$11,000 U

For the single product, the sales volume variance is the change in contribution
margin caused by the difference between the actual and budgeted volume
(holding price constant).
a) In the example, it equals $7,000 F (1,000-unit increase in volume ×
Operating Income sales
$7 budgeted UCM). Volume variance

sales volume variance $7,000 F

The sales mix variance is zero because the firm sells only one product. Hence,
the sales volume variance equals the sales quantity variance.
5) The sales price variance ($11,000 U) combined with the sales volume variance
($7,000 F) equals the total change in the contribution margin ($4,000 U).

equals the total change in the contribution margin ($4,000 U).


-$4,000

If a company produces two or more products, the multiproduct sales variances reflect not
only the effects of the change in total unit sales, but also any difference in the mix of
products sold.

Plastic Metal
Comprehensive example:

Budgeted selling price per unit $6.00 $10.00 $6.0 $10.0


Budgeted variable cost per unit 3.00 7.50 $3.0 $7.5
Budgeted contribution margin per unit $3.00 $ 2.50 $3.0 $2.5
Budgeted unit sales 300 200 500 300 200
Budgeted mix percentage 60% 40% 100% 60% 40%
Actual units sold 260 260 520 260 260
Actual selling price per unit $6.00 $9.50 $6.0 $9.5

Budgeted mix percentage X Actual sales 312 208

As shown below (000 omitted), the total contribution margin variance was
$100 unfavorable ($130 unfavorable sales price variance – $30 favorable sales
volume variance).
Sales price variance:
Plastic 260 × ($6.00 – $6.00) $ 0 0 -130
Metal 260 × ($10 – $9.50) (130) $130 unfavorable
Sales volume variance:
Plastic (260 – 300) × $3.00 $(120) -$120.0 U $150.0
Metal (260 – 200) × $2.50 150 $ 30 favorable
Total contribution margin variance $100 unfavorable -$100.0

The sales volume variance may be broken down as follows: Sales quantity
variance:
Sales quantity variance:
Plastic [(520 × .6) – 300] × $3.00 $ 36 $36.0 F $20.0
Metal [(520 × .4) – 200] × $2.50 20 $ 56 favorable
Sales mix variance:
Plastic [260 – (520 × .6)] × $3.00 $(156) -$156.0 U $130.0
Metal [260 – (520 × .4)] × $2.50 130 $ 26 unfavorable
Sales volume variance $ 30 favorable
e. The sales quantity variance may be broken down into the market size and market
share variances.
1) The market size variance measures the effect on the contribution margin of the
difference between the actual market size in units and the budgeted market
size in units, assuming the market share percentage and the budgeted
weighted-average UCM are constant.
a) It equals the budgeted market share percentage, times the difference
between the actual market size in units and the budgeted market size in
units, times the budgeted weighted-average UCM.

The market share variance measures the effect on the contribution margin of
the difference between the actual and budgeted market share percentages,
assuming the actual market size in units and the budgeted weighted-average
UCM are constant.
a) It equals the difference between the actual market share percentage and
the budgeted market share percentage, times the actual market size in
units, times the budgeted weighted-average UCM.
3) EXAMPLE: Assume that a company’s budgeted and actual market sizes and
market shares are as follows:
Budget Actual
Market size in units 60,000 50,000
Market share 9% 10%

a) Assuming a budgeted weighted-average UCM of $3, the market size


variance is $2,700 U [(60,000 units – 50,000 units) × 9% × $3]. The
variance is unfavorable because market size diminished.
b) The market share variance is $1,500 F [50,000 units × $3 × (10% – 9%)].
The variance is favorable because market share increased. Thus, the
sales quantity variance is $1,200 U ($2,700 U – $1,500 F).
$176,000

$110,000

$16,000

Selling Price = (Actual selling - Flexible budget selling) x Unit sold


Variance price per Unit price per unit

Operating Income sales = (Actual Units sold - unites Budgeted) X Master budget CM per unit
Volume variance to be sold
Total

500
100%
520

520

Sales price variance:


U -$130.0

Sales volume variance:


F $30.0
Total contribution margin variance $100 unfavorable
-$100.0

F $56.0 F

F -$26.0 U
the market size variance is
W av. UCM
$3.0
1% $2,700.0 U

The market share variance is


$1,500.0 F

sales quantity variance is


$1,200.0 U
1. Account for all units (physical flow of quantities).

Beginning WIP Total Units to Account for =


Started Units this period Finished or Transferred-out Goods
Total Units to Account for - Ending WIP
Spoilage (lost)

Step 2: compute Equivalent Units of Production


EUP Computation under FIFO (If materials are added as work in process (continually):
Material Conversion

Total units Completed


- Beginning WIP (regardless of % of completion)
Units Started and Completed this Period - -
+ Amount needed to complete Beginning WIP
+ Amount Completed on Ending WIP
EUP under FIFO - -

If materials are added at the beginning of a process :


Total Units Completed
+ Amount of materials Needed to Complete BWIP zero
.+ Amount of materials Added to Date on EWIP 100%
EUP for Materials -

If materials are added at the end of a process


Total Units Completed
+ Amount of materials Needed to Complete BWIP
+ Amount of materials Added to Date on EWIP zero
EUP for Materials -

EUP : Weighted Average


Ü  EUP under weighted average costing may be computed as follows:
Total Units Completed this period
Work to date on Ending WIP
EUP under weighted average -
-
d-out Goods
[Fact Pattern #24]
Goggle-eyed Old Snapping Turtle, a sporting goods Costs for the Forming Department for the m
manufacturer, buys wood as a direct material for baseball bats. were as follows:
The Forming Department processes the baseball bats, and the Direct materials
bats are then transferred to the Finishing Department where a Conversion costs
sealant is applied. The Forming Department began Total
manufacturing 10,000 “Casey Sluggers” during the month of A total of 8,000 bats were completed and tr
May. There was no beginning inventory. Finishing Department; the remaining 2,000
the forming process at the end of the mont
Forming Department’s direct materials were
process, but, on average, only 25% of the co
was applied to the ending work-in-process i

1. Account for all units (physical flow of quantities).

Beginning WIP Total Units to Account for =


Started Units this period 10,000.00 Finished or Transferred-out Goods
Total Units to Account for 10,000.00 Ending WIP
Spoilage (lost)

cost 33,000.00 17,000.00


Step 2: compute Equivalent Units of Production
EUP Computation under FIFO (If materials are added as work in process (continually):
Material Conversion

Total units Completed 8,000.00 8,000.00


- Beginning WIP (regardless of % of completion)
Units Started and Completed this Period 8,000.00 8,000.00
+ Amount needed to complete Beginning WIP
+ Amount Completed on Ending WIP 2,000.00 500.00
EUP under FIFO 10,000.00 8,500.00

3.30 2.00 5.30

6,600.00 1,000.00

[Fact Pattern #25]


Levittown Company employs a process cost system for its
manufacturing operations. All direct materials are added at Work-in-process on November
the beginning of the process and conversion costs are added (60% complete as to conversio
proportionately. Levittown’s production quantity schedule for Units started during November
November is reproduced in the next column. Total units to account for
Units completed and transferre
from beginning inventory
Units started and completed d
Work-in-process on November
(20% complete as to conversio
Total units accounted for

1. Account for all units (physical flow of quantities).

Beginning WIP 1,000.00 Total Units to Account for =


Started Units this period 5,000.00 Finished or Transferred-out Goods
Total Units to Account for 6,000.00 Ending WIP
Spoilage (lost)

Step 2: compute Equivalent Units of Production


EUP Computation under FIFO (If materials are added as work in process (Beg):
Material Conversion

Total units Completed 4,000.00 4,000.00


- Beginning WIP (regardless of % of completion) 1,000.00 1000
Units Started and Completed this Period 3,000.00 3,000.00
+ Amount of materials Needed to Complete BWIP zero 400
.+ Amount of materials Added to Date on EWIP 100% 2,000.00 400
EUP under FIFO 5,000.00 3,800.00

(Refers to Fact Pattern #25)


Using the FIFO method, Levittown’s equivalent units for direct materials for November are

EUP : Weighted Average Material Conversion


Ü  EUP under weighted average costing may be computed as follows:
Total Units Completed this period 4,000.00 4,000.00
Work to date on Ending WIP 2,000.00 400.00
EUP under weighted average 6,000.00 4,400.00

(Refers to Fact Pattern #25)


Using the weighted-average method, Levittown’s equivalent units for direct materials for November are
e Forming Department for the month of May

33,000.00
17,000.00
50,000.00
000 bats were completed and transferred to the 8000
epartment; the remaining 2,000 bats were still in 2000
process at the end of the month. All of the
partment’s direct materials were placed in
t, on average, only 25% of the conversion cost
to the ending work-in-process inventory.

nt for = 10,000.00
nsferred-out Goods 8,000.00
2,000.00

(Refers to Fact Pattern #24)


The cost of the units transferred to Snapping Turtle’s Finishing Department is
42,400.00
(Refers to Fact Pattern #24)
7,600.00 The cost of the work-in-process inventory in Snapping Turtle’s Forming Department at the end o

Units
Work-in-process on November 1
(60% complete as to conversion costs) 1000
Units started during November 5000
Total units to account for 6000
Units completed and transferred out
from beginning inventory 1000
Units started and completed during November 3000
Work-in-process on November 30
(20% complete as to conversion costs) 2000
Total units accounted for 6000

nt for = 6,000.00
nsferred-out Goods 4,000.00
2,000.00

(Refers to Fact Pattern #25)


Using the FIFO method, Levittown’s equivalent units for conversion costs for November are

as follows:

(Refers to Fact Pattern #25)


Using the weighted-average method, Levittown’s equivalent units for conversion costs for November are

November are
Department at the end of May is
1. Account for all units (physical flow of quantities).

Beginning WIP 16,000.00 Total Units to Account for =


Started Units this period 100,000.00 Finished or Transferred-out Goods
Total Units to Account for 116,000.00 Ending WIP
Spoilage (lost)

Step 2: compute Equivalent Units of Production


EUP Computation under FIFO (If materials are added as work in process (continually):
Material Conversion

Total units Completed 92,000.00 92000


- Beginning WIP (regardless of % of completion) 16,000.00 16000
Units Started and Completed this Period 76,000.00 76,000.00
+ Amount needed to complete Beginning WIP 6,400.00 12800
+ Amount Completed on Ending WIP 21,600.00 9600
EUP under FIFO 104,000.00 98,400.00

Using the FIFO method, Kimbeth’s equivalent unit cost of materials for May is

$ 4.50 $ 5.83 5.83

Using the FIFO method, Kimbeth’s the total cost of units in the ending work-in-process inventory at May 31 is

97,200.00 $ 55,968.00 $ 153,168

EUP : Weighted Average


  EUP under weighted average costing may be computed as follows:

Material Conversion
Total Units Completed this period 92,000.00 92000
Work to date on Ending WIP 21,600.00 9600
EUP under weighted average 113,600.00 101,600.00

Using the weighted-average method, Kimbeth’s equivalent unit cost of materials for May is

$ 4.60 $ 6.00
116,000.00
rred-out Goods 92,000.00
24,000.00

The beginning inventory was 60% complete for materials and 20% complete for
conversion costs. The ending inventory was 90% complete for materials and
40% complete for conversion costs.

Beginning WIP 16000


Started in production during May 100000
Completed production during May 92000
Ending work-in-process inventory, 24000

Beginning inventory costs for materials $ 54,560


Beginning inventory costs for conversion $ 35,560
Costs incurred during May for materials $ 468,000
Costs incurred during May for conversion $ 574,040
The Raymar Company example

The firm has established a $200,000 line of credit with its bank at a 12% annual rate of interest on which borrowin
$10,000 increments. There is no outstanding balance on the line of credit loan on April 1. Principal repayments ar
there is a surplus of cash.
Interest is to be paid monthly. If there are no outstanding balances on the loans, Raymar will invest any cash in ex
balance in U.S. Treasury bills. Raymar intends to maintain a minimum balance of $100,000 at the end of each mo
below the minimum balance or investing any excess cash. Expected monthly collection and disbursement pattern

• Collections. 50% of the current month’s sales budget and 50% of the previous month’s sales budget.
• Accounts payable disbursements. 75% of the current month’s accounts payable budget and 25% of the
previous month’s accounts payable budget.
• All other disbursements occur in the month in which they are budgeted.

March April May


Sales 40,000 50,000 100,000
Accounts payable 30,000 40,000 40,000
Payroll 60,000 70,000 50,000
Other disbursements 25,000 30,000 10,000

(Refers to Fact Pattern #13)


In April, Raymar’s budget will result in a need to borrow $100,000 on its line of credit for the cash defic

Budget Information
Total Cash receipt
March April May
20,000.00 25,000.00 50,000.00
20,000.00 25,000.00
Total Cash disb
22,500.00 30,000.00 30,000.00
7,500.00 10,000.00

Total Cash receipt 20,000.00 45,000.00 75,000.00


Total Cash disb 107,500 137,500.00 101,000.00
Net cash flow (87,500) (92,500) (26,000)

interest on April loan will be paid on May


(Refers to Fact Pattern #13)
In May, Raymar will be required to Borrow an additional $20,000 and pay $1,000 interest.

1) calculate the excess cash that result because Raymer borrow with $10,000 increment as it will be the beginning balance in M
According to the loan provisions Raymer must borrow an amount of $100,000 to cover the $92,500 deficit
thus it will have an excess balance with an amount of $7,500
The loan amount 100,000
The deficit to be covered (92,500)
the excess cash in April 7,500

2)Calculate the interest to be paid in May


Interest for one month to be paid in May 1,000

May total cash receipts 75,000


May total cash disbursements 101,000
Thus, disbursements exceed receipts by (26,000)
cash has a beginning surplus balance of 7,500
As a result, the company needs to borrow an additional of $18,500 eliminate its cash defi (18,500)
its cash deficit.

Total cash needed in May (18,500)

Karmee Company has been accumulating operating data in


order to prepare an annual profit plan. Details regarding
Karmee’s sales for the first 6 months of the coming year are
as follows:

Estimated Monthly Sales Type of Monthly Sale Collection Pattern for Credit Sales
Jan 600,000 Cash sales 20% Month of sale
feb 650,000 Credit sales 80% One month following sale
march 700,000 Second month following sale
april 625,000
may 720,000
june 800,000

Karmee’s cost of goods sold averages 40% of the sales value. Karmee’s objective is to maintain a target inventory equal to 30
Purchases of merchandise for resale are paid for in the month following the sale.
Karmee’s cost of goods sold averages 40% of the sales value. Karmee’s objective is to maintain a target inventory equal to 30
Purchases of merchandise for resale are paid for in the month following the sale.

The variable operating expenses (other than cost of goods sold) for Karmee are 10% of sales and are paid for in the month
following the sale. The annual fixed operating expenses are presented below. All of these are incurred uniformly
throughout the year and paid monthly except for insurance and property taxes. Insurance is paid quarterly in January, April,
July, and October. Property taxes are paid twice a year in April and October.

1) Calculate the COGS/ Target E.inv/V.operating cost/fixed cost


Estimated Monthly Sales COGS Target E.inventory The variable operating expenses
Jan 600,000 240,000 78,000.0 60,000
feb 650,000 260,000 84,000.0 65,000
march 700,000 280,000 75,000.0 70,000
april 625,000 250,000 86,400.0 62,500
may 720,000 288,000 96,000.0 72,000
june 800,000 320,000 - 80,000

Jan feb march april may


Cash sales 120,000 130,000 140,000 125,000 144,000
Credit sales 480,000 520,000 560,000 500,000 576,000
Collection from credit sales
Month of sale 144,000 156,000 168,000 150,000 172,800
One month following sale 192,000 208,000 224,000 200,000
Second month following sale 120,000 130,000 140,000

Collection from total sales 264,000 478,000 636,000 629,000 656,800

(Refers to Fact Pattern #14)


The amount of cash collected in March for Karmee Company from the sales made during March w

308,000

(Refers to Fact Pattern #14)


Karmee Company’s total cash receipts for the month of April will be

629,000
(Refers to Fact Pattern #14)
The purchase of merchandise that Karmee Company will need to make during February will be

Purchase Budget= 266,000.0


Purchase for the month's sale 260,000
.+ .Target Ending Inv. 84,000.0
.-Beginning Inv. 78,000.0
266,000.0

(Refers to Fact Pattern #14)


The amount for cost of goods sold that will appear on Karmee Company’s pro forma income state
February will be
260,000

(Refers to Fact Pattern #14)


The total cash disbursements that Karmee Company will make for the operating expenses (expen
goods sold) during the month of April will be
385,000

Historically, Pine Hill Wood Products has had no significant bad debt experience with its customers.

Cash sales have accounted for 10% of total sales, 10%


payments for credit sales have been received as follows:
40% of credit sales in the month of the sale 40%
30% of credit sales in the first subsequent month 30%
25% of credit sales in the second subsequent month 25%
5% of credit sales in the third subsequent month 5%
January February March April May
Cash sales 9500 6500 7000 8000 8500
Collection from credit sales
40% of credit sales in the month 34200 23400 25200 28800 30600
30% of credit sales in the first subsequent mo 25650 17550 18900 21600
25% of credit sales in the second subsequent month 21375 14625 15750
5% of credit sales in the third subsequent month 4275 2925
43700 55550 71125 74600 79375

(Refers to Fact Pattern #15)


What is the forecasted cash inflow for Pine Hill Wood Products for May?

79,375

(Refers to Fact Pattern #15)


Due to deteriorating economic conditions, Pine Hill Wood Products has now decided that its cash forecast should include
a bad debt adjustment of 2% of credit sales, beginning with sales for the month of April. The 5% collection in the fourth
month should be reduced to reflect the bad debt. Because of this policy change, the total expected cash inflow in April
related to sales made in April will Be unchanged.

The estimated collections in July related to April credit sales will be reduced by $1,440.
Estimated collections in the month of the sale will be unchanged.

[Fact Pattern #18]


Super Drive, a computer disk storage and back-up company, uses accrual accounting. The company’s Statement of Financial
Position for the year ended November 30, is as follows:

Super Drive
Statement of Financial Position
Position November 30

Assets
Cash 52,000
Accounts receivable, net 150,000
Inventory 315,000
Property, plant, and equipment 1,000,000
Total assets 1,517,000

Additional information regarding Super Drive’s operations include the following:


Sales are budgeted at $520,000 for December and $500,000 for January of the next year.
Collections are expected to be 60% in the month of sale and 40% in the month following the sale.
80% of the disk drive components are purchased in the month prior to the month of sale, and 20% are purchased in the
month of sale. Purchased components are 40% of the cost of goods sold.
Payment for the components is made in the month following the purchase.
Cost of goods sold is 80% of sales.

Dec Jan
Sales 520,000 500,000
Collections are expected to be 60% in the month of sale 312,000 300,000
Collections are expected to be 40% in the month following the sale. 150,000 208,000

COGS 416,000 400,000


Purchased components are 40% of the cost of goods sold. 166,400 160,000
80% of the disk drive components are purchased in the month prior to the 128,000
20% are purchased in the month of sale. 33,280

Payment for the components is made in the month following the purchase.

(Refers to Fact Pattern #18)


Super Drive’s budgeted cash collections for the month of December are

462,000

(Refers to Fact Pattern #18)


Super Drive’s projected balance in accounts payable on December 31 is

161,280

(Refers to Fact Pattern #18)


Super Drive’s projected gross profit for the month ending December 31 is

104,000

[Fact Pattern #19]


Kelly Company is a retail sporting goods store that uses accrual accounting for its records. Facts regarding Kelly’s operations are
as follows:

Sales are budgeted at:


December Year 1 January Year 2.
220,000 200,000
Collections are expected to be 60% in the month of sale
132,000 120,000
Collections are expected to be 38% in the month following the sale.
83,600
Gross margin is 25% of sales. 55,000 50,000

COGS 165,000 150,000

A total of 80% of the merchandise held for resale is purchased in the month prior to the month of sale
120000

20% is purchased in the month of sale.


33000
Payment for merchandise is made in the month following the purchase.
153000
Other expected monthly expenses to be paid in cash are $22,600.
22600 22600
Annual depreciation is $216,000. 18000

2% BAD DEBT EXP 4,400.00

(Refers to Fact Pattern #19)


Kelly’s budgeted cash collections for December Year 1 are

208,000

(Refers to Fact Pattern #19)


Kelly’s pro forma income (loss) before income taxes for December Year 1 is

10,000

(Refers to Fact Pattern #19)


Kelly’s projected balance in accounts payable on December 31, Year 1, is

153,000

Kelly’s projected balance in inventory on December 31, Year 1, is

120,000
ate of interest on which borrowings for cash deficits must be made in
April 1. Principal repayments are to be made in any month in which

Raymar will invest any cash in excess of its desired end-of-month cash
$100,000 at the end of each month by either borrowing for deficits
ction and disbursement patterns are shown in the column to the right.

’s sales budget.
et and 25% of the

its line of credit for the cash deficit.

The net negative cash flow (amount to be borrowed to reach the required
minimum cash balance of $100,000) is $92,500 ($137,500 – $45,000). Bec
the line of credit must be drawn upon in $10,000 increments, the loan mu
for $100,000.
The net negative cash flow (amount to be borrowed to reach the required
minimum cash balance of $100,000) is $92,500 ($137,500 – $45,000). Bec
the line of credit must be drawn upon in $10,000 increments, the loan mu
for $100,000.
and pay $1,000 interest.

t will be the beginning balance in May


$92,500 deficit

Given the requirement that loans be in $10,000 increments, the May


loan must be for $20,000.

ollection Pattern for Credit Sales


onth of sale 30%
ne month following sale 40%
cond month following sale 25%

ntain a target inventory equal to 30% of the next month’s sales in units.
ntain a target inventory equal to 30% of the next month’s sales in units.

Annual Fixed Operating Costs Monthly Quarterly Half-Year


les and are paid for in the month Advertising 720,000 60,000
are incurred uniformly
is paid quarterly in January, April, Depreciation 420,000 35,000
Insurance 180,000 45,000
Property taxes 240,000 120,000
Salaries 1,080,000 90,000

erating expenses The outflow of variable operating expenses Fixed Operating Costs
150,000 45,000
60,000 150,000
65,000 150,000
70,000 150,000 45,000 120,000
62,500 150,000
72,000 150,000
80,000 150,000

june
160,000
640,000

192,000
230,400
125,000

707,400

sales made during March will be


Cash sales are 20% of monthly sales, credit sales are 80% of monthly sales, and collectio
on credit sales are 30% in the month of sale. Consequently, cash collected during a mon
equals 44% [20% + (30% × 80%)] of sales for that month. Cash collections in March on
March sales were therefore $308,000 ($700,000 × 44%).

Cash collected during a month on sales for that month equals 44% of total sales. Hence, cash
receipts in April on April’s sales are $275,000 ($625,000 × 44%). April collections on March
credit sales equal $224,000 ($700,000 × 40% × 80%). April collections on February credit sales
equal $130,000 ($650,000 ×
25% × 80%). Thus, total cash receipts for April were $629,000 ($275,000 + $224,000 +
$130,000).
receipts in April on April’s sales are $275,000 ($625,000 × 44%). April collections on March
credit sales equal $224,000 ($700,000 × 40% × 80%). April collections on February credit sales
equal $130,000 ($650,000 ×
25% × 80%). Thus, total cash receipts for April were $629,000 ($275,000 + $224,000 +
$130,000).

e during February will be

Purchases equal cost of goods sold, plus ending inventory, minus beginning inventory.
Estimated cost of goods sold for February equals $260,000 ($650,000 sales × 40%). Ending
inventory is given as 30% of sales in units. Stated at cost, this amount equals $84,000
($700,000 March sales × 30% × 40%).
Furthermore, beginning inventory is $78,000 ($260,000 CGS for February × 30%). Thus,
purchases equal $266,000 ($260,000 + $84,000 – $78,000).

ny’s pro forma income statement for the month of


Cost of goods sold is expected to be 40% of sales. Thus, cost of goods sold is $260,000
($650,000 February sales × 40%).

operating expenses (expenses other than the cost of

Cash disbursements for variable operating expenses in April (excluding cost of goods sold)
equal $70,000 ($700,000 March sales × 10%). Cash disbursements for fixed operating
expenses (excluding depreciation, a noncash expense) include advertising ($720,000 ÷ 12 =
$60,000), salaries ($1,080,000 ÷ 12 = $90,000), insurance ($180,000 ÷ 4 = $45,000), and
property taxes ($240,000 ÷ 2 = $120,000). Hence, cash payments for April operating expenses
are $385,000 ($70,000 + $60,000 + $90,000 + $45,000 + $120,000).

The forecast for both cash and credit sales is as follows:


Month Sales CR sales
January 95000 85500
February 65000 58500
March 70000 63000
April 80000 72000
May 85000 76500
The cash inflows for May will come from May cash sales of $8,500 ($85,000 × 10%), May
credit sales of $30,600 ($85,000 × 90% × 40%), April sales of $21,600 ($80,000 × 30% ×
90%), March sales of $15,750 ($70,000 × 25% × 90%), and February sales of $2,925
($65,000 × 5% × 90%). The total is $79,375.

s cash forecast should include


e 5% collection in the fourth
xpected cash inflow in April

ompany’s Statement of Financial

Liabilities and shareholders’ equity


Accounts payable 175000
Common stock 900000
Retained earnings 442000
Total liabilities and
shareholders’ equity 1,517,000

nd 20% are purchased in the

Collections are expected to be 60% in the month of sale and 40% in the month
following the sale. Thus, collections in December consist of the $150,000 of
receivables at November 30, plus 60% of December sales. Total collections are
therefore $462,000 [$150,000 + ($520,000 × 60%)].

Payments are made in the month following purchase. The balance in accounts
payable on November 30 is $175,000; this amount will be paid in December. The
account is credited for purchases of a portion of components to be used for sales in
December (20% of December components) and for sales in January (80% of January
components). Cost of goods sold is 80% of sales, and components are 40% of cost of
goods sold. Thus, December component needs are $166,400 ($520,000 sales × 80% ×
40%), and January component needs are $160,000 ($500,000 sales × 80% × 40%). The
December purchases of December component needs equal $33,280 ($166,400 ×
20%).
December purchases of January component needs are $128,000 ($160,000 × 80%).
Hence, the total of December purchases (ending balance in accounts payable) equals
$161,280 ($33,280 + $128,000).

Given that cost of goods sold is 80% of sales, gross profit is 20% of sales.
Consequently, pro forma gross profit is $104,000 ($520,000 × 20%).
Given that cost of goods sold is 80% of sales, gross profit is 20% of sales.
Consequently, pro forma gross profit is $104,000 ($520,000 × 20%).

acts regarding Kelly’s operations are

Below is Kelly Company’s statement of financial position at November 30, Year 1.


Assets
Cash 22,000
Accounts receivable (net of $4,000 76,000
allowance for uncollectible accounts)
Inventory 132,000
Property, plant, and equipment (net of
$680,000 accumulated deprecation) 870,000
Total assets 1,100,000
Liabilities and Stockholders’ Equity
Accounts payable 162,000
Common stock 800,000
Retained earnings 138,000
Total liabilities and stockholders’ equity 1,100,000

Since collections are 60% of the current month’s sales and 38% of the
previous month’s sales, total collections should be

Sales are budgeted at $220,000. Given that cost of goods sold is 75% of sales, or
$165,000, gross profit is $55,000. Deduct cash expenses of $22,600, depreciation of
$18,000 ($216,000 ÷ 12), and bad debt expense of $4,400 ($220,000 × .02). This
leaves an income of $10,000
Sales are budgeted at $220,000. Given that cost of goods sold is 75% of sales, or
$165,000, gross profit is $55,000. Deduct cash expenses of $22,600, depreciation of
$18,000 ($216,000 ÷ 12), and bad debt expense of $4,400 ($220,000 × .02). This
leaves an income of $10,000

The balance is equal to the purchases made during December since all purchases are paid for in the
month following purchase. Purchases for December is given as 20% of December’s sales and 80% of
January’s sales. Thus, of the $220,000 of merchandise sold during December, 20%, or $44,000, would have
been purchased during the month. January’s sales are expected to be $200,000, so 80% of that amount,
or $160,000, would have been purchased during December. December purchases are thus estimated as
$204,000 at the company’s selling prices. The merchandise costs only 75% of the marked selling prices,
however. Therefore, the balance in the purchases account at month-end is projected to be $153,000
($204,000 × 75%).

The inventory is expected to be 80% of January’s needs. $200,000 projected January sales ×
80% = $160,000. Thus, the ending inventory would be goods that the company could sell for $160,000.
Given a gross margin of 25%, cost would only be 75% of sales, and ending inventory would be $120,000
($160,000 × 75%).
reach the required
00 – $45,000). Because
ments, the loan must be
reach the required
00 – $45,000). Because
ments, the loan must be

crements, the May


nthly sales, and collections
h collected during a month
ollections in March on
ods sold)
ting
000 ÷ 12 =
0), and
ng expenses
aid for in the
s and 80% of
44,000, would have
% of that amount,
hus estimated as
ed selling prices,
be $153,000
CoGs Total
Beg.RM 100,000
Add: Purchase 2,000,000
Less : Returns and discounts (150,000)
Net Purchase 1,850,000
Add: Freight-in 120,000
RM Available for use 2,070,000
Less : End.RM (500,000)
DM used in Production 2,570,000
DL 330,000
MOH 1,000,000
Total manufacturing costs for the period 3,900,000
Add: Beg. WIP -
Less: End. WIP -
Costs of goods Manufacturing 3,900,000
Add: Beg.Finished 850,000
Goods Available for sale 4,750,000
Less: End. Finished (350,000)
Costs of goods Sold 4,400,000

If the beginning balance for May of the materials inventory account was $27,500, the ending balance for May is $28,750,
and $128,900 of materials were used during the month, the materials purchased during the month cost

CoGs Total
Beg.RM 27,500
Add: Purchase
Less : Returns and discounts
Net Purchase -
Add: Freight-in
RM Available for use 27,500
Less : End.RM 28,750
DM used in Production (1,250)
DL
MOH
Total manufacturing costs for the period (1,250)
Add: Beg. WIP -
Less: End. WIP -
Costs of goods Manufacturing (1,250)
Add: Beg.Finished
Goods Available for sale (1,250)
Less: End. Finished
Costs of goods Sold (1,250)
balance for May is $28,750,

128900 130150
Job 298
Actual DM 4606 2003 Actual DL hours 27,000.00
Actual DL 1579 Actual MOH 1,215,000.00
6185 Actual rate $ 45.00
Actual DL hours for Job 298 88
Actual Allocated OH 88 X $45 $ 3,960.00 2003 Budgeted DL hours 28,000.00
Budgeted MOH 1,120,000.00
Total Manf. Cost $ 10,145.00 Budgeted rate $ 40.00
Actual Price $ 15,000.00
Gross Margin $ 4,855.00
Gross Margin % 32%
Applied MOH 1,080,000.00
Net (Dr) Balance 135,000.00 12.05%
Job 298Applied MOH 88X$40 $ 3,520.00
Total Manf. Cost(normal costing) $ 9,705.00 Dr CoGS 135,000.00
DR Applied 1,080,000.00
Diff $ 440.00 CR MOH control 1,215,000.00

DR Cogs 2,025.00 2%
DR WIP 3,915.00 3%
DR FG 129,060.00 96%
Dr Applied OH 1,080,000.00 135,000.00
CR MOH control 1,215,000.00

99. From the following budgeted data, calculate the budgeted indirect cost rate that would be
used in a normal costing system
Total direct labor hours 250,000 250,000
Direct costs $10,000,000 $ 10,000,000
Total indirect labor hours 50,000 50,000
Total indirect-labor-related costs $ 5,000,000 $ 5,000,000
Total indirect non-labor related costs $ 7,000,000 $ 7,000,000

Total indirect costs $ 12,000,000


Total allocation base 250,000
$ 48

100. Loyal Co. produces three types of men’s undershirts: T-shirts, V-neck shirts, and athletic 100. D
shirts. In the Folding and Packaging Department, operations costing is used to apply
costs to individual units, based on the standard time allowed to fold and package each
type of undershirt. The standard time to fold and package each type of undershirt is as
follows.
Allocation base
T-shirt 40 seconds per shirt 40 2,000,000.0 50,000 56%
V-neck shirt 40 seconds per shirt 40 1,200,000 30,000 33%
Athletic shirt 20 seconds per shirt 20 400,000 20,000 11%
100 3,600,000 100,000 1.0

During the month of April, Loyal produced and sold 50,000 T-shirts, 30,000 V-neck
shirts, and 20,000 athletic shirts. If costs in the Folding and Packaging Department were $ 78,200
$78,200 during April, how much folding and packaging cost should be applied to each Tshirt?

Budgeted rate $ 0.022


Applied OH 43,444 $ 43,444.44
per shirt 0.8689
99. D d. $48.
d. $.8689.
Goggle-eyed Old Snapping Turtle, a sporting goods
manufacturer, buys wood as a direct material for baseball bats.
The Forming Department processes the baseball bats, and the
bats are then transferred to the Finishing Department where a
sealant is applied. The Forming Department began
manufacturing 10,000 “Casey Sluggers” during the month of
May. There was no beginning inventory.
Costs for the Forming Department for the month of May
were as follows:
Direct materials $33,000
Conversion costs 17,000
Total $50,000
A total of 8,000 bats were completed and transferred to the
Finishing Department; the remaining 2,000 bats were still in
the forming process at the end of the month. All of the
Forming Department’s direct materials were placed in
process, but, on average, only 25% of the conversion cost
was applied to the ending work-in-process inventory.

Beginning WIP 0
Started Units this period 10000

Total Units to Account for 10000 10000

1. Finished or Transferred-out 8000


2. Ending WIP 2000

M C
Total units Completed
Beginning WIP (regardless of % of completion)
Units Started and Completed this Period 8000 8000
+ Amount needed to complete Beginning WIP
+ Amount Completed on Ending WIP 2000 500
10000 8500
3.30 2.00

The total cost transferred was therefore $42,400 26,400.00 16,000.00 42,400.00

6,600.00 1,000.00 7,600.00

New MCQ
Mack Inc. uses a weighted-average process costing system. Direct materials and
conversion costs are incurred evenly during the production process. During the month of
October, the following costs were incurred.
Direct materials $39,700
Conversion costs 70,000
The work-in-process inventory as of October 1 consisted of 5,000 units, valued at $4,300,
that were 20% complete. During October, 27,000 units were transferred out. Inventory
as of October 31 consisted of 3,000 units that were 50% complete. The weighted-average
inventory cost per unit completed in October was

Beginning WIP 5000 $4,300 20%


Started Units this period 25000

Total Units to Account for 30000 30000

1. Finished or Transferred-out 27000


2. Ending WIP 3000 50%

During December, Krause Chemical Company had the following selected data
concerning the manufacture of Xyzine, an industrial cleaner.
Production Flow Physical Units
Completed and transferred to the next department 100
Add: Ending work-in-process inventory 10 (40% complete as
to conversion)
Total units to account for 110
Less: Beginning work-in-process inventory 20 (60% complete as
to conversion)
Units started during December 90
All material is added at the beginning of processing in this department, and conversion
costs are added uniformly during the process. The beginning work-in-process inventory
had $120 of raw material and $180 of conversion costs incurred. Material added during
December was $540 and conversion costs of $1,484 were incurred. Krause uses the
weighted-average process-costing method. The total raw material costs in the ending
work-in-process inventory for December is
M CONV
Beginning WIP 20 60% conv $120 $180
Started Units this period 90 $540 $1,484
$660 $1,664
Total Units to Account for 110 110

1. Finished or Transferred-out 100


2. Ending WIP 10 40% conv

M conv
Ü  EUP under weighted average costing may be computed as follows:
Total Units Completed this period XX 100 100
Work to date on Ending WIP XX 10 4
_____
EUP under weighted average XXXX 110 104

6.00 16.00

The total raw material costs in the ending work-in-process invent 60.00

Southwood Industries uses a process costing system and inspects its goods at the end of
manufacturing. The inspection as of June 30 revealed the following information for the
month of June.
Good units completed 16,000 16000
Normal spoilage (units) 300
Abnormal spoilage (units) 100
Unit costs were: materials, $3.50; and conversion costs, $6.00. The number of units that
Southwood would transfer to its finished goods inventory and the related cost of these

M CONV
Beginning WIP XX
Started Units this period XX $3.5 $6.0
$3.5 $6.0 $9.5
Total Units to Account for 16000 0

1. Finished or Transferred-out 16000 $152,000.0


2. Ending WIP
3. NormalSpoilage (lost) 300 $2,850.0 product cost
4. Abnormal Spoilage (lost) 100 $950.0 period cost
The number of units that Southwood would transfer to its finished goods inventory and the related cost of these
16000

Colt Company uses a weighted-average process cost system to account for the cost of
producing a chemical compound. As part of production, Material B is added when the
goods are 80% complete. Beginning work-in-process inventory for the current month
was 20,000 units, 90% complete. During the month, 70,000 units were started in process,
and 65,000 of these units were completed. There were no lost or spoiled units. If the
ending inventory was 60% complete, the total equivalent units for Material B for the
month was

M CONV
Beginning WIP 20000 90%
Started Units this period 70000
$0.0 $0.0
Total Units to Account for 90000 90000

1. Finished or Transferred-out 65000


2. Ending WIP 25000 0

M conv
Ü  EUP under weighted average costing may be computed as follows:
Total Units Completed this period XX 65000 65000
Work to date on Ending WIP XX 0 15000
_____
EUP under weighted average XXXX 65000 80000
DM 33000
Conv 17000
50000

The cost of the work-in-process inventory in Snapping Turtle’s Forming Department at the end of May is
Total Units Completed this period XX 27000
Current cost Work to date on Ending WIP XX 1500
DM $39,700 _____
Conv $70,000 EUP under weighted average XXXX 28500
$109,700

$114,000 $4.0
related cost of these
$154,850.0
Price $200.00 100% $ 1.00
VC $120.00 60% $ 0.60
CM $80.00 40% $ 0.40
FC $2,000.00
CM% = CM/Price 40%
Break=FC/CM 25 Units
0 1 5 25
Revenues $0.00 $200.00 $1,000.00 $5,000.00
VC $0.00 $120.00 $600.00 $3,000.00
CM $0.00 $80.00 $400.00 $2,000.00
FC $2,000.00 $2,000.00 $2,000.00 $2,000.00
OI ($2,000.00) ($1,920.00) ($1,600.00) $0.00

CM% = CM/Price #DIV/0! 40% 40% 40%


VC%=VC/Price #DIV/0! 60% 60% 60%
Break $=FC/CM% $ 5,000.00

Q Margin of safety
$ Margin of safety

Target OI $ 1,200.00 TOI=Rev-VC-FC


TOI Q=FC+TOI/$CM 40 How many units must be sold to generate enough CM to cover FC and
.=since each $ of revenue earn 40 cent of CM What is the revenue in$ required to generate $1200 TOI??
thus to earn $3200 CM rev must =
3200/.4 $ 8,000.00 .=FC+TOI/CM%

Target NI= TOI-(TOI x%T)


Target NI= TOI x (1-T%)
Target OI= TNI / (1-T%)
Since, TOI=Rev-VC-FC
thus, Rev-VC-FC= TNI / (1-T%)
TNI $ 960.00 How many units must be sold to generate enough CM to cover FC and
TNI Q=FC+(TNI/1-T%)/$CM 45.00
TNI Revenue=FC+(TNI/1-T%)/%CM $ 9,000.00

Consider a decision about which features to add to an existing


product. Different choices can affect selling prices, variable cost
per unit, fixed costs, units sold, and operating income.

Strategic decisions invariably entail risk. CVP analysis evaluates how operating
income will be affected if the original predicted data are not achieved - say, if
sales are 10%lower than estimated. Evaluating this risk affects other strategic
decisions a company might make. For example, if the probability of a decline in
sales seems high, a manager may take actions to shift the cost structure to
have more variable costs and fewer fixed ,) costs.
Strategic decisions invariably entail risk. CVP analysis evaluates how operating
income will be affected if the original predicted data are not achieved - say, if
sales are 10%lower than estimated. Evaluating this risk affects other strategic
decisions a company might make. For example, if the probability of a decline in
sales seems high, a manager may take actions to shift the cost structure to
have more variable costs and fewer fixed ,) costs.

Decision to Advertise
New Q will be 44 instead of 40
New FC will be $500 higher than old FC due to Adv.
Selling Price will be the same

Units without Adv. Units with Adv. Diff


40 44 4
CM $3,200.00 $3,520.00 $ 320.00
FC $ (2,000.00) $ (2,500.00) $ (500.00)
OI $1,200.00 $1,020.00 $ (180.00)

Operating income decreases by $180, so Mary should not advertise.

Decision to Reduce Selling Price from $200 to $175


New VC will be $115 instead of $120
New CM will be $175-$115=$60, instead of $80 60
FC will be the same

Units without reducing. Units with reducing. Diff


40 50 10
CM $3,200.00 $3,000.00 $ (200.00)
FC $ (2,000.00) $ (2,000.00) 0
OI $1,200.00 $1,000.00 $ (200.00)

Contribution margin from lowering price to $175: ($175 - $115) per unit x 50 units $ 3,000.00
Contribution margin from maintaining price at $200: ($200 - $120) per unit x 40 units $ 3,200.00
Change in contribution margin from lowering price $ (200.00)

Mary can immediately see the revenues that need to be generated to reach particular operating-income levels, given a
costs and variable cost per unit. For example, revenues of $6,40.0 ($2Q{) per unit x 32 units) are required to earn an oper
fIxed/costs are $2,000 and variable cost per unit is $100. Mary can also use Exhibit 3-4 to assess what revenues she needs
operating income of $0) if, for example, the booth rental at the Chicago convention is raised to $2,800 (increasing fIxed co
software supplier raises its price to $150 (increasing variable cost to $150 per unit).

Rev
must
be
Rev
must
be
200
FC VC CM CM%
1 2000 $ 100.00 $ 100.00 50% rev must be
2 2000 $ 120.00 $ 80.00 40% rev must be
3 2000 $ 150.00 $ 50.00 25% rev must be
4 2400 $ 100.00 $ 100.00 50% rev must be
5 2400 $ 120.00 $ 80.00 40% rev must be
6 2400 $ 150.00 $ 50.00 25% rev must be
7 2800 $ 100.00 $ 100.00 50% rev must be
8 2800 $ 120.00 $ 80.00 40% rev must be
9 2800 $ 150.00 $ 50.00 25% rev must be

COST PLANNING AND CVP


Alternative fixed-Cost/Variable-Cost Structures

Compare line 2 (fIxed costs, $2,000; variable cost per unit, $120)-and
line 7 (fIxed costs, $2,800; variable cost per unit, $100) See how the
revenues required to break even are higher for line 7 ($5,600 versus
$5,000 in line 2) ..whereas the revenues required to earn $2,000 of
operating income are lower in line T ($9,600 versus $10,000 in line 2).
Line 7, with higher fIxed costs, has more risk of loss (has a higher
breakeven point) but offers a greater return (more profits) as revenues
increase.

CVP analysis can help managers evaluate various fIXed-costjvariable-cost


structures. To consider these choices in more detail, let's return to Do-All
Software. Mary is paying a $2,000 booth-rental fee. Suppose Computer
Conventions offers Mary three rental alternatives:
Option 1: $2,000 fIXed fee
Option 2: $800 fIXed fee plus 15% of convention revenues
Option 3: 25% of convention revenues with no fIxed fee

If Mary sells 40 units, she should be indifferent across t


various options. Each option results in operating incom
$1,200.

Quantity sold 40 Option 1


Price $ 200.00 $ 200.00
VC $ 120.00 VC 120
incremental VC in option 2 (15% from rev) $ 30.00 CM per unit $ 80.00
incremental VC in option 3 (25% from rev) $ 50.00 Quantity sold 40
CM $ 3,200.00
CM% 40%
FC 2000
OI $ 1,200.00

The CVP analysis, however, highlights the different risks of loss and different return
option if sales differ from 40 units. The higher risk of a loss in option 1 is because o
($2,000), which result in a higher breakeven point (25 units) and a lower margin of
units) relative to the other options. The line representing option 1 intersects the
horizontal axis farther to the right than the lines representing options 2 and 3.

Quantity sold 25 Option 1


Price $ 200.00 $ 200.00
VC $ 120.00 VC 120
incremental VC in option 2 (15% from rev) $ 30.00 CM per unit $ 80.00
incremental VC in option 3 (25% from rev) $ 50.00 Quantity sold 25
CM $ 2,000.00
CM% 40%
FC 2000
OI $ -
Q Break 25.00

Consider operating income under each option if the number of units sold dr
20. Exhibit 3-5 shows that option 1 leads to an operating loss, whereas optio
continue to generate operating incomes. (A vertical line from X = 20 units so
option 1 line below the horizontal axis in the mauve area and cuts the optio
lines above the horizontal axis in the blue-green area.) The higher risk ofloss
however, must be evaluated against its potential benefits. Option 1 has the
margin per unit because of its low variable costs.

Quantity sold 20 Option 1


Price $ 200.00 $ 200.00
VC $ 120.00 VC 120
incremental VC in option 2 (15% from rev) $ 30.00 CM per unit $ 80.00
incremental VC in option 3 (25% from rev) $ 50.00 Quantity sold 20
CM $ 1,600.00
CM% 40%
FC 2000
OI $ (400.00)
Q Break 25.00

Once fixed costs are recovered at sales of 25 units, each additional unit sold adds $80 of contribution margin and, therefo
$80 of operating income per unit. For example, at sales of 60 units, option 1 shows an operating income of $2,800, greate
60 units under options 2 and 3. By moving from option 1 toward option 3, Mary faces less risk of loss when demand is lo
because she loses less contribution margin per unit. She must, however, accept less operating income when
demand is high because of the higher variable costs of option 3 compared with options 1 and 2. The choice among option
confidence in the level of demand for the software package and her willingness to risk losses if demand
is low.
Once fixed costs are recovered at sales of 25 units, each additional unit sold adds $80 of contribution margin and, therefo
$80 of operating income per unit. For example, at sales of 60 units, option 1 shows an operating income of $2,800, greate
60 units under options 2 and 3. By moving from option 1 toward option 3, Mary faces less risk of loss when demand is lo
because she loses less contribution margin per unit. She must, however, accept less operating income when
demand is high because of the higher variable costs of option 3 compared with options 1 and 2. The choice among option
confidence in the level of demand for the software package and her willingness to risk losses if demand
is low.

Quantity sold 60 Option 1


Price $ 200.00 $ 200.00
VC $ 120.00 VC 120
incremental VC in option 2 (15% from rev) $ 30.00 CM per unit $ 80.00
incremental VC in option 3 (25% from rev) $ 50.00 Quantity sold 60
CM $ 4,800.00
CM% 40%
FC 2000
OI $ 2,800.00
Q Break 25.00

The risk-return tradeoff across alternative cost structures can be measured as operating leverage. Operating le
costs have on changes in operating income as changes occur in units sold and, hence, in contribution margin.
Organizations with a high proportion of fixed costs in their cost structures, as is the case under option 1, have h
representing option 1 in Exhibit 3-5 is the steepest of the three lines. Small increases in sales lead to large incre
incomes. Small decreases in sales result in relatively large decreases in operating incomes, leading to a greater r
level of sales, the degree of operating leverage equals contribution margin divided by operating income.

If Mary sells 40 units, she should be indifferent across t


various options. Each option results in operating incom
$1,200.

Quantity sold 40 Option 1


Price $ 200.00 $ 200.00
VC $ 120.00 VC 120
incremental VC in option 2 (15% from rev) $ 30.00 CM per unit $ 80.00
incremental VC in option 3 (25% from rev) $ 50.00 Quantity sold 40
CM $ 3,200.00
CM% 40%
FC 2000
OI $ 1,200.00

Operating Leverage=CM/OI $ 2.67

These numbers indicate that, when sales are 40 units, a percentage change in sales and contribution margin will result in
operating income for option 1, but the same percentage change in operating income for option 3.
Consider, for example, a sales increase of 50% from 40 to 60 units. Contribution margin will increase by 50% under each
increase by 2.67 x 50% = 133% from $1,200 to $2,800 in option 1, but it will increase only by 1.00 x 50% = 50% from $1,2
The degree of operating leverage at a given level of sales helps managers calculate the effect of fluctuations in sales
These numbers indicate that, when sales are 40 units, a percentage change in sales and contribution margin will result in
operating income for option 1, but the same percentage change in operating income for option 3.
Consider, for example, a sales increase of 50% from 40 to 60 units. Contribution margin will increase by 50% under each
increase by 2.67 x 50% = 133% from $1,200 to $2,800 in option 1, but it will increase only by 1.00 x 50% = 50% from $1,2
The degree of operating leverage at a given level of sales helps managers calculate the effect of fluctuations in sales
on operating incomes.

Quantity sold 60 Option 1


Price $ 200.00 $ 200.00
VC $ 120.00 VC 120
incremental VC in option 2 (15% from rev) $ 30.00 CM per unit $ 80.00
incremental VC in option 3 (25% from rev) $ 50.00 Quantity sold 60
CM $ 4,800.00
CM% 40%
FC 2000
OI $ 2,800.00
50 % in units sold will raise the CM with 50% in each option 50%

50 % in units sold will raise the OI with diff. % 133%


it is the same like%increase in CM x O.leverage 133%

Operating Leverage=CM/OI or CM/CM-FC $ 1.71

These numbers indicate that, when sales are 40 units, a percentage change in sales and contribution margin will result in
operating income for option 1, but the same percentage change in operating income for option 3.
Consider, for example, a sales increase of 50% from 40 to 60 units. Contribution margin will increase by 50% under each o
increase by 2.67 x 50% = 133% from $1,200 to $2,800 in option 1, but it will increase only by 1.00 x 50% = 50% from $1,20
The degree of operating leverage at a given level of sales helps managers calculate the effect of fluctuations in sales
on operating incomes.

old sales new sales


(1) % change in EBIT 133% $ 8,000.00 $ 12,000.00
(2) % change in output (sales) 50% 50%
DOL (1)/ (2) 2.67

(1) EBIT + FC it the same as CM $ 3,200.00


(2) EBIT $ 1,200.00
DOL (1)/ (2) 2.67

What does "DOLs 40 units = "2.67" really mean?


It means that a 1 percent change in SALES from the 40-unit sales position causes a 2.67 percen
percentage change in SALES from the 40-unit position causes a percentage change in EBIT tha
example, a 3 percent decrease in SALES causes a 8.01 percent decrease in EBIT, while a 4 perce
10.68 percent increase in EBIT.
What does "DOLs 40 units = "2.67" really mean?
It means that a 1 percent change in SALES from the 40-unit sales position causes a 2.67 percen
percentage change in SALES from the 40-unit position causes a percentage change in EBIT tha
example, a 3 percent decrease in SALES causes a 8.01 percent decrease in EBIT, while a 4 perce
10.68 percent increase in EBIT.

How would knowledge of a firm's DOL be of use to a financial manager?


The manager would know in advance what impact a potential change in sales would have on
response to this advance knowledge, the firm may decide to make some changes in its sales p
general rule, firms do not like to operate under conditions of a high degree of operating levera
drop in sales may lead to an operating loss.

The degree of operating leverage should thus be viewed as a measure of "potential risk" whic
presence of sales and production cost variability.The degree of operating leverage itself is not

12 20
8 15
RC HS
CM $4.0 $5.0
units 10000 15000 25000
40% 60%
$1.6 $3.0 $4.6

fc $94,000.0 20,434.78

8,173.91 12,260.87 20,434.78


$12.0 $20.0
$8.0 $15.0 If the product sales mix were t
RC HS the breakeven volume for each
CM $4.0 $5.0
units 6250 18750 25000
25% 75%
$1.0 $3.8 $4.8

fc $94,000.0 19,789.47

4,947.37 14,842.11 19,789.47


40 45
$8,000.00 $9,000.00
$4,800.00 $5,400.00
$3,200.00 $3,600.00
$2,000.00 $2,000.00
$1,200.00 $1,600.00

40% 40%
60% 60%

15 20
3,000 4,000

enough CM to cover FC and TOI???


ate $1200 TOI??

enough CM to cover FC and TNI???


ating-income levels, given alternative levels of fIxed
are required to earn an operating income of $1,200 if
ess what revenues she needs to break even (earn
o $2,800 (increasing fIxed costs to $2,800) or if the

Rev
must
be
Rev
must
be
To earn OI
zero $ 1,200.00 $ 1,600.00 $ 2,000.00
$ 4,000.00 $ 6,400.00 $ 7,200.00 $ 8,000.00
$ 5,000.00 $ 8,000.00 $ 9,000.00 $ 10,000.00
$ 8,000.00 $ 12,800.00 $ 14,400.00 $ 16,000.00
$ 4,800.00 $ 7,200.00 $ 8,000.00 $ 8,800.00
$ 6,000.00 $ 9,000.00 $ 10,000.00 $ 11,000.00
$ 9,600.00 $ 14,400.00 $ 16,000.00 $ 17,600.00
$ 5,600.00 $ 8,000.00 $ 8,800.00 $ 9,600.00
$ 7,000.00 $ 10,000.00 $ 11,000.00 $ 12,000.00
$ 11,200.00 $ 16,000.00 $ 17,600.00 $ 19,200.00

hould be indifferent across the


n results in operating income of

Option 2 Option 3
$ 200.00 $ 200.00
$ 150.00 $ 170.00
$ 50.00 $ 30.00
40 40
$ 2,000.00 $ 1,200.00
25% 15%
800 0
$ 1,200.00 $ 1,200.00

s of loss and different returns associated with each


loss in option 1 is because of its higher fixed costs
units) and a lower margin of safety (40 - 25 = 15
ng option 1 intersects the
enting options 2 and 3.

Option 2 Option 3
$ 200.00 $ 200.00
$ 150.00 $ 170.00
$ 50.00 $ 30.00
25 25
$ 1,250.00 $ 750.00
25% 15%
800 0
$ 450.00 $ 750.00
16.00 -

the number of units sold drops to


operating loss, whereas options 2 and 3
tical line from X = 20 units sold cuts the
auve area and cuts the options 2 and 3
area.) The higher risk ofloss in option 1,
l benefits. Option 1 has the highest contribution
s.

Option 2 Option 3
$ 200.00 $ 200.00
$ 150.00 $ 170.00
$ 50.00 $ 30.00
20 20
$ 1,000.00 $ 600.00
25% 15%
800 0
$ 200.00 $ 600.00
16.00 -

ribution margin and, therefore,


ting income of $2,800, greater than the operating incomes for sales of
k of loss when demand is low, both because of lower fixed costs and
g income when
2. The choice among options 1, 2, and 3 will be:;influenced by her
if demand
ribution margin and, therefore,
ting income of $2,800, greater than the operating incomes for sales of
k of loss when demand is low, both because of lower fixed costs and
g income when
2. The choice among options 1, 2, and 3 will be:;influenced by her
if demand

Option 2 Option 3
$ 200.00 $ 200.00
$ 150.00 $ 170.00
$ 50.00 $ 30.00
60 60
$ 3,000.00 $ 1,800.00
25% 15%
800 0
$ 2,200.00 $ 1,800.00
16.00 -

ng leverage. Operating leverage describes the effects that fixed


in contribution margin.
se under option 1, have high operating leverage. The line
n sales lead to large increases in operating
mes, leading to a greater risk of operating losses. At any given
operating income.

hould be indifferent across the


n results in operating income of

Option 2 Option 3
$ 200.00 $ 200.00
$ 150.00 $ 170.00
$ 50.00 $ 30.00
40 40
$ 2,000.00 $ 1,200.00
25% 15%
800 0
$ 1,200.00 $ 1,200.00

$ 1.67 $ 1.00

ribution margin will result in 2.67 times that percentage change in


on 3.
ncrease by 50% under each option. Operating income, however, will
1.00 x 50% = 50% from $1,200 to $1,800 in option 3 (see Exhibit 3-5).
of fluctuations in sales
ribution margin will result in 2.67 times that percentage change in
on 3.
ncrease by 50% under each option. Operating income, however, will
1.00 x 50% = 50% from $1,200 to $1,800 in option 3 (see Exhibit 3-5).
of fluctuations in sales

Option 2 Option 3
$ 200.00 $ 200.00
$ 150.00 $ 170.00
$ 50.00 $ 30.00
60 60
$ 3,000.00 $ 1,800.00 50%
25% 15%
800 0
$ 2,200.00 $ 1,800.00
50% 50%

83% 50%
83% 50%

$ 1.36 $ 1.00

ibution margin will result in 2.67 times that percentage change in


on 3.
crease by 50% under each option. Operating income, however, will
1.00 x 50% = 50% from $1,200 to $1,800 in option 3 (see Exhibit 3-5).
of fluctuations in sales

n causes a 2.67 percent change in EBIT. In fact, any


age change in EBIT that is 2.67 times as large. For
n EBIT, while a 4 percent increase in SALES causes a
n causes a 2.67 percent change in EBIT. In fact, any
age change in EBIT that is 2.67 times as large. For
n EBIT, while a 4 percent increase in SALES causes a

er?
n sales would have on operating profit. Sometimes, in
e changes in its sales policy and/ or cost structure. As a
ree of operating leverage. Since, in that situation, a small

f "potential risk" which becomes "active" only in the


g leverage itself is not the source of the variability.

Specialty Cakes Inc. produces two types of cakes, a 2 lbs. round cake and a 3 lbs. heartshaped
cake. Total fixed costs for the firm are $94,000. Variable costs and sales data for
the two types of cakes are presented below.
2 lbs. 3 lbs.
Round Cake Heart-shape Cake
Selling price per unit $12 $20
Variable cost per unit $8 $15
Current sales (units) 10,000 15,000
the product sales mix were to change to three heart-shaped cakes for each round cake,
e breakeven volume for each of these products would be
Price $200.00 100% $ 1.00
VC $120.00 60% $ 0.60
CM $80.00 40% $ 0.40
FC $2,000.00
CM% = CM/Price 40%
Q.Break=FC/CM 25
Units
0 1 5
Revenues $0.00 $200.00 $1,000.00
VC $0.00 $120.00 $600.00
CM $0.00 $80.00 $400.00
FC $2,000.00 $2,000.00 $2,000.00
OI ($2,000.00) ($1,920.00) ($1,600.00)

CM% = CM/Price #DIV/0! 40% 40%


VC%=VC/Price #DIV/0! 60% 60%
Break $=FC/CM%

Q Margin of safety =current Q- BE.Q


$ Margin of safety =current Rev-BE.Rev

Target OI $ 1,200.00 TOI=Rev-VC-FC


TOI Q=FC+TOI/$CM 40 How many units must be sold to generate enough CM to
.=since each $ of revenue earn 40 cent of CM What is the revenue in$ required to generate $1200 TOI?
thus to earn $3200 CM rev must =
3200/.4 $ 8,000.00 .=FC+TOI/CM%

Target NI= TOI-(TOI x%T)


Target NI= TOI x (1-T%)
Target OI= TNI / (1-T%)
Since, TOI=Rev-VC-FC
thus, Rev-VC-FC= TNI / (1-T%)
TNI $ 960.00 How many units must be sold to generate enough CM to
TNI Q=FC+(TNI/1-T%)/$CM 45.00
TNI Revenue=FC+(TNI/1-T%)/%CM $ 9,000.00

Consider a decision about which features to add to an existing product.


Different choices can affect selling prices, variable cost per unit, fixed costs,
units sold, and operating income.

Strategic decisions invariably entail risk. CVP analysis evaluates how operating income will be
affected if the original predicted data are not achieved - say, if sales are 10%lower than
estimated. Evaluating this risk affects other strategic decisions a company might make. For
example, if the probability of a decline in sales seems high, a manager may take actions to
shift the cost structure to have more variable costs and fewer fixed ,) costs.
Strategic decisions invariably entail risk. CVP analysis evaluates how operating income will be
affected if the original predicted data are not achieved - say, if sales are 10%lower than
estimated. Evaluating this risk affects other strategic decisions a company might make. For
example, if the probability of a decline in sales seems high, a manager may take actions to
shift the cost structure to have more variable costs and fewer fixed ,) costs.

Decision to Advertise
New Q will be 44 instead of 40
New FC will be $500 higher than old FC due to Adv.
Selling Price will be the same

Units without Adv. Units with Adv.


40 44
CM $3,200.00 $3,520.00
FC $ (2,000.00) $ (2,500.00)
OI $1,200.00 $1,020.00

Operating income decreases by $180, so Mary should not advertise.

Decision to Reduce Selling Price from $200 to $175


New VC will be $115 instead of $120
New CM will be $175-$115=$60, instead of $80 60
FC will be the same

Units without reducing. Units with reducing.


40 50
CM $3,200.00 $3,000.00
FC $ (2,000.00) $ (2,000.00)
OI $1,200.00 $1,000.00

Contribution margin from lowering price to $175: ($175 - $115) per unit x 50 units
Contribution margin from maintaining price at $200: ($200 - $120) per unit x 40 units
Change in contribution margin from lowering price

Mary can immediately see the revenues that need to be generated to reach particular operating-income levels, given a
variable cost per unit. For example, revenues of $6,40.0 ($2Q{) per unit x 32 units) are required to earn an operating inco
and variable cost per unit is $100. Mary can also use Exhibit 3-4 to assess what revenues she needs to break even (earn o
the booth rental at the Chicago convention is raised to $2,800 (increasing fIxed costs to $2,800) or if the software supplier
variable cost to $150 per unit).
Rev
must
be
200
FC VC CM CM%
1 2000 $ 100.00 $ 100.00 50%
2 2000 $ 120.00 $ 80.00 40%
3 2000 $ 150.00 $ 50.00 25%
4 2400 $ 100.00 $ 100.00 50%
5 2400 $ 120.00 $ 80.00 40%
6 2400 $ 150.00 $ 50.00 25%
7 2800 $ 100.00 $ 100.00 50%
8 2800 $ 120.00 $ 80.00 40%
9 2800 $ 150.00 $ 50.00 25%

COST PLANNING AND CVP


Alternative fixed-Cost/Variable-Cost Structures

Compare line 2 (fIxed costs, $2,000; variable cost per unit, $120)-and line 7 (fIxed
costs, $2,800; variable cost per unit, $100) See how the revenues required to break
even are higher for line 7 ($5,600 versus $5,000 in line 2) ..whereas the revenues
required to earn $2,000 of operating income are lower in line T ($9,600 versus
$10,000 in line 2). Line 7, with higher fIxed costs, has more risk of loss (has a higher
breakeven point) but offers a greater return (more profits) as revenues increase.

CVP analysis can help managers evaluate various fIXed-costjvariable-cost structures. To


consider these choices in more detail, let's return to Do-All Software. Mary is paying a
$2,000 booth-rental fee. Suppose Computer
Conventions offers Mary three rental alternatives:
Option 1: $2,000 fIXed fee
Option 2: $800 fIXed fee plus 15% of convention revenues
Option 3: 25% of convention revenues with no fIxed fee

If Mary sells 40 units, she should be indiff


various options. Each option results in ope
$1,200.

Quantity sold 40
Price $ 200.00
VC $ 120.00 VC
incremental VC in option 2 (15% from rev) $ 30.00 CM per unit
incremental VC in option 3 (25% from rev) $ 50.00 Quantity sold
CM
CM%
FC
OI

The CVP analysis, however, highlights the different risks of loss and different return
differ from 40 units. The higher risk of a loss in option 1 is because of its higher fixe
higher breakeven point (25 units) and a lower margin of safety (40 - 25 = 15 units) r
representing option 1 intersects the
horizontal axis farther to the right than the lines representing options 2 and 3.

Quantity sold 25
Price $ 200.00
VC $ 120.00 VC
incremental VC in option 2 (15% from rev) $ 30.00 CM per unit
incremental VC in option 3 (25% from rev) $ 50.00 Quantity sold
CM
CM%
FC
OI
Q Break

Consider operating income under each option if the number of units sold dr
20. Exhibit 3-5 shows that option 1 leads to an operating loss, whereas optio
continue to generate operating incomes. (A vertical line from X = 20 units so
option 1 line below the horizontal axis in the mauve area and cuts the optio
lines above the horizontal axis in the blue-green area.) The higher risk ofloss
however, must be evaluated against its potential benefits. Option 1 has the
margin per unit because of its low variable costs.

Quantity sold 20
Price $ 200.00
VC $ 120.00 VC
incremental VC in option 2 (15% from rev) $ 30.00 CM per unit
incremental VC in option 3 (25% from rev) $ 50.00 Quantity sold
CM
CM%
FC
OI
Q Break

Once fixed costs are recovered at sales of 25 units, each additional unit sold adds $80 of contribution margin and, therefo
$80 of operating income per unit. For example, at sales of 60 units, option 1 shows an operating income of $2,800, greate
under options 2 and 3. By moving from option 1 toward option 3, Mary faces less risk of loss when demand is low, both b
contribution margin per unit. She must, however, accept less operating income when
demand is high because of the higher variable costs of option 3 compared with options 1 and 2. The choice among option
the level of demand for the software package and her willingness to risk losses if demand
is low.
Once fixed costs are recovered at sales of 25 units, each additional unit sold adds $80 of contribution margin and, therefo
$80 of operating income per unit. For example, at sales of 60 units, option 1 shows an operating income of $2,800, greate
under options 2 and 3. By moving from option 1 toward option 3, Mary faces less risk of loss when demand is low, both b
contribution margin per unit. She must, however, accept less operating income when
demand is high because of the higher variable costs of option 3 compared with options 1 and 2. The choice among option
the level of demand for the software package and her willingness to risk losses if demand
is low.

Quantity sold 60
Price $ 200.00
VC $ 120.00 VC
incremental VC in option 2 (15% from rev) $ 30.00 CM per unit
incremental VC in option 3 (25% from rev) $ 50.00 Quantity sold
CM
CM%
FC
OI
Q Break

The risk-return tradeoff across alternative cost structures can be measured as operating leverage. Operating le
on changes in operating income as changes occur in units sold and, hence, in contribution margin.
Organizations with a high proportion of fixed costs in their cost structures, as is the case under option 1, have h
option 1 in Exhibit 3-5 is the steepest of the three lines. Small increases in sales lead to large increases in operati
incomes. Small decreases in sales result in relatively large decreases in operating incomes, leading to a greater r
the degree of operating leverage equals contribution margin divided by operating income.

If Mary sells 40 units, she should be indiff


various options. Each option results in ope
$1,200.

Quantity sold 40
Price $ 200.00
VC $ 120.00 VC
incremental VC in option 2 (15% from rev) $ 30.00 CM per unit
incremental VC in option 3 (25% from rev) $ 50.00 Quantity sold
CM
CM%
FC
OI

Operating Leverage=CM/OI

These numbers indicate that, when sales are 40 units, a percentage change in sales and contribution margin will result in
for option 1, but the same percentage change in operating income for option 3.
Consider, for example, a sales increase of 50% from 40 to 60 units. Contribution margin will increase by 50% under each
2.67 x 50% = 133% from $1,200 to $2,800 in option 1, but it will increase only by 1.00 x 50% = 50% from $1,200 to $1,800
These numbers indicate that, when sales are 40 units, a percentage change in sales and contribution margin will result in
for option 1, but the same percentage change in operating income for option 3.
Consider, for example, a sales increase of 50% from 40 to 60 units. Contribution margin will increase by 50% under each
2.67 x 50% = 133% from $1,200 to $2,800 in option 1, but it will increase only by 1.00 x 50% = 50% from $1,200 to $1,800
leverage at a given level of sales helps managers calculate the effect of fluctuations in sales
on operating incomes.

Quantity sold 60
Price $ 200.00
VC $ 120.00 VC
incremental VC in option 2 (15% from rev) $ 30.00 CM per unit
incremental VC in option 3 (25% from rev) $ 50.00 Quantity sold
CM
CM%
FC
OI
50 % in units sold will raise the CM with 50% in each option

50 % in units sold will raise the OI with diff. %


it is the same like%increase in CM x O.leverage

Operating Leverage=CM/OI or CM/CM-FC

These numbers indicate that, when sales are 40 units, a percentage change in sales and contribution margin will result in
for option 1, but the same percentage change in operating income for option 3.
Consider, for example, a sales increase of 50% from 40 to 60 units. Contribution margin will increase by 50% under each o
2.67 x 50% = 133% from $1,200 to $2,800 in option 1, but it will increase only by 1.00 x 50% = 50% from $1,200 to $1,800
leverage at a given level of sales helps managers calculate the effect of fluctuations in sales
on operating incomes.

old sales
(1) % change in EBIT 133% $ 8,000.00
(2) % change in output (sales) 50% 50%
DOL (1)/ (2) 2.67

(1) EBIT + FC it the same as CM $ 3,200.00


(2) EBIT $ 1,200.00
DOL (1)/ (2) 2.67

What does "DOLs 40 units = "2.67" really mean?


It means that a 1 percent change in SALES from the 40-unit sales position causes a 2.67 percen
change in SALES from the 40-unit position causes a percentage change in EBIT that is 2.67 tim
What does "DOLs 40 units = "2.67" really mean?
It means that a 1 percent change in SALES from the 40-unit sales position causes a 2.67 percen
change in SALES from the 40-unit position causes a percentage change in EBIT that is 2.67 tim
decrease in SALES causes a 8.01 percent decrease in EBIT, while a 4 percent increase in SALES c

How would knowledge of a firm's DOL be of use to a financial manager?


The manager would know in advance what impact a potential change in sales would have on
this advance knowledge, the firm may decide to make some changes in its sales policy and/ or
not like to operate under conditions of a high degree of operating leverage. Since, in that situa
operating loss.

The degree of operating leverage should thus be viewed as a measure of "potential risk" whic
sales and production cost variability.The degree of operating leverage itself is not the source o

12 20
8 15
RC HS
CM $4.0 $5.0
units 10000 15000 25000
40% 60%
$1.6 $3.0 $4.6

fc $94,000.0 20,434.78

8,173.91 12,260.87 20,434.78


$12.0 $20.0
$8.0 $15.0
RC HS
CM $4.0 $5.0
units 6250 18750 25000
25% 75%
$1.0 $3.8 $4.8

fc $94,000.0 19,789.47

4,947.37 14,842.11 19,789.47


Units
25 40 45
$5,000.00 $8,000.00 $9,000.00
$3,000.00 $4,800.00 $5,400.00
$2,000.00 $3,200.00 $3,600.00
$2,000.00 $2,000.00 $2,000.00
$0.00 $1,200.00 $1,600.00
25
40% 40% 40%
60% 60% 60%
$ 5,000.00

15 20
3,000 4,000 4800

ld to generate enough CM to cover FC and TOI???


uired to generate $1200 TOI??

ld to generate enough CM to cover FC and TNI???

1,600.00 2000 3,600.00


45.00
Diff
4
$ 320.00
$ (500.00)
$ (180.00)

se.

Diff
10
$ (200.00)
0
$ (200.00)

$ 3,000.00
$ 3,200.00
$ (200.00)

rating-income levels, given alternative levels of fIxed costs and


red to earn an operating income of $1,200 if fIxed/costs are $2,000
needs to break even (earn operating income of $0) if, for example,
00) or if the software supplier raises its price to $150 (increasing
Rev
must
be
To earn OI
zero $ 1,200.00 $ 1,600.00 $ 2,000.00
rev must be $ 4,000.00 $ 6,400.00 $ 7,200.00 $ 8,000.00
rev must be $ 5,000.00 $ 8,000.00 $ 9,000.00 $ 10,000.00
rev must be $ 8,000.00 $ 12,800.00 $ 14,400.00 $ 16,000.00
rev must be $ 4,800.00 $ 7,200.00 $ 8,000.00 $ 8,800.00
rev must be $ 6,000.00 $ 9,000.00 $ 10,000.00 $ 11,000.00
rev must be $ 9,600.00 $ 14,400.00 $ 16,000.00 $ 17,600.00
rev must be $ 5,600.00 $ 8,000.00 $ 8,800.00 $ 9,600.00
rev must be $ 7,000.00 $ 10,000.00 $ 11,000.00 $ 12,000.00
rev must be $ 11,200.00 $ 16,000.00 $ 17,600.00 $ 19,200.00

40 units, she should be indifferent across the


ns. Each option results in operating income of

Option 1 Option 2 Option 3


$ 200.00 $ 200.00 $ 200.00
120 $ 150.00 $ 170.00
$ 80.00 $ 50.00 $ 30.00
40 40 40
$ 3,200.00 $ 2,000.00 $ 1,200.00
40% 25% 15%
2000 800 0
$ 1,200.00 $ 1,200.00 $ 1,200.00

ks of loss and different returns associated with each option if sales


1 is because of its higher fixed costs ($2,000), which result in a
of safety (40 - 25 = 15 units) relative to the other options. The line

senting options 2 and 3.

Option 1 Option 2 Option 3


$ 200.00 $ 200.00 $ 200.00
120 $ 150.00 $ 170.00
$ 80.00 $ 50.00 $ 30.00
25 25 25
$ 2,000.00 $ 1,250.00 $ 750.00
40% 25% 15%
2000 800 0
$ - $ 450.00 $ 750.00
25.00 16.00 -

f the number of units sold drops to


operating loss, whereas options 2 and 3
rtical line from X = 20 units sold cuts the
auve area and cuts the options 2 and 3
n area.) The higher risk ofloss in option 1,
al benefits. Option 1 has the highest contribution
ts.

Option 1 Option 2 Option 3


$ 200.00 $ 200.00 $ 200.00
120 $ 150.00 $ 170.00
$ 80.00 $ 50.00 $ 30.00
20 20 20
$ 1,600.00 $ 1,000.00 $ 600.00
40% 25% 15%
2000 800 0
$ (400.00) $ 200.00 $ 600.00
25.00 16.00 -

tribution margin and, therefore,


ting income of $2,800, greater than the operating incomes for sales of 60 units
s when demand is low, both because of lower fixed costs and because she loses less

d 2. The choice among options 1, 2, and 3 will be:;influenced by her confidence in


tribution margin and, therefore,
ting income of $2,800, greater than the operating incomes for sales of 60 units
s when demand is low, both because of lower fixed costs and because she loses less

d 2. The choice among options 1, 2, and 3 will be:;influenced by her confidence in

Option 1 Option 2 Option 3


$ 200.00 $ 200.00 $ 200.00
120 $ 150.00 $ 170.00
$ 80.00 $ 50.00 $ 30.00
60 60 60
$ 4,800.00 $ 3,000.00 $ 1,800.00
40% 25% 15%
2000 800 0
$ 2,800.00 $ 2,200.00 $ 1,800.00
25.00 16.00 -

ting leverage. Operating leverage describes the effects that fixed costs have
ution margin.
ase under option 1, have high operating leverage. The line representing
to large increases in operating
omes, leading to a greater risk of operating losses. At any given level of sales,
ome.

40 units, she should be indifferent across the


ns. Each option results in operating income of

Option 1 Option 2 Option 3


$ 200.00 $ 200.00 $ 200.00
120 $ 150.00 $ 170.00
$ 80.00 $ 50.00 $ 30.00
40 40 40
$ 3,200.00 $ 2,000.00 $ 1,200.00
40% 25% 15%
2000 800 0
$ 1,200.00 $ 1,200.00 $ 1,200.00

$ 2.67 $ 1.67 $ 1.00

tribution margin will result in 2.67 times that percentage change in operating income

increase by 50% under each option. Operating income, however, will increase by
= 50% from $1,200 to $1,800 in option 3 (see Exhibit 3-5). The degree of operating
tribution margin will result in 2.67 times that percentage change in operating income

increase by 50% under each option. Operating income, however, will increase by
= 50% from $1,200 to $1,800 in option 3 (see Exhibit 3-5). The degree of operating

Option 1 Option 2 Option 3


$ 200.00 $ 200.00 $ 200.00
120 $ 150.00 $ 170.00
$ 80.00 $ 50.00 $ 30.00
60 60 60
$ 4,800.00 $ 3,000.00 $ 1,800.00 50%
40% 25% 15%
2000 800 0
$ 2,800.00 $ 2,200.00 $ 1,800.00
50% 50% 50%

133% 83% 50%


133% 83% 50%

$ 1.71 $ 1.36 $ 1.00

ribution margin will result in 2.67 times that percentage change in operating income

ncrease by 50% under each option. Operating income, however, will increase by
= 50% from $1,200 to $1,800 in option 3 (see Exhibit 3-5). The degree of operating

new sales
$ 12,000.00

on causes a 2.67 percent change in EBIT. In fact, any percentage


in EBIT that is 2.67 times as large. For example, a 3 percent
on causes a 2.67 percent change in EBIT. In fact, any percentage
in EBIT that is 2.67 times as large. For example, a 3 percent
cent increase in SALES causes a 10.68 percent increase in EBIT.

er?
n sales would have on operating profit. Sometimes, in response to
its sales policy and/ or cost structure. As a general rule, firms do
age. Since, in that situation, a small drop in sales may lead to an

of "potential risk" which becomes "active" only in the presence of


tself is not the source of the variability.

Specialty Cakes Inc. produces two types of cakes, a 2 lbs. round cake and a 3 lbs. heartshaped
cake. Total fixed costs for the firm are $94,000. Variable costs and sales data for
the two types of cakes are presented below.
2 lbs. 3 lbs.
Round Cake Heart-shape Cake
Selling price per unit $12 $20
Variable cost per unit $8 $15
Current sales (units) 10,000 15,000
If the product sales mix were to change to three heart-shaped cakes for each round cake,
the breakeven volume for each of these products would be
lbs. heartshaped
Firm F Firm V
Sales 10000 11000
Operating costs
FC 7000 2000
VC 2000 7000
Operating Profit (EBIT) $ 1,000.00 $ 2,000.00
Operating leverage ratios
FC/total costs 0.78 0.22
FC/sales 0.70 0.18

After 50% increase in sales 50% 50%


1.5 1.5
Sales 15000 16500
Operating costs
FC 7000 2000
VC 3000 10500
Operating Profit (EBIT) $ 5,000.00 $ 4,000.00
Operating leverage ratios
FC/total costs
FC/sales
CM 12000 6000
DOL 2.40 1.50
Percent change in EBIT
(EBIT - EBIT t-I)/EBITt-I 400% 100%

All firms show the effects of operating leverage (that is, changes in sales result in more than proportional changes in oper
But, Firm F proves to be the most sensitive firm with a 50 percent increase in sales leading to a 400 percent increase in op
As we have
just seen, it would be an error to assume that the firm with the largest absolute or relative amount of fixed costs automati
the most dramatic effects of operating leverage. Later, we will come up with an easy way to determine which firm is most
the presence of operating leverage. But before we can do so, we need to learn how to study operating leverage by means
analysis.

Price $ 50.00
Annual FC $ 100,000.00
VC $ 25.00
CM $ 25.00
CM% 50%
Q Break 4,000
$ Break $ 200,000
Long-Term Financing (Common Equity) $ 10,000,000
Need another 5$ m for expansion $ 5,000,000
Current EBIT without the expansion $ 1,500,000
New EBIT with the expansion $ 2,700,000
Tax 40%
Common stock outstanding 200,000

We have 3 financing options


option 1
All common stock 100,000.00
Price of each one $ 50

option 2
All debt 12% I
I $ 600,000

option 3
All P.Stock 11% Div $ 550,000

All common stock All debt 12% I


option 1 option 2
EBIT $ 2,700,000 $ 2,700,000
Interest (I) $ 600,000
EBT $ 2,700,000 $ 2,100,000
Income TAX $ 1,080,000.00 $ 840,000.00
EAT $ 1,620,000.00 $ 1,260,000.00
PD
Earning available to C.S $ 1,620,000.00 $ 1,260,000.00
No. of shares outstanding 300,000 200,000
EPS $ 5.4 $ 6.3

DFL 1.00 1.29

All common stock All debt 12% I


option 1 option 2
EBIT $ 1,800,000 $ 1,800,000
Interest (I) $ 600,000
EBT $ 1,800,000 $ 1,200,000
Income TAX $ 720,000.00 $ 480,000.00
EAT $ 1,080,000.00 $ 720,000.00
PD
Earning available to C.S $ 1,080,000.00 $ 720,000.00
No. of shares outstanding 300,000 200,000
EPS (Measure of the ROE) $ 3.6 $ 3.6

% change in EPS
% change in EBIT

We have seen in our example that if EBIT is above $1.8 million, debt financing is the preferred alternative from the stan
earnings per share. We know from our earlier discussion, however, that the impact on expected return is only one side
coin. The other side is the effect that financial leverage has on risk.

For one thing, the financial manager should compare the indifference point between two alternatives, like debt financin
stock financing, with the most likely level of EBIT .The higher the expected level of EBIT,assuming that
exceeds the indifference point, the stronger the case that can be made for debt financing, all other
same.

In addition, the financial manager should assess the likelihood of future EBITs actually falling below the indifference po
estimate of expected EBIT is $2.7million. Given the business risk of the company and the resulting possible fluctuations
in EBIT,the financial manager should assess the probability of EBITsfalling below $1.8 million. If the probability is neg
debt alternative will be supported. On the other hand, if EBIT is presently only slightly above the indiffe
the probability of EBITs falling below this point is high, the financialmanager may conclude
alternative is too risky.

In summary, the greater the level of expected EBIT above the indifference point and the lower
fluctuation, the stronger the case that can be made for the use of debt financing. EBIT-EPS bre
several methods used for determining the appropriate amount of debt a firm might carry. No o
satisfactory by itself. When several methods of analysis are undertaken simultaneously, howev

A quantitative measure of the sensitivity of a firm's earnings per share to a change in the firm's operating profit is ca
financial leverage (DFL). The degree of financial leverage at a particular level of operating profit is simply the percent
per share over the percentage change in operating profit that causes the change in earnings per share.
2700000
1.5

All common stock All debt 12% I


option 1 option 2
EBIT $ 2,700,000.00 $ 4,050,000.00
Interest (I) 600000
EBT 2700000 3450000
Income TAX 1080000 840000
EAT 1620000 2610000
PD
Earning available to C.S 1620000 $ 2,610,000
No. of shares outstanding 300000 200000
EPS 5.4 13.05

DFL 1 1.17
Firm 2F
19500

14000
3000
$ 2,500.00

0.82
0.72

50%
1.5
29250

14000
4500
$ 10,750.00

24750
2.30

330%

n proportional changes in operating profits).


to a 400 percent increase in operating profit.

amount of fixed costs automatically shows


o determine which firm is most sensitive to
dy operating leverage by means of break-even
All P.Stock 11% Div
option 3
$ 2,700,000

$ 2,700,000
$ 1,080,000.00
$ 1,620,000.00 Financial leverage is employed in
the hope of increasing the return to
$ 550,000 common shareholders.
$ 1,070,000.00
200,000
$ 5.4

1.51

All P.Stock 11% Div


option 3
$ 1,800,000

$ 1,800,000
$ 720,000.00
$ 1,080,000.00
$ 550,000
$ 530,000.00
200,000
$ 2.7

ferred alternative from the standpoint of


xpected return is only one side of the

o alternatives, like debt financing versus common


evel of EBIT,assuming that it
debt financing, all other things the

alling below the indifference point. As before, our


e resulting possible fluctuations
million. If the probability is negligible, the use of the
slightly above the indifference point and
manager may conclude that the debt

nce point and the lower the probability of downside


t financing. EBIT-EPS break-even analysis is but one of
t a firm might carry. No one method of analysis is
n simultaneously, however, generalizations are possible.

the firm's operating profit is called the degree of


ting profit is simply the percentage change in earnings
arnings per share.

states that DFL at a particular level of operating profit is calculated


by dividing operating profit by the dollar difference between
operating profit and the amount of before-tax operating profit
necessary to cover total fixed financing costs.
states that DFL at a particular level of operating profit is calculated
by dividing operating profit by the dollar difference between
operating profit and the amount of before-tax operating profit
necessary to cover total fixed financing costs.
(Remember, it takes more in before-tax earnings to cover
preferred dividends than it does to cover interest; hence, we need
to divide preferred dividends by 1 minus the tax rate in our
formula.)

4,050,000.00

All P.Stock 11% Div


option 3
$ 4,050,000.00

4050000
1080000
2970000
550000 916666.7
$ 2,420,000
200000
12.1

1.29
RC HS
CM 4 5
units 10000 15000 25000
0.4 0.6
1.6 3 4.6

fc 94000 20434.78261

8173.913
Products Asphalt Fuel oil
Quantity 300 300
Q based Allocation 30% 30%
Common Costs $ 100,000 $ 30,000 $ 30,000
1,000 barrels of crude oil
Products Asphalt Fuel oil
Quantity 300 300
Price per unit $ 60 $ 180
Market Valueat split $ 18,000 $ 54,000
Market Value % 14% 42%
Market based Allocation $ 13,846 $ 41,538

Final Price 70 200


Final Price X Q $ 21,000 $ 60,000
separable costs 1000 1000
estimated net realizable value (NRV) $ 20,000 $ 59,000
estimated net realizable value (NRV) % 14% 43%
estimated net realizable value (NRV) Allocation $ 14,492.75 $ 42,753.62

Final Price X Q $ 21,000 $ 60,000


$ 1,000 $ 1,000

1 gross margin
2 gross margin%
3 gross margin to each product $ 5,503.45 $ 15,724.14
4 (final price - Gross margin) $ 15,496.55 $ 44,275.86
Deduct gross margin from each product to arrive at a cost of goods sold

Deduct the separable costs from each product


to arrive at the allocated joint costs $ 14,496.55 $ 43,275.86
Diesel fuel Kerosene Gasoline
200 100 100 1000
20% 10% 10%
$ 20,000 $ 10,000 $ 10,000 $ 100,000

Diesel fuel Kerosene Gasoline


200 100 100 1000
$ 160 $ 80 $ 180
$ 32,000 $ 8,000 $ 18,000 $ 130,000
25% 6% 14%
$ 24,615 $ 6,154 $ 13,846 $ 100,000

180 90 190
$ 36,000 $ 9,000 $ 19,000 $ 145,000
1000 2000 2000 $ 7,000.00
$ 35,000 $ 7,000 $ 17,000 $ 138,000
25% 5% 12%
$ 25,362.32 $ 5,072.46 $ 12,318.84 $ 100,000

$ 36,000 $ 9,000 $ 19,000 $


145,000
$ 1,000 $ 2,000 $ 2,000 $ 7,000
$
100,000
$ 38,000
26%
$ 9,434.48 $ 2,358.62 $ 4,979.31 $ 38,000.00
$ 26,565.52 $ 6,641.38 $ 14,020.69 $ 107,000.00

$ 25,565.52 $ 4,641.38 $ 12,020.69 $ 100,000


Quantity 400,000
Price 1
Cost 0.84
FC 28000
Annual Interest 6000
Prefered Stock Dividends per year 2000
Tax 40%

CM $ 0.16
CM 64,000 10% 6,400.00 70,400.00
FC 28000 28000
EBIT $ 36,000 42,400.00
I 6000 6000
EBT $ 30,000 36,400.00
T 12000 14,560.00
EAT $ 18,000 21,840.00
P.Div 2000 $ 3,333.33 2000
EA p.s $ 16,000 19,840
DFL denemenator $ 26,667 24,506.67
C>S 1000 1000
DOL 1.78
DFL 1.35
DTL 2.40

DFL denemenator without P.S $ 30,000


DFL without P.S 1.20
DTL 2.13

EPS 16 19.84

We can note when the DFL =1.35, with 18% increase in EBIT , EPS will increase with (18% x
1.35)=24%

2009 2010 %
sales revenues 1,400,000.00 1,800,000.00 29%
VC 800,000.00 900,000.00 13%
CM 600,000.00 900,000.00 50%
FC 530,000.00 750,000.00 42%
OI 70,000.00 150,000.00 114%
S,G,A 25,000.00 65,000.00 160%
EBIT 45,000.00 85,000.00 89%
I 10,000.00 15,000.00 50%
EBT 35,000.00 70,000.00 100%
T 14,000.00 28,000.00 100%
NI 21,000.00 42,000.00 100%

DOL 8.57 6.00


DFL 3.33 3.57
6,400 10%

6,400 18%

$ 3,840 21%
3333.333
$ 3,840 24% 24%

$ 4 0.24

with (18% x
Quantity 30000
Price $ 100 DOL=3, an increase (or decrease )in sales with 1%
will increase (or decrease) EBIT with 3%
VC $ 40
CM $ 60
FC $ 1,200,000
FFC(interest) $ 200,000
With 10% DFL=1.5, an increase (or decrease )in EBIT with
1% will increase (or decrease) NI with 1.5%
sales $ 3,000,000
VC $ 1,200,000
CM $ 1,800,000 $ 1,980,000.00 10%
FC 1200000 $ 1,200,000.00
EBIT $ 600,000 $ 780,000.00 30% DTL=4.5, an increase (or decrease )in sales with
I 200000 200000 1% will increase (or decrease) NI with 1.5%
EBT $ 400,000 $ 580,000
T $ 160,000 $ 232,000
EAT (NI) $ 240,000 $ 348,000 45%

DOL = CM/EBIT 3.00


FDL=EBIT/EBIT-I 1.50
DTL=CM/EBIT-I 4.50
ROI is commonly based on before-
tax income.

Net sales 4,000


Cost of goods sold 3,525
General and administrative expenses 75
EBIT 400
Average plant and equipment 1,775
Average working capital 625
Av total Inv. 2,400

beforetax
return on investment (ROI) for last year 16.67%
.=EBIT/Av total Inv.

Sales 450,000
Operating income 25,000
Net profit after taxes 8,000
Total assets 500,000
Shareholders’ equity 200,000
Cost of capital 6% 30,000

ROI 6%
RI = Income- %*Inv (5,000)

Return on investment is commonly calculated by dividing pretax income by total assets available.
Residual income is the excess of the return on investment over a targeted amount equal to an imputed interest charge on
invested capital. The rate used is ordinarily the weighted-average cost of capital. Some companies measure managerial
performance in terms of the amount of residual income rather than the percentage return on investment. Because REB has
assets of $500,000 and a cost of capital of 6%, it must earn $30,000 on those assets to cover the cost of capital. Given that
operating income was only $25,000, it had a negative residual income of $5,000.
The following selected information is from the financial statements of Bishop
Corporation for the last fiscal year.
Current assets $ 500,000 500,000
Fixed assets 250,000 250,000
Current liabilities 100,000 100,000
Long-term debt 300,000 300,000
Shareholders' equity 350,000 350,000
Operating profit 1,000,000 1,000,000
Income taxes 400,000 400,000
Net income 600,000 600,000

Total ass 750,000


Total ass-CL 650,000
65,000
535,000

Bishop has a cost of capital of 10%. The company's economic value added (EVA) for
last year was
ble.
imputed interest charge on
nies measure managerial
investment. Because REB has
he cost of capital. Given that
Economic Value Added (sp
version of residual incom
Economic value added (EVA®)

= After-tax operating income

– [Weighted-average cost of capital

× (Total assets – current liabilities)]


dded (specific
al income).
ed (EVA®)

g income

ost of capital

nt liabilities)]
Consider Bushells Company, which produces 12-ounce bottles of iced tea at ils
Sydney bottling plant. The annual fixed manufacturing costs of the bottling plant are
Bushells currently uses absorption costing with a standard-costing systemfor . $5,400,000
external reporting purposes, and it calculates its budgeted fixed manufacturing rate ana
per case basis (one case is twenty-four 12-ounce bottles of iced tea). We will now examine
four different capacity levels used as the denominator to compute the budgeted fixed
~'manufacturing overhead cost rate: theoretical capacity, practical capacity, normal capaci
.utilization, and master-budget capacity utilization

In business and accounting, capacity ordinarily means "constraint," an "upper limit.'


Theoretical capacity is the level of capacity based on producing at full efficiency all the time. Bushells can produce 10,000
operating at maximum speed. Assuming 360 days per year, the theoretical annual capacity for three 8-hour shifts per da

Theoretical capacity is theoretical in the sense


10,000 cases per shift 10,000.00 interruptions because of bottle breakage on th
3 shifts per day 3.00 represents an ideal goal of capacity usage. The
360 days 360.00
= 10,800,000 cases 10,800,000.00

Practical capacity is the level of capacity that reduces theoretical capacity by unavoidable operating interruptions, such a
so on. Assume that the pra ical capacity is the practical production rate of 8,000 cases per shift for three shifts per day for

8,000 cases per shift 8000


3 shifts per day 3
300 days 300
= 7,200,000 cases 7,200,000.00

Both theoretical capacity and practical capacity measure capacity levels in terms of what a plant can supply- available cap
budget capacity utilization measure capacity levels in terms of demand for the output of the plant - the amount of the av
demand for its products. In many cases, budgeted demand is well below the production capacity available.

Normal capacity utilization is the level of capacity utilization that satisfies average customer demand over a period (say,
Master-budget capacity utilization is expected level of capacity utilization for the current budget period, typically one ye
when an industry has cyclical periods of high and low demand or
when management believes that the budgeted production for the coming period is not representativeof long-run deman

ConsiderBushells' master budget for 2004, based on production of 4,000,000 cases of tea per year.Despite using this mas
2004,top management believes that over the next three years the normal (average) annual production level will be 5,000
4,000,000 cases to be "abnormally" low. Why? Because a major competitor (Tea- Mania)has been sharply reducing its sel
expects that the competitor's lower price and advertising blitz will not be a long-run phenomenon and that, in 2005, Bus
ConsiderBushells' master budget for 2004, based on production of 4,000,000 cases of tea per year.Despite using this mas
2004,top management believes that over the next three years the normal (average) annual production level will be 5,000
4,000,000 cases to be "abnormally" low. Why? Because a major competitor (Tea- Mania)has been sharply reducing its sel
expects that the competitor's lower price and advertising blitz will not be a long-run phenomenon and that, in 2005, Bus

Master budget for 2004 VS.


8,000 cases per shift 8000
one shift working 300 days 300
2,400,000.00

8,000 cases per shift 8000


another shift working 200 days 200
1,600,000.00

master budget for 2004 4,000,000.00

Effect on Budgeted Fixed Manufacturing Overhead Cost Rate


Wenow illustrate how each of these four denominator levels affect the budgeted fixed manufacturing overhead cost rate.
5,400,000 for 2004. This lump-sum amount is incurred to provide the capacity to bottle iced tea.This lump sum, includes,
the compensation of the plant manager. The budgeted fixed manufacturing overhead cost rates for 2004 for each of the f

fixed manufacturing costs of 5,400,000 for 2004 $5,400,000.00

Budgeted Fixed Manufacturing Overhead Cost per Case

Theoreticalcapacity $0.50
Practicalcapacity $0.75
Normalcapacity utilization $1.08
Master-budgetcapacity $1.35 170%
utilization

Now assume that the Standard variable manufacturing cost is $5.20 per case. The total standard manufacturing cost per c
concepts is

Total
Standard variable manufacturing cost is $5.20 Manufacturing
Cost per Case

Theoreticalcapacity $5.70
Practicalcapacity $5.95
Normalcapacity utilization $6.28
Master-budgetcapacity $6.55
utilization
Effect on Financial Statements
The magnitude of the favorable/unfavorable production-volume variance under absorption costing will be affected by the
fixed manufacturing cost per case. Assume Bushells' actual production in 2004 is 4,400,000 cases of iced tea. Actual sales
inventory for 2004 and no price variances, spending variances, or efficiency variances in manufacturing costs. Those assum
and actual fixed manufacturing overhead costs are both $5,400,000. The average selling price per case of iced tea is $8.00

actual production in 2004 is 4,400,000.00


Actual sales for 2004 are 4,200,000.00 price
budgeted fixed manufacturing overhead costs and actual fix $5,400,000.00

The production-volume variance :

FOH allocated

Theoreticalcapacity $2,200,000.00
Practicalcapacity $3,300,000.00
Normalcapacity utilization $4,752,000.00
Master-budgetcapacity $5,940,000.00
utilization

How Bushells handles its end-of-period variances will determine the effect these p
will have on the company's operating income. We now discuss the three alternati
use to handle the production-volume varianc

1. Adjusted allocation-rate approach. This approach restates all amounts in the general and su
budgeted cost rates. Given that actual fIXed manufacturing overhead costs are $5,400,000 and
recalculated fixed manufacturing overhead cost is $1.23 per case ($5,400, 000 /4,4OO,OOO ca
allocation-rate approach results in the choice of the capacity level used to calculate the budge
having no effect on end-of-period financial statements.
In effect,an actual costing system is adopted at the en
Actual Rate $1.23

2. Proration approach. The underallocated or overallocated overhead is spread among (a)endin


(c) cost of goods sold. The proration restates the ending balances of (a), (b), and (c) to what the
used rather than budgeted cost rates. The proration approach also results in the choice of the c
fixedmanufacturing overhead cost per case having no effect on end-of-period financial stateme

3. Write-off variances to cost of goods sold approach. Exhibit 9-7 shows how use of this appro
Recall, Bushells had no beginning inventory,production of 4,400,000 cases, and sales of 4,200,0
31, 2004, is 200,000 cases. Using master-budget capacity utilizationas the denominator results
manufacturing overheadcost per case to the 200,000 cases in ending inventory. Accordingly, op
capacity utilization concept. The differences in operating income for the four denominator-level
amounts of fixed manufacturing overhead being inventoried at the end of2004:

Fixed Manufacturing Overhead in Dec. 31, 2004,Inventory 200,000.00

Theoreticalcapacity $100,000.00
Practicalcapacity $150,000.00
Normalcapacity utilization $216,000.00
Master-budgetcapacity $270,000.00
utilization

Income Statement Effects of Using Alternative Capacity-level Concepts Bushells Company for 2004

Theoretical Practical
10,800,000.00 7,200,000.00
sales rev $33,600,000.00 $33,600,000.00
Cost of Goods Sold
Beginning inventory $0.00 $0.00
Variable manufacturing costs $22,880,000.00 $22,880,000.00
Fixed manufacturing costs $2,200,000.00 $3,300,000.00
Cost for goods available for sale $25,080,000.00 $26,180,000.00
deduct ending inventory $1,140,000.00 $1,190,000.00
Total COGS(at standard costs) $23,940,000.00 $24,990,000.00
Adjustment for manufacturing variances $3,200,000.00 $2,100,000.00
Total COGS $27,140,000.00 $27,090,000.00
Gross Margin $6,460,000.00 $6,510,000.00
Operating cost $2,810,000.00 $2,810,000.00
Operating income $3,650,000.00 $3,700,000.00
the $54,000 difference ($3,820,000 - $3,766,000) in operating income betweenthe master-budget
capacity and the normal capacity utilization concepts is due to thedifference in fixed manufacturing
overhead inventoried ($270,000- $216,000)

Suppose Bushells Company is computing its operating income for 2006. That year's results are identical to the results for 2
utilization for 2006 is 6,000,000 cases instead of 4,000,000 cases. Production in 2006 is 4,400,000 cases. There is no begin
other than the production-volume variance.Bushells writes off this variance to cost of goods sold. Sales in 2006 are 4,200

master-budget capacity utilization for 2006 is 6,000,000.00


Production in 2006 is 4,400,000.00
Sales in 2006 are 4,200,000.00
Ending Inventory 200,000.00

Theonly change in Exhibit 9-7 results would be for the master-budget capacity utilization level. The
budgeted fIXed manufacturing overhead cost rate for 2006 is

fixed manufacturing costs of 5,400,000 for 2004

Budgeted Fixed Manufacturing Overhead Cost per Case

Master-budgetcapacity $0.90
allocated fixed cost $3,960,000.00
PVV $1,440,000.00 $1,440,000.00

Themanufacturing cost per case is $6.10 ($5.20 + $0.90). So, the production-volume variance for
2006is
$6.10

Income Statement 2006

sales rev 33,600,000.00


Cost of Goods Sold
Beginning inventory 0
Variable manufacturing costs $22,880,000.00
Fixed manufacturing costs $3,960,000.00
Cost for goods available for sale $26,840,000.00
deduct ending inventory $1,220,000.00
Total COGS(at standard costs) $25,620,000.00
Adjustment for manufacturing variances $1,440,000.00
Total COGS $27,060,000.00
Gross Margin $6,540,000.00
Operating cost $2,810,000.00
Operating income $3,730,000.00 -$90,000.00

The higher denominator level used to calculate budgeted fixed manufacturing cost per case inthe 2006 master budget
that fewer fixed manufacturing overhead costs are inventoried in 2006 than in 2004, given identical sales and producti
and assuming the production-volumevariance is written off to cost of goods sold.
e
00,000
ana
xamine
ed
capaci

.'
the time. Bushells can produce 10,000 cases of iced tea per shift when the bottling lines are
capacity for three 8-hour shifts per day is

tical capacity is theoretical in the sense that it does not allow for any plant mainte. nance,
ptions because of bottle breakage on the filling lines, or any other factor. Theoretical capacity
nts an ideal goal of capacity usage. Theoretical capacity is unattainable in the real world.

oidable operating interruptions, such as scheduled maintenance time, shutdowns for holidays, and
ses per shift for three shifts per day for 300 days a year. The practical annual capacity

f what a plant can supply- available capacity. In contrast, normal capacity utilization and master.
put of the plant - the amount of the available capacity that the plant expects to use based on the
uction capacity available.

customer demand over a period (say, 2 to 3 years) that includes seasonal, cyclical, and trendfactors.
current budget period, typically one year. These two capacity utilization levels can differ-for example,

s not representativeof long-run demand.

s of tea per year.Despite using this master-budget capacity utilization level of 4,000,000 cases for
e) annual production level will be 5,000,000 cases. They view 2004's budgeted production level of
Mania)has been sharply reducing its selling price and spending large amounts on advertising. Bushells
un phenomenon and that, in 2005, Bushells' production and sales will be higher.
s of tea per year.Despite using this master-budget capacity utilization level of 4,000,000 cases for
e) annual production level will be 5,000,000 cases. They view 2004's budgeted production level of
Mania)has been sharply reducing its selling price and spending large amounts on advertising. Bushells
un phenomenon and that, in 2005, Bushells' production and sales will be higher.

normal (average) annual production level will be 5,000,000 cases.


5,000,000.00

head Cost Rate


xed manufacturing overhead cost rate. Bushells has budgeted fixed manufacturing costs of
ottle iced tea.This lump sum, includes, among other costs, lease costs for bottling equipment ,and
ad cost rates for 2004 for each of the four capacity-level concepts are

Thebudgeted fixed manufacturing overhead cost rate based on master-budget capacity


utilization, $1.35 per case, is 170% higher than the cost rate based on theoretical capacity,
$0.50per case. This big difference in cost rates is because theoretical capacity is much
largerthan the master-budget capacity utilization.

otal standard manufacturing cost per case with alternative capacity-level


sorption costing will be affected by the choice of the denominator used to calculate the budgeted
400,000 cases of iced tea. Actual sales for 2004 are 4,200,000 cases. Also assume no beginning
es in manufacturing costs. Those assumptions mean budgeted fixed manufacturing overhead costs
elling price per case of iced tea is $8.00. Operating costs equal $2,810,000.

$8.00

PVV The higher the denominator level, (1) the lower the budgeted FMOH cost rate,
and the lower the amount of FMOH allocated to output produced (because
the budgeted FMOH cost rate is lower), and (3) the higher the unfavorable PVV
$3,200,000.00 U (because the higher the denominator level, the more likely actual output will
$2,100,000.00 U fall short of that level).
$648,000.00 U
-$540,000.00 F

etermine the effect these production-volume variances


discuss the three alternative approaches Bushells can

amounts in the general and subsidiary ledgers by using actual rather than
head costs are $5,400,000 and actual production is 4,400,000 cases,the
e ($5,400, 000 /4,4OO,OOO cases, rounded up to the nearest cent). The adjusted
el used to calculate the budgeted fixed manufacturingoverhead cost per case

adopted at the end of the period.


head is spread among (a)ending work in process, (b) ending finished goods, and
s of (a), (b), and (c) to what they would have been if actual cost rates had been
so results in the choice of the capacity level used to calculate the budgeted
nd-of-period financial statements .

7 shows how use of this approch affects Bushells' operating income for 2004.
000 cases, and sales of 4,200,000 cases. Hence, the ending inventory on December
tionas the denominator results in assigning the highest amount of fixed
ding inventory. Accordingly, operating income is highest using the master-budget
for the four denominator-level concepts in Exhibit 9-7 are due to different
he end of2004:

pany for 2004

Normal Master-budget
5,000,000.00 4,000,000.00
$33,600,000.00 $33,600,000.00

$0.00 $0.00
$22,880,000.00 $22,880,000.00
$4,752,000.00 $5,940,000.00
$27,632,000.00 $28,820,000.00
$1,256,000.00 $1,310,000.00
$26,376,000.00 $27,510,000.00
$648,000.00 -$540,000.00
$27,024,000.00 $26,970,000.00
$6,576,000.00 $6,630,000.00
$2,810,000.00 $2,810,000.00
$3,766,000.00 $3,820,000.00
$54,000.00
$54,000.00
master-budget
d manufacturing

results are identical to the results for 2004, shown in Exhibit 9-7, except that master-budget capacity
06 is 4,400,000 cases. There is no beginning inventory on January 1, 2006, and there are no variances
of goods sold. Sales in 2006 are 4,200,000 cases.

tion level. The

variance for
ost per case inthe 2006 master budget means
004, given identical sales and production levels
d.
budgeted FMOH cost rate,
tput produced (because
higher the unfavorable PVV
re likely actual output will
Product D Product R
Units 5,000 15,000
Price $ 400 $ 200
Prime $ 200 $ 80
DL hours 25,000 75,000 100,000

Budgeted OH Pool $ 2,000,000


OH assigned 500,000 1,500,000 2,000,000
Tradittional OH rate $ 20

After anlysis made to OH Pool Activity Consumption ABC rate


Engineering $ 125,000 E.hours 12500 $ 10
Setups $ 300,000 No. of setups 300 $ 1,000
Machine running $ 1,500,000 M.hours 150000 $ 10
Packing $ 75,000 No.of packing orders 15000 $ 5
$ 2,000,000

All we need is to know the number of activities consumed in each product


Product D Product R Total
E.hours 5,000 7,500 12,500
No. of setups 200 100 300
M.hours 50,000 100,000 150,000
No.of packing orders 5,000 10,000 15,000
$
E.hours $ 50,000 $ 75,000
No. of setups $ 200,000 $ 100,000
M.hours $ 500,000 $ 1,000,000
No.of packing orders $ 25,000 $ 50,000
$ 775,000 $ 1,225,000 $ 2,000,000
Units 5,000 15,000
$ per unit 155 82
Butteco has the following cost components for 100,000 units of product for the year:
Direct materials $200,000
Direct labor 100,000
Manufacturing overhead 200,000
Selling and administrative expense 150,000
All costs are variable except for $100,000 of manufacturing overhead and $100,000 of selling and administrative
expenses. The total costs to produce and sell 110,000 units for the year are

units 100,000
Direct materials $200,000
Direct labor $100,000
V.Manufacturing overhead $100,000
F.Manufacturing overhead $100,000
Selling and administrative expense $50,000
F.Selling and administrative expense $100,000

V.Manuf.
F.manf
V.SGA
F.SGA

The total costs to produce and sell 110,000 units for the year are
110000

Madtack Company’s beginning and ending inventories for the


month of November are
November. 1
Direct materials $67,000
Work-in-process $145,000
Finished goods $85,000

Production data for the month of November follows:


Direct labor $200,000
Actual overhead $132,000
Direct materials purchased $163,000
Transportation in $4,000
Purchase returns and allowances $2,000

Madtack uses one overhead control account and charges


overhead to production at 70% of direct labor cost. The
company does not formally recognize over/underapplied
overhead until year-end.

$370,000

CoGs
Beg.RM
Add: Purchase 163,000
Less : Returns and discounts 2,000
Net Purchase 161,000
Add: Freight-in 4,000
RM Available for use
Less : End.RM
DM used in Production
DL
MOH
Total manufacturing costs for the period
Add: Beg. WIP
Less: End. WIP
Costs of goods Manufacturing
Add: Beg.Finished
Goods Available for sale
Less: End. Finished
Costs of goods Sold

Lucy Sportswear manufactures a specialty line of T-shirts using a job-order costing system. During March, the following
costs were incurred in completing job ICU2: direct materials, $13,700; direct labor, $4,800; administrative, $1,400; and
selling, $5,600. Overhead was applied at the rate of $25 per machine hour, and job ICU2 required 800 machine hours. If
job ICU2 resulted in 7,000 good shirts, the cost of goods sold per unit would be

CoGs
Beg.RM
Add: Purchase
Less : Returns and discounts
Net Purchase -
Add: Freight-in
RM Available for use
Less : End.RM
DM used in Production
DL
MOH
Total manufacturing costs for the period
Add: Beg. WIP
Less: End. WIP
Costs of goods Manufacturing
Add: Beg.Finished
Goods Available for sale
Less: End. Finished
Costs of goods Sold

Zeta Company is preparing its annual profit plan. As part of


its analysis of the profitability of individual products, the
controller estimates the amount of overhead that should be
allocated to the individual product lines from the
information given as follows:

Units produced 25 25
Material moves per product line 5 15
Direct labor hours per unit 200 200
Budgeted materials handling costs $50,000 $50,000

Under a costing system that allocates overhead on the basis of direct labor hours, Zeta Company’s materials handling cos
allocated to one unit of wall mirrors would be
allocated to 25
allocated to one unit

Under activity-based costing (ABC), Zeta’s materials handling costs allocated to one unit of wall mirrors would be

allocated to 25
allocated to one unit

New-Rage Cosmetics has used a traditional cost accounting system to apply quality control costs uniformly to all products
at a rate of 14.5% of direct labor cost. Monthly direct labor cost for Satin Sheen makeup is $27,500. In an attempt to
distribute quality control costs more equitably, New-Rage is considering activity-based costing. The monthly data shown
in the chart below have been gathered for Satin Sheen makeup.

0.145 27500 3987.5


85. A
Taylor Corporation is determining the cost behavior of several items in order to budget
for the upcoming year. Past trends have indicated the following dollars were spent at
three different levels of output.

In establishing a budget for 14,000 units, Taylor should treat Costs A, B, and C,
respectively, as

Unit Levels Unit Levels


10000 per unit 12000 per unit 15000
A $25,000 #DIV/0! $29,000 $2.42 $35,000
B $10,000 #DIV/0! $15,000 $1.25 $15,000
C $15,000 #DIV/0! $18,000 $1.50 $22,500

97. A company employs a just-in-time (JIT) production system and utilizes backflush
accounting. All acquisitions of raw materials are recorded in a raw materials control
account when purchased. All conversion costs are recorded in a control account as
incurred, while the assignment of conversion costs are from an allocated conversion cost
account. Company practice is to record the cost of goods manufactured at the time the
units are completed using the estimated budgeted cost of the goods manufactured.
The budgeted cost per unit for one of the company's products is as follows

Direct Materials $15.00 15


Conversion costs 35.00 35
Total budgeted unit cost $50.00 50

During the current accounting period, 80,000 units of product were completed, and
75,000 units were sold. The entry to record the cost of the completed units for the period
would be
b. Finished Goods - Control 4,000,000
Raw Material - Control 1,200,000
Conversion Cost Allocated 2,800,000.

99. From the following budgeted data, calculate the budgeted indirect cost rate that would be
used in a normal costing system

Total direct labor hours 250,000


Direct costs $10,000,000
Total indirect labor hours 50,000
Total indirect-labor-related costs $ 5,000,000
Total indirect non-labor related costs $ 7,000,000

Total indirect costs


Total allocation base

104. B
The marketing manager of Ames Company has learned the following about a new
product that is being introduced by Ames. Sales of this product are planned at $100,000
for the first year. Sales commission expense is budgeted at 8% of sales plus the
marketing manager's incentive budgeted at an additional ½%. The preparation of a
product brochure will require 20 hours of marketing salaried staff time at an average rate
of $100 per hour, and 10 hours, at $150 per hour, for an outside illustrator's effort. The
variable marketing cost for this new product will be

Sales commission expense 8000


500
$ 8,500 variable marketing cost for this new prod

110. A
110. Bethany Company has just completed the first month of producing a new product but has
not yet shipped any of this product. The product incurred variable manufacturing costs of
$5,000,000, fixed manufacturing costs of $2,000,000, variable marketing costs of
$1,000,000, and fixed marketing costs of $3,000,000.

If Bethany uses the variable cost method to value inventory, the inventory value of the new product would be

[34] Gleim #: 3.2.34 -- Source: CMA 692 3-1


The controller of JoyCo has requested a quick estimate of the manufacturing supplies needed for the Morton Plant for the
month of July when production is expected to be 470,000 units to meet the ending inventory requirements and sales of
475,000 units.

production 470,000.00
sales 475,000.00

JoyCo’s budget analyst has the following actual data for the last 3 months:

Production Manufacturing
Month in Units Supplies

March 450,000 723,060 1.45


April 540,000 90,000 853,560 130,500 1.45
May 480,000 766,560

Using these data and the high-low method to develop a cost estimating equation, the estimate of needed manufacturing
supplies for July would be 752,060.00

Answer (D) is correct. The fixed and variable portions of mixed costs may be estimated by identifying the highest
and the lowest costs within the relevant range. The difference in cost divided by the difference in activity is the
variable rate. Once the variable rate is found, the fixed portion is determinable. April and March provide the
highest and lowest amounts. The difference in production was 90,000 units (540,000 April – 450,000 March), and
the difference in the cost of supplies was $130,500 ($853,560 – $723,060). Hence, the unit variable cost was $1.45
($130,500 ÷ 90,000 units). The total variable costs for March must have been $652,500 (450,000 units × $1.45 VC
per unit), and the fixed cost must therefore have been $70,560 ($723,060 – $652,500). The probable costs for July
equal $681,500 (470,000 units × $1.45 VC per unit), plus $70,560 of fixed costs, a total of $752,060.

[37] Gleim #: 3.2.37 -- Source: CMA 1292 3-11


Butteco has the following cost components for 100,000 units of product for the year:
Direct materials $200,000 $ 200,000 2
Direct labor 100,000 $ 100,000 1
Manufacturing overhead 200,000 $ 200,000 100000
Selling and administrative expense 150,000 $ 150,000 100000
All costs are variable except for $100,000 of manufacturing overhead and $100,000 of selling and administrative
expenses. The total costs to produce and sell 110,000 units for the year are

VC 5 495000
FC 200000 $ 695,000

Answer (C) is correct. Direct materials unit costs are strictly variable at $2 ($200,000 ÷ 100,000 units). Similarly,
direct labor has a variable unit cost of $1 ($100,000 ÷ 100,000 units). The $200,000 of manufacturing overhead for
100,000 units is 50%. The variable unit cost is $1. Selling costs are $100,000 fixed and $50,000 variable for
production of 100,000 units, and the variable unit selling expenses is $.50 ($50,000 ÷ 100,000 units). The total unit
variable cost is therefore $4.50 ($2 + $1 + $1 + $.50). Fixed costs are $200,000. At a production level of 110,000
units, variable costs are $495,000 (110,000 units × $4.50). Hence, total costs are $695,000 ($495,000 + $200,000).

[45] Gleim #: 3.2.45 -- Source: CMA Sample Q 02/2005 2-17


Parker Company pays each member of its sales staff a salary as well as a commission on each unit sold. For the coming
year, Parker plans to increase all salaries by 5% and to keep unchanged the commission paid on each unit sold. Because of
increased demand, Parker expects the volume of sales to increase by 10%. How will the total cost of sales salaries and
commissions change for the coming year?
Q price. REV comm VC Salary FC Tc
100 $ 5 $ 500 0.08 $ 1,000 $ 1,040
10 50
110 $ 5 $ 550 0.08 $ 1,050 $ 1,094
$ 54

B. Increase by more than 5% but less than 10%.

Sales salaries will increase by exactly 5%. The per-unit commission amount will remain
constant, but sales commissions in total are expected to increase by 10%. Thus, total sales salaries and
commissions will increase somewhere between 5% and 10%.

[46] Gleim #: 3.2.46 -- Source: CMA Sample Q 02/2005 2-23


Ace, Inc. estimates its total materials handling costs at two production levels as follows:
Cost Gallons
$ 160,000 80000 $ 28,000 1.4 $ 112,000 $ 48,000
$ 132,000 60000 20000 1.4 $ 84,000 $ 48,000

What is the estimated total cost for handling 75,000 gallons? 75000
$ 153,000
[63] Gleim #: 3.3.63 -- Source: CMA 696 3-19
If the beginning balance for May of the materials inventory account was $27,500, the ending balance for May is $28,750,
and $128,900 of materials were used during the month, the materials purchased during the month cost
BI $ 27,500
Pur $ 130,150
Avi $ 157,650
EI $ 28,750
Used $ 128,900

[Fact Pattern #3]


Alex Company had the following inventories at the beginning and end of the month of January.

Jan-01 Jan-31
Finished goods $125,000 $117, $ 125,000 $ 117,000
Work-in-process 235,000 251,0 $ 235,000 $ 251,000 DM availabe
Direct materials 134,000 124,0 $ 134,000 $ 124,000 $ 325,000

The following additional manufacturing data were available for the month of January:
Direct materials purchased $189,000 $ 189,000
Purchase returns and allowances 1,000 $ 1,000
Transportation-in 3,000 $ 3,000
Direct labor 300,000 $ 300,000
Actual factory overhead 175,000 $ 175,000

Alex Company applies factory overhead at a rate of 60% of direct labor cost, and any overapplied or underapplied factory
overhead is deferred until the end of the year, December 31.

[75] Gleim #: 3.3.75 -- Source: CMA 690 4-1


(Refers to Fact Pattern #3)
Alex Company’s prime cost for January was $ 501,000

[76] Gleim #: 3.3.76 -- Source: CMA 690 4-2


(Refers to Fact Pattern #3)
Alex Company’s total manufacturing cost for January was

DM $ 201,000
DL $ 300,000
APP.OH $ 180,000
$ 681,000
[77] Gleim #: 3.3.77 -- Source: CMA 690 4-3
(Refers to Fact Pattern #3)
Alex Company’s cost of goods manufactured for January was $ 665,000

[78] Gleim #: 3.3.78 -- Source: CMA 690 4-4


(Refers to Fact Pattern #3)
Alex Company’s cost of goods sold for January was $ 673,000

[79] Gleim #: 3.3.79 -- Source: CMA 690 4-5


(Refers to Fact Pattern #3)
Alex Company’s balance in factory overhead control for January was

Actual factory overhead 175,000 $ 175,000


APP.OH $ 180,000 $ 5,000

Dr Cr
$ 175,000 $ 180,000

$ 5,000 Thus, the net effect is a $5,000 credit bal

The factory overhead control account should have a debit of $175,000 for the actual costs
incurred and a credit for the $180,000 (60% of direct labor) applied to production.

[80] Gleim #: 3.3.80 -- Source: CMA Sample Q 02/2005 2-15


The schedule of cost of goods manufactured of Gruber Fittings, Inc. shows the following balances for its fiscal year-end:
Direct manufacturing labor $ 280,000 $ 280,000
Manufacturing overhead 375,000 $ 375,000
Ending work-in-process inventory 230,000 $ 230,000
Raw materials used in production 450,000 $ 450,000
Cost of goods manufactured 1,125,000 $ 1,125,000

The value of the work-in-process inventory at the beginning of the fiscal year was

Manuf cost $ 1,105,000


BWIP $ 250,000
Ending work-in-process inventory 230,000
Cost of goods manufactured 1,125,000
$ 1,355,000
[114] Gleim #: 4.1.2 -- Source: Publisher
Felicity Corporation manufactures a specialty line of dresses using a job-order costing system. During January, the
following costs were incurred in completing job J-1:
Direct materials $27,400 27400
Direct labor 9,600 9600
Administrative costs 2,800 2800
Selling costs 11,200 11200

Factory overhead was applied at the rate of $50 per direct labor hour, and job J-1 required 400 direct labor hours. If job J-
1 resulted in 4,000 good dresses, the cost of goods sold per unit is

OH App 20000
57000
$ 14.25

[118] Gleim #: 4.1.6 -- Source: CMA Sample Q 02/2005 2-22


Kepler Optics makes lenses for telescopes. Because Kepler will only sell lenses of the highest quality, the normal spoilage
during a reporting period is 1,000 units. 1000

At the beginning of the current reporting period, Kepler had 2,200 units in inventory,
and during the period, production was started and completed on 4,000 units.
Units in inventory at the end of the current reporting period were 1,500,
and the units transferred out were 3,000.

During this period, the abnormal spoilage for Kepler’s lens production was

Answer (A) is correct. Kepler’s abnormal spoilage for the period can be calculated as follows:
Beginning work-in-process 2,200
Add: started and completed 4,000
Less: transferred out (3,000)
Less: ending work-in-process (1,500)
Total spoilage for period 1,700
Less: normal spoilage (1,000)
Abnormal spoilage for period 700

[154] Gleim #: 4.2.42 -- Source: CMA 690 4-10


During the month of May, Mercer Company completed 50,000 units costing $600,000, exclusive of spoilage allocation.
50000 600000
Of these completed units, 25,000 were sold during the month. An additional 10,000 units, costing $80,000, were 50%
complete at May 31. All units are inspected between the completion of manufacturing and transfer to finished goods
inventory. Normal spoilage for the month was $20,000, and abnormal spoilage of $50,000 was also incurred during the
month. The portion of total spoilage that should be charged against revenue in May is

compl 50000 600000


sold 25000 300000
ewip 10000 80000 50%

Answer (D) is correct. Normal spoilage is an inventoriable cost of production that is charged to cost of goods sold
when the units are sold. Abnormal spoilage is a period cost recognized when incurred.

The $50,000 of abnormal spoilage is therefore expensed during May.


In addition, 50% of the normal spoilage is debited to cost of goods sold because 50% (25,000 ÷ 50,000) of the units comple

No spoilage is allocated to work-in-process because inspection occurs after completion.

Thus, the normal spoilage expensed during the month is $10,000 ($20,000 × 50%). Total spoilage charged against revenue

[Fact Pattern #16]


At the end of its fiscal year, Jubal Manufacturing recorded the data below:
Prime cost $800,000 $ 800,000
Variable manufacturing overhead 100,000 $ 100,000
Fixed manufacturing overhead 160,000 $ 160,000
Variable selling and other expenses 80,000 $ 80,000
Fixed selling and other expenses 40,000 $ 40,000

[195] Gleim #: 5.1.10 -- Source: CMA 1286 4-18


(Refers to Fact Pattern #16)
If Jubal uses variable costing, the inventoriable costs for the fiscal year are

[196] Gleim #: 5.1.11 -- Source: CMA 1286 4-19


(Refers to Fact Pattern #16)
Using absorption (full) costing, Jubal’s inventoriable costs are

[Fact Pattern #19]


The following is taken from Fortech Company’s records for the fiscal year just ended:
Direct materials used $300,000 $ 300,000
Direct labor 100,000 $ 100,000
Variable manufacturing overhead 50,000 $ 50,000
Fixed manufacturing overhead 80,000 $ 80,000
Selling and admin. costs--variable 40,000 $ 40,000
Selling and admin. costs--fixed 20,000 $ 20,000

[207] Gleim #: 5.1.22 -- Source: CMA 1286 4-18


(Refers to Fact Pattern #19)
If Fortech Company uses variable costing, the inventoriable costs for the fiscal year are

[208] Gleim #: 5.1.23 -- Source: CMA 1286 4-19


(Refers to Fact Pattern #19)
Using absorption (full) costing, Fortech Company’s inventoriable costs are

[Fact Pattern #20] B. $41


Estimated unit costs for Cole Lab using full absorption
costing and operating at a production level of 12,000 units 12000
per month:
Estimated
Cost Item Unit Cost
Direct material $32 32
Direct labor 20 20
Variable manufacturing overhead 15 15
Fixed manufacturing overhead 6 6 72000
Variable selling 3 3
Fixed selling 4 4

[210] Gleim #: 5.1.25 -- Source: CMA 1283 4-1


(Refers to Fact Pattern #20)
Cole Lab’s estimated conversion costs per unit are $ 41

[211] Gleim #: 5.1.26 -- Source: CMA 1283 4-2


(Refers to Fact Pattern #20)
Cole Lab’s estimated prime costs per unit are $ 52
[Fact Pattern #21]
Farber Company employs a normal (nonstandard) absorption cost system. The following information is from the financial
records of the company for the year.
• Total manufacturing costs were $2,500,000.
• Cost of goods manufactured was $2,425,000.
• Applied factory overhead was 30% of total manufacturing costs.
• Factory overhead was applied to production at a rate of 80% of direct labor cost.
• Work-in-process inventory at January 1 was 75% of work-in-process inventory at December 31.

[213] Gleim #: 5.1.28 -- Source: CMA 1285 4-26


(Refers to Fact Pattern #21)
Total cost of direct material used by Farber Company for the year is

DM
DL
App.OH
TMC 2,500,000.00
BWIP 75,000.00
EWIP (100,000.00)
2,425,000.00 (75,000.00) 2,475,000.00 50,000.00
ing and administrative

per unit

$2.00
$1.00
$1.00

$0.50

$4.00
$100,000
$0.50
$100,000

$695,000.00

November .30
$62,000
$171,000
$78,000
Total
67,000

232,000
62,000
170,000
200,000 370,000 Madtack Company’s prime cost for November is
140,000
510,000 Madtack Company’s total manufacturing cost for November is
145,000
171,000
484,000 Madtack Company’s cost of goods transferred to finished goods inven
85,000
569,000
78,000
491,000 Madtack Company’s cost of goods sold for November is

During March, the following


; administrative, $1,400; and
equired 800 machine hours. If

Total
13,700

13,700

13,700
4,800
20,000
38,500
-
-
38,500

38,500

38,500 7000 5.5

Wall Mirrors Specialty Windows total


25 25 50
5 15 20
200 200 400

ompany’s materials handling costs

25000
$1,000

of wall mirrors would be

12500
$500

l costs uniformly to all products


$27,500. In an attempt to
ting. The monthly data shown

11.5 12 138
0.14 17500 2450
77 25 1925
4513
525.5

per unit
$2.33 semi
$1.00 fixed Fixed cost fixed in total &
variable cost fixed per unit
$1.50 VC per unit fixed

97. B

80000
4000000
1200000
2800000

99. D d. $48.

250,000
$ 10,000,000
50,000
$ 5,000,000
$ 7,000,000

$ 12,000,000
250,000
$ 48

b. $8,500.

100000
8.0%
0.5%
20
100

marketing cost for this new product will be

$5,000,000
$2,000,000
$1,000,000 $3,000,000

w product would be $ 5,000,000

cost behaiv

ded for the Morton Plant for the


ory requirements and sales of

VC FC
652,500 70,560
783,000 70,560

mate of needed manufacturing

identifying the highest


ence in activity is the
March provide the
– 450,000 March), and
t variable cost was $1.45
50,000 units × $1.45 VC
e probable costs for July

cost behaiv

100,000

100000 1
50000 0.50
ing and administrative
110000

,000 units). Similarly,


nufacturing overhead for
000 variable for
000 units). The total unit
ction level of 110,000
($495,000 + $200,000).

cost behaiv

ach unit sold. For the coming


id on each unit sold. Because of 5%
tal cost of sales salaries and 10%

5.19%

salaries and
ng balance for May is $28,750,
he month cost

DM used in prod
$ 201,000

applied or underapplied factory 60%


over app D. $5,000 credit - overapplied.

e net effect is a $5,000 credit balance resulting from the overapplication of overhead.

alances for its fiscal year-end:

B. $250,000
B. $14.25

em. During January, the

400 direct labor hours. If job J- 50 400


4000

est quality, the normal spoilage

2200
4000 6200
1500
3000 4500
1700
700

lusive of spoilage allocation.


costing $80,000, were 50%
d transfer to finished goods
was also incurred during the

d to cost of goods sold

000 ÷ 50,000) of the units completed were sold during the period.

poilage charged against revenue is $60,000 ($50,000 + $10,000).

B. $900,000

$ 900,000

$ 1,060,000
$ 450,000

$ 530,000
nformation is from the financial

2500000
2425000
30%
80%
st for November is

st for November is

ed to finished goods inventory for November is

November is
Gregg Industries manufactures molded chairs. The three models of molded chairs, which are all variations of the same des
are Standard (can be stacked), Deluxe (with arms), and Executive (with arms and padding). The company uses batch
manufacturing and has an operation costing system.
Gregg has an extrusion operation and subsequent operations to form, trim, and finish the chairs.
Plastic sheets are produced by the extrusion operation, some of which are sold directly to other manufacturers.
During the forming operation, the remaining plastic sheets are molded into chair seats and the legs are added; the standar
During the trim operation, the arms are added to the deluxe and executive models and the chair edges are smoothed.
Only the executive model enters the finish operation where the padding is added.
All of the units produced are subject to the same steps within each
operation, and no units are in process at the end of the period. The units of production and direct materials costs were as f

Units Extrusion Form


Produced Materials Materials
Plastic sheets 5,000.00 $60,000
Standard mode 6,000.00 $72,000 $24,000
Deluxe model 3,000.00 $36,000 $12,000
Executive model 2,000.00 $24,000 $8,000
16,000.00 $192,000.00 $44,000.00

Units attributable to each stage 16,000.00 11,000.00


Unit material cost $12.00 $4.00

Plastic sheets are produced by the During the forming operation, the remaining
extrusion operation, some of which plastic sheets are molded into chair seats and
are sold directly to other the legs are added; the standard model is sold
manufacturers. after this operation.

Manufacturing costs applied during the month were:


Extrusion Form
Operation Operation
Direct labor $152,000 $60,000
Overhead $240,000 $72,000
$392,000 $132,000

Units attributable to each stage 16,000.00 11,000.00


Unit conversion cost $24.50 $12.00

(Refers to Fact Pattern #33)


Gregg Industries’ unit cost of a standard model is $52.50

A standard model passes through the extrusion and form operations. Thus, its unit cost includes the
materials and conversion costs for both operations. The unit materials and conversion costs for the extrusion
operation are $12.00 ($192,000 ÷ 16,000 units) and $24.50 [($152,000 + $240,000) ÷ 16,000 units], respectively. The
unit materials and conversion costs for the form operation are $4.00 [$44,000 ÷ (16,000 – 5,000) units] and $12.00
[($60,000 + $72,000) ÷ (16,000 – 5,000) units], respectively. Accordingly, the unit cost of a standard model is $52.50
($12.00 + $24.50 + $4.00 + $12.00).
A standard model passes through the extrusion and form operations. Thus, its unit cost includes the
materials and conversion costs for both operations. The unit materials and conversion costs for the extrusion
operation are $12.00 ($192,000 ÷ 16,000 units) and $24.50 [($152,000 + $240,000) ÷ 16,000 units], respectively. The
unit materials and conversion costs for the form operation are $4.00 [$44,000 ÷ (16,000 – 5,000) units] and $12.00
[($60,000 + $72,000) ÷ (16,000 – 5,000) units], respectively. Accordingly, the unit cost of a standard model is $52.50
($12.00 + $24.50 + $4.00 + $12.00).

(Refers to Fact Pattern #33)


Gregg Industries’ unit cost of a deluxe model is $69.30

(Refers to Fact Pattern #33)


Gregg Industries’ total product cost of the executive model is

Total DM 50,000.00
Total Conv. $142,600.00
Total cost $192,600

An executive model passes through all four operations. Thus, its unit cost equals that of the deluxe model plus the
unit costs incurred in the finish operation. The unit cost of the deluxe model is $69.30. The unit materials and
conversion costs for the finish operation are $6.00 [$12,000 ÷ (16,000 – 14,000) units] and $21.00 [($18,000 + $24,000)
÷ (16,000 – 14,000) units], respectively. Consequently, the unit cost of the executive model is $96.30 ($69.30 + $6.00 +
$21.00), and the total product cost is $192,600 (2,000 units × $96.30).

Refers to Fact Pattern #33)


Assume that 1,000 units of Gregg Industries’ deluxe model remained in work-in-process at the end of the period and that
these units were 100% complete as to materials and 60% complete as to trim operation conversion. What is the balance of
work-in-process?
No. of units 1000 Total DM $19,000.00
Total Conv. $36,500.00
Total TRIM Conv. $9,000.00
Total cost $64,500.00
EUP Conv
Completed Deluxe mo 2000
partial Deluxe model 600
completed Executive 2000
EUP 4600
l variations of the same design,
company uses batch

r manufacturers.
legs are added; the standard model is sold after this operation.
ir edges are smoothed.

ect materials costs were as follows

Trim Finish
Materials Materials

$9,000
$6,000 $12,000
$15,000.00 $12,000.00

5,000.00 2,000.00
$3.00 $6.00

ation, the remaining During the trim operation, the arms are
into chair seats and added to the deluxe and executive Only the executive model enters
andard model is sold models and the chair edges are the finish operation where the
eration. smoothed. padding is added.

Trim Finish
Operation Operation
$30,000 $18,000
$39,000 $24,000
$69,000 $42,000

5,000.00 2,000.00
$13.80 $21.00
EUP 4600
cost 15.00

ost includes the


osts for the extrusion
nits], respectively. The
00) units] and $12.00
ndard model is $52.50
ost includes the
osts for the extrusion
nits], respectively. The
00) units] and $12.00
ndard model is $52.50

deluxe model plus the


nit materials and
1.00 [($18,000 + $24,000)
$96.30 ($69.30 + $6.00 +

end of the period and that


sion. What is the balance of
CMA Part 1: Financial Planning, Performance, and Control, 15th Ed. - New 2-p
Irvin N. Gleim, Dale L. Flesher

1 Budgeting Concepts and Forecasting Techniques


2 Budget Methodologies and Budget Preparation
3 Cost Management Terminology and Concepts
4 Cost Accumulation Systems
5 Cost Allocation Techniques
6 Operational Efficiency and Business Process Performance
7 Cost and Variance Measures
8 Responsibility Accounting and Performance Measures
9 Internal Controls I
10 Internal Controls II and Ethics for Management Accountants
Control, 15th Ed. - New 2-part Exam
78. The Mountain Mule Glove Company is in its first year of business. Mountain Mule had a
beginning cash balance of $85,000 for the quarter. The company has a $50,000 shortterm
line of credit. The budgeted information for the first quarter is shown below.

January
Sales $60,000.00
Purchases $35,000.00
Operating costs $25,000.00

All sales are made on credit and are collected in the second month following the sale.
Purchases are paid in the month following the purchase, while operating costs are paid in
the month that they are incurred. How much will Mountain Mule need to borrow at the
end of the quarter if the company needs to maintain a minimum cash balance of $5,000 as
required by a loan covenant agreement?

January
Coll Sales
Pyt. Purchases
Operating costs $25,000.00
cash coll - disb -$25,000.00

NET
minimum cash balance
Mule need to borrow

77. Data regarding Johnsen Inc.’s forecasted dollar sales for the last seven months of the year
and Johnsen’s projected collection patterns are as follows.

Cash sales 30%


Forecasted sales
June $700,000 $700,000.00 $210,000.00
July 600,000 $600,000.00 $180,000.00
August 650,000 $650,000.00 $195,000.00
September 800,000 $800,000.00 $240,000.00
October 850,000 $850,000.00 $255,000.00
November 900,000 $900,000.00 $270,000.00
December 840,000 $840,000.00 $252,000.00
Types of sales
Cash sales 30% 0.3
Credit sales 70% 0.7

Collection pattern on credit sales (5% determined to be uncollectible)


During the month of sale 20% 0.2
During the first month following the sale 50% 0.5
During the second month following the sale 25% 0.25

Johnsen’s budgeted cash receipts from sales and collections on account for September are

Sep cash sales $240,000.00


coll from sep cr sales $112,000.00
coll from aug sales $227,500.00
coll from july sales $105,000.00
$684,500.00

76. Tidwell Corporation sells a single product for $20 per unit. All sales are on account, with
60% collected in the month of sale and 40% collected in the following month. A
schedule of cash collections for January through March of the coming year reveals the
following receipts for the period.

Cash Receipts
January February
December receivables $32,000 $32,000.00
From January sales 54,000 $36,000 $54,000.00 $36,000.00
From February sales 66,000 $44,000 $66,000.00
From March sales 72,000

Other information includes the following

• Inventories are maintained at 30% of the following month’s sales.


• Tidwell desires to keep a minimum cash balance of $15,000. Total
payments in January are expected to be $106,500, which excludes $12,000
of depreciation expense. Any required borrowings are in multiples of 1000
• The December 31 balance sheet for the preceding year revealed a cash
balance of $24,900.

Ignoring income taxes, the financing needed in January to maintain the firm’s minimum
cash balance is
cash coll $110,900.00
cash disbur. $121,500.00
-$10,600.00 since , Any required borrowings are in multiples of 1

75. Health Foods Inc. has decided to start a cash budgeting program to improve overall cash
management. Information gathered from the past year reveals the following cash
collection trends.

40% of sales are on credit 40%


50% of credit sales are collected in month of sale 50%
30% of credit sales are collected first month after sale 30%
15% of credit sales are collected second month after sale 15%
5% of credit sales result in bad debts 5%

Gross sales for the last five months were as follows


January $220,000 $220,000.00
February 240,000 $240,000.00
March 250,000 $250,000.00
April 230,000 $230,000.00
May 260,000 $260,000.00
$1,200,000.00

Sales for June are projected to be $255,000. Based on this information, the expected cash
receipts for March would be

march cash sales $150,000.00


march cash coll from cr sales $50,000.00
feb coll in march $28,800.00
jan coll in march $13,200.00
$242,000.00

74. Brooke Company’s management team is preparing a cash budget for the coming quarter.
The following budgeted information is under review.
January February
Revenue $700,000 $800,000 $500,000 $700,000.00 $800,000.00
Inventory purchases 350,000 425,000 225,000 $350,000.00 $425,000.00
Other expenses 150,000 175,000 175,000 $150,000.00 $175,000.00

The company expects to collect 40% of its monthly sales in the month of sale and 60% in
the following month. 50% of inventory purchases are paid in the month of purchase, and
50% in the following month. Payments for all other expenses are made in the month
incurred.

Brooke forecasts the following account balances at the beginning of the quarter.

Cash $200,000 $200,000.00


Accounts receivable 300,000 $300,000.00
Accounts payable (Inventory) 400,000 $400,000.00

Given the above information, the projected ending cash balance for February will be

January February
40% in the same mnth $280,000.00 $320,000.00
60% follow $300,000.00 $420,000.00
cash coll $580,000.00 $740,000.00
pyt of 50% purchase $175,000.00 $212,500.00
pyt of 50% purchase in the follow $400,000.00 $175,000.00
pyt purchase $575,000.00 $387,500.00
Other expenses $150,000.00 $175,000.00
total disb $725,000.00 $562,500.00

Cash B.B $200,000.00 $55,000.00


Net coll - disbur -$145,000.00 $177,500.00
net $55,000.00 $232,500.00

73. ANNCO sells products on account, and experiences the following collection schedule.

In the month of sale 10% 10%


In the month after sale 60% 60% 67%
In the second month after sale 30% 30% 33%

At December 31, ANNCO reports accounts receivable of $211,500. Of that amount,


$162,000 is due from December sales, and $49,500 from November sales. ANNCO is
budgeting $170,000 of sales for January. If so, what amount of cash should be collected
in January?
In the month of sale 10% $17,000.00
In the month after sale 60% $108,000.00 108000
In the second month after sale 30% $49,500.00
$174,500.00

72. Prudent Corporation’s budget for the upcoming accounting period reveals total sales of
$700,000 in April and $750,000 in May. The sales cash collection pattern is

20% of each month’s sales are cash sales.


5% of a month’s credit sales are uncollectible.
70% of a month’s credit sales are collected in the month of sale.
25% of a month’s credit sales are collected in the month following the sale.

If Prudent anticipates the cash sale of a piece of old equipment in May for $25,000,
May’s total budgeted cash receipts would be

$700,000.00 $750,000.00
April May
cash sales $140,000.00 $150,000.00
Cr sales $560,000.00 $600,000.00

cash coll from cash sales $150,000.00


cash coll from cr sales in the same month $420,000.00
cash coll from cr from previous $140,000.00
other coll $25,000.00
$735,000.00

70. Monroe Products is preparing a cash forecast based on the following information.
• Monthly sales: December $200,000; January $200,000; February $350,000;
March $400,000.
• All sales are on credit and collected the month following the sale.
• Purchases are 60% of next month’s sales and are paid for in the month of
purchase.
• Other monthly expenses are $25,000, including $5,000 of depreciation.
If the January beginning cash balance is $30,000, and Monroe is required to maintain a
minimum cash balance of $10,000, how much short-term borrowing will be required at
the end of February?
DEC January
sales $200,000.00 $200,000.00
pur $120,000.00 $210,000.00
coll from sales $200,000.00
pyt pur $120,000.00 $210,000.00
cash exp 20000 20000

cash BB 30000
minm
$0.00

69. Tip-Top Cleaning Supply carries a large number of different items in its inventory, giving
the firm a competitive advantage in its industry. Below is part of Tip-Top’s budget for
the first quarter of next year.

Sales $855,000 $855,000.00 $812,250.00


Cost of goods sold 425,000 $425,000.00
Rent and salary expenses 375,000 $375,000.00

Historically, all of the sales are on account and are made evenly over the quarter. 5% of
all sales are determined to be uncollectible and written off. The balance of the
receivables is collected in 50 days. This sales and collection experience is expected to
continue in the first quarter. The projected balance sheet for the first day of the quarter
includes the following account balances

Cash $ 10,000 $10,000.00


Accounts receivable (net) 450,000 $450,000.00
Inventory 900,000 $900,000.00
Accounts payable 800,000 $800,000.00

How much cash can Tip-Top anticipate collecting in the first quarter (based on a 360-day year)?

$450,000.00
$361,000.00
$811,000.00

68. Projected monthly sales of Wallstead Corporation for January, February, March, and
April are as follows.

January $300,000 $300,000.00


February 340,000 $340,000.00
March 370,000 $370,000.00
April 390,000 $390,000.00

• The company bills each month's sales on the last day of the month.
• Receivables are booked gross and credit terms of sale are: 2/10, n/30.
• 50% of the billings are collected within the discount period, 30% are collected by
the end of the month, 15% are collected by the end of the second month, and 5%
become uncollectible.

Budgeted cash collections for Wallstead Company during April would be

Jan
50% of the billings are collected within the discount period 0.5
30% are collected by the end of the month, 0.3
15% are collected by the end of the second month, 0.15
5% become uncollectible.

67. Bootstrap Corporation anticipates the following sales during the last six months of the
year.
CASH SALES CR SALES NET CR SALES 5%
July $460,000 $460,000.00 $92,000.00 $368,000.00 $349,600.00 $18,400.00
August 500,000 $500,000.00 $100,000.00 $400,000.00 $380,000.00 $20,000.00
September 525,000 $525,000.00 $105,000.00 $420,000.00 $399,000.00 $21,000.00
October 500,000 $500,000.00 $100,000.00 $400,000.00 $380,000.00 $20,000.00
November 480,000 $480,000.00 $96,000.00 $384,000.00 $364,800.00 $19,200.00
December 450,000 $450,000.00 $90,000.00 $360,000.00 $342,000.00 $18,000.00
$2,915,000.00 $583,000.00 $2,332,000.00 $2,215,400.00 $116,600.00
20% of Bootstrap’s sales are for cash.

The balance is subject to the collection pattern shown below.


Percentage of balance collected in the month of sale 40% 40.0%
Percentage of balance collected in the month following sale 30% 30.0%
Percentage of balance collected in the second month following 25.0%
sale 25%
Percentage of balance uncollectible 5% 5.0%

What is the planned net accounts receivable balance as of December 31?


July

20% of Bootstrap’s sales are for cash. $92,000.00


Percentage of balance collected in the month of sale 40% $147,200.00
Percentage of balance collected in the month following sale 30%
Percentage of balance collected in the second month following
sale 25% $239,200.00

Sales $2,915,000.00
coll $2,504,400.00
gross acc/rec $410,600.00
tWO
5% $116,600.00 WAYS
net Acc/Rec $294,000.00

66. Cooper Company’s management team is preparing a cash budget for the coming quarter.
The following budgeted information is under review.

January February
Revenue $700,000 $800,000 $500,000 $700,000.00 $800,000.00
Inventory purchases 350,000 425,000 225,000 $350,000.00 $425,000.00
Other expenses 150,000 175,000 175,000 $150,000.00 $175,000.00

The company expects to collect 40% of its monthly sales in the month of sale and 60% in
the following month. 50% of inventory purchases are paid in the month of purchase, and
50% in the following month. Payments for all other expenses are made in the month
incurred.

Brooke forecasts the following account balances at the beginning of the quarter.

Cash $100,000 $100,000.00


Accounts receivable 300,000 $300,000.00
Accounts payable (Inventory) 500,000 $500,000.00

Given the above information, the projected change in cash during the coming quarter will
be

January February
40% in the same mnth $280,000.00 $320,000.00
60% follow $300,000.00 $420,000.00
cash coll $580,000.00 $740,000.00
pyt of 50% purchase $175,000.00 $212,500.00
pyt of 50% purchase in the follow $500,000.00 $175,000.00
pyt purchase $675,000.00 $387,500.00
Other expenses $150,000.00 $175,000.00
total disb $825,000.00 $562,500.00

Cash B.B $100,000.00 -$145,000.00


Net coll - disbur -$245,000.00 $177,500.00
net -$145,000.00 $32,500.00

Given the above information, the projected change in cash during the coming quarter will

65. Brown Company estimates that monthly sales will be as follows.

January $100,000 $100,000.00


February 150,000 $150,000.00
March 180,000 $180,000.00

Historical trends indicate that 40% of sales are collected during the month of sale, 50%
are collected in the month following the sale, and 10% are collected two months after the
sale.
Brown’s accounts receivable balance as of December 31 totals $80,000 ($72,000
from December’s sales and $8,000 from November’s sales). The amount of cash Brown
can expect to collect during the month of January is

40% of sales are collected during the month of sale $40,000.00


50% are collected in the month following the sale $60,000.00
10% are collected two months after the $8,000.00
$108,000.00

62. Granite Company sells products exclusively on account, and has experienced the
following collection pattern: 60% in the month of sale, 25% in the month after sale, and
15% in the second month after sale. Uncollectible accounts are negligible. Customers
who pay in the month of sale are given a 2% discount. If sales are $220,000 in January,
$200,000 in February, $280,000 in March, and $260,000 in April, Granite’s accounts
receivable balance on May 1 will be
jan feb mar
$220,000.00 $200,000.00 $280,000.00

60% in the month of sale 60%


25% in the month after sale 25%
15% in the second month after sale 15%

jan feb mar


$220,000.00 $200,000.00 $280,000.00

60% in the month of sale 60%


25% in the month after sale 25%
15% in the second month after sale 15%

61. Tut Company’s selling and administrative costs for the month of August, when it sold
20,000 units, were as follows. 20,000.00

Costs
Per Unit
Variable costs $18.60 $372,000 $18.60
Step costs 4.25 85,000 $4.25
Fixed costs 8.80 176,000 $8.80
Total selling and
administrative costs $31.65 $633,000 $31.65

The variable costs represent sales commissions paid at the rate of 6.2% of sales.
The step costs depend on the number of salespersons employed by the company.
In August there were 17 persons on the sales force. However, two members have taken early retirement effective August 3
It is anticipated that these positions will remain vacant for several months.
Total fixed costs are unchanged within a relevant range of 15,000 to 30,000 units per month.
Tut is planning a sales price cut of 10%, which it expects will increase sales volume to 24,000 units per month.

If Tut implements the sales price reduction, the total budgeted selling and administrative costs for the month of Septem

61. Tut Company’s selling and administrative costs for the month of August, when it sold
20,000 units, were as follows. 24,000.00

Costs
Per Unit
Variable costs $18.60 $372,000 $16.74
Step costs 4.25 85,000 $3.13
Fixed costs 8.80 176,000 $8.80
Total selling and
administrative costs $31.65 $633,000 $28.67
78. C c. $10,000.
85000

February March
$40,000.00 $50,000.00
$40,000.00 $75,000.00
$25,000.00 $25,000.00

5000

February March
$60,000.00
$35,000.00 $40,000.00
$25,000.00 $25,000.00
-$60,000.00 -$5,000.00 -$90,000.00

-$5,000.00
-$5,000.00
-$10,000.00

77. B b. $684,500.

Credit sales 70%

$490,000.00
$420,000.00
$455,000.00
$560,000.00
$595,000.00
$630,000.00
$588,000.00
$20.00 76. C c. $11,000.

March

$44,000.00
$72,000.00

30%
$15,000.00
$106,500.00

$24,900.00
borrowings are in multiples of 1000

75. C c. $242,000.

Cash sales 60% Credit sales 40%


60% 40%
$132,000.00 $88,000.00
$144,000.00 $96,000.00
$150,000.00 $100,000.00
$138,000.00 $92,000.00
$156,000.00 $104,000.00

74. C c. $232,500
March
$500,000.00
$225,000.00
$175,000.00

40% 60%
50%

March
$200,000.00
$480,000.00
$680,000.00
$112,500.00
$212,500.00
$325,000.00
$175,000.00
$500,000.00

$180,000.00

73. B b. $174,500

$211,500.00 $162,000.00 $49,500.00


$170,000.00 180000
nov dec dec acc /rec
$49,500.00 $162,000.00 $211,500.00
the amount of $162000 represents 90% from Dec sales , thus the portion to be

72. C c. $735,000

20.00%
5.00% 0.95
70.00%
25.00%

$25,000.00

70. B b. $70,000

60%

25000 5000 20000


30000
10000
February March
$350,000.00 $400,000.00
$240,000.00
$200,000.00 $350,000.00
$240,000.00 $0.00
20000 20000

10000
-$70,000.00

69. A a. $811,000.

Q
90 40
44%

68. A a. $343,300
feb march april
$181,300.00
$111,000.00
$51,000.00

$343,300.00

67. B b. $294,000.

40% 30% 25%


$147,200.00 $110,400.00 $92,000.00
$160,000.00 $120,000.00 $100,000.00
$168,000.00 $126,000.00 $105,000.00
$160,000.00 $120,000.00 $100,000.00
$153,600.00 $115,200.00 $96,000.00
$144,000.00 $108,000.00 $90,000.00
$932,800.00 $699,600.00 $583,000.00
August September October November December

$100,000.00 $105,000.00 $100,000.00 $96,000.00 $90,000.00 $583,000.00 20% of Bootstrap’s sales are for
$160,000.00 $168,000.00 $160,000.00 $153,600.00 $144,000.00 Percentage of balance collected
$110,400.00 $120,000.00 $126,000.00 $120,000.00 $115,200.00 Percentage of balance collected
$92,000.00 $100,000.00 $105,000.00 $100,000.00 Percentage of balance collected
$370,400.00 $485,000.00 $486,000.00 $474,600.00 $449,200.00 $2,504,400.00

66. C c. $112,500.

March
$500,000.00
$225,000.00
$175,000.00

40% 60%
50%

March
$200,000.00
$480,000.00
$680,000.00
$112,500.00
$212,500.00
$325,000.00
$175,000.00
$500,000.00

$32,500.00
$180,000.00
$212,500.00

$112,500.00

65. C c. $108,000.

40%
50% 83%
10%
72000 8000

62. C c. $146,000.
april may
$260,000.00

$70,000.00 $65,000.00
$30,000.00 $42,000.00 $39,000.00

april may june


$260,000.00

$65,000.00
$42,000.00 $39,000.00

$146,000.00

61. A a. $652,760.

price 300 6,000,000.00


372000

Total
$372,000.00
$85,000.00 17 5000
$176,000.00

$633,000.00

ly retirement effective August 31.

0 units per month.

costs for the month of September would be


price 270 6,480,000.00
$401,760.00

Total
$401,760.00
$75,000.00 17 4411.76471
$176,000.00 3.75

$652,760.00
simple
Dec sales , thus the portion to be collected =60% from the 90%
July August September October

20% of Bootstrap’s sales are for cash. $92,000.00 $100,000.00 $105,000.00 $100,000.00
Percentage of balance collected in the month of sale 40% $147,200.00 $160,000.00 $168,000.00 $160,000.00
Percentage of balance collected in the month following sale 30% $110,400.00 $120,000.00 $126,000.00
Percentage of balance collected in the second month following $92,000.00 $100,000.00
November December JAN FEB MARCH

$96,000.00 $90,000.00
$153,600.00 $144,000.00
$120,000.00 $115,200.00 $108,000.00
$105,000.00 $100,000.00 $96,000.00 $90,000.00

$294,000.00
Section A: Budget Preparation
35. Stumphouse Cheese is in the process of implementing a cost improvement system with
kaizen costing as the basis for budgeting all manufacturing activities. This will be
utilized over the next four years in an attempt to become more profitable. The target
reduction rate has been set at 5% of fixed overhead costs. Total fixed overhead costs for
this year were $900,000. What is the budgeted amount for the next two years using
kaizen costing?

Current Year +1 Current Year +2

855,000.00 812,250.00

36. Netco’s sales budget for the coming year is as follows.

Item Volume in Units Sales Price


1 tea 200,000 50
2 sugar 150,000 10
3 cars 300,000 30
Total sales revenue

Items 1 and 3 are different models of the same product. Item 2 is a complement to Item1.
Past experience indicates that the sales volume of Item 2 relative to the sales volume of Item 1 is fairly constant.
Netco is considering an 10% price increase for the coming year for Item 1,
which will cause sales of Item 1 to decline by 20%, while simultaneously
causing sales of Item 3 to increase by 5%. If Netco institutes the price increase for Item
1, total sales revenue will decrease by
$ (1,050,000)

37. Troughton Company manufactures radio-controlled toy dogs. Summary budget financial
data for Troughton for the current year are as follows.
Sales (5,000 units at $150 each) 5000
Variable manufacturing cost
Fixed manufacturing cost
Variable selling and administrative cost
Fixed selling and administrative cost

Troughton uses an absorption costing system with overhead applied based on the number of units produced, with a denom
Underapplied or overapplied manufacturing overhead is written off to cost of goods sold in the year incurred.
The $20,000 budgeted operating income from producing and selling 5,000 toy dogs planned for this year is of concern to Tr
She believes she could increase operating income to $50,000 (her bonus threshold) if Troughton produces more units than
How much of an increase in the number of units in the finished goods inventory would be needed to generate the $50,000

Targeted OI
Curr OI
Diff

38. Ace Manufacturing plans to produce two products, Product C and Product F, during the
next year, with the following characteristics.
Product C
Selling price per unit $10 $15 $ 10
Variable cost per unit $ 8 $10 $ 8
Expected sales (units) 20,000 5,000 20,000
80%
CM $ 2
$ 1.60

Total projected fixed costs for the company are $30,000.


Assume that the product mix would be the same at the breakeven point as at the expected level of sales of both products.
What is the projected number of units (rounded) of Product C to be sold at the
breakeven point?
9,231

39. B b. $540,000.
39. Hannon Retailing Company prices its products by adding 30% to its cost.
Hannon anticipates sales of $715,000 in July, $728,000 in August, and $624,000 in September.
Hannon’s policy is to have on hand enough inventory at the end of the month to cover 25% of the next month’s sales.
What will be the cost of the inventory that Hannon should budget for purchase in August?

July Aug Sep


COGS $ 550,000 $ 560,000 $ 480,000
EI $ 140,000 $ 120,000 0
Purchase(prod+ei-bi) $ 540,000
40. Streeter Company produces plastic microwave turntables. Sales for the next year are
expected to be 65,000 units in the first quarter, 72,000 units in the second quarter, 84,000
units in the third quarter, and 66,000 units in the fourth quarter. Streeter maintains a
finished goods inventory at the end of each quarter equal to one half of the units expected
to be sold in the next quarter. How many units should Streeter produce in the second
quarter?
Q1 Q2 Q3
Units 65000 72000 84000
EI 36,000 42,000 33,000
Pro 78,000

41. Tyler Company produces one product and budgeted 220,000 units for the month of
August with the following budgeted manufacturing costs.
Total Costs Cost Per Unit
Variable costs $ 1,408,000.0 $ 6.4
Batch set-up cost $ 880,000.0 $ 4.0
Fixed costs $ 1,210,000.0 $ 5.5
Total $ 3,498,000.0 $ 15.9

The variable cost per unit and the total fixed costs are unchanged within a production range of 200,000 to 300,000 units pe
The total for the batch set-up cost in any month depends on the number of production batches that Tyler runs.
A normal batch consists 50,000 units unless production requires less volume.
In the prior year, Tyler experienced a mixture of monthly batch sizes of 42,000 units, 45,000 units, and 50,000 units.
Tyler consistently plans production each month in order to minimize the number of batches.
For the month of September, Tyler plans to manufacture 260,000 units.
What will be Tyler’s total budgeted production costs for September?

$ 1,664,000
$ 1,210,000 $ 2,874,000
$ 2,874,000
No. of batches to produce Aug 4.40
= 5.00
Cost per batch 176,000.00

The cost per set-up is $176,000, which is computed from the August budget of 220,000 units. The 220,000 units for August
No. of batches to produce Sep 5.20
= 6.00
What will be Tyler’s total budgeted production costs for September?
$ 1,664,000
$ 1,210,000 $ 2,874,000
Batch set-up cost $ 1,056,000
$ 3,930,000

42. C c. 7,133.
42. Ming Company has budgeted sales at 6,300 units for the next fiscal year, and desires to
have 590 good units on hand at the end of that year. Beginning inventory is 470 units.
Ming has found from past experience that 10% of all units produced do not pass final
inspection, and must therefore be destroyed. How many units should Ming plan to
produce in the next fiscal year?

Prod. Represents 90% only 6420


100% prod. 7,133

43. B b. 71,700.

43. Savior Corporation assembles backup tape drive systems for home microcomputers. For
the first quarter, the budget for sales is 67,500 units. Savior will finish the fourth quarter
of last year with an inventory of 3,500 units, of which 200 are obsolete. The target
ending inventory is 10 days of sales (based upon 360 days). What is the budgeted production for the first quarter?

Sales per day (total sales / days on Quarter) 750


Sales on 10 days EI 7500
71,700

d. 86,000 units.
44. Streeter Company produces plastic microwave turntables. Sales for the next year are
expected to be 65,000 units in the first quarter, 72,000 units in the second quarter, 84,000
units in the third quarter, and 66,000 units in the fourth quarter. Streeter usually
maintains a finished goods inventory at the end of each quarter equal to one half of the
units expected to be sold in the next quarter. However, due to a work stoppage, the
finished goods inventory at the end of the first quarter is 8,000 units less than it should
be. How many units should Streeter produce in the second quarter?
Q1 Q2 Q3 Q4
65,000 72,000 84,000 66,000
EI 28,000 42,000 33,000 -
86,000

45. Data regarding Rombus Company's budget are shown below.


Planned sales 4,000 units 4000
Material cost $2.50 per pound 2.5
Direct labor 3 hours per unit 3
Direct labor rate $7 per hour 7
Finished goods beginning inventory 900 units 900
Finished goods ending inventory 600 units 600
Direct materials beginning inventory 4,300 units 4300
Direct materials ending inventory 4,500 units 4500
Materials used per unit 6 pounds 6
Rombus Company's production budget will show total units to be produced of
3700

46. Krouse Company is in the process of developing its operating budget for the coming
year. Given below are selected data regarding the company’s two products, laminated
putter heads and forged putter heads, that are sold through specialty golf shops.

Forged
Raw materials
Steel 2 pounds @ $5/lb.
10
Copper None
-
Direct labor 1/4 hour @ $20/hr.
5
Expected sales (units) 8,200 2,000 8200
Selling price per unit $30 $80 $ 30
Ending inventory target (units) 100 60 100
Beginning inventory (units) 300 60 300
Beginning inventory (cost) $5,250 $3,120 $ 5,250

Production 8,000
Production hours 2,000
Manufacturing overhead is applied to units produced on the basis of direct labor hours.
Variable manufacturing overhead is projected to be $25,000, and fixed manufacturing
overhead is expected to be $15,000.

The estimated cost to produce one unit of the laminated putter head is

OH rate Num $ 40,000


den 4,000 $ 10.00

47. Tidwell Corporation sells a single product for $20 per unit. All sales are on account, with
60% collected in the month of sale and 40% collected in the following month. A partial
schedule of cash collections for January through March of the coming year reveals the
following receipts for the period

January
December receivables $32,000 $ 32,000
From January sales 54,000 $36,000 $ 54,000
From February sales 66,000 $44,000

Other information includes the following.


• Inventories are maintained at 30% of the following month’s sales.
• Assume that March sales total $150,000.

The number of units to be purchased in February is

January
sales 90000
33000

The number of units to be purchased in February is

48. Stevens Company manufactures electronic components used in automobile


manufacturing. Each component uses two raw materials, Geo and Clio. Standard usage
of the two materials required to produce one finished electronic component, as well as the
current inventory, are shown below.

Standard
Material Per Unit Price /ib Current Inventory
Geo 2.0 pounds $15/lb. 5,000 pound 2 15 5000
Clio 1.5 pounds $10/lb. 7,500 pound 1.5 10 7500

Stevens forecasts sales of 20,000 components for the next two production periods.
Company policy dictates that 25% of the raw materials needed to produce the next
period’s projected sales be maintained in ending direct materials inventory

Based on this information, the budgeted direct material purchases for the coming period
would be
prod ei units to be purchse
Geo 2.0 pounds 40000 10000 45000
Clio 1.5 pounds 30000 7500 30000

49. Petersons Planters Inc. budgeted the following amounts for the coming year.
Beginning inventory, finished goods $ 10,000 $ 10,000
Cost of goods sold 400,000 $ 400,000
Direct material used in production 100,000 $ 100,000
Ending inventory, finished goods 25,000 $ 25,000
Beginning and ending work-in-process inventory Zero $ -

Overhead is estimated to be two times the amount of direct labor dollars. The amount
that should be budgeted for direct labor for the coming year is

$ 415,000
OH+DL $ 315,000
$ 105,000

dm $ 100,000
dl
oh
cogm $ 100,000
bf $ 10,000
ef $ (25,000)
cogs $ 400,000

50. Over the past several years, McFadden Industries has experienced the following
regarding the company’s shipping expenses.
Fixed costs $16,000 16000
Average shipment 15 pounds 15
Cost per pound $.50 $ 0.50

Shown below are McFadden’s budget data for the coming year.
Number of units shipped 8,000 8000
Number of sales orders 800 800
Number of shipments 800 800
Total sales $1,200,000 1200000
Total pounds shipped 9,600 9600

McFadden’s expected shipping costs for the coming year are


$ 4,800.00
$ 20,800.00

51. Swan Company is a maker of men's slacks. The company would like to maintain
20,000 yards of fabric in ending inventory. The beginning fabric inventory is expected to
contain 25,000 yards. The expected yards of fabric needed for sales is 90,000. Compute
the yards of fabric that Swan needs to purchase.

52. Manoli Gift Shop maintains a 35% gross profit percentage on sales, and carries an ending
inventory balance each month sufficient to support 30% of the next month’s expected
sales. Anticipated sales for the fourth quarter are as follows. ei

cost @ (100%-35%) ei
October $42,000 42000 27300 11310
November 58,000 58000 37700 14430
December 74,000 74000 48100 0
What amount of goods should Manoli Gift Shop plan to purchase during the month of
November?
$ 40,820

53. In preparing the direct material purchases budget for next quarter, the plant controller has
the following information available.
Budgeted unit sales 2,000 2000
Pounds of materials per unit 4 4 8000
Cost of materials per pound $3 $ 3.00
Pounds of materials on hand 400 400
Finished units on hand 250 250
Target ending units inventory 325 325
Target ending inventory of pounds of materials 800 800

How many pounds of materials must be purchased?

Prod 2075
pounds for prod 8300
8,700

54. Playtime Toys estimates that it will sell 200,000 dolls during the coming year. The
beginning inventory is 12,000 dolls; the target ending inventory is 15,000 dolls. Each
doll requires two shoes which are purchased from an outside supplier. The beginning
inventory of shoes is 20,000; the target ending inventory is 18,000 shoes. The number of
shoes that should be purchased during the year is

sales fg 200000
bfg 12000
efg 15000
purchase fg 203000 406000

bs 20000
es 18000
404,000

55. Maker Distributors has a policy of maintaining inventory at 15% of the next month’s
forecasted sales. The cost of Maker’s merchandise averages 60% of the selling price.
The inventory balance as of May 31 is $63,000, and the forecasted dollar sales for the last
seven months of the year are as follows
cost
June $700,000 $ 700,000 420000
July 600,000 $ 600,000 360000
August 650,000 $ 650,000 390000
September 800,000 $ 800,000 480000
October 850,000 $ 850,000 510000
November 900,000 $ 900,000 540000
December 840,000 $ 840,000 504000
What is the budgeted dollar amount of Maker’s purchases for July?

58. Using the following budget data for Valley Corporation, which produces only one
product, calculate the company’s predetermined factory overhead application rate for
variable overhead.

Units to be produced 11,000 11000


Units to be sold 10,000 10000
Indirect materials, varying with production $ 1,000 $ 1,000
Indirect labor, varying with production 10,000 $ 10,000
Factory supervisor’s salary,
incurred regardless of production 20,000 $ 20,000
Depreciation on factory building and equipment 30,000 $ 30,000
Utilities to operate factory machines 12,000 $ 12,000
Security lighting for factory 2,000 $ 2,000
Selling, general and administrative expenses 5,000 $ 5,000

Num $ 23,000
Den 11000
$ 2.09

60. Given the following data for Scurry Company, what is the cost of goods sold?
Beginning inventory of finished goods $100,000 $ 100,000
Cost of goods manufactured 700,000 $ 700,000
Ending inventory of finished goods 200,000 $ 200,000
Beginning work-in-process inventory 300,000 $ 300,000
Ending work-in-process inventory 50,000 $ 50,000

600,000

61. Tut Company’s selling and administrative costs for the month of August, when it sold
20,000 units, were as follows. 20000 sales

Costs
Per Unit
Variable costs $18.60 $372,000 $ 18.6
Step costs 4.25 85,000 $ 4.3
Fixed costs 8.80 176,000 $ 8.8
Total selling and
administrative costs $31.65 $633,000 $ 31.65

The variable costs represent sales commissions paid at the rate of 6.2% of sales.
The step costs depend on the number of salespersons employed by the company.
In August there were 17 persons on the sales force. However, two members have taken early retirement effective August 3
It is anticipated that these positions will remain vacant for several months.
Total fixed costs are unchanged within a relevant range of 15,000 to 30,000 units per month.
Tut is planning a sales price cut of 10%, which it expects will increase sales volume to 24,000 units per month.
If Tut implements the sales price reduction, the total budgeted selling and administrative costs for the month of Septem

sales rev $ 24,000 $ 270

Variable costs
Step costs
Fixed costs

61. Tut Company’s selling and administrative costs for the month of August, when it sold
20,000 units, were as follows.

Variable costs $18.60 $372,000


Step costs 4.25 85,000
Fixed costs 8.80 176,000
Total selling and
administrative costs $31.65 $633,000

The variable costs represent sales commissions paid at the rate of 6.2% of sales.
The step costs depend on the number of salespersons employed by the company.
In August there were 17 persons on the sales force. However, two members have taken early retirement effective August 3
It is anticipated that these positions will remain vacant for several months.
Total fixed costs are unchanged within a relevant range of 15,000 to 30,000 units per month.
Tut is planning a sales price cut of 10%, which it expects will increase sales volume to 24,000 units per month.

If Tut implements the sales price reduction, the total budgeted selling and administrative costs for the month of Septem
61. Tut Company’s selling and administrative costs for the month of August, when it sold
20,000 units, were as follows.

Variable costs $18.60 $372,000


Step costs 4.25 85,000
Fixed costs 8.80 176,000
Total selling and
administrative costs $31.65 $633,000

62. Granite Company sells products exclusively on account, and has experienced the
following collection pattern: 60% in the month of sale, 25% in the month after sale, and
15% in the second month after sale. Uncollectible accounts are negligible. Customers
who pay in the month of sale are given a 2% discount. If sales are $220,000 in January,
$200,000 in February, $280,000 in March, and $260,000 in April, Granite’s accounts
receivable balance on May 1 will be

jan feb
220000 200000
60% in the month of sale,
25% in the month after sale
15% in the second month after sale.

62. Granite Company sells products exclusively on account, and has experienced the
following collection pattern: 60% in the month of sale, 25% in the month after sale, and
15% in the second month after sale. Uncollectible accounts are negligible. Customers
who pay in the month of sale are given a 2% discount. If sales are $220,000 in January,
$200,000 in February, $280,000 in March, and $260,000 in April, Granite’s accounts
receivable balance on May 1 will be
jan
$220,000.00

60% in the month of sale 60%


25% in the month after sale 25%
15% in the second month after sale 15%

jan
$220,000.00

60% in the month of sale 60%


25% in the month after sale 25%
15% in the second month after sale 15%

65. Brown Company estimates that monthly sales will be as follows.

January $100,000 $ 100,000


February 150,000 $ 150,000
March 180,000 $ 180,000

Historical trends indicate that 40% of sales are collected during the month of sale, 50%
are collected in the month following the sale, and 10% are collected two months after the
sale. Brown’s accounts receivable balance as of December 31 totals $80,000 ($72,000
from December’s sales and $8,000 from November’s sales). The amount of cash Brown
can expect to collect during the month of January is

dec sales 120000


Nov sales 80000

Acc rec scs1 scs2


dec coll in jan 60,000 60000
nov coll in jan 8000 8000
jan coll in jan 40000 40000
108,000 108,000

65. Brown Company estimates that monthly sales will be as follows.

January $100,000 $100,000.00


February 150,000 $150,000.00
March 180,000 $180,000.00

Historical trends indicate that 40% of sales are collected during the month of sale, 50%
are collected in the month following the sale, and 10% are collected two months after the
sale.
Brown’s accounts receivable balance as of December 31 totals $80,000 ($72,000
from December’s sales and $8,000 from November’s sales). The amount of cash Brown
can expect to collect during the month of January is

40% of sales are collected during the month of sale


50% are collected in the month following the sale
10% are collected two months after the

66. Cooper Company’s management team is preparing a cash budget for the coming quarter.
The following budgeted information is under review.
January
Revenue $700,000 $800,000 $500,000 $ 700,000
Inventory purchases 350,000 425,000 225,000 $ 350,000
Other expenses 150,000 175,000 175,000 $ 150,000

The company expects to collect 40% of its monthly sales in the month of sale and 60% in
the following month. 50% of inventory purchases are paid in the month of purchase, and
the other 50% in the following month. All payments for other expenses are made in the
month incurred.

coll 40% $ 280,000


coll 60% $ 300,000

dis 50% 175000


dis 50% $ 500,000

Cooper forecasts the following account balances at the beginning of the quarter.

Cash $ 100,000
Accounts receivable $ 300,000
Accounts payable (Inventory) $ 500,000
Given the above information, the projected change in cash during the coming quarter will
be

66. Cooper Company’s management team is preparing a cash budget for the coming quarter.
The following budgeted information is under review.

January
Revenue $700,000 $800,000 $500,000 $700,000.00
Inventory purchases 350,000 425,000 225,000 $350,000.00
Other expenses 150,000 175,000 175,000 $150,000.00

The company expects to collect 40% of its monthly sales in the month of sale and 60% in
the following month. 50% of inventory purchases are paid in the month of purchase, and
50% in the following month. Payments for all other expenses are made in the month
incurred.

Brooke forecasts the following account balances at the beginning of the quarter.

Cash $100,000 $100,000.00


Accounts receivable 300,000 $300,000.00
Accounts payable (Inventory) 500,000 $500,000.00

Given the above information, the projected change in cash during the coming quarter will
be

January
40% in the same mnth $280,000.00
60% follow $300,000.00
cash coll $580,000.00
pyt of 50% purchase $175,000.00
pyt of 50% purchase in the follow $500,000.00
pyt purchase $675,000.00
Other expenses $150,000.00
total disb $825,000.00

Cash B.B $100,000.00


Net coll - disbur -$245,000.00
net -$145,000.00
Given the above information, the projected change in cash during the coming quarter will

65. Brown Company estimates that monthly sales will be as follows.

January $100,000 $100,000.00


February 150,000 $150,000.00
March 180,000 $180,000.00

Historical trends indicate that 40% of sales are collected during the month of sale, 50%
are collected in the month following the sale, and 10% are collected two months after the
sale.
Brown’s accounts receivable balance as of December 31 totals $80,000 ($72,000
from December’s sales and $8,000 from November’s sales). The amount of cash Brown
can expect to collect during the month of January is

40% of sales are collected during the month of sale


50% are collected in the month following the sale
10% are collected two months after the

67. Bootstrap Corporation anticipates the following sales during the last six months of the
year. cr sales
July $460,000 $ 460,000 $ 368,000
August 500,000 $ 500,000 $ 400,000
September 525,000 $ 525,000 $ 420,000
October 500,000 $ 500,000 $ 400,000
November 480,000 $ 480,000 $ 384,000 96000
December 450,000 $ 450,000 $ 360,000 198000

20% of Bootstrap’s sales are for cash. The balance is subject to the collection pattern
shown below.

Percentage of balance collected in the month of sale 40%


Percentage of balance collected in the month following sale 30%
Percentage of balance collected in the second month following sale 25%
Percentage of balance uncollectible 5%

What is the planned net accounts receivable balance as of December 31?


$ 294,000

67. Bootstrap Corporation anticipates the following sales during the last six months of the
year.
CASH SALES CR SALES
July $460,000 $460,000.00 $92,000.00 $368,000.00
August 500,000 $500,000.00 $100,000.00 $400,000.00
September 525,000 $525,000.00 $105,000.00 $420,000.00
October 500,000 $500,000.00 $100,000.00 $400,000.00
November 480,000 $480,000.00 $96,000.00 $384,000.00
December 450,000 $450,000.00 $90,000.00 $360,000.00
$2,915,000.00 $583,000.00 $2,332,000.00
20% of Bootstrap’s sales are for cash.

The balance is subject to the collection pattern shown below.


Percentage of balance collected in the month of sale 40%
Percentage of balance collected in the month following sale 30%
Percentage of balance collected in the second month following
sale 25%
Percentage of balance uncollectible 5%

What is the planned net accounts receivable balance as of December 31?

20% of Bootstrap’s sales are for cash.


Percentage of balance collected in the month of sale 40%
Percentage of balance collected in the month following sale 30%
Percentage of balance collected in the second month following
sale 25%

Sales $2,915,000.00
coll $2,504,400.00
gross acc/rec $410,600.00
5% $116,600.00
net Acc/Rec $294,000.00

68. Projected monthly sales of Wallstead Corporation for January, February, March, and
April are as follows.

January $300,000 $300,000.00


February 340,000 $340,000.00
March 370,000 $370,000.00
April 390,000 $390,000.00

• The company bills each month's sales on the last day of the month.
• Receivables are booked gross and credit terms of sale are: 2/10, n/30.
• 50% of the billings are collected within the discount period, 30% are collected by
the end of the month, 15% are collected by the end of the second month, and 5%
become uncollectible.

Budgeted cash collections for Wallstead Company during April would be

50% of the billings are collected within the discount period


30% are collected by the end of the month,
15% are collected by the end of the second month,
5% become uncollectible.

69. Tip-Top Cleaning Supply carries a large number of different items in its inventory, giving
the firm a competitive advantage in its industry. Below is part of Tip-Top’s budget for
the first quarter of next year.

Sales $855,000 $855,000.00


Cost of goods sold 425,000 $425,000.00
Rent and salary expenses 375,000 $375,000.00

Historically, all of the sales are on account and are made evenly over the quarter. 5% of
all sales are determined to be uncollectible and written off. The balance of the
receivables is collected in 50 days. This sales and collection experience is expected to
continue in the first quarter. The projected balance sheet for the first day of the quarter
includes the following account balances

Cash $ 10,000 $10,000.00


Accounts receivable (net) 450,000 $450,000.00
Inventory 900,000 $900,000.00
Accounts payable 800,000 $800,000.00

How much cash can Tip-Top anticipate collecting in the first quarter (based on a 360-day year)?

$450,000.00
$361,000.00
$811,000.00

70. Monroe Products is preparing a cash forecast based on the following information.
• Monthly sales: December $200,000; January $200,000; February $350,000;
March $400,000.
• All sales are on credit and collected the month following the sale.
• Purchases are 60% of next month’s sales and are paid for in the month of
purchase.
• Other monthly expenses are $25,000, including $5,000 of depreciation.
If the January beginning cash balance is $30,000, and Monroe is required to maintain a
minimum cash balance of $10,000, how much short-term borrowing will be required at
the end of February?

DEC
sales $200,000.00
pur $120,000.00
coll from sales
pyt pur $120,000.00
cash exp 20000

cash BB
minm

72. Prudent Corporation’s budget for the upcoming accounting period reveals total sales of
$700,000 in April and $750,000 in May. The sales cash collection pattern is

20% of each month’s sales are cash sales.


5% of a month’s credit sales are uncollectible.
70% of a month’s credit sales are collected in the month of sale.
25% of a month’s credit sales are collected in the month following the sale.

If Prudent anticipates the cash sale of a piece of old equipment in May for $25,000,
May’s total budgeted cash receipts would be

$700,000.00
April
cash sales $140,000.00
Cr sales $560,000.00

cash coll from cash sales


cash coll from cr sales in the same month
cash coll from cr from previous
other coll

73. ANNCO sells products on account, and experiences the following collection schedule.

In the month of sale 10% 10%


In the month after sale 60% 60%
In the second month after sale 30% 30%

At December 31, ANNCO reports accounts receivable of $211,500. Of that amount,


$162,000 is due from December sales, and $49,500 from November sales. ANNCO is
budgeting $170,000 of sales for January. If so, what amount of cash should be collected
in January?

In the month of sale 10% $17,000.00


In the month after sale 60% $108,000.00
In the second month after sale 30% $49,500.00
$174,500.00

74. Brooke Company’s management team is preparing a cash budget for the coming quarter.
The following budgeted information is under review.

January
Revenue $700,000 $800,000 $500,000 $700,000.00
Inventory purchases 350,000 425,000 225,000 $350,000.00
Other expenses 150,000 175,000 175,000 $150,000.00

The company expects to collect 40% of its monthly sales in the month of sale and 60% in
the following month. 50% of inventory purchases are paid in the month of purchase, and
50% in the following month. Payments for all other expenses are made in the month
incurred.

Brooke forecasts the following account balances at the beginning of the quarter.

Cash $200,000 $200,000.00


Accounts receivable 300,000 $300,000.00
Accounts payable (Inventory) 400,000 $400,000.00

Given the above information, the projected ending cash balance for February will be

January
40% in the same mnth $280,000.00
60% follow $300,000.00
cash coll $580,000.00
pyt of 50% purchase $175,000.00
pyt of 50% purchase in the follow $400,000.00
pyt purchase $575,000.00
Other expenses $150,000.00
total disb $725,000.00
Cash B.B $200,000.00
Net coll - disbur -$145,000.00
net $55,000.00

75. Health Foods Inc. has decided to start a cash budgeting program to improve overall cash
management. Information gathered from the past year reveals the following cash
collection trends.

40% of sales are on credit


50% of credit sales are collected in month of sale
30% of credit sales are collected first month after sale
15% of credit sales are collected second month after sale
5% of credit sales result in bad debts

Gross sales for the last five months were as follows


January $220,000
February 240,000
March 250,000
April 230,000
May 260,000

Sales for June are projected to be $255,000. Based on this information, the expected cash
receipts for March would be

march cash sales


march cash coll from cr sales
feb coll in march
jan coll in march
76. Tidwell Corporation sells a single product for $20 per unit. All sales are on account, with
60% collected in the month of sale and 40% collected in the following month. A
schedule of cash collections for January through March of the coming year reveals the
following receipts for the period.

January
December receivables $32,000 $32,000.00
From January sales 54,000 $36,000 $54,000.00
From February sales 66,000 $44,000
From March sales 72,000

Other information includes the following

• Inventories are maintained at 30% of the following month’s sales.


• Tidwell desires to keep a minimum cash balance of $15,000. Total
payments in January are expected to be $106,500, which excludes $12,000
of depreciation expense. Any required borrowings are in multiples of 1000
• The December 31 balance sheet for the preceding year revealed a cash
balance of $24,900.

Ignoring income taxes, the financing needed in January to maintain the firm’s minimum
cash balance is

cash coll $110,900.00


cash disbur. $121,500.00
-$10,600.00

77. Data regarding Johnsen Inc.’s forecasted dollar sales for the last seven months of the year
and Johnsen’s projected collection patterns are as follows.

Forecasted sales
June $700,000 $700,000.00
July 600,000 $600,000.00
August 650,000 $650,000.00
September 800,000 $800,000.00
October 850,000 $850,000.00
November 900,000 $900,000.00
December 840,000 $840,000.00

Types of sales
Cash sales 30% 0.3
Credit sales 70% 0.7

Collection pattern on credit sales (5% determined to be uncollectible)


During the month of sale 20%
During the first month following the sale 50%
During the second month following the sale 25%

Johnsen’s budgeted cash receipts from sales and collections on account for September are

Sep cash sales $240,000.00


coll from sep cr sales $112,000.00
coll from aug sales $227,500.00
coll from july sales $105,000.00
$684,500.00

78. The Mountain Mule Glove Company is in its first year of business. Mountain Mule had a
beginning cash balance of $85,000 for the quarter. The company has a $50,000 shortterm
line of credit. The budgeted information for the first quarter is shown below.

Sales
Purchases
Operating costs

All sales are made on credit and are collected in the second month following the sale.
Purchases are paid in the month following the purchase, while operating costs are paid in
the month that they are incurred. How much will Mountain Mule need to borrow at the
end of the quarter if the company needs to maintain a minimum cash balance of $5,000 as
required by a loan covenant agreement?

Coll Sales
Pyt. Purchases
Operating costs
cash coll - disb

[Fact Pattern #15]

Daffy Tunes manufactures a toy rabbit with moving


parts and a built-in voice box. Projected sales in units
for the next 5 months are as follows:

Projected
Month Sales in Units
January 30,000
February 36,000
March 33,000
April 40,000
May 29,000

[156] Gleim #: 2.3.42 -- Source: CMA 1296 3-4


(Refers to Fact Pattern #15)
Daffy Tunes’ unit production budget for toy rabbits for January is 54,000.0
A. 45,000 units.
B. 16,500 units.
C. 54,000 units.
D. 14,500 units.

Answer (C) is correct. January’s production should be 1.5 times February’s sales. Thus, the production budget for
January should be 54,000 units (36,000 units of February sales × 1.5).

[157] Gleim #: 2.3.43 -- Source: CMA 1296 3-5


(Refers to Fact Pattern #15)
Daffy Tunes’ dollar production budget for toy rabbits for February is
A. $327,000
B. $390,000
C. $113,500
D. $127,500
Daffy Tunes’ unit production budget for toy rabbits for FEB is 16,500

Answer (D) is correct. The units to be produced in February equal 50% of March sales, or 16,500 units (33,000 ×
.5). The unit variable cost is $7.00 ($3.50 + $1.00 + $2.00 + $.50), so total variable costs are $115,500 (16,500 ×
$7). Thus, the dollar production budget for February is $127,500 ($115,500 VUC + $12,000 FC).

[161] Gleim #: 2.3.47 -- Source: CMA 696 3-7


For the month of December, Crystal Clear Bottling expects to sell 12,500 cases of Cranberry Sparkling Water at $24.80
per case and 33,100 cases of Lemon Dream Cola at $32.00 per case. Sales personnel receive 6% commission on each case
of Cranberry Sparkling Water and 8% commission on each case of Lemon Dream Cola. In order to receive a commission
on a product, the sales personnel team must meet the individual product revenue quota. The sales quota for Cranberry
Sparkling Water is $500,000, and the sales quota for Lemon Dream Cola is $1,000,000. The sales commission that should
be budgeted for December is

Q P sales rev
Cranberry Sparkling Water 12,500 $ 24.8 $ 310,000.0
Lemon Dream Cola 33,100 $ 32.0 $ 1,059,200.0

VC

A. $4,736 Answer (C) is correct. The sale of 12,500 cases of Cranberry at $24.80 per case produces revenue of $
B. $82,152 amount that does not qualify for commissions. The sale of 33,100 cases of Lemon at $32 per case pro
C. $84,736 of $1,059,200. This amount is greater than the minimum and therefore qualifies for a commission of $
D. $103,336 ($1,059,200 × 8%). This calculation assumes that commissions are paid on all sales if the revenue quo

[166] Gleim #: 2.3.52 -- Source: Publisher


Harvin, Inc. pays out sales commissions to its sales team in the month the company receives cash for payment. These
commissions equal 5% of total (monthly) cash inflows as a result of sales. Harvin has budgeted sales of $300,000 for
August, $400,000 for September, and $200,000 for October. Approximately half of all sales are on credit, and the other
half are cash sales. Experience indicates that 70% of the budgeted credit sales will be collected in the month following the
sale, 20% the month after that, and 10% of the sales will be uncollectible. Based on this information, what should be the
total amount of sales commissions paid out by Harvin in the month of October?

budgeted sales cash inflows fm cash sales


aug $ 300,000 $ 150,000
sep $ 400,000 $ 200,000
oct $ 200,000 $ 100,000

Answer (B) is correct. Cash sales for Harvin, Inc. for the month of October are budgeted at $100,000 (half of
$200,000 overall sales). Projections for collections of credit sales in August indicate that 20% will be cash inflows
in October, or ($150,000 × 20%) = $30,000. Projections for collections of credit sales in September indicate that
70% will be cash inflows in October, or ($200,000 × 70%) = $140,000. Therefore, total cash inflows projected for
the month of October equal $100,000 + $30,000 + $140,000 = $270,000. Because sales commissions are set at 5%
of monthly cash inflows, the sales commissions for October equal ($270,000 × 5%) = $13,500.

[167] Gleim #: 2.3.53 -- Source: Publisher


Nick Klaus is a divisional manager for Tot Toys. He has been assigned the task of creating a production budget for his
division, which produces the company’s most popular stuffed animal. Budgeted sales for this toy for the next year have
been set at 650,000 units, desired ending finished goods inventory is 200,000 units, and Klaus would like there to be
100,000 equivalent units in ending work-in-process inventory. The starting finished goods inventory for the next year will
be 300,000 units, with 75,000 equivalent units in beginning work-in-process inventory. How many equivalent units should
Klaus plan for his division to produce?
A. 550,000
B. 575,000
C. 725,000
D. 925,000

Budgeted sale 650,000


Ei 200,000
E wip 100,000
BI 300,000
B wip 75,000

Prd 575,000

Answer (B) is correct. Using production-related budgets, units to produce equals budgeted sales + desired ending
finished goods inventory + desired equivalent units in ending work-in-process inventory – beginning finished
goods inventory – equivalent units in beginning work-in-process inventory. Therefore, in this case, units to
produce is equal to 650,000 + 200,000 + 100,000 – 300,000 – 75,000 = 575,000.

[Fact Pattern #16]


Mountain Corporation manufactures cabinets but outsources the handles.
Eight handles are needed for a cabinet, with assembly requiring 30 minutes of direct labor per unit.
Ending finished goods inventory is planned to consist of 50% of projected unit sales for the next month,
and ending handles inventory is planned to be 80% of the requirement for the next month’s projected unit output of finish

50%
Mountain’s projected unit sales: Ending finished goods inventory
October 4600 2500
November 5000 2100
December 4200 3000
January 6000
Mountain’s ending inventories in units at September 30:
Finished goods 3800
Handles 16000

[170] Gleim #: 2.3.56 -- Source: Publisher


(Refers to Fact Pattern #16)
The number of units that Mountain finished during December is
A. 3,000
B. 5,100
C. 4,200
D. 5,000

Answer (B) is correct. The company will need 4,200 finished units to meet the sales estimate for December. In
addition, 3,000 finished units (6,000 unit sales in January × 50%) should be in inventory at the end of December.
The total requirement is therefore 7,200 units (4,200 + 3,000). Of these units, 2,100 (4,200 unit sales in December
× 50%) should be available from November’s ending inventory. Consequently, production in December should be
5,100 units (7,200 – 2,100).

[171] Gleim #: 2.3.57 -- Source: Publisher


(Refers to Fact Pattern #16)
The number of handles Mountain should purchase in October is
A. 39,840
B. 76,800
C. 40,000
D. 36,800

the first step is to calculate Oct. prd 3300

the first step is to calculate nov. prd 4600

[173] Gleim #: 2.3.59 -- Source: CMA 694 3-7


Zohar Company’s budget contains the following information:
Zohar Company
Units
Beginning finished goods inventory 85 85
Beginning work-in-process in equivalent units 10 10
Desired ending finished goods inventory 100 100
Desired ending work-in-process in
equivalent units 40 40
Projected sales 1,800 1800
How many equivalent units should Zohar plan to produce?
A. 1,800 1845
B. 1,565
C. 1,815
D. 1,845

[Fact Pattern #17]


Rokat Corporation is a manufacturer of tables sold to schools, restaurants, hotels, and other institutions.
The table tops are manufactured by Rokat, but the table legs are purchased from an outside supplier.
The Assembly Department takes a manufactured table top and attaches the four purchased table legs.
It takes 20 minutes of labor to assemble a table.
The company follows a policy of producing enough tables to ensure that 40% of next month’s sales are in the finished good
Rokat also purchases sufficient raw materials to ensure that direct materials inventory is 60% of the following month’s sche

0.4
Rokat’s sales budget in units for the next quarter is as follows: ei
July 2300 1000
August 2500 840
September 2100
Rokat’s ending inventories in units for June 30 are
Finished goods 1900
Direct materials (legs) 4000

[175] Gleim #: 2.3.61 -- Source: CMA 695 3-14


(Refers to Fact Pattern #17)
The number of tables to be produced by Rokat during August is 2340
A. 1,400 tables.
B. 2,340 tables.
C. 1,440 tables.
D. 1,900 tables.

Answer (B) is correct. The company will need 2,500 finished units for August sales. In addition, 840 units (2,100
September unit sales × 40%) should be in inventory at the end of August. August sales plus the desired ending
inventory equals 3,340 units. Of these units, 40% of August’s sales, or 1,000 units, should be available from
beginning inventory. Consequently, production in August should be 2,340 units.
[176] Gleim #: 2.3.62 -- Source: CMA 695 3-15
(Refers to Fact Pattern #17)
Assume Rokat’s required production for August and September is 1,600 and 1,800 units, respectively, and the July 31
direct materials inventory is 4,200 units. The number of table legs to be purchased in August is

PRD
AUG 1600 6400
SEP 1800
BEG. DM INV AUG 4200
Targeted EI DM in Au 4320
6520

A. 6,520 legs.
B. 9,400 legs.
C. 2,200 legs.
D. 6,400 legs.

[Fact Pattern #18]


Jordan Auto has developed the following production plan: Each unit contains 3 pounds of direct material
The desired direct materials ending inventory
Month Units next month’s production, plus 500 pounds. (Th
January 10,000 Jordan has developed the following direct lab
February 8,000
March 9,000
April 12,000 Hours per unit 2.0 0.5
Hourly rate $6.75 $12.00

[178] Gleim #: 2.3.64 -- Source: CMA 697 3-14


(Refers to Fact Pattern #18)
How much direct materials should Jordan Auto purchase in March?
A. 27,000 pounds.
B. 32,900 pounds.
C. 36,000 pounds.
D. 37,800 pounds.

Answer (D) is correct. Jordan needs 27,000 pounds (3 × 9,000 units) of materials for March production. It also
needs 43,700 pounds {[(3 × 12,000 units to be produced in April) × 120%] + 500} for ending inventory. Given a
beginning inventory of 32,900 pounds {[(3 × 9,000 units to be produced in March) × 120%] + 500}, required
purchases equal 37,800 pounds (27,000 pounds + 43,700 pounds – 32,900 pounds).

[179] Gleim #: 2.3.65 -- Source: CMA 697 3-15


(Refers to Fact Pattern #18)
Jordan Auto’s total budgeted direct labor dollars for February usage should be
A. $156,000
B. $165,750
C. $175,500
D. $210,600

Answer (A) is correct. The standard unit labor cost is $19.50 [($6.75 × 2 hours in Department 1) + ($12 × .5 hour
in Department 2)], so the total budgeted direct labor dollars for February equal $156,000 (8,000 units × $19.50).

[194] Gleim #: 2.3.80 -- Source: CMA 687 4-17


Selo Imports uses flexible budgeting for the control of costs.
The company’s annual master budget includes $324,000 for fixed production supervisory salaries at a volume of 180,000 u
Supervisory salaries are expected to be incurred uniformly throughout the year.
During the month of September, 15,750 units were produced,
and production supervisory salaries incurred were $28,000.
A performance report for September would reflect a budget variance of
A. $350 favorable.
B. $350 unfavorable.
C. $1,000 unfavorable.
D. $1,000 favorable.

Answer (C) is correct. The $324,000 for supervisory salaries is a fixed cost, at a rate of $27,000 per month. Since
these costs are fixed, volume is irrelevant. Thus, the variance is the difference between actual costs of $28,000 and
the budgeted costs of $27,000, which equals $1,000 unfavorable

[195] Gleim #: 2.3.81 -- Source: CMA 687 4-18


Baxter Corporation’s master budget calls for the production of 5,000 units of product monthly.
The master budget includes indirect labor of $144,000 annually; Baxter considers indirect labor to be a variable cost.
During the month of April, 4,500 units of product were produced,
and indirect labor costs of $10,100 were incurred.
A performance report utilizing flexible budgeting would report a budget variance for indirect labor of

A. $1,900 unfavorable.
B. $700 favorable.
C. $1,900 favorable.
D. $700 unfavorable.

Answer (B) is correct. The $144,000 annual amount equals $12,000 per month. Since volume is expected to be
5,000 units per month, and the $12,000 is considered a variable cost, budgeted cost per unit is $2.40 ($12,000 ÷
5,000 units). If 4,500 units are produced, the total variable costs should be $10,800 (4,500 units × $2.40).
Subtracting the $10,100 of actual costs from the budgeted figure results in a favorable variance of $700.

[196] Gleim #: 2.3.82 -- Source: CMA 1287 4-29


The Jung Corporation’s budget calls for the following production:
BEG EI prd
Qtr 1 -- 45,000 units 45000 40500 34200 135000
Qtr 2 -- 38,000 units 38000 34200 30600 114000
Qtr 3 -- 34,000 units 34000 30600 43200 102000
Qtr 4 -- 48,000 units 48000 43200 0 144000

Each unit of product requires three pounds of direct material.


The company’s policy is to begin each quarter with an inventory of direct materials equal to 30% of that quarter’s direct ma
Budgeted direct materials purchases for the third quarter would be
A. 114,600 pounds.
B. 43,200 pounds.
C. 38,200 pounds.
D. 30,600 pounds.

Answer (A) is correct. Beginning inventory should be 30,600 pounds (34,000 units of budgeted sales × 3 pounds
× 30%). Ending inventory should be 43,200 pounds (48,000 units of budgeted sales for Quarter 4 × 3 pounds ×
30%). Since BI plus purchases minus EI equals Quarter 3 budgeted sales, purchases must be 114,600.
30,600 + X – 43,200 = 3 × 34,000
X – 12,600 = 102,000
X = 114,600

[200] Gleim #: 2.3.86 -- Source: CMA 691 3-4


DeBerg Company has developed the following sales projections for the calendar year.
cash 50% cash 45%
May $100,000 100000 50000
June 120,000 120000 60000 45000
July 140,000 140000 70000 54000 124000
August 160,000 160000 80000 63000 143000
September 150,000 150000 75000 72000 147000
October 130,000 130000 65000 67500
Normal cash collection experience has been that 50% of sales are collected during the month of sale and 45% in the month
following sale. The remaining 5% of sales is never collected. DeBerg’s budgeted cash collections for the third calendar
quarter are
A. $427,500
B. $422,500
C. $414,000 $ 414,000
D. $450,000
Answer (C) is correct. If 50% of sales are collected in the month of sale and 45% in the next month, with the
balance uncollectible, collections during the third quarter will be based on sales during June, July, August, and
September. As calculated below, total budgeted collections are $414,000.
June $120,000 × 45% = $ 54,000
July 140,000 × (50% + 45%) = 133,000
August 160,000 × (50% + 45%) = 152,000
September 150,000 × 50% = 75,000
$414,000

[Fact Pattern #22] 402000


Superflite expects April sales of its deluxe model airplane, the C-14, to be 402,000 units at $11 each. Each C-14 requires thr
purchased components shown below.
Number Needed
Purchase Cost for each C-14 Unit
A-9 $ .50 1 $ 0.50 1
B-6 .25 2 $ 0.25 2
D-28 1.00 3 $ 1.00 3
Factory direct labor and variable overhead per unit of C-14 totals $3.00.
Fixed factory overhead is $1.00 per unit at a production level of 500,000 units.

Superflite plans the following beginning and ending inventories for the month of April and uses standard absorption costin

Part No. Units at April 1 Units at April 30


C-14 12,000 10,000
A-9 21,000 9,000
B-6 32,000 10,000
D-28 14,000 6,000

[211] Gleim #: 2.3.97 -- Source: CMA 1293 3-10


(Refers to Fact Pattern #22)
Superflite’s C-14 production budget for April should be based on the manufacture of
A. 390,000 units.
B. 400,000 units. 400,000
C. 402,000 units.
D. 424,000 units.

Answer (B) is correct. Sales are expected to be 402,000 units in April. The beginning inventory is 12,000 units,
and the ending inventory is expected to be 10,000 units, a decline in inventory of 2,000 units. Thus, the budget
should be based on production of 400,000 units (402,000 units to be sold – 12,000 units BI + 10,000 units EI).
[212] Gleim #: 2.3.98 -- Source: CMA 1293 3-11
(Refers to Fact Pattern #22)
Assume Superflite plans to manufacture 400,000 units in April. Superflite’s April budget for the purchase of A-9 should
be
A. 379,000 units.
B. 388,000 units.
C. 402,000 units.
D. 412,000 units.

Answer (B) is correct. Each of the 400,000 units to be produced in April will require one unit of A-9, a total
requirement of 400,000 units. In addition, ending inventory is expected to be 9,000 units. Hence, 409,000 units
must be supplied during the month. Of these, 21,000 are available in the beginning inventory. Subtracting the
21,000 beginning inventory from 409,000 leaves 388,000 to be purchased.

[Fact Pattern #24]


Simpson, Inc. is in the process of preparing its annual budget. The following beginning and ending inventory levels (in units
planned for the year ending December 31.
Beginning Ending
Inventory Inventory
Raw material* 40000 50000
Work-in-process 10000 10000
Finished goods 80000 50000
*Two units of raw material are needed to produce each unit of finished product.

[217] Gleim #: 2.3.103 -- Source: CMA 1294 3-17


(Refers to Fact Pattern #24)
If Simpson, Inc. plans to sell 480,000 units during the year, the number of units it would have to manufacture during the
year would be
A. 440,000 units.
B. 480,000 units.
C. 510,000 units.
D. 450,000 units. 450,000

Answer (D) is correct. The finished units needed for sales (480,000), plus the units desired for ending inventory
(50,000), minus beginning inventory (80,000), equals the necessary production of 450,000 units.

[218] Gleim #: 2.3.104 -- Source: CMA 1294 3-18


(Refers to Fact Pattern #24)
If 500,000 finished units were to be manufactured for the year by Simpson, Inc., the units of raw material that must be
purchased would be
A. 1,000,000 units.
B. 1,020,000 units.
C. 1,010,000 units. 1,010,000
D. 990,000 units.

Answer (C) is correct. The 500,000 finished units to be manufactured require 1,000,000 units of raw material (2 ×
500,000). In addition, the inventory of raw material is planned to increase by 10,000 units. Consequently,
1,010,000 units of raw material should be purchased.

[226] Gleim #: 2.3.112 -- Source: CMA 684 4-26


Each unit of Product XK-46 requires three direct labor hours. Employee benefit costs are treated as direct labor costs.
Data on direct labor are
Number of direct employees 25 25
Weekly productive hours per employee 35 35
Estimated weekly wages per employee $245 $ 245
Employee benefits (related to weekly wages) 25% 25%
The standard direct labor cost per unit of Product XK-46 is
A. $21.00
B. $26.25
C. $29.40
D. $36.75

Answer (B) is correct. The standard direct labor unit cost equals 3 hours times the cost per DLH. This amount is
determined by adding employee benefits to weekly wages and dividing by hours per week.
Weekly wages $245.00
Add: benefits ($245 × .25) 61.25
Weekly total compensation $306.25
Divided by: hours/week ÷ 35
Cost per DL hour $ 8.75
Times: DL hours per unit × 3
Unit DL cost $ 26.25

[240] Gleim #: 2.4.126 -- Source: CMA 696 3-9


Trumbull Company budgeted sales on account of $120,000 for July, $211,000 for August, and $198,00
Collection experience indicates that 60% of the budgeted sales will be collected the month after the s
collected the second month, and 4% will be uncollectible. The cash receipts from accounts receivable
budgeted for September would be
july
aug
sep

A. $169,800
B. $147,960
C. $197,880
D. $194,760
Answer (A) is correct. The budgeted cash collections for September are $169,800 [($120,000 July sale
($211,000 August sales × 60%)].

[245] Gleim #: 2.4.131 -- Source: CMA Sample Q 02/2005 2-11


Holland Company is in the process of projecting its cash position at the end of the second quarter. Sho
pertinent information from Holland’s records.
Cash balance at end of 1st quarter $ 36,000
Cash collections from customers for 2nd quarter 1,300,000
Accounts payable at end of 1st quarter 100,000
Accounts payable at end of 2nd quarter 75,000
All 2nd quarter costs and expenses (accrual basis) 1,200,000
Depreciation (accrued expense included above) 60,000
Purchases of equipment (for cash) 50,000
Gain on sale of asset (for cash) 5,000
Net book value of asset sold 35,000
Repayment of notes payable 66,000

From the data above, determine Holland’s projected cash balance at the end of the second quarter.

Answer (D) is correct. The change in Holland’s cash balance can be calculated as follows:
Beginning cash balance $ 36,000
Add: cash collections 1,300,000
Less: net change in accounts payable (25,000)
Less: accrual-basis costs and expenses (1,200,000)
Add: depreciation expense (noncash) 60,000
Less: payment for equipment (50,000)
Add: net cash received from asset sale 40,000
Less: retirement of notes payable (66,000)
Ending cash balance $ 95,000
[256] Gleim #: 2.4.142 -- Source: CMA Sample Q 02/2005 2-10
Steers Company has just completed its prospective financial statements for the coming year. Relevant
summarized below:
Projected net income $100,000 100000
Anticipated capital expenditures 50,000 50000
Increase in working capital 25,000 25000
Depreciation expense 15,000 15000
From the information provided above, the increase in Steers’ cash account for the coming year will be

Answer (B) is correct. The change in cash can be calculated as follows:


GAAP-basis net income $100,000
Less: capital outlays (50,000)
Less: increase in working capital (25,000)
Add: depreciation expense (noncash) 15,000
Increase in cash $ 40,000

[Fact Pattern #23]


Birch Corporation has the following historical pattern on its credit sales:
70% collected in month of sale 70%
15% collected in the first month after sale 15%
10% collected in the second month after sale 10%
4% collected in the third month after sale 4%
1% uncollectible 1%

The sales on open account have been budgeted for the first 6 months of the year as follows:
January $ 70,000 $ 70,000
February 90,000 $ 90,000
March 100,000 $ 100,000
April 120,000 $ 120,000
May 100,000 $ 100,000
June 90,000 $ 90,000

[257] Gleim #: 2.4.143 -- Source: CMA 1289 4-24


(Refers to Fact Pattern #23)
Birch’s estimated total cash collections during April from accounts receivable are

70% collected in month of sale $ 84,000


15% collected in the first month after sale $ 15,000
10% collected in the second month after sale $ 9,000
4% collected in the third month after sale $ 2,800
$ 110,800
[258] Gleim #: 2.4.144 -- Source: CMA 1289 4-25
(Refers to Fact Pattern #23)
Birch’s estimated total cash collections during the second calendar quarter from sales made on open
second calendar quarter are

April
70% collected in month of sale $ 84,000

$ 84,000
May
70% collected in month of sale $ 70,000
15% from april sales collected in the first month after sa $ 18,000
$ 88,000

June
70% collected in month of sale $ 63,000
15% from may collected in the first month after sale $ 15,000
10% from april collected in the second month after sale $ 12,000
$ 90,000
35. B

5%
900,000.00

36. A a. $1,050,000.

Sales Revenue Item Volume in Units


$ 10,000,000 1 tea 160,000
$ 1,500,000 2 sugar 120,000
$ 9,000,000 3 cars 300,000
$ 20,500,000 Total sales revenue

me of Item 1 is fairly constant.

37. C c. 1,500 units.

150 $ 750,000
$ 400,000
$ 20 $ 100,000
$ 80,000
$ 150,000

number of units produced, with a denominator level of activity of 5,000 units.


ds sold in the year incurred.
s planned for this year is of concern to Trudy George, Troughton’s president.
d) if Troughton produces more units than it sells, thus building up thefinished goods inventory.
would be needed to generate the $50,000 budgeted operating income?

$ 50,000
$ 20,000
$ 30,000
1,500

38. B b. 9,231 units.

Product F
$ 15
$ 10
5,000 25,000
20%
$ 5
$ 1.00 $ 2.60

30,000
expected level of sales of both products.

2,307.69 11,538.46

0.3
n September. $ 715,000 $ 728,000 $ 624,000
over 25% of the next month’s sales.
40. C c. 78,000 units.

Q4
66000
-

41. B b. $3,930,000 3930000


220,000

4.4 50000 200,000

tion range of 200,000 to 300,000 units per month.


uction batches that Tyler runs.

nits, 45,000 units, and 50,000 units.


r of batches.
260,000

$ 1,056,000 $ 176,000.0

0,000 units. The 220,000 units for August would require a minimum of 5 batch set-ups.
6300
590 470
10%

71700
71,700

67,500
3,500 200 3,300
production for the first quarter?

44. D
a. 3,700. 45. A

c. $52. 46. C

Putter Heads
Laminated

unds @ $5/lb. 1 pound @ $5/lb.


5
1 pound @ $15/lb.
15
hour @ $20/hr. 1 hour @ $22/hr.
22
2000
80
$ 60
60
$ 3,120

2,000
2,000 4,000
$ 25,000
$ 15,000

$ 52.00

c. 6,100 units 47. C

Cash Receipts
February March

$ 36,000
$ 66,000 $ 44,000

February March
110000 150000
45000
$ 122,000
6,100

48. B
b. $675,000 $300,000
nt Inventory

20000
25%

to be purchse
$ 675,000
$ 300,000

d. $105,000. 49. D

d. $20,800 50. D
51. A a. 85,000.

20000
25000
90000
85000

a. $40,820 52. A

c. 8,700. 53. C
ontroller has
d. 404,000 shoes. 54. D
200000 400000
12000 24000
15000 30000
20000
18000

55. C c. $364,500
15%
60%
63000

ei
54000
58500
72000
76500
81000
75600
0
$ 364,500

a. $2.09. 56. D

b. $600,000 60. B

a. $652,760. 61. A

6,000,000 $ 300.00

Total
$ 372,000
$ 85,000 5000
$ 176,000

$ 633,000.00

6.2%

taken early retirement effective August 31.

e to 24,000 units per month. 24000


nistrative costs for the month of September would be

$ 6,480,000

$ 401,760
$ 75,000
$ 176,000
$ 652,760

61. A a. $652,760.

20,000.00 price 300 ###


372000
Costs
Per Unit Total
$18.60 $372,000.00
$4.25 $85,000.00 17 5000
$8.80 $176,000.00

$31.65 $633,000.00

taken early retirement effective August 31.

to 24,000 units per month.

nistrative costs for the month of September would be


24,000.00 price 270 ###
$401,760.00
Costs
Per Unit Total
$16.74 $401,760.00
$3.13 $75,000.00 17 4411.7647058824
$8.80 $176,000.00 3.75

$28.67 $652,760.00

c. $146,000. 62. C

0.6
0.25
0.15
0.02
220000

mar apr
280000 260000

42000
104000
$ 146,000

62. C c. $146,000.

feb mar april may


$200,000.00 $280,000.00 $260,000.00

$70,000.00 $65,000.00
$30,000.00 $42,000.00 $39,000.00

feb mar april may june


$200,000.00 $280,000.00 $260,000.00

$65,000.00
$42,000.00 $39,000.00

$146,000.00

65. C c. $108,000.

72000
8000

0.8333333333

65. C c. $108,000.
40%
50% 83%
10%
72000 8000

$40,000.00
$60,000.00
$8,000.00
$108,000.00

9/8/2010.

66. C c. $112,500.

February March
$ 800,000 $ 500,000 $ 2,000,000
$ 425,000 $ 225,000 $ 1,000,000
$ 175,000 $ 175,000 $ 500,000

40% 60%
50% 50%

$ 320,000 $ 200,000 $ 800,000


$ 420,000 $ 480,000 $ 1,200,000
$ -
212500 112500 $ 500,000
175000 212500 $ 887,500
$ 500,000
$ 112,500
$ 112,500

66. C c. $112,500.

February March
$800,000.00 $500,000.00
$425,000.00 $225,000.00
$175,000.00 $175,000.00

40% 60%
50%

February March
$320,000.00 $200,000.00
$420,000.00 $480,000.00
$740,000.00 $680,000.00
$212,500.00 $112,500.00
$175,000.00 $212,500.00
$387,500.00 $325,000.00
$175,000.00 $175,000.00
$562,500.00 $500,000.00

-$145,000.00 $32,500.00
$177,500.00 $180,000.00
$32,500.00 $212,500.00
$112,500.00

65. C c. $108,000.

40%
50% 83%
10%
72000 8000

$40,000.00
$60,000.00
$8,000.00
$108,000.00

67. B b. $294,000.

40%
30%
25%
5%

67. B b. $294,000.

NET CR SALES 5% 40% 30% 25%


$349,600.00 $18,400.00 $147,200.00 $110,400.00 $92,000.00
$380,000.00 $20,000.00 $160,000.00 $120,000.00 $100,000.00
$399,000.00 $21,000.00 $168,000.00 $126,000.00 $105,000.00
$380,000.00 $20,000.00 $160,000.00 $120,000.00 $100,000.00
$364,800.00 $19,200.00 $153,600.00 $115,200.00 $96,000.00
$342,000.00 $18,000.00 $144,000.00 $108,000.00 $90,000.00
$2,215,400.00 $116,600.00 $932,800.00 $699,600.00 $583,000.00

40.0%
30.0%
25.0%

5.0%

July August September October November

$92,000.00 $100,000.00 $105,000.00 $100,000.00 $96,000.00


$147,200.00 $160,000.00 $168,000.00 $160,000.00 $153,600.00
$110,400.00 $120,000.00 $126,000.00 $120,000.00
$92,000.00 $100,000.00 $105,000.00
$239,200.00 $370,400.00 $485,000.00 $486,000.00 $474,600.00

TWO
WAYS
TWO
WAYS

68. A a. $343,300

Jan feb march april


0.5 $181,300.00
0.3 $111,000.00
0.15 $51,000.00

$343,300.00

69. A

Q
90 40
$812,250.00 44%
60-day year)?

70. B b. $70,000

60%

25000 5000 20000


30000
10000

January February March


$200,000.00 $350,000.00 $400,000.00
$210,000.00 $240,000.00
$200,000.00 $200,000.00 $350,000.00
$210,000.00 $240,000.00 $0.00
20000 20000 20000

30000
10000
$0.00 -$70,000.00

20.00%
5.00% 0.95
70.00%
25.00%

$25,000.00

$750,000.00
May
$150,000.00
$600,000.00

$150,000.00
$420,000.00
$140,000.00
$25,000.00
$735,000.00

73. B

67%
33%

$211,500.00 $162,000.00 $49,500.00


$170,000.00
nov dec dec acc /rec
$49,500.00 $162,000.00 $211,500.00
108000 the amount of $162000 re

74. C

February March
$800,000.00 $500,000.00
$425,000.00 $225,000.00
$175,000.00 $175,000.00

40% 60%
50%

February March
$320,000.00 $200,000.00
$420,000.00 $480,000.00
$740,000.00 $680,000.00
$212,500.00 $112,500.00
$175,000.00 $212,500.00
$387,500.00 $325,000.00
$175,000.00 $175,000.00
$562,500.00 $500,000.00
$55,000.00
$177,500.00 $180,000.00
$232,500.00

75. C c. $242,000.

40%
50%
30%
15%
5%

Cash sales 60% Credit sales 40%


60% 40%
$220,000.00 $132,000.00 $88,000.00
$240,000.00 $144,000.00 $96,000.00
$250,000.00 $150,000.00 $100,000.00
$230,000.00 $138,000.00 $92,000.00
$260,000.00 $156,000.00 $104,000.00
$1,200,000.00

$150,000.00
$50,000.00
$28,800.00
$13,200.00
$242,000.00
$20.00

Cash Receipts
February March

$36,000.00
$66,000.00 $44,000.00
$72,000.00

30%
$15,000.00
$106,500.00

$24,900.00

since , Any required borrowings are in multiples of 1000

Cash sales 30% Credit sales 70%

$210,000.00 $490,000.00
$180,000.00 $420,000.00
$195,000.00 $455,000.00
$240,000.00 $560,000.00
$255,000.00 $595,000.00
$270,000.00 $630,000.00
$252,000.00 $588,000.00

0.2
0.5
0.25

85000

January February March


$60,000.00 $40,000.00 $50,000.00
$35,000.00 $40,000.00 $75,000.00
$25,000.00 $25,000.00 $25,000.00

5000

January February March


$60,000.00
$35,000.00 $40,000.00
$25,000.00 $25,000.00 $25,000.00
-$25,000.00 -$60,000.00 -$5,000.00 -$90,000.00

NET -$5,000.00
minimum cash balance -$5,000.00
Mule need to borrow -$10,000.00

Each rabbit requires basic materials that Daffy purchases from a single supplier at $3.50 per rabbit.
Voice boxes are purchased from another supplier at $1.00 each.
Assembly labor cost is $2.00 per rabbit,
and variable overhead cost is $.50 per rabbit.
Fixed manufacturing overhead applicable to rabbit production is $12,000 per month.
Daffy’s policy is to manufacture 1.5 times the coming month’s projected sales every other month, startin
and to manufacture 0.5 times the coming month’s projected sales in alternate months (i.e., even-numbe
This allows Daffy to allocate limited manufacturing resources to other products as needed during the ev

Thus, the production budget for 17 b - This is a very long question, but with only a few important pieces of informatio
$ 127,500.00 18 b - In February the production will be equal to 1 - 2 of March sales. March sales a
$.50), so total variable costs will be $115,500. We need to add to this the $12,000 of
ales, or 16,500 units (33,000 ×
costs are $115,500 (16,500 ×
$12,000 FC).

Cranberry Sparkling Water at $24.80


nel receive 6% commission on each case
ola. In order to receive a commission
quota. The sales quota for Cranberry
000. The sales commission that should

VC
6% less than
8% $ 84,736.0

t $24.80 per case produces revenue of $310,000, an


3,100 cases of Lemon at $32 per case produces revenue
therefore qualifies for a commission of $84,736
s are paid on all sales if the revenue quota is met.

y receives cash for payment. These


as budgeted sales of $300,000 for
all sales are on credit, and the other
be collected in the month following the 70%
n this information, what should be the 20%
10%

5%
bud cr sales cash inflows fm cr sales cash inflows com
$ 150,000
$ 200,000 $ 105,000
$ 100,000 $ 140,000 $ 30,000 $ 270,000 $ 13,500

geted at $100,000 (half of


e that 20% will be cash inflows
es in September indicate that
otal cash inflows projected for
sales commissions are set at 5%
= $13,500.

eating a production budget for his


les for this toy for the next year have
s, and Klaus would like there to be
d goods inventory for the next year will
tory. How many equivalent units should

udgeted sales + desired ending


ntory – beginning finished
ore, in this case, units to

ect labor per unit. 8


s for the next month,
t month’s projected unit output of finished goods. 80%

production in December should be 5100

s estimate for December. In


ntory at the end of December.
0 (4,200 unit sales in December
duction in December should be

The calculation to determine the number of handles to be purchased is as follows:


Handles for October production
(3,300 × 8 handles per cabinet) 26,400 26400
Handles for November production
required in October ending inventory
(4,600 × 8 × 80%) 29,440 29440
Less: Beginning inventory (16,000)
Total handles to be purchased in October 39,840 39,840
and other institutions.
an outside supplier.
urchased table legs. 4

ext month’s sales are in the finished goods inventory. 40%


tory is 60% of the following month’s scheduled production 60%

In addition, 840 units (2,100


ales plus the desired ending
should be available from
units, respectively, and the July 31
d in August is

unit contains 3 pounds of direct materials. 3


esired direct materials ending inventory each month is 120% of the 1.2 500
month’s production, plus 500 pounds. (The beginning inventory meets this requirement.)
an has developed the following direct labor standards for production of these units:

Department 1 Department 2
s per unit 2.0 0.5 2.0 0.5
y rate $6.75 $12.00 $ 6.75 $ 12.00

prd bi ei
27000 32900 43700 37800

or March production. It also


or ending inventory. Given a
× 120%] + 500}, required
$ 108,000 $ 48,000 $ 156,000

epartment 1) + ($12 × .5 hour


56,000 (8,000 units × $19.50).

ervisory salaries at a volume of 180,000 units. $ 324,000 180,000 27000

15750
28000 (1,000.00)

e of $27,000 per month. Since


ween actual costs of $28,000 and

Annual prd
uct monthly. 5,000 60000
ndirect labor to be a variable cost. $ 144,000 VC per unit $ 2.40
4,500 $ 10,800
$ 10,100
or indirect labor of $ 700

nce volume is expected to be


st per unit is $2.40 ($12,000 ÷
0 (4,500 units × $2.40).
able variance of $700.

Purchase
128700
110400
114,600
100800

3
s equal to 30% of that quarter’s direct material requirements. 0.3

of budgeted sales × 3 pounds


s for Quarter 4 × 3 pounds ×
s must be 114,600.

the month of sale and 45% in the month


sh collections for the third calendar
the next month, with the
uring June, July, August, and

$ 11
units at $11 each. Each C-14 requires three

$ 3.00
$ 1.00 500,000 $ 500,000

April and uses standard absorption costing for valuing inventory.

ng inventory is 12,000 units,


2,000 units. Thus, the budget
0 units BI + 10,000 units EI).
udget for the purchase of A-9 should

388,000

e one unit of A-9, a total


0 units. Hence, 409,000 units
g inventory. Subtracting the

ning and ending inventory levels (in units) are

would have to manufacture during the 480000

desired for ending inventory


450,000 units.

he units of raw material that must be 500000


0,000 units of raw material (2 ×
00 units. Consequently,

sts are treated as direct labor costs. 3

$ 7.00 per hour


$ 1.75
$ 8.75
$ 26.25

cost per DLH. This amount is

or July, $211,000 for August, and $198,000 for September. $ 120,000 $ 211,000 $ 198,000
es will be collected the month after the sale, 36% will be
e cash receipts from accounts receivable that should be
60% 36% 4%
$ 120,000
$ 211,000 72000
$ 198,000 126600 43200 $ 169,800

ember are $169,800 [($120,000 July sales × 36%) +

tion at the end of the second quarter. Shown below is

$ 36,000 $ 36,000
$ 1,300,000 $ 1,300,000
$ 100,000
$ 75,000 $ (25,000)
$ 1,200,000
$ 60,000 $ (1,140,000)
$ 50,000 $ (50,000)
$ 5,000
$ 35,000 $ 40,000
$ 66,000 $ (66,000)
$ 95,000

ance at the end of the second quarter.

an be calculated as follows:
statements for the coming year. Relevant information is

100,000.00
(50,000.00)
(25,000.00)
15,000.00
cash account for the coming year will be

$ 40,000

6 months of the year as follows:

ounts receivable are


endar quarter from sales made on open account during the

Answer (A) is correct. The second calendar quarter consists of April, May, and June. For April’s sales of $1
collections should be 95% (70% + 15% + 10%), or $114,000. For May’s sales of $100,000, collections sho
(70% + 15%), or $85,000. For June’s sales of $90,000, collections should be 70%, or $63,000. The quarter
$262,000 ($114,000 + $85,000 + $63,000).

$ 262,000

Karmee Company
YummyDog BoneCompany
b. $855,000 $812,250

ume in Units Sales Price Sales Revenue


55 $ 8,800,000
10 $ 1,200,000
30 $ 9,450,000
$ 19,450,000
December July

### ### 20% of Bootstrap’s sales are for cash. ###


### Percentage of balance collected in the month of sale 40% ###
### Percentage of balance collected in the month following sale 30%
### Percentage of balance collected in the second month following
### ###
a. $811,000.
72. C c. $735,000

b. $174,500

180000
e amount of $162000 represents 90% from Dec sales , thus the portion to be collected =60% from the 90%

c. $232,500
edit sales 40%
76. C c. $11,000.

77. B b. $684,500. simple


78. C c. $10,000.
t $3.50 per rabbit. $ 3.50
$ 1.00
$ 2.00
$ 0.50 $ 7.00
$ 12,000.00
ery other month, starting with January (i.e., odd-numbered months) for February sales, 1.5
onths (i.e., even-numbered months). 0.5
as needed during the even-numbered months.

tant pieces of information. In January, the production will be equal to 1.5 times the expected sales in February. Expected February sale
rch sales. March sales are expected to be 33,000, so February will see production of 16,500 units. The variable cost per unit is $7 ($3.5
dd to this the $12,000 of fixed costs giving us a total production cost of $127,500.
ne. For April’s sales of $120,000,
100,000, collections should be 85%
or $63,000. The quarterly total is

rmee Company
mmyDog BoneCompanyis
August SeptemberOctober NovemberDecemberJAN FEB MARCH

### ### ### ### ###


### ### ### ### ###
### ### ### ### ### ###
### ### ### ### ### ###

###
ebruary. Expected February sales are 36,000 so in January the company will produce 54,000 units.
variable cost per unit is $7 ($3.50 + $1 + $2 +
[803] Gleim #: 7.8.163 -- Source: CMA Sample Q 02/2005 2-40

Teaneck, Inc. sells two products, Product E and Product F, and had the following data for last month:

Product E Product F
Budget Actual Budget Actual
Unit sales 5500 6000 4500 6000
Unit contribution margin $ 4.50 $ 4.80 $ 10.00 $ 10.50

The company’s sales mix variance is

Answer (A) is correct. The first step is to calculate the contribution margin (CM) for a “composite” unit using
budgeted mix percentages and budgeted margins:

budgeted mix budgeted margins

Product E 55% $ 2.475

Product F 45% $ 4.50

Composite Budget UCM $ 6.98

This process is repeated using actual mix percentages and budgeted margins:

Actual mix budgeted margins

Product E 50% $ 2.25

Product F 50% $ 5.00

Composite Actual UCM $ 7.25 $ 0.28

The difference between the two is multiplied by the number of units sold to arrive at the sales mix variance [(6,000
+ 6,000) × ($7.250 actual – $6.975 budget) = (12,000 × $0.275) = $3,300 favorable].

[794] Gleim #: 7.8.154 -- Source: CMA 687 4-16

The following information is available for the Mitchelville Products Company for the month of July.

Master
Budget Actual
Units 4,000 3,800 4000 3800 -200
unit price $ 15.00 $ 14.00
Sales revenue $60,000 $53,200 $ 60,000 $ 53,200
Variable manufacturing costs 16,000 19,00 $ 16,000 $ 19,000
$ 4 $ 5
Fixed manufacturing costs 15,000 16,000 $ 15,000 $ 16,000
Variable selling and administrative
expense 8,000 7,600 $ 8,000 $ 7,600
$ 2 $ 2
Fixed selling and administrative
expense 9,000 10,000 $ 9,000 $ 10,000

CM $ 9 $ 7

The contribution margin volume variance for the month of July would be $ (1,800)

[Fact Pattern #64]


Product E&F Product E&F
Budget Actual
10000 12000
$ 14.50 $ 15.30

$ 3,300 $3,300 favorable.

x variance [(6,000
B. $1,800 unfavorable.
[76] Gleim #: 1.5.76 -- Source: CMA 697 4-22

Philip Enterprises, distributor of video discs, is developing its budgeted cost of goods sold for next year. Philip has
developed the following range of sales estimates and associated probabilities for the year:

Sales Estimate Probability COGS


60,000 25% 15,000 0.8
85,000 40% 34,000 0.8
100,000 35% 35,000 0.8
84,000

Philip’s cost of goods sold averages 80% of sales. What is the expected value of Philip’s budgeted cost of goods sold?

80% 67,200

Answer (D) is correct. The expected value is calculated by weighting each sales estimate by the probability of its
occurrence. Consequently, the expected value of sales is $84,000 [$60,000 × .25) + ($85,000 × .40) + ($100,000 × .35)].
Cost of goods sold is therefore $67,200 ($84,000 × .80).

[Fact Pattern #4]


A beverage stand can sell either soft drinks or coffee on any given day. If the stand sells soft drinks and the weather is hot, i
make $2,500; if the weather is cold, the profit will be $1,000. If the stand sells coffee and the weather is hot, it will make $1
if the weather is cold, the profit will be $2,000. The probability of cold weather on a given day at this time is 60%.

Hot
Soft Hot 40% $ 2,500 $ 1,000 40%
Cold 60% $ 1,000 $ 600
$ 1,600

Coffee Hot 40% $ 1,900 $ 760


Cold 60% $ 2,000 $ 1,200
Coffee $ 1,960

[81] Gleim #: 1.5.81 -- Source: CMA 1292 4-21


(Refers to Fact Pattern #4)
The expected payoff for selling coffee is

Answer (D) is correct. The expected payoff calculation for coffee is


Expected payoff = Prob. hot (Payoff hot) + Prob. cold (Payoff cold)
= .4($1,900) + .6($2,000)
= $1,960 $ 1,960
[82] Gleim #: 1.5.82 -- Source: CMA 1292 4-22
(Refers to Fact Pattern #4)
The expected payoff if the vendor has perfect information is

$ 2,200

Answer (B) is correct. The vendor would like to sell coffee on cold days ($2,000) and soft drinks on hot days
($2,500). Hot days are expected 40% of the time. Hence, the probability is 40% of making $2,500 by selling soft
drinks. The chance of making $2,000 by selling coffee is 60%. The payoff equation is:
Exp. payoff with perf. info. = Prob. hot (Payoff soft drinks) +
Prob. cold (Payoff coffee)
= .4($2,500) + .6($2,000)
= $2,200

Hot Cold
Soft Hot 0% $ 2,500 $ -
Cold 0% $ 1,000 $ -
EX VL Soft $ -

Coffee Hot 0% $ 1,900 $ -


Cold 0% $ 2,000 $ -
EX VL Coffee $ -

[83] Gleim #: 1.5.83 -- Source: CMA 1292 4-23


(Refers to Fact Pattern #4)
If the probability of hot weather, given a hot weather forecast, is 50%, how much would the vendor be willing to pay for
the forecast?

$ 1,900

$ 2,500 ‫ل( هفدم سوفت واكسب‬

$ 600

$ 300 50%
Answer (B) is correct. If the weather is hot and coffee is served, the vendor earns $1,900. If the vendor knows the
weather will be hot, (s)he would sell soft drinks and make $2,500, a $600 increase. Thus, the vendor should be willing to
pay up to $600 for perfect information regarding hot weather. However, if the forecasts are only 50% accurate, the
information is not perfect. Accordingly, the vendor should be willing to pay only $300 (the $600 potential increase in
profits × 50%) for the sometimes accurate forecasts

[Fact Pattern #5]


Butler and Burnside are projecting market conditions for the upcoming month. They have prepared the following payoff tab

Demand in Units
0 2 4 6
Probability of Demand
Supply in Units 10% 30% 40% 20%
0 - - - -
2 (80) 40 40 40
4 (160) (40) 80 80
6 (240) (120) - 120

[84] Gleim #: 1.5.84 -- Source: CMA 689 5-28


(Refers to Fact Pattern #5)
Butler and Burnside’s expected profit when supply equals 4 units is

Probability of Demand
Supply in Units 10% 30% 40% 20%
4 (160) (40) 80 80
-16 -12 32 16 20

[85] Gleim #: 1.5.85 -- Source: CMA 689 5-29


(Refers to Fact Pattern #5)
Butler and Burnside’s expected profit with perfect information is
0 12 32 24 68

[86] Gleim #: 1.5.86 -- Source: CMA 689 5-30


(Refers to Fact Pattern #5)
The price Butler and Burnside are willing to pay for perfect information is
Demand in Units
0 2 4 6
Probability of Demand
Supply in Units 10% 30% 40% 20%
0 - - - - 0
2 (80) 40 40 40 -8
4 (160) (40) 80 80 -16
6 (240) (120) - 120 -24

Ex Vl with - 12 32 24 68

Ex Vl without 28

Ex Vl of 40

Answer (B) is correct. The maximum amount the seller should pay for perfect information is the difference
between the expected profit with perfect information ($68) and the expected profit if demand is not known.
Without the perfect information, the seller should purchase the supply that will result in the maximum long-run
profit. Using the information given, it can be determined that the profit will be $20 when the supply is 4 units. It is
also evident that the profit is zero when the supply is zero. The expected profit must also be calculated for supply
levels of 2 and 6 units. For a supply of 2 units, the expected profit is
.1(–$80) + .3($40) + .4($40) + .2($40) = $28
For a supply of 6 units, the expected loss is
.1(–$240) + .3(–$120) + .4($0) + .2($120) = $(36)
Thus, without perfect information, profits are maximized at $28 when the supply is 2 units. However, with perfect
information, profits will be $68. Thus, a rational seller should therefore be willing to pay up to $40 ($68 – $28).
[Fact Pattern #6]
The College Honor Society sells hot pretzels at the home
football games. The pretzels are sold for $1.00 each, and
the cost per pretzel is $.30. Any unsold pretzels are
discarded because they will be stale before the next home
game.
The frequency distribution of the demand for pretzels per
game is presented as follows.
Unit Sales Volume Probability
2000 pretzels 10% 200
3000 pretzels 15% 450
4000 pretzels 20% 800
5000 pretzels 35% 1750
6000 pretzels 20% 1200
4400

[90] Gleim #: 1.5.90 -- Source: CMA 1289 5-20


(Refers to Fact Pattern #6)
The estimated demand for pretzels at the next home football game using an expected value approach is

91] Gleim #: 1.5.91 -- Source: CMA 1289 5-21


(Refers to Fact Pattern #6)
The estimated demand for pretzels at the next home football game using a deterministic approach based on the most likely
outcome is C. 5,000 pretzels.

Answer (C) is correct. A deterministic approach assumes that a value is known with certainty. If that value is deemed to
be the most likely outcome, assumed demand will be 5,000 pretzels, the volume with the highest probability (35%).

[92] Gleim #: 1.5.92 -- Source: CMA 1289 5-22


(Refers to Fact Pattern #6)
The conditional profit per game of having 4,000 pretzels available but only selling 3,000 pretzels is

selling 3000 $ 3,000.00


cost of 4000 $ 1,200.00
$ 1,800.00
Answer (A) is correct. Each pretzel costs $.30. Thus, the cost of 4,000 pretzels is $1,200 (4,000 × $.30). Selling
3,000 pretzels at $1 each produces revenue of $3,000. Subtracting the $1,200 of costs from the $3,000 of revenue
results in a conditional profit of $1,800.

93] Gleim #: 1.5.93 -- Source: CMA 1289 5-23


(Refers to Fact Pattern #6)
The conditional profit per game of having 4,000 pretzels available and selling all 4,000 pretzels is

selling 4000 $ 4,000.00


cost of 4000 $ 1,200.00
$ 2,800.00

Answer (C) is correct. Each pretzel costs $.30. Thus, the cost of 4,000 pretzels is $1,200 (4,000 × $.30). Selling
4,000 pretzels at $1 each produces revenue of $4,000. Subtracting the $1,200 of costs from the $4,000 of revenue
results in a conditional profit of $2,800.

[97] Gleim #: 1.5.97 -- Source: CMA 688 5-20


The following table contains the profit outcomes for each state of nature and decision combination for a firm

States of Nature
S1 S2 S3
Decision 1 24 14 -6
2.4 7 -2.4 7
Decision 2 20 10 5
2 5 2 9
Decision 3 -20 8 15
-2 4 6 8
Probabilities 10% 50% 40%

The expected value of perfect information for this firm in this case is

2.4 7 6 15.4 6.4

Answer (A) is correct. The first step is to determine the expected value without perfect information by formulating
a payoff matrix. For example, the expected payoff for the combination of State of Nature S1 and Decision 1 is
$2.40 (10% probability × $24 outcome). The entire payoff matrix is
S1 S2 S3 Total
Decision 1 $ 2.40 $7.00 $(2.40) $7.00
Decision 2 2.00 5.00 2.00 9.00
Decision 3 (2.00) 4.00 6.00 8.00
Thus, the best decision under conditions of uncertainty is Decision 2 (expected value = $9). If the decision maker
knew exactly when each state of nature would occur, the decision would correspond to the maximum profit
opportunity for that state of nature. For instance, if S1 is certain, the most profitable decision is Decision 1 ($24).
Thus, the expected payoff given perfect information is $15.40.

[Fact Pattern #8]


A company is considering three alternative machines to produce a new product. The cost structures (unit variable costs plu
avoidable fixed costs) for the three machines are shown as follows. The selling price is unaffected by the machine used.

Single purpose machine $.60x + $20,000


Semiautomatic machine $.40x + $50,000
Automatic machine $.20x + $120,000

The demand for units of the new product is described by the following probability distribution.

Demand Probability %xQ


200,000 40% 80,000
300,000 30% 90,000
400,000 20% 80,000
500,000 10% 50,000
300,000

[102] Gleim #: 1.5.102 -- Source: CMA 689 5-26


(Refers to Fact Pattern #8)
Ignoring the time value of money, the expected cost of using the semiautomatic machine is

Semiautomatic machine $.40x + $50,000 170,000.0

Answer (A) is correct. The expected demand is 300,000 units [(.4 × 200,000) + (.3 × 300,000) + (.2 × 400,000) +
(.1 × 500,000)]. Total expected cost is therefore $170,000 [$50,000 fixed cost + ($.40 × 300,000) variable cost].
[Fact Pattern #2]
In preparing the annual profit plan for the coming year,
Wilkens Company wants to determine the cost behavior
pattern of the maintenance costs. Wilkens has decided to use
linear regression by employing the equation y = a + bx for
maintenance costs. The prior year’s data regarding
maintenance hours and costs and the results of the regression
analysis are as follows.

Average cost per hour $ 9.00 9


a 684.65 684.65
b 7.2884 7.2884
Standard error of a 49.515
Standard error of b .12126
Standard error of the estimate 34.469
r 2 .99724

[35] Gleim #: 1.2.35 -- Source: CMA 1290 4-29


(Refers to Fact Pattern #2)
Based upon the data derived from the regression analysis, 420 maintenance hours in a month would mean that Wilkens
Co.’s maintenance costs (rounded to the nearest dollar) would be budgeted at 420
A. $3,780
B. $3,600
C. $3,790
D. $3,746
Answer (A) is incorrect because The budgeted maintenance costs are $3,746.
Answer (B) is incorrect because The budgeted maintenance costs are $3,746.
Answer (C) is incorrect because The budgeted maintenance costs are $3,746.
Answer (D) is correct. Substituting the given data into the regression equation results in a budgeted cost of $3,746
(rounded to the nearest dollar).
y = a + bx
y = 684.65 + 7.2884(420) $ 3,746
y = $3,746

[36] Gleim #: 1.2.36 -- Source: CMA 1290 4-30


(Refers to Fact Pattern #2)
The percentage of Wilkens Co.’s total variance that can be explained by the regression equation is
A. 99.724%
B. 69.613%
C. 80.982%
D. 99.862%
Answer (A) is correct. The coefficient of determination (r 2) measures the percentage of the total variance in cost
that can be explained by the regression equation. If the coefficient of determination is .99724, 99.724% of the
variance is explained by the regression equation. Thus, the values in the regression equation explain virtually the
entire amount of total cost.
Answer (B) is incorrect because The percentage of the total variance explained by the regression equation is
99.724%, which corresponds to the coefficient of determination (r 2), or .99724.
Answer (C) is incorrect because The percentage of the total variance explained by the regression equation is
99.724%, which corresponds to the coefficient of determination (r 2), or .99724.
Answer (D) is incorrect because The percentage of the total variance explained by the regression equation is
99.724%, which corresponds to the coefficient of determination (r 2), or .99724.

[37] Gleim #: 1.2.37 -- Source: Publisher


(Refers to Fact Pattern #2)
If Wilkens Company uses the high/low method of analysis, the equation for the relationship between hours of activity and
maintenance cost would be
A. y = 400 + 9.0x
B. y = 570 + 7.5x
C. y = 3,600 + 400x
D. y = 570 + 9.0x
Answer (A) is incorrect because The fixed cost and variable cost are wrong.
Answer (B) is correct. First, determine the months with the highest (520 hours in August) and lowest (300 hours in
April) levels of activity.
Hours Dollars
August 520 $ 4,470
April (300) (2,820)
Difference 220 $ 1,650
As the hours increased by 220, cost increased by $1,650, which is $7.50 per hour. Thus, at 300 hours of activity,
the total variable costs are $2,250 ($7.50 × 300 hours). Since the total cost was $2,820, the $570 above the variable
costs must be fixed costs. Substituting into the standard regression equation of y = a + bx gives y = $570 + $7.50x.
Answer (C) is incorrect because Both fixed and variable costs are wrong.
Answer (D) is incorrect because The variable cost is wrong.

[45] Gleim #: 1.3.45 -- Source: CMA 1293 4-24


The average labor cost per unit for the first batch produced by a new process is $120. The cumulative average labor cost
after the second batch is $72 per product. Using a batch size of 100 and assuming the learning curve continues, the total
labor cost of four batches will be $
A. $4,320 1 120
B. $10,368 2 72
C. $2,592 4 43.2
D. $17,280

Answer (A) is incorrect because The cost of the items in the fourth batch equals $4,320.
Answer (B) is incorrect because The amount of $10,368 is based on the assumption that the cumulative average
unit labor cost is reduced by the learning curve percentage with each batch, not each doubling of output.
Answer (C) is incorrect because The amount of $2,592 represents the labor cost of 100 units at the unit rate
expected after another doubling of production to eight batches.
Answer (D) is correct. The learning curve reflects the increased rate at which people perform tasks as they gain
experience. The time required to perform a given task becomes progressively shorter. Ordinarily, the curve is
expressed in a percentage of reduced time to complete a task for each doubling of cumulative production. One
common assumption in a learning curve model is that the cumulative average time (and labor cost) per unit is
reduced by a certain percentage each time production doubles. Given a $120 cost per unit for the first 100 units and
a $72 cost per unit when cumulative production doubled to 200 units, the learning curve percentage must be 60%
($72 ÷ $120). If production is again doubled to 400 units (four batches), the average unit labor cost should be
$43.20 ($72 × 60%). Hence, total labor cost for 400 units is estimated to be $17,280 (400 units × $43.20).

Fact Pattern #4]


Moss Point Manufacturing recently completed and sold an Direct materials $ 1,500
order of 50 units that had costs as shown in the next Direct labor (1,000 hours × $8.50) 8,500
column. Variable overhead (1,000 hours × $4.00)* 4,00
The company has now been requested to prepare a bid for Fixed overhead** 1,400
150 units of the same product. $15,400
*Applied on the basis of direct labor hours.
**Applied at the rate of 10% of variable cost

[46] Gleim #: 1.3.46 -- Source: CMA 1288 5-19


(Refers to Fact Pattern #4)
If an 80% learning curve is applicable, Moss Point’s total cost on this order would be estimated at
A. $26,400 doubl
B. $32,000 100
C. $38,000 200
D. $41,800 Direct materials
Direct labor
Variable overhead
Fixed overhead

[47] Gleim #: 1.3.47 -- Source: CMA 1288 5-20


(Refers to Fact Pattern #4)
If Moss Point had experienced a 70% learning curve, the bid for the 150 units would
A. Show a 30% reduction in the total direct labor hours required with no learning curve.
B. Include increased fixed overhead costs.
C. Be 10% lower than the total bid at an 80% learning curve. doubl
D. Include 6.40 direct labor hours per unit at $8.50 per hour. 100
200
Direct labor

46. A learning curve of 80% assumes that direct 80%


labor costs are reduced by 20% for each doubling of 20%
output. What is the incremental cost of the sixteenth
unit produced as an approximate percentage of the
first unit produced?
1 1
2 1.6
4 2.56
51.20% 8 4.096
40.96% 40.96% 16 6.5536 8 2.4576

Answer (B) is correct. With an 80% learning curve, the average cost after 8 units is 51.20% of the cost of the first
unit (100% × 80% × 80% × 80%). After 16 units, the average cost is 40.96% (51.20% × 80%). Thus, the average
cost of units in the last batch (units 9 through 16) must have been 30.72% [(40.96% × 2) – 51.20%].

[53] Gleim #: 1.3.53 -- Source: CMA 692 4-5


Lake Corporation manufactures specialty components for the electronics industry in a highly labor intensive environment.
Arc Electronics has asked Lake to bid on a component that Lake made for Arc last month. The previous order was for 80
units and required 120 hours of direct labor to manufacture. Arc would now like 240 additional components. Lake
experiences an 80% learning curve on all of its jobs. The number of direct labor hours needed for Lake to complete the
240 additional components is
A. 360.0 doub hours
B. 187.2 80 120
C. 307.2 160 192
D. 256.0 320 307.2
240 187.2

54] Gleim #: 1.3.54 -- Source: CMA 696 4-7


It is estimated that a particular manufacturing job is subject to an 80% learning curve. The first unit required 50 labor
hours to complete. What is the cumulative average time per unit after completing four units?
A. 50.0 hours.
B. 40.0 hours. 80%
C. 32.0 hours. 1 50
D. 30.0 hours. 2 80
4 128 32
[Fact Pattern #5]
LCB, Inc. is preparing a bid to the Department of the Navy to produce engines for rescue boats. The company has manufact
these engines for the Navy for the past 3 years on an exclusive contract and has experienced the following costs:

Cumulative Total Cumulative Costs


Units Produced Materials Labor
10 60000 120000
20 120000 192000 240000 80%
40 240000 307200

At LCB, variable overhead is applied on the basis of $1.00 per direct labor dollar. $ 1
Based on historical costs, LCB knows that the production of 40 engines will incur $100,000 of fixed overhead costs.all comp
The bid request is for an additional 40 units; submitting bids are allowed to charge a maximum of 25% above full cost for e

[58] Gleim #: 1.3.58 -- Source: CMA 688 5-9


(Refers to Fact Pattern #5)
In order to ensure that the company would not lose money on the project, LCB’s minimum bid for the 40 units would be

80 $ 480,000 $ 491,520
DM DL VOH
40 increm $ 240,000 $ 184,320 $ 184,320 $ 608,640

[59] Gleim #: 1.3.59 -- Source: CMA 688 5-7


(Refers to Fact Pattern #5)
LCB’s rate of learning on the 3-year engine contract is 80%

[60] Gleim #: 1.3.60 -- Source: CMA 688 5-8


(Refers to Fact Pattern #5)
The maximum bid price that LCB, Inc. could submit to the Department of the Navy for the 40 units is
DM DL VOH FOH TC TC + profit
40 increm $ 240,000 $ 184,320 $ 184,320 $ 100,000 $ 708,640 $ 885,800
r next year. Philip has

12,000
27,200
28,000
67,200

eted cost of goods sold?

the probability of its


× .40) + ($100,000 × .35)].

drinks and the weather is hot, it will


e weather is hot, it will make $1,900;
ay at this time is 60%.

Cold
60%
‫‪nks on hot days‬‬
‫‪2,500 by selling soft‬‬

‫‪vendor be willing to pay for‬‬

‫لو الجو حار و قدمت قهوة هكسب‬

‫لكن لو انا عارف ان الجو حار )عن طريق دراسة مثل( هفدم سوفت واكسب‬

‫يعنى الدراسة مفيدة وهتفرق معايا ب‬

‫لو الدراسة دقيقة ب ‪%50‬ل بس‬


the vendor knows the
e vendor should be willing to
only 50% accurate, the
600 potential increase in

repared the following payoff table:


Ex val
0 0 0 0
12 16 8 28
-12 32 16 20
-36 0 24 -36

the difference
nd is not known.
e maximum long-run
e supply is 4 units. It is
calculated for supply

However, with perfect


to $40 ($68 – $28).
selling price cost
$ 1.00 $ 0.30

approach is B. 4,400 pretzels.

proach based on the most likely

y. If that value is deemed to


ghest probability (35%).
00 × $.30). Selling A. $1,800
the $3,000 of revenue

C. $2,800

00 × $.30). Selling
the $4,000 of revenue

ination for a firm

A. $6.40

mation by formulating
and Decision 1 is

If the decision maker


maximum profit
n is Decision 1 ($24).

ructures (unit variable costs plus


ected by the machine used.

) + (.2 × 400,000) +
000) variable cost].
Hours of Maintenance
Activity Costs
January 480 $ 4,200 480 4200
February 320 3,000 320 3000
March 400 3,600 400 3600
April 300 2820
May 500 4,350 500 4350
June 310 2,960 310 2960
July 320 3,030 320 3030
August 520 4470 220 1650 7.5
September 490 4,260 490 4260 3900
October 470 4,050 470 4050 570
November 350 3,300 350 3300
December 340 3,160 340 3160
Sum 4,800 $43,200 4800 43200
Average 400 $ 3,600 400 3600

th would mean that Wilkens

udgeted cost of $3,746

total variance in cost


24, 99.724% of the
n explain virtually the

ssion equation is
ssion equation is

ssion equation is

between hours of activity and

d lowest (300 hours in

00 hours of activity,
$570 above the variable
ves y = $570 + $7.50x.

umulative average labor cost


ng curve continues, the total
units
100
60% 200
400 $ 17,280.00

cumulative average
ing of output.
s at the unit rate

m tasks as they gain


arily, the curve is
ve production. One
or cost) per unit is
or the first 100 units and
rcentage must be 60%
or cost should be
its × $43.20).

1500
000 hours × $8.50) 8,500 1000 8.5 8500
ad (1,000 hours × $4.00)* 4,000 1000 4 4000 14000
1400
15400
e basis of direct labor hours.
e rate of 10% of variable cost 10%

80%
hours
1600
2560
4 6000
2560 21760
2560 10240 38000
3800
$ 41,800
$ 26,400

70% 150

hours
1400
1960 hours required to produce 200 units
$ 16,660 DL cost to produce 200 units
$ 8,160 960 $ 8.50 6.40

B. 31%
30.72%

f the cost of the first


Thus, the average

y labor intensive environment.


e previous order was for 80
nal components. Lake 240
ed for Lake to complete the 80%

rst unit required 50 labor


ats. The company has manufactured
d the following costs:

f fixed overhead costs.all companies 40 100000


mum of 25% above full cost for each order.
125%

bid for the 40 units would be


Question JOB-ORDER COSTING
ErgoFurn Inc. manufactures economically designed furniture for computer stations. ErgoFurn
uses a job order cost system and employs the full absorption accounting method for cost accumulation.

ErgoFurn's work-in-process inventory at April 30, 1993, consisted of the following jobs.

Job No. Item Units_

CC723 Computer caddy 20,000


CH291 Chair 15,000
PS812 Printer stand 25,000

At April 30, 1993, the company's finished goods inventory, which is evaluated using the flrstin,
first-out (FIFO) method, consisted of four items.

_____Item____ Quantity &


______Unit Cost______

Computer caddy 7,500 units @ $64 each


Chair 19400 19,400 units @ $35 each
Printer stand 21,000 units @ $55 each
Desk 11,200 units @ $102 each

At the end of April, the balance in ErgoFurn's Materials Inventory account, which includes
both raw materials and purchased parts, was $668,000. Additions to, and requisitions from,
the materials inventory during the month of May included the following.

Raw Materials
Additions $242,000 $396,000 $ 242,000
Requisitions:
Job CC723 $ 51,000
Job Ch291 $ 3,000
Job PS812 $ 124,000
Job DS444 $ 65,000
(5,000 desks) 5000

ErgoFurn applies factory overhead on the basis of machine hours.


The company's factory overhead budget for the fiscal year ending May 31, 1993, totals $4,500,000,
and the company plans to expend 900,000 machine hours during this period.
Through the first eleven months of the year, a total of 830,000 machine hours were used,
and total factory overhead amounted to $4,274,500.
During the month of May, machine hours and labor hours consisted of the following.

Account Machine Hours Labor Hours


CC723 12,000 11,600
CH291 4,400 3,600
PS812 19,500 14,300
DS444 14,000 12,500
Indirect labor 3,000
Supervision

Listed below are the Jobs that were completed and the unit sales for the month of May.
Requisitions: Item Quantity Completed
CC723 Computer caddy 20,000 20000
CH291 Chair 15,000 15000
DS444 Desk 5,000 5000

_Item_ Quantity Completed


Computer caddy 17,500
Chair 21,000
Printer stand 18,000
Desk 6,000

Required
A. Describe when it is appropriate for a company to use a job order cost system.
B. Calculate the dollar balance in ErgoFurn Inc.'s work-in-process inventory account as of
May 31, 1993.
C. Calculate the dollar value of the chairs In ErgoFurn Inc.'s finished goods Inventory as of
May 31, 1993.
D. Explain the proper accounting treatment for overapplied or underapplied overhead balances
when using a job order cost system.

B. Calculate the dollar balance in ErgoFurn Inc.'s work-in-process inventory account as of


May 31, 1993.

Dr. units Cr. units


CC723 20,000 $ 1,133,400 CC723 20,000
CH291 15,000 $ 510,000 CH291 15,000
PS812 25,000 $ 759,000
Job DS444 5000 $ 273,000 Job DS444 5,000
$ 2,675,400
C. Calculate the dollar value of the chairs In ErgoFurn Inc.'s finished goods Inventory as of
May 31, 1993.

current mc $ 34 15,000 $ 510,000


bI 35 19400 $ 679,000
sales $ (679,000)
sales $ 34 1,600 $ (54,400)
total sales $ (733,400)
EI 13,400 $ 455,600

SECTION B – ABC AND STANDARD COSTING

Alaire Corporation manufactures several different types of printed circuit boards; however,
two of the boards account for the majority of the company's sales.
The first of these boards,a television (TV) circuit board, has been a standard in the industry for several years.

The market for this type of board is competitive and, therefore, price-sensitive.
Alaire plans to sell 65,000 of the TV boards in 1993 at a price of $150 per unit.
65,000 $ 150

The second high-volume product , a personal computer (PC) circuit board, is a recent addition to Alaire's product line.
Because the PC board incorporates the latest technology, it can be sold at a premium price; the
1993 plans include the sale of 40.000 PC boards at $300 per unit.
40,000 $ 300

Alaire’s management group is meeting to discuss strategies for 1993, and the current topic of conversation is how to spend
The sales manager believes that the market share for the TV board could be expanded by concentrating
Alaire's promotional efforts in this area. In response to this suggestion, the production manager
said, "Why don't you go after a bigger market for the PC board? The cost sheets that I
get show that the contribution from the PC board is more than double the contribution from
the TV board. I know we get a premium price for the PC board; selling it should help overall
profitability."

Alaire uses a standard cost system, and the following data apply to the TV and PC boards.
TV Board PC Board
Direct material $80 $140 $ 80 $ 140
Direct labor 1.5 hours 4 hours 1.50 4.00
Machine time .5 hours 1.5 hours 0.50 1.50

Variable factory overhead is applied on the basis of direct labor hours.


For 1993, variable factory overhead is budgeted at $ 1,120,000, and
direct labor hours are estimated at 280,000.
The hourly rates for machine time
The hourly rates for direct labor

Alaire applies a material handling charge at 10 percent of material cost; this material handling charge is not included in v
10%
Total 1993 expenditures for material are budgeted at $10,600,000.

Budgeted Cost
Material overhead:
Procurement $400,00 $ 400,000
Production schedulin $ 220,000
Packaging and shippi $ 440,000
$ 1,060,000
Variable overhead:
Machine set-up $ 446,000
Hazardous waste disp $ 48,000
Quality control $ 560,000
General supplies $ 66,000
$ 1,120,000
Manufacturing:
Machine insertion $ 1,200,000
Manual insertion $ 4,000,000
Wave soldering $ 132,000
$ 5,332,000

_Required per unit_


Parts
Machine insertion
Manual insertion
Machine set-ups
Hazardous waste
Inspections

Ed Welch, Alaire's controller, believes that before the management group proceeds with the
discussion about allocating sales and promotional dollars to individual products, it might be
worthwhile to look at these products on the basis of the activities involved in their production.
As Welch explained to the group, "Activity-based costing integrates to cost of all activities,
known as cost drivers. Into individual product costs rather than including these costs in overhead
pools." Welch has prepared the schedule shown above to help the management group
understand this concept.

Using this information, Welch explained, "we can calculate an activity-based cost for each
TV board and each PC board and then compare it to the standard cost we have been using.
The only cost that remains the same for both cost methods is the cost of direct material. The
cost drivers will replace the direct labor, machine time, and overhead costs in the standard
cost."

Required
A. Identify at least four general advantages that are associated with activity-based costing.
B. On the basis of standard costs, calculate the total contribution expected in 1993 for Alaire Corporation's
1) TV board.
2) PC board.
C. On the basis of activity-based costs, calculate the total contribution expected in 1993 for Alaire Corporation's
1) TV board.
2) PC board.
D. Explain how the comparison of the results of the two costing methods may impact the decisions made by Alaire Corpo

Solution
A. At least four general advantages associated with activity-based costing include the following.
􀁹 Provides management with a more thorough understanding of complex product costs
and product profitability for improved resource management and pricing decisions.
􀁹 Allows management to focus on value-added and nonvalueadded activities so that
nonvalueadded activities can be controlled or eliminated, thus streamlining production
processes.
􀁹 Highlights the Interrelationship (cause and effect) of activities which identifies opportunities
to reduce costs, i.e., designing products with fewer parts in order to reduce
the cost of the manufacturing process.
􀁹 Provides more appropriate means of charging overhead costs to products.

B. 1. Using standard costs, the total contribution expected in 1993 by Allaire Corporation
from the TV Board is $1,950,000, calculated as follows.

Per Unit
Revenue $ 150
Direct material $ 80
Material overhead (10% of material) $ 8
Direct labor ($14 x 1.5 hours) $ 21
Variable overhead ($4x 1.5hours)* $ 6
Machine time ($10 x.5) $ 5
Total cost $ 120
Unit contribution $ 30 Total contribution (65,000 x $30) $ $ 30

2. Using standard costs, the total contribution expected in 1993 by Allaire Corporation
from the PC Board is $2,360,000, calculated as follows.

Per Unit

Revenue $ 300
Direct material $ 140
Material overhead (10% of material) $ 14
Direct labor ($14 x 4 hours) $ 56
Variable overhead ($4x 4 hours)* $ 16
Machine time ($10 x.5) $ 15
Total cost $ 241
Unit contribution $ 30 Total contribution (65,000 x $30) $ $ 59

C. Shown below are the calculations of the cost drivers which apply to both C. 1. and C. 2.

1) Using activity-based costing, the total contribution expected in 1993 by Allaire Corporation
from the TV Board is $2,557,100 calculated as follows.

Per Unit
Revenue $150.00 $9,750,000 $ 150
Direct material 80.00 5,200,000 $ 80
Material overhead
Procurement ($.10x25) 2.50 162,500 $ 2.50
Production scheduling 2.00 130,000 $ 2.00
Packaging & shipping 4.00 260,000 $ 4.00
Variable overhead
Machine set-ups ($1.60x2) 3.20 208,000 $ 3.20
Waste disposal ($3 x .02) .06 3,900 $ 0.06
Quality control 3.50 227,500 $ 3.50
General supplies .60 39,000 $ 0.60
Manufacturing
Machine insertion ($.40 x 24) 9.60 624,000 $ 9.60
Manual insertion 4.00 260,000 $ 4.00
Wave soldering 1.20 78,000 $ 1.20
Total cost 110.66 7,192,900 $ 110.66
Unit contribution $39.34 $ 39.34
Total contribution

2) Using activity-based costing, the total contribution expected in 1993 by Allaire Corporation
from the PC Board is $1,594,000 calculated as follows.

Per Unit
Revenue $300.00 $12,000,000 $ 300
Direct material 140.00 5,600,000 $ 140
Material overhead
Procurement ($.10x55) 5.50 220,000 5.5
Production scheduling 8 0,0 00 Packaging & 2.00
shipping 4.00 160,000 4.00
Variable overhead
Machine set-ups ($1.60x3) 4.80 192,000 4.8
Waste disposal ($3 x .35) 1.05 42,000 1.05
Quality control ($3.50 x 2) 7.00 280,000 7
General supplies .60 24,000 0.60
Manufacturing
Machine insertion ($.40 x 35) 14.00 560,000 14
Manual insertion ($4 x 20) 80.00 3,200,000 80
Wave soldering 1.20 48,000 1.20
Total cost 260.15 10,406,000 $ 260
Unit contribution $ 39.85 $ 39.85
Total contribution $1,594,000

D. The analysis using standard costs shows that the unit contribution of the PC Board is almost
double that of the TV Board. On this basis, Alaire's management is likely to accept
the suggestion of the production manager and concentrate promotional efforts on expanding
the market for the PC Boards.
However, the analysis using activity-based costs does not support this decision. This
analysis shows that the unit dollar contribution from each of the boards is almost equal,
and the total contribution from the TV Board exceeds that of the PC Board by almost
$1,000,000. As a percentage of selling price, the contribution from the TV Board is double
that of the PC Board, e.g., 26 percent versus 13 percent.

Hawthorn Company

Question
Hawthorn Company manufactures three lawncare component parts: fuel systems, transmission
assemblies, and electrical systems. For the past five years, manufacturing overhead has
been applied to products on standard direct labor hours for the units actually produced. The
standard cost information is shown below.
The current direct labor rate is $10 per hour. New machinery that highly automates the production
process, was installed two years ago and greatly reduced the direct labor time to
produce the three products. The selling price for each of the three products is 125 percent of
the manufacturing cost.
Hawthorn’s segment of the lawncare component industry has become very competitive, and
the company’s profits have been decreasing. Jim Briggs, controller, has been asked by the
president of the company to analyze the overhead allocations and pricing structure. Briggs
thinks that future allocations should be based on machine hours and direct labor hours rather
than the current allocation method which is based on direct labor hours, only. Briggs has determined
the additional product information shown below.

Standard Cost Information


Fuel Transmission Electrical
Systems Assemblies Systems
Units produced and sold 10,00 10,000 20,000 30,000
Standard labor hours 2.0 1.5 1. 2.00 1.50 1.00
Standard direct material cost $ 25 $ 36 $ 30

Budgeted and actual manufacturing overhead $3,920,000 $ 3,920,000 $ 49.00

Additional Product Information


Fuel Transmission Electrical
Systems Assemblies Systems
Standard machine hours 2.0 4.0 2 4 6

Manufacturing overhead:
Direct labor hours 560,000 560,000
Machine hours 3,360,000 3,360,000

Required
A. By allocating all of the budgeted overhead based on direct labor hours, calculate the unit
manufacturing cost and unit sales price for each of the three products manufactured at
Hawthorn Company.
B. Prepare an analysis for Hawthorn Company using the appropriate cost driver(s) determined
by Jim Briggs for manufacturing overhead. Calculate the unit manufacturing cost
and unit sales price for each of the three products.
C. Based on your calculations in Requirements A and B, prepare a recommendation for the
president at Hawthorn Company to increase the firm's profitability.

Solution
A. The allocation of all of Hawthorn Company's budgeted manufacturing overhead based on
direct labor hours, results in the unit manufacturing costs and unit sales prices for its
three products, calculated as follows.

Fuel Transmission Electrical


Systems Assemblies Systems
Units 10,000 20,000 30,000
Standard labor hour/unit 2.0 1.5 1.0
Total standard labor 20,000 30,000 30,000 80,000
Direct material t $25.00 $36.0 $ 25.0 $ 36.0 $ 30.0
Direct labor at $10/hour 20.00 $ 20.0 $ 15.0 $ 10.0
Overhead a $49/DLH (1) 98.00 $ 98.0 $ 73.5 $ 49.0
Total cost $143.00 $124.50 $8 $ 143.0 $ 124.5 $ 89.0
Sales price at 125% of cost $1 $ 178.75 $ 155.63 $ 111.25

Note:
(1) Total manufacturing overhead labor hour of $3,920,000 ÷ 80,000 total direct labor
hours = $49.00 per direct labor hour

B. When the cost drivers, identified by Jim Briggs, are used to allocate manufacturing overhead,
the unit manufacturing costs and unit sales prices for the three products manufactured
at Hawthorn Company are calculated as follows.

Fuel Transmission Electrical


Systems Assemblies Systems
Units 10,000 20,000 30,000
Standard labor hour/unit 2.00 1.50 1.00
Total standard labor 20,000 30,000 30,000 80,000

Machine hours units 2.0 4.0 6. 2.00 4.00 6.00


Total machine hours 20.000 80 20,000 80,000 180,000 280,000

Direct material t $25.00 $36.0 $ 25 $ 36 $ 30


Direct labor at $10/hour 20.00 $ 20.0 $ 15.0 $ 10.0
Overhead - DLH at $7/hours (1) 14.0 10.5 7.0
Overhead – Machine hours 24.00 48.00 72.00
at $12/hour (2) 24.00 48.00 72.00
Total cost $83.00 $109.50 $119 $ 83 $ 110 $ 119
Sales price at 125% of cost $1 $ 103.75 $ 136.88 $ 148.75

Note:
(1) Direct labor overhead of $560,000 * 80,000 total direct labor hours = $7.00 per direct
labor hour
(2) Machine overhead of $3,360,000 * 280,000 total machine hours = $12.00 per machine
hour

C. Presented below is a summary of the revised margins for each of Hawthorn Company's
three products assuming the sales prices developed in Requirement A (allocation of all
manufacturing overhead based on direct labor hours) is compared to revised costs developed
in Requirement B (allocation of manufacturing overhead based on cost drivers).

Fuel Transmission Electrical


Systems Assemblies Systems
Current price $178.75 $155.63 $ 178.75 $ 155.63 $ 111.25
Revised Cost 83.00 109.50 119 $ 83.00 $ 109.50 $ 119.00
Gross $95.75 $46.13 $(7.75) $ 95.75 $ 46.13 $ (7.75)
Margin 54% 30% N/A 54% 30% -7%
N/A
Based upon the above analysis, fuel systems and transmission assemblies are producing
a higher return than Hawthorn Company previously thought. In particular, fuel systems
are especially profitable with a 54 percent gross margin. The electrical systems, in
contrast, are losing money on a full-cost basis.
Recommendations for improving profitability include the following.
􀁹 Increase emphasis on fuel systems, which could include increasing marketing expenditures
and reducing the price to increase sales.
􀁹 Improve profitability of electrical systems, which could include improving the manufacturing
process to reduce the machine hours required. Also, decrease marketing
emphasis of this product, and increase the selling price if the market will bear it.

JOINT AND BY-PRODUCTS


Question
Princess Corporation grows, processes, packages, and sells three apple products—sliced apples
that are used in frozen pies, applesauce, and apple juice. The outside skin of the apple,
which is removed in the Cutting Department and processed as animal feed, is treated as a
by-product. Princess uses the net realizable (relative sales) value method to assign costs of
the joint process to its main products. The by-product is inventoried at its market value, and
the net realizable value of the by-product is used to reduce the joint production costs prior to
allocation to the main products. Details of Princess' production process are presented below.

􀁹 The apples are washed and the outside skin is removed in the Cutting Department.
The apples are then cored and trimmed for slicing. The three main products and the
by-product are recognizable after processing in the Cutting Department. Each product
is then transferred to a separate department for final processing.
􀁹 The trimmed apples are forwarded to the Slicing Department where they are sliced
and frozen. Any juice generated during the slicing operation is frozen with the slices.
􀁹 The pieces of apple trimmed from the fruit are processed into applesauce in the
Crushing Department. Again, the juice generated during this operation is used in the
applesauce.
􀁹 The core and any surplus apple generated from the Cutting Department are pulverized
into a liquid in the Juicing Department. There is a loss equal to eight percent of
the weight of the good output produced in this department.
􀁹 The outside skin is chopped into animal feed and packaged in the Feed Department.

A total of 270,000 pounds of apples were entered into the Cutting Department during November.
The schedule presented below shows the costs incurred in each department, the
proportion by weight transferred to the four final processing departments, and the selling
price of each end product.

Required
A. Princess Corporation uses the net realizable value method to determine inventory values
for its main products and by-products. For the month of November 1991, calculate the
1) resulting output for apple slices, applesauce, apple juice, and animal feed, in
pounds.
2) net realizable value at the split-off point for each of the three main products.
3) amount of the cost of the Cutting Department assigned to each of the three main
products and to the by-product in accordance with corporate policy.
4) gross margins in dollars for each of the three main products.

B. Comment on the significance to management of the gross margin dollar information by


main product for planning and control purposes, as opposed to inventory valuation.

Processing Data and Costs


Nov-91
Proportion of Product by
Weight
Department Costs Incurred Transferred to Departments
Cutting $60,000 none none $60,000.00 NONE
Slicing 11,280 33% $.80 $11,280.00 33%
Crushing 8,550 30 .55 $8,550.00 30%
Juicing 3,000 27 .40 $3,000.00 27%
Feed ___700 10 .10 $700.00 10%
Total $83.530 100% $83,530.00 100%

Solution
A. 1. For the month of November, 1991, Princess Corporation's resulting pounds of apple
slices, applesauce, apple juice, and animal feed were 89,100, 81,000, 67,500, and
27,000, respectively, as calculated in Exhibit 1.

Exhibit 1
Princess Corporation
Product Output in Pounds
Nov-91
Product Input Proportion Total Pounds Net
Pounds Lost Pounds
Slices 270, 270,000.00 33% 89,100.00 89,100.00
Sauce 270, 270,000.00 30% 81,000.00 81,000.00
Juice 270, 270,000.00 27% 72,900.00 5,400.00 67,500.00
Feed 270,0 270,000.00 10% 27,000.00 27,000.00
100% 270,000.00 270,000.00

Note(1)
Net Pounds: = 72,900 - (.08 x net pounds.)
= 72,900+ 1.08
= 67,500 net pounds

2. The net realizable value for each of the three main products is calculated below.

Calculation of net Realizable Value


at the Split-off Point
Net
Net Selling Separable Realizable
Product Pounds Price Revenue Costs Value

Slices 89, 89,100.00 $0.80 71,280.00 11280 60,000.00


Sauce 81,0 81,000.00 $0.55 44,550.00 8550 36,000.00
Juice 67,5 67,500.00 $0.40 27,000.00 3000 24,000.00
$142,830 $22,830 $120,000 142830 22830 120000

3. The net realizable value of the by-product is deducted from the production costs prior
to allocation to the main products, as presented below.
Allocation of Cutting Department Costs To Main and By-Products
Net realizable value (NRV) of by-product = By-product revenue -Separable costs
= $.10(270,000x10%)-$700 2,000.00
= $2,700 - $700
= $2,000
Costs to be allocated = Joint cost - NRV of by-product
= $60,000 - $2,000 58,000.00
= $58,000

4. The gross margin in dollars for each of Princess Corporation's three main products is
reflected in Exhibit 2 below.

Exhibit 2
Princess Corporation
Gross Margin in Dollars
Nov-91
Product Revenue Separable Joint Gross
__Cost___ __Cost__ __Margin_

Slices $ 7 71,280.00 11280 29,000.00 31,000.00


Sauce 44,5 44,550.00 8550 17,400.00 18,600.00
Juice __27 27,000.00 3000 11,600.00 12,400.00
$142,830 $ 142,830.00 22830 58,000.00 62,000.00

B. The gross margin dollar information by main product is determined by the arbitrary allocation
of joint production costs. As a result, these cost figures and the resulting gross
margin information are of little significance for planning and control purposes. The allocation
is made only for purposes of inventory valuation and income determination.
1. Account for all units (physical flow of quantities).

Beginning WIP 4,000.00


Started Units this period 21,000.00
Total Units to Account for 25,000.00

Step 2: compute Equivalent Units of Production


EUP Computation under FIFO (If materials are added as work in process (continually):
Material

Total units Completed 20,000.00


- Beginning WIP (regardless of % of completion) 4,000.00
Units Started and Completed this Period 16,000.00
+ Amount needed to complete Beginning WIP
+ Amount Completed on Ending WIP 5,000.00
EUP under FIFO 21,000.00

If materials are added at the beginning of a process :


Total Units Completed
+ Amount of materials Needed to Complete BWIP zero
.+ Amount of materials Added to Date on EWIP 100%
EUP for Materials -

If materials are added at the end of a process


Total Units Completed
+ Amount of materials Needed to Complete BWIP
+ Amount of materials Added to Date on EWIP zero
EUP for Materials -

EUP : Weighted Average


Ü  EUP under weighted average costing may be computed as follows:
Total Units Completed this period
Work to date on Ending WIP
EUP under weighted average -

Question
Kristina Company, which manufactures quality paint sold at premium prices, uses a single
production department. Production begins with the blending of various chemicals, which are
added at the beginning of the process, and ends with the canning of the paint.

Canning occurs when the mixture reaches the 90 percent stage of completion. The gallon cans are then
transferred to the Shipping Department for crating and shipment. Labor and overhead are
added continuously throughout the process. Factory overhead is applied on the basis of direct
labor hours at the rate of $3.00 per hour.

Prior to May, when a change in the process was implemented, work-in-process inventories
were insignificant. The change in the process enables greater production but results in material
amounts of work-in-process for the first time. The company has always used the
weighted average method to determine equivalent production and unit costs. Now, production
management is considering changing from the weighted average method to the first-in,
first-out method.
The following data relate to actual production during the month of May.

Costs for May


Work-in-process inventory. May 1
(4,000 gallons 25% complete) 4000 0.25
Direct materials-chemicals $ 45,600 45600
Direct labor ($10 per hour) 6,250 6250
Factory overhead 1,875 1875
May costs added
Direct materials-chemicals 228,400 228400
Direct materials-cans 7,000 7000
Direct labor ($10 per hour) 35,000 35000
Factory overhead 10,500 10500
Units for May
Gallons
Work-in-process inventory.
May 1 (25% complete) 4,000 4000 0.25
Sent to Shipping Department 20,000 20000
Started in May 21.000 21000
Work-in-process inventory,
May 31 (80% complete) 5,000 5000
or cost accumulation.

Accumulated
___Cost___

$ 900,000
$ 431,000
$ 250,000
$ 1,581,000

ng the flrstin,

Accumulated
_____Cost_____

$ 480,000
$ 679,000
$ 1,155,000
$ 1,142,400
$ 3,456,400

$ 668,000

Purchased Parts
$ 396,000

$ 104,000
$ 10,800
$ 87,000
$ 187,000

otals $4,500,000, $ 4,500,000


900,000 $ 5.00
830,000
$ 4,274,500
Labor Cost OH applied
$ 122,400 $ 60,000.00
$ 43,200 $ 22,000.00
$ 200,500 $ 97,500.00
$ 138,000 $ 70,000.00
$ 29,400
$ 57,600
$ 591,100

uantity Completed

uantity Completed

ead balances

$ (1,133,400)
$ (510,000)

$ (273,000)
$ (1,916,400) $ 759,000
BWIP $ 431,000
DM $ 13,800
DL $ 43,200
OH $ 22,000
TC $ 510,000

industry for several years.

ent addition to Alaire's product line.


um price; the

ent topic of conversation is how to spend the sales and promotion dollars for next year.
nded by concentrating
tion manager
$ 1,120,000
280,000 $ 4.00
$ 10
$ 14

erial handling charge is not included in variable factory overhead.

$ 10,600,000

Annual Activity C. Shown below are the calculations


Cost Driver for Cost Driver

Number of parts 4,000,000 parts 0.10 per


Number of boards 110,000 boards 2.00 per
Number of boards 110,000 boards 4.00 per

Number of set-ups 278,750 set-ups 1.60 per


Pounds of waste 16,000 pounds 3.00 per
Number of inspections 160,000 inspections 3.50 per
Number of boards 110,000 boards 0.60

Number of parts 3,000,000 parts 0.40 per


Number of parts 1,000,000 parts 4.00 per
Number of boards 110,000 boards 1.20 per

TV Board Pc Board
25 55
24 35
1 20
2 3
0.02 lb. 0.35 lb.
1 2

r production.

sts in overhead
93 for Alaire Corporation's

n 1993 for Alaire Corporation's

mpact the decisions made by Alaire Corporation's management group.

the following.

pportunities

Totals for
65,000 units
$ 9,750,000
$ 5,200,000
$ 520,000
$ 1,365,000
$ 390,000
$ 325,000
$ 7,800,000
$ 1,950,000

Totals for
40,000 units
$ 12,000,000
$ 5,600,000
$ 560,000
$ 2,240,000
$ 640,000
$ 600,000
$ 9,640,000
$ 2,360,000

e Corporation

Totals for
65,000 units
$ 9,750,000
$ 5,200,000

$ 162,500
$ 130,000
$ 260,000

$ 208,000
$ 3,900
$ 227,500
$ 39,000

$ 624,000
$ 260,000
$ 78,000
$ 7,192,900

$ 2,557,100

e Corporation

Totals for
40,000 units
$ 12,000,000
$ 5,600,000

$ 220,000
$ 80,000
$ 160,000
$ 192,000
$ 42,000
$ 280,000
$ 24,000

$ 560,000
$ 3,200,000
$ 48,000
$ 10,406,000

$ 1,594,000

ard is almost

n expanding

transmission

s the production $ 10

hours rather
ggs has determined
determined
ing overhead,

7.00

12.00

s developed
anufacturing

—sliced apples

osts prior to

8%

ring November. 270,000.00


Selling Price per Pound
of Final Product
NONE
$0.80
$0.55
$0.40
$0.10

67500
50%
30%
20%

rary allocation
Total Units to Account for = 25,000.00
Finished or Transferred-out Goods 20,000.00
Ending WIP 5,000.00
Spoilage (lost)

Conversion

20,000.00
4,000.00
16,000.00
3000

19,000.00

puted as follows:

e gallon cans are then


asis of direct

ts in material

w, production
below are the calculations of the cost drivers which apply to both C. 1. and C. 2.

parts
boards
boards

set-ups
pounds
inspections

parts
parts
boards
Watkins Machinery Company uses a normal job costing system. The company has this
partial trial balance information on March 1, 2001, the last month of its fiscal year:

Materials Inventory (X, $3,000; Y, $2,000; Indirect materials, $5,000)


Work·in-Process Inventory-Job 101
Finished Goods Inventory-Job 100
Total
These transactions relate to the month of March:
a. Purchased direct materials and indirect materials with the following summary of receiving reports:

Material X
Material Y
Indirect materials
Total

b. Issued direct materials and indirect materials with this summary of requisition forms:

Job 101 Job 102


Material X 5000 3000
Material Y 4000 3000
Subtotal 9000 6000
Indirect materials
Total

c. Factory labor incurred is summarized by these time tickets:


Job 101 12000
Job 102 8000
Indirect labor 5000
Total $ 25,000

d. Factory utilities, factory depreciation, and factory insurance incurred are summarized by these factory vouchers, invoic

Utilities 500
Depreciation 15000
Insurance 2500
Total $ 18,000

e. Factory overhead costs were applied to jobs at the predetermined rate of $15 per machine-hour. Job 101 incurred 1,20
f. Job 101 was completed; job 102 was still in process at the end of March.

f. Finished Goods Inventory 45,000


Work-in- Process Inventory 45,000
To record the job finished.
Totalmanufacturing cost for job 101
Beginninginventory $ 6,000
Direct materials added 9,000
Direct labor incurred 12,000
Factory overhead applied 18,000
Total $45,000

g. Job 100 and job 101 were shipped to customers during March. Both jobs had gross margins of 20 percent based on ma

g. Accounts Receivable 66,000


Sales 66,000
To record the total sales revenue of two jobs.

Cost of Goods Sold 55,000


Finished Goods Inventory 55,000
To record the total cost of goods sold.

Summaryofthe total cost in shipping orders


Job 100 $10,000 10000
Job 101 45,000 45000
Total $55,000 55000
Sales = $55,000 x 120% = $66,000

The company closed the overapplied or underapplied overhead to the Cost of Goods Sold account at the end of March.

Required
1. Prepare journal entries to record the transactions and events. Letter your
entries from a to g.
2. Compute the ending balance of the Work-in-Process Inventory account.
3. Compute the overhead variance and indicate whether it is overapplied or underapplied.
4 Close the overhead variance to the Cost of Goods Sold account.
2. Compute the ending balance of the Work-in-Process Inventory account.
2. Ending balance of the Work-in- Process Inventory account for Job 102:
Direct materials 6000
Direct labor 8000
Factory overhead applied 12000
Total ending balance 26000

3. Compute the overhead variance and indicate whether it is overapplied or underapplied.

Factory overhead variance:


Actual factory overhead
Indirect materials 8000
Indirect labor 5000
Utilities 500
Depreciation 15000
Insurance 2500 31000
Applied factory overhead 30000
Underapplied factory overhead -1000

4. To record the disposition of underapplied factory overhead by closing both


Factory Overhead and Factory Overhead Applied accounts to the Cost of Goods Sold account.

Factory Overhead Applied 30000


Cost of Goods Sold 1000
Factory Overhead 31000

Volume-Based Costing Versus ABC

Carter Company manufactures two products, Deluxe and Regular, and uses a traditional two-stage cost allocation system.
The first stage assigns all factory overhead costs to two production departments A and B, based on machine-hours.
The second stage uses direct labor-hours to allocate overhead to individual products.
For 2006, the fi rms budgeted $1,000,000 total factory overhead cost for these operations

Production Department A
Machine-hour 4,000 0.2
Direct labor-hour 20,000

The following information relates to the fi rm’s operations for the month of January, 2006:

Deluxe
Units produced and sold 200 800 200
Unit cost of direct materials $100 $ 50 100
Hourly direct labor wage rate $ 25 $ 20 25
Direct labor-hours in Department A per unit 2 2 2 400
Direct labor-hours in Department B per unit 1 1 1 200

Carter Company is considering implementing an activity-based costing system. Its management


accountant has collected the following information for activity cost analysis:
Budgeted Cost Driver
Activity Overhead

Material movement 7,000 Number of production runs 350 15


Machine setups 400,000 Number of setups 500 25 50
Inspections 588,000 Number of units 19,600 200 800
Shipment 5,000 Number of shipments 250 50 100
1,000,000

1. Volume-based costing system


Stage 1 Allocation
Total overhead allocated to Department A
$ 1,000,000 × ( 4,000 / 20,000 ) = $ 200,000 $ 200,000
Total overhead allocated to Department B
$ 1,000,000 × ( 16,000 / 20,000 ) = $ 800,000 $ 800,000

Stage 2 Allocation
Per Unit Cost
Deluxe Regular
Overhead allocated to Department A
($200,000/20,000) × 2 = $ 20 $ 10 $ 20
($200,000/20,000) × 2 = $ 20 $ 10 $ 20

Department B
($800,000/10,000) × 1 = 80 80 80
($800,000/10,000) × 1 = 80 80 80
Total $ 100 $ 100
Product cost per unit: Deluxe Regular
Direct materials $100 $ 50 100 50
Direct labor 75 60
$25 × (2 + 1) = 75
$20 × (2 + 1) = 60
Factory overhead 100 100 $ 100 $ 100
Unit cost $275 $210 $ 275 $ 210

Material movement 300 400


Machine setups 20000 40000
Inspections 6000 24000
Shipment 1000 2000
27300 66400
136.5 83
175 110
311.50 193.00

A Comparison of Volume-Based and Activity-Based Costing

The following example contrasts Steps 2 and 3 of the volume-based costing system using
direct labor-hours as the cost driver with an activity-based costing system that uses both
volume-based and activity-based cost drivers.
Haymarket BioTech, Inc. (HBT) produces and sells two secure communication systems,
AW (Anywhere) and SZ (SecureZone). AW uses satellite technology and allows parties whose
DNA is implanted in the device to communicate anywhere on the earth. SZ uses similar technology
except it allows communication between two parties who are within 10 miles of each
other. HBT has the following operating data for the two products:

AW SZ
Production volume 5000 20000
Selling price $ 400 $ 200
Unit direct materials and labor $ 200 $ 80
Direct labor-hours 25000 75000
Direct labor-hours per unit 5 3.75

Volume-Based Costing
The volume-based costing system that the fi rm uses assigns factory overhead (OH) based on
direct labor-hours (DLH). The fi rm has a total budgeted overhead of $2,000,000. Since the
fi rm budgeted 100,000 direct labor hours for the year, the overhead rate per direct labor hour
is $20 per direct labor hour.

Total overhead $2,000,000


Total DLH 25,000 + 75,000 = 100,000
Overhead rate per DLH $ 20.00

Since the fi rm uses 25,000 direct labor hours to manufacture 5,000 units of AW, the factory
overhead assigned to AW is $500,000 in total and $100 per unit:

Total OH assigned to AW $20 × 25,000 = $500,000


Number of units of AW 5,000
Factory overhead per unit of AW $ 100.00

The factory overhead for SZ is $1,500,000 in total and $75 per unit since the fi rm spent
75,000 direct labor hours to manufacture 20,000 units of SZ:

Total OH assigned to SZ $20 × 75,000 = $1,500,000


Number of units of SZ 20,000
Factory overhead per unit of SZ $ 75.00

In Exhibit 5.5 we show a product profi tability analysis using the fi rm’s volume based
costing system.

AW SZ
Unit selling price $400 $200 $ 400 $ 200
Unit product cost:
Direct materials and labor $200 $80 $ 200 $ 80
Factory overhead 100 75 $ 100 $ 75
Cost per unit 300 155 $ 300 $ 155
Unit margin $100 $ 45 $ 100 $ 45

Activity-Based Costing
In using an activity-based costing, HBT has identifi ed the following activities, budgeted costs,
and activity consumption cost drivers:

Activity Budgeted Cost Activity Consumption Cost Driver


Engineering $ 125,000 $ 125,000 Engineering hours
Setups 300,000 $ 300,000 Number of setups
Machine running 1,500,000 $ 1,500,000 Machine-hours
Packing 75,000 $ 75,000 Number of packing orders
Total $2,000,000 $ 2,000,000

HBT also has gathered the following operating data pertaining to each of its products:
AW AW SZ SZ
Engineering hours 5,000 7,500 12,500 5000 $ 50,000 $ 75,000 7500
Number of setups 200 100 300 200 $ 200,000 $ 100,000 100
Machine-hours 50,000 100,000 150,0 50000 $ 500,000 $ 1,000,000 100000
Number of packing orders 5,000 10,00 5000 $ 25,000 $ 50,000 10000
$ 775,000 $ 1,225,000
$ 10,000
6000
10000
$ 26,000

Journal Entries and Accounting for Overhead

$10,000 Materials Inventory 25,000


10,000 Accounts Payable 25,000
5,000 To record the purchase of direct materials and
$25,000 indirect materials.

Work-in- Process Inventory 15,000 15000


Total Factory Overhead 8,000 8000
8000 Materials Inventory 2 $ 23,000
7000 To record direct and indirect materials issued.
15000
8000
$ 23,000

Work-in-Process Inventory 20,000 20000


Factory Overhead 5,000 5000
Accrued Payroll 25,000
To record factory labor incurred.

these factory vouchers, invoices, and cost memos:

Factory Overhead 18,000 $ 18,000


Accounts Payable 500
Accumulated Depreciation-Factory 15,000
Prepaid Insurance 2,500
To record actual overhead costs incurred, including
factory utilities, depreciation, and insurance.

ne-hour. Job 101 incurred 1,200 machine-hours; job 102 used 800 machine-hours.
15 1200 800
e. e. Work-in-Process Inventory 30,000
Factory Overhead Applied 30,000
To record the application of factory overhead to jobs.
Summaryoffactory overhead applied
Job 1 ($15 x 1,2001 18000
Job 2 ($15 x 800) 12000
Total 30000

45000
45000

anufacturing cost for job 101 ..


inginventory $ 6,000 6000
materials added 9,000 9000
labor incurred 12,000 12000
y overhead applied 18,000 18000
45000

ns of 20 percent based on manufacturing cost.

ccount at the end of March.


stage cost allocation system.
sed on machine-hours.
Production Department B
16000 0.8 20000
10000

Regular
800
50
20
2 1600 2000
1 800 1000

Budgeted Driver Consumption


Quantity Deluxe Regular

350 15 20 20
500 25 50 800
19600 200 800 30
250 50 100 20
Note that the volume-based costing system overcosts the high-volume regular product a
the low-volume deluxe product
$ 2,000,000
100,000
$ 20.00

$ 500,000
5000
$ 100

$ 1,500,000
20000
$ 75

EXHIBIT 5.5
Product Profi tability Analysis
under Volume-Based Costing
Using the gathered data, the cost driver rate for each activity consumption cost driver is
Total calculated as follows:
12,500 $ 10
300 $ 1,000
150,000 $ 10
15,000 $ 5
500
15000
2500
-volume regular product and under costs
cost driver is
The following data are used in developing Stylistic's 2004 budget:

a. Each coffee table has the following product specifications:

Direct Materials
Particle board (PB) 9 board feet (b.f.) per table
Red oak (RO) 10 board feet (b.f.) per table
Direct manufacturing labor
Laminating labor 0.25 hours per table
Machining labor 3.75 hours per table

b. Inventory information in physical units for 2004 is Ending inventory is not just a leftover

Beginning Target Ending


Inventory Inventory
Direct materials
Particle board 20000 b.f 18000 b.f
Red oak 25000 b.f 22000 b.f

Finished goods
Coffee tables 5000 units 3000 units

c. Coffee table revenues expected for 2004 are

Selling price $ 392 per table


Units sold 52,000 coffee tables 13000

d. Costs expected for 2004 include:


2003 2004
Particle board (per b.f) $ 3.90 $ 4.00
Red oak (per b.f) $ 5.80 $ 6.00
Laminating labor (per hour) $ 24.00 $ 25.00
Machining labor (per hour) $ 29.00 $ 30.00

e. Other budgeted cost rates and amounts for 2004 are

Variablemanufacturing overhead costs- $ 9.5 per direct manufacturing laborhour


Variable non manufacturing costs-13.5% of revenues 13.5%
Fixed manufacturing overhead costs - $1,600,000 $ 1,600,000
Fixed non manufacturing costs - $1,400,000 $ 1,400,000

f. The inventoriable (manufacturing) cost is $275 $ 275.0 per coffee table in 2003.

Step 1: Prepare the Revenues Budget.


Schedule 1: Revenues Budget
For the Year Ended December 31, 2002
Selling Units Total
Price Sold Revenues
Coffee tables $ 392 52,000 $ 20,384,000

Step 2: Prepare the Production Budget (in Units).

Schedule 2: Production Budget (in Units)


For the Year Ended December 31, 2004
Coffee Tables
Budgeted unit sales (Schedule 1) 52,000.00
Add target ending finished goods inventory 3,000.00
Total requirements 55,000.00
Deduct beginning finished goods inventory 5,000.00
Units to be produced 50,000.00

The number of units to be produced, calculated in Schedule 2, isthe k


to computing the usage of direct materials in quantities and in dollars.

Prepare the Direct Materials Usage Budget and Direct Materials Purcnas Budget.

Schedule 3A: Direct Materials Usage Budget


For the Year Ended December 31,2004

Particle
Board (PB) Red Oak (RO)
Phvsical Units Budget
PB: 50,000 units x 9.00 b.f. per unit 450,000
RO: 50,000 units x 10.00 b.f. per unit 500,000
To be used in production, b.f. 450,000 500,000

Cost Budget
(Available trom beginning inventory)
PB: $3.90 per b.t. x 20,000 b.f. $ 78,000
RO: $5.80 per b.f. x 25,000 b.f. $ 145,000
To be obtained from purchases of this period:
PB: $4.00 per b.f. x (450,000 b.f. - 20,000 b.f.) $ 1,720,000
RO: $6.00 per b.t. x (500,000 b.f. - 25,000 b.f.) $ 2,850,000

Direct materials to be used $ 1,798,000 $ 2,995,000

Schedule 3B computes the budget for direct materials purchases, which depends on the bu
materials to be used, the beginning inventory of direct materials, and the target ending inv
materials
Schedule 3B computes the budget for direct materials purchases, which depends on the bu
materials to be used, the beginning inventory of direct materials, and the target ending inv
materials

Schedule 3B: Direct Materials Purchases Budget


For the Year Ended December 31, 2004

Particle Red Oak


Board (PB) (RO)
Phvsical Units Budget
Production usage
(from Schedule 3A) 450,000 500,000
Add target ending inventory 18000 22000
Total requirements 468,000 522,000
Deduct beginning inventory 20000 25000
Purchases 448,000 497,000

Cost Budget
PB: 448,000 b.f. x $4.00 per b.f. $ 1,792,000.00
RO: 497,000 b.t. x $6.00 per b.f. $ 2,982,000.00
Purchases $ 1,792,000.00 $ 2,982,000.00

Step 4: Prepare the Direct Manufacturing Labor Budget.


The computations of budgeted direct manufacturing labor costs appear in Schedule 4:

Schedule 4: Direct Manufacturing labor Budget


Forthe Year Ended December 31, 2004

Laminating Machining
Labor Labor
(LL) (ML)
Labor-Hours Budget
LL:50,000 units x 0.25 hours/unit 12,500 hours
ML: 50,000 units x 3.75 hours/unit 187,500
12,500 187,500
Cost Budget
LL:$25.00 per hour x 12,500 hours 312,500
ML: $30.00 per hour x 187,500 hours 5,625,000
312,500 5,625,000

Step 5: Prepare the Manufacturing Overhead Budget.

It inventories manufacturing overhead at the budgeted rate of $17.50 per direct manufacturing labor-hour
(total budgeted manufacturing overhead, $3,500,000 + 200,000 budgeted direct manufacturing labor-hours).

total budgeted manufacturing overhead (FIXED + V PER HOUR) $ 3,500,000


budgeted direct manufacturing labor-hours 200,000
Budgeted rate $ 17.50

The budgeted manufacturing overhead cost per coffee table

total budgeted manufacturing overhead (FIXED + V PER HOUR) $ 3,500,000


coffee tables budgeted to be produced in 2004. 50,000.00
The budgeted manufacturing overhead cost per coffee table $ 70.00

The $70 budgeted manufacturing overhead cost per coffee table can also be calculated as $17.50 budgeted cost
per direct manufacturing labor-hour x 4 budgeted direct manufacturing labor hours per coffee table = $70.

budgeted direct manufacturing labor hours per coffee table 4.00

Schedule 5: Direct Manufacturing Overhead Budget


For the Year Ended December 31, 2004

At Budgeted level of 200,000


Direct Manufacturing labor-Hours
Variable manufacturing overhead costs
Supplies $ 240,000
Indirect manufacturing labor $ 620,000
Power and energy $ 460,000
Maintenance $ 300,000
Miscellaneous $ 280,000
Fixed manufacturing overhead costs
Depreciation $ 500,000
Property taxes $ 350,000
Property insurance $ 260,000
Plant supervision $ 210,000
Miscellaneous $ 280,000
Total manufacturing overhead costs

Step 6: Prepare the Ending Inventories Budget.


Schedule 6A shows the computation of the unit cost of coffee tables started and completed in 2004
Under the fIrst-in,first-out method (FIFO), this unit cost is used to calculate the cost of target
ending inventories of finished goods in Schedule GB

Schedule 6A: Computation of Unit Costs of Ending Inventory of FinishedGoods


December 31.2004

Cost per
Unit of Input Input
Direct materials
Particle board (per b.f) $ 4.00 per b.f 9
Red oak (per b.f) $ 6.00 per b.f 10
Direct manufaturing labor
Laminating labor (per hour) $ 25.00 per hour 0.25
Machining labor (per hour) $ 30.00 per hour 3.75
Manufacturing Overhead $ 17.50 per hour 4.00
Total

This $284.75 unit cost for 2004 compares to $275.00 unit cost for 2003

Schedule 6B: Ending Inventories Budget


31-Dec-04

Cost
per Unit Units
Direct materials
Particle board (per b.f) $ 4.00 18000 $ 72,000
Red oak (per b.f) $ 6.00 22000 $ 132,000

Finished goods Coffee tables $ 284.75 3000 $ 854,250


Total ending inventory

Step 7: Prepare the Cost of Goods Sold Budget.


The information from Schedules 3 through 6 is used in Schedule 7

Schedule 7: Cost of Goods Sold Budget


For the Year Ended December 31,2004

Beginning finished goods inventory,


January 1, 2004, $275 x 5,000 Given $ 1,375,000
Direct materials used Schedule 3A $ 4,793,000
Direct manufacturing labor Schedule 4 5,937,500
Manufacturing overhead Schedule 5 $ 3,500,000 $ 14,230,500
Cost of goods manufactured 15,605,500
Cost of goods available for sale
Deduct ending finished goods inventory, Schedule 6B $ 854,250
December 31,2004
Cost of goods sold $ 14,751,250

Step 8: Prepare the Nonmanufacturing Costs Budget.

$
Variable non manufacturing costs-13.5% of revenues 13.5% $ 2,751,840
Fixed non manufacturing costs - $1,400,000 $ 1,400,000 $ 1,400,000
Step 9: Prepare the Budgeted Income Statement.

Revenues Schedule 1
Costof goods sold Schedule 7
Gross margin
Operating costs
R& D/Product design Schedule 8 $ 555,760
Marketingcosts Schedule 8 $ 1,920,720
Distributioncosts Schedule 8 $ 729,600
Customer-service costs Schedule 8 $ 504,992
Administrativecosts Schedule 8 $ 440,768
Operating income

Schedule 8: Nonmanufacturing Costs Budget


For the Year Ended December 31. 2004

Variable Fixed
Costs Costs
Value-Chain Function
R& D/Product design $ 305,760 $ 250,000
Marketing $ 1,630,720 $ 290,000
Distribution $ 509,600 $ 220,000
Customer service $ 264,992 $ 240,000
Administrative $ 40,768 $ 400,000
$ 2,751,840 $ 1,400,000

To see how sensitivity analysis works, let's consider two parameters in Stylistic
Furniture's budget model for 2004:
1. Selling price per table of $392.
2. Direct material prices of $4 per bJ. for particle board and $6 per bJ. for red oak.
What if either or both of these parameters were to change? Exhibit 6-4 presents the budgeted
operating income for nine combinations of different inputs for parameters 1 and 2:
8. Selling prices per table of (i) $431.20 (10% increase), (ii) $392.00 (original budgeted
price), and (iii) $352.80 (10% decrease).
b. Direct material purchase prices (i) decreasing by 5% to $3.80 per b.f. for particle board
and $5.70 per bJ. for red oak, (ii) remaining at the original budgeted price of $4.00 per
bJ. for particle board and $6.00 per b.f. for red oak, and (iii) increasing by 5% to $4.20
per bJ. for particle board and $6.30 per bJ. for red oak.
The nine combinations in Exhibit 6-4 show how budgeted operating income will change
considerably with changes in selling prices and direct material costs.

Effect of Changes in
Budget Assumptions on
Budgeted Operating
Income for Stylistic
Furniture

Balance Sheel lor


Slylislic Furnilure,
31-Dec-06

Current Assets
Cash $ 500,000
Accounts receivable $ 1,881,600
Direct materials inventory 223,000 $ 223,000
Finished goods inwntory 1,375,000 $3,979,600 $ 1,375,000 $ 3,979,600
Property, plant and equipment
Land $ 1,200,000
Building and equipment $ 2,300,000
Accumulated depreciation $ (800,000) $ 1,500,000 $ 2,700,000
Total $ 6,679,600

Liabilities and Stockholders Equity


Current Liabilities
Accounts payable $ 384,000
Income t8JreSpayable $ 20,460
Total cmrent liabilities $ 404,460
Long-term debt (interest at 10% per year) $ 2,400,000
Total cmrent and long-term liabilities $ 2,804,460
Stockholders' equity
Common stock, $0.01 parvalue, 300,000 sheres outstanding 3,000 $ 3,000
Retained earnings $ 3,872,140
Total $ 3,875,140
$ 6,679,600

Suppose Stylistic Furniture had the balance sheet for the year ended December 31,2006, shown
in Exhibit 6-5. The budgeted cash nows for 2007 are:
Quarters
1 2 3 4
Collections from customers $ 5,331,220 $ 4,704,000 $ 4,704,000 $ 6,272,000
Disbursements
Direct materials $ 960,000 $ 1,152,000 $ 1,152,000 $ 1,536,000
Payroll $ 1,626,300 $ 1,626,300 $ 1,888,600 $ 1,626,300
Other costs $ 1,580,460 $ 1,580,460 $ 1,580,460 $ 1,580,460
Machinery purchese $ - $ - $ 1,800,000 $ -
Interest expense on long-term deb$ 60,000 $ 60,000 $ 60,000 $ 60,000
Income taxes $ 100,000 $ 120,460 $ 100,000 $ 100,000

The quanerly data are based on the budgeted cash effects of the operations formulated in Sd1cdules
1 through 8 in the chapter, but the details of that formulation are not shown here to keep this illustration
as brief and as focused as possible.
Long-term debt is $2.4 million at an annual interest rate of 10%, with $60,000 interest payable
every quarter. The company wants to maintain a $100,000 minimum cash balance at the end of
each quarter. The company can borrow or repay money at an interest rate of 12% per year.

Management does not want to borrow any more short-term cash than is necessary. By special
arrangement, interest is computed and paid when the principal is repaid. Assume, for simplicity,
that borrowing takes place (in multiples of $1,000) at the beginning and repayment at the end of
the quarter under consideration. Interest is computed to the nearest dollar.
Suppose the management accountant at Stylistic is given the preceding data and the other data
contained in the budgets in the chapter (pp. 188-193). She is instructed as follows:

1. Prepare a cash budget for 2007 by quarter. That is, prepare a statemenl of cash receipts and disbursements
by quarter, including details of borrowing, repayment. and interest.
2. Prepare a budgeted balance sheel on December 31, 2007.
3. Prepare a budgeted income stalement for the year ended December 31, 2007. This statement
should include interest expense and income taxes (at a rate of 36% of operating income). In
April 2007, Stylistic will pay $120,640 of income taxes. This amount is the remaining payment
due for the 2006 income tax year togelher with tlle $100,000 Stylistic pays each quarter of
2007 toward its 2007 income tax bill. Any remaining amount due is paid in April 2008.

Quarters
1 2 3 4
Cash balance, beginnir1g $ 500,000 $ 1,504,440 $ 1,669,220 $ 100,160
Add receipts
Collections from customers $ 5,331,220 $ 4,704,000 $ 4,704,000 $ 6,272,000
Total cash available for needs (x) $ 5,831,220 $ 6,208,440 $ 6,373,220 $ 6,372,160
Deduct disbursements
Direct materials $ 960,000 $ 1,152,000 $ 1,152,000 $ 1,536,000
Payroll $ 1,626,300 $ 1,626,300 $ 1,888,600 $ 1,626,300
Other costs $ 1,580,460 $ 1,580,460 $ 1,580,460 $ 1,580,460
Machinery purchese $ - $ - $ 1,800,000 $ -
Interest expense on long-term de $ 60,000 $ 60,000 $ 60,000 $ 60,000
Income taxes $ 100,000 $ 120,460 $ 100,000 $ 100,000
Total disbursements (y) $ 4,326,760 $ 4,539,220 $ 6,581,060 $ 4,902,760
Minimum cash balance desired $ 100,000 $ 100,000 $ 100,000 $ 100,000
Total cash needed $ 4,426,760 $ 4,639,220 $ 6,681,060 $ 5,002,760
Cash excess(defIciency)a $ 1,404,460 $ 1,569,220 $ (307,840) $ 1,369,400
Financing
Bonowing (at beginning) $ - $ - $ 308,000
Repayment (at end) $ - $ - $ - $ (308,000)
Interest (at 12% per annum) b $ - $ - $ - $ (18,480)
Total effects of fmancing $ - $ - $ 308,000 $ (326,480)
Cash balance, ending c $ - $ - $ 160
g inventory is not just a leftover; it is a budgeted amount

195,000

irect manufacturing laborhour

The $20,384,000 is the amount of revenues in the budgeted income statement.


The revenues budget is often the outcome of elaborate information gathering and discussions am
sales representatives who have a detailed understanding of customer needs, market potential, a
Statistical approaches such as regression and trend analysis can also help in sales forecasting. Th
of economic activity and past sales data to forecast future sales. Managers should use statistical
forecast sales. In the final analysis, the sales forecast should represent the collective experience
The usual starting point for step 1 is to base revenues on expected demand. Occasionally, a facto
budgeted revenues. For example, when demand is greater than available production capacity or
supply, the revenues budget would be based on the maximum units that could be produced. Wh
limited by the amount produced.
sales representatives who have a detailed understanding of customer needs, market potential, a
Statistical approaches such as regression and trend analysis can also help in sales forecasting. Th
of economic activity and past sales data to forecast future sales. Managers should use statistical
forecast sales. In the final analysis, the sales forecast should represent the collective experience
The usual starting point for step 1 is to base revenues on expected demand. Occasionally, a facto
budgeted revenues. For example, when demand is greater than available production capacity or
supply, the revenues budget would be based on the maximum units that could be produced. Wh
limited by the amount produced.

ule 2, isthe k
nd in dollars.

Once we determine the number


of units to be produced from the
production budget (Schedule 2l,
we can budget manufacturing
inputs-direct materials, direct
labor, and overhead.

Total

$ 4,793,000

hases, which depends on the budgeted direct


erials, and the target ending inventory of direct
hases, which depends on the budgeted direct
erials, and the target ending inventory of direct

$ 4,774,000

These costs depend on wage rates, production methods, and hiring plans.

Total

hours
200,000

5,937,500

The total of these costs depends on how individual overhead costs vary wi
The calculations of budgeted manufacturing overhead costs appear in Sche
manufacturing overhead costs are based on input from Stylistic's operating
uring labor-hour and prior years. Management makes adjustments for cost changes expecte
manufacturing overhead as inventoriable costs. Stylistic does not use a sep
uring labor-hours). overhead rate and a separate fIxed manufacturing overhead rate.
per direct manufacturing labor-hour

Total variable overhead costs fluctuate in


proportion to the quantity of the cost-
allocation base (direct manufacturing labor-
hours in the Stylistic example!. whereas
total fixed overhead costs remains constant
per coffee table over a relevant range of output.

$17.50 budgeted cost


ffee table = $70.

Budgeted hours divided by budgeted prod.


per coffee table

dgeted level of 200,000


t Manufacturing labor-Hours

$ 1,900,000

$ 1,600,000
$ 3,500,000

Total

$ 36.00
$ 60.00 $ 96.00
$ 6.25
$ 112.50 $ 118.75
$ 70.00
$ 284.75

Total

$ 204,000

$ 854,250
$ 1,058,250

The cost of goods manufactured portion of the cost of


goods sold budget in Schedule 7 is a summarized
form ofthe cost of goods manufactured schedule (for a
different company) in Exhibit 2-6, Panel 8, p. 41. (Note,
the Stylistic example assumes there is no beginning or
ending work-inprocess inventory.)

Schedules 2 through 7 cover budgeting for Stylistic's production function part of the value chain
brevity, other parts of the value chain are combined into a single schedule Variable
nonmanufacturing costs are variable with respect to the amount of revenues at the rate of 13.5
revenues: $20,384,000 from Schedule 1 x 0.135 = $2,751,840. For example, variable product de
costs represent roy.
alty payments of 1.5% of revenues paid to the company that designed the table; variable marke
costs are 8% sales commission on revenues paid to salespersons; variable distribution costs are
of revenues for insurance and freight; and variable customer service costs equal 1.3% of revenu
paidto an outside party to service all warranty claims. The individual fixed COSI
amounts are based on input from Stylistic's business function managers in different parts of its
chain.
Schedules 1, 7, and 8 provide the information needed to complete the budgeted
Exhibit 6-3. More details could be included in the income statement: The more
$ 20,384,000 details put in the income statement, the fewer supporting schedules needed for
$ 14,751,250 the income statement.
$ 5,632,750 Top management's strategies for achieving revenue and operating income
goals influence the costs planned for the different business functions of the value
chain. As strategies change, the budgeted costs for different elements of the valu
chain will also change. For example, a shift in strategy toward emphasizing produ
development and customer service will result in increased costs in these parts
of the operating budget.

$ 4,151,840
$ 1,480,910

Total
Costs

$ 555,760
$ 1,920,720
$ 729,600
$ 504,992
$ 440,768
$ 4,151,840
his illustration

and disbursements

Years as whole
$ 500,000

$ 21,011,220
$ 21,511,220

$ 4,800,000
$ 6,767,500
$ 6,321,840
$ 1,800,000
$ 240,000
$ 420,460
$ 20,349,800
$ 100,000
$ 20,449,800
$ 1,061,420
ement.
ering and discussions among sales managers and
eds, market potential, and competitors' products.
in sales forecasting. These techniques use indicators
rs should use statistical analysis only as one input to
e collective experience and judgement of managers.
nd. Occasionally, a factor other than demand limits
production capacity or a resource factor is in short
could be produced. Why? Because sales would be
eds, market potential, and competitors' products.
in sales forecasting. These techniques use indicators
rs should use statistical analysis only as one input to
e collective experience and judgement of managers.
nd. Occasionally, a factor other than demand limits
production capacity or a resource factor is in short
could be produced. Why? Because sales would be
overhead costs vary with respect to the cost driver - direct manufacturing labor-hours, in this example.
ead costs appear in Schedule 5. The individual amounts for variable manufacturing overhead costs and fIxed
rom Stylistic's operating personnel. The starting point for these amounts is Stylistic's costs in the current
or cost changes expected in the future. Stylistic treats both variable manufacturing overhead and fIxed
listic does not use a separate variable manufacturing
overhead rate.
uate in
ost-
ing labor-
ereas
constant
cost of
d
e (for a
. (Note,
ning or

n part of the value chain. For


ule Variable
enues at the rate of 13.5% of
ple, variable product design

he table; variable marketing


le distribution costs are 2.5%
sts equal 1.3% of revenues
ed COSI
in different parts of its value
omplete the budgeted income statement, shown in
statement: The more
schedules needed for

perating income
s functions of the value
nt elements of the value
ard emphasizing product
d costs in these parts
Exhibit 9-4 Sales Budget Kerry sets the sales levels shown and a selling price of $30 per unit.

KERRY INDUSTRIAL COMPANY


Sales Budget
For the Quarter Ended June 30, 2002
April May June
Sales in units 20,000 25,000 35,000
Selling price per unit $ 30 $ 30 $ 30
Total sales $ 600,000 $ 750,000 $ 1,050,000

To illustrate, Kerry expects to have 5,000 units on hand at the beginning of the quarter, April 1.
The firm's operation guideline requires the inventory on hand at the end of each month to be 30 percent of the following mon
Kerry therefore has a production budget to manufacture 22,500 units in April as calculated

1. Determine the desired amount of ending inventory (April 30)


Expected sales in May
x Desired percentage of the next month's sales to be on hand April 30
Desired ending inventory on April 30

2. Calculate the production units budgeted for April:


Number of sales units budgeted for April
+ Desired ending inventory
Total number of units needed in April
- Beginning inventory on hand on April 1
Number of production units budgeted for April

Exhibit 9-5 shows Kerry's production budget for the first quarter of 2002. This budget
is based on the sales budget in Exhibit 9-4 and the expected sales of 40,000 units in July.
Kerry expects its total sales in May 2002 to be 25,000 units. The desired ending
inventory on April 30, therefore, is 7,500 units as shown in step 1 below. Step 2 shows
the calculation that determines the number of units to be manufactured in April.

Exhibit 9-5 Production Budget


KERRY INDUSTRIAL COMPANY
Production Budget
For the Quarter Ended June 30, 2002
April May June
Budgeted sales in units 20,000 25,000 35,000
Add: Desired ending inventory 7,500 10500 12000
of finished units
Total units needed 27,500 35,500 47,000
Less: Beginning inventory
of finished units 5000 7,500 10500
Budgeted production in units 22,500 28,000 36,500

Direct Materials Usage and Purchases


The information in the production budget becomes the basis for preparing several
other budgets for the period. The first is the direct materials usage budget (Exhibit 9-6),
which shows the amount of direct materials required for production and their budgeted cost.

The last line of the production budget (Exhibit 9-5) shows the number of units of its only product that Kerry Industrial Compan
This amount becomes line 1 of Exhibit 9-6, Kerry's direct materials usage budget.

This budget shows that production of one unit requires 3 pounds of aluminum alloy for a total of

The remainder of the direct materials usage budget (Exhibit 9-6, part B) identifies the cost
of direct materials for the budget period, but it can be completed after Kerry prepares
the second budget, its direct materials purchase budget (Exhibit 9-7). A direct materials
purchase budget shows the amount of direct materials (in units and cost) to be purchased
during the period to meet production needs.

Kerry prepares its direct materials purchase budget to identify the purchases that it must make to ensure that it has sufficient d
needs and to maintain the amount of direct materials ending inventory required by the firm's operation guidelines.

Line 3 of Exhibit 9-6 shows that the firm needs 67,500,84,000, and 109,500 pounds of aluminum alloy to meet the productio

In addition, operation guidelines require an ending direct materials inventory of


Ending inventory of aluminum alloy for April is 8400 pounds (line 2 of Exhibit 9-7, which is 10% of May's pro

The sum oflines 1 and 2 (Exhibit 9-7) identifies the total amount of direct materials needed for April,
75,900 pounds. The firm expects to have 7,000 pounds (March's ending inventory) as
the beginning balance in April; this amount must be subtracted from the amount to

Exhibit 9-6 Direct Materials Usage Budget


KERRY INDUSTRIAL COMPANY
Direct Materials Usage Budget
For the Quarter Ended June 3D, 2002

Line Item April May


A Production Requirement
1 Budgeted production 22,500 28,000
2 Pounds of aluminum alloy
for one unit of product 3 3
3 Total pounds of aluminum
alloy needed in production 67,500 84,000

B Cost of Direct Materials


4 Pounds of aluminum alloy
from beginning inventory 7,000 8,400
5 Cost per pound 2.40 2.45
6 Total cost of aluminum alloy
beginning inventory $ 16,800 $ 20,580
7 Total cost of aluminum alloy
purchases
8 Total cost of aluminum alloy
available
9 Desired ending inventory 8,400 10,950
of aluminum alloy in units
10 Cost per unit 2.45 2.5
11 Aluminum alloy ending
inventory 20,580 27,375
12 Total cost of aluminum alloy
used in production
Quarter
80,000
$ 30
$ 2,400,000

5000
o be 30 percent of the following month's predicted sales. 30%
here. 22,500

25,000
30%
7,500

20,000
7,500
27,500
5000
22,500

40000
25,000
Quarter july
80,000 40000
12,000

92,000

5000 10500
87,000 (10,500)

product that Kerry Industrial Company plans to manufacture in April: 22500 units.

um alloy for a total of 67,500 pounds (line 3) to produce the 22,500 units planned.

make to ensure that it has sufficient direct materials available to meet production
m's operation guidelines.

uminum alloy to meet the production budget for April, May, and June, respectively.

10% percent of the next period's production.


Exhibit 9-7, which is 10% of May's production need of 84,000 pounds; see line 3 of Exhibit 9-6).
7000

June Quarter

36,500 87,000

3 3 Kerry's purchasing department estimates t


pound in April, for a total cost of $168,805
109,500 261,000 Following the procedure just described, the
for May and June. June's direct materials e
based on the 36,000 units to be manufactu
36,000 = 108,000 pounds; 10% X 108,000 =
10950 7,000
2.5 2.4 Following the procedure just described, the
for May and June. June's direct materials e
$ 27,375 $ 16,800 based on the 36,000 units to be manufactu
36,000 = 108,000 pounds; 10% X 108,000 =
Ex.* 9-7

10,800 10,800

2.6 2.6

28,080 28,080
urchasing department estimates the cost of aluminum alloy to be $2.45 per 2.45
April, for a total cost of $168,805 for the 68,900 pounds it must purchase.
the procedure just described, the firm completes the purchase budgets
nd June. June's direct materials ending inventory of 10,800 pounds is determined
the 36,000 units to be manufactured in July (3 pounds per unit X
108,000 pounds; 10% X 108,000 = 10,800 pounds).

the procedure just described, the firm completes the purchase budgets
nd June. June's direct materials ending inventory of 10,800 pounds is determined 10,800
the 36,000 units to be manufactured in July (3 pounds per unit X 36,000
108,000 pounds; 10% X 108,000 = 10,800 pounds). 108,000
VARIABLE COSTING AND ABSORPTION COSTING
Data for One- Year Example
Stassen Company manufactures and markets optical consumer products. Stassen uses a standard costing system in which:

a. Direct costs are traced to products using standard prices and standard inputs allowed for actual outputs produced.
b. Indirect (overhead) costs are allocated using standard indirect rates times the standard inputs allowed for the actual outputs

The allocation base for all indirect manufacturing costs is budgeted units produced; the allocation base for all indirect marketin

Stassen wants you to prepare an income statement for 2003 (the year just ended) for the telescope product line. The operating

Units
Beginning inventory 0
Production 800
Sales 600
Ending inventory 200

Actual price and cost data for 2003 are


Selling price $ 100
Variable manufacturing cost per unit produced
Direct material cost $ 11
Direct manufacturing labor cost $ 4
Indirect manufacturing cost $ 5
Total variable manufacturing cost per unit produced $ 20
Variable marketing cost per unit sold (all indirect) $ 19
Fixed manufacturing costs (all indirect) $ 12,000 $ 15
Fixed marketing costs (all indirect) $ 10,800

We assume the following at Stassen:


1. Thecost driver for all variable manufacturing costs is units produced; the cost driver for
variable marketing costs is units sold. There are no batch-level costs and no product sustaining costs.

2. Workin process is zero.

3. The budgeted level of production for 2003 is 800 units, which is used to calculate the budgeted fIxed-manufacturing cost pe
The actual production for 2003 is 800 units.
4. Stassen budgeted sales of 600 units for 2003, which is the same as the actual sales for 2003
5. There are no price variances, no efficiency variances, no spending variances. Hence, the
budgeted (standard) price and cost data for 2003 are the same as the actual price and cost
data as given. Our fIrst example (2003) has no production-volume variance for manufacturing
costs. Later examples (for 2004 and 2005) have production-volume variances.
6. All variances are written off to cost of goods sold in the period (year) in which they
occur.
The accounting for fixed manufacturing costs is the main difference between variable costing and absorption costing.

Under variable costing, fixed manufacturing costs are treated as an expense of the period .
Under absorption costing, fIxed manufacturing costs are inventoriable costs.
In our example, the standard fixed manufacturing overhead cost is $15 per unit ($12,000 / 800 units) produced.

Inventoriable costs per unit produced in 2003 under the two methods for Stassen are

Variable Absorption
Costing Costing
Variable manufacturing cost per unit produced
Direct material $ 11 $ 11
Direct manufacturing labor $ 4 $ 4
Indirect manufacturing cost $ 5 $ 20 $ 5
Fixed indirect manufacturing cost per unit produced 0
Total inventoriable cost per unit produced $ 20

9-1 PANEL A: VARIABLE COSTING

Revenues: $100 x 600 units $ 60,000


Variable costs
Beginning inventory $ 0 $ -
Variable manufacturing costs: $20 x 800 units 16,000 $ 16,000
Cost of goods available for sale 16,000 $ 16,000
Deduct ending inventory: $20 x 200 units (4,000) $ 4,000
Variable cost of goods sold 12,000 $ 12,000
Variable marketing costs: $19 x 600 units 11,400 $ 11,400
Adjustment for variable-cost variances 0 $ -
Total variable costs $ 23,400
Contribution margin $ 36,600
Fixed costs
Fixed manufacturing costs 12,000 $ 12,000
Fixed marketing costs 10,800 $ 10,800
Adjustment for fixed-cost variances 0 0
Total fixed costs $ 22,800
Operating income $ 13,800

PANEL B: ABSORPTION COSTING

Revenues: $100 x 600 units $60,000 $ 60,000


Cost of goods sold
Beginning inventory $ 0 0
Variable manufacturing costs: $20 x 800 units 16,000 $ 16,000
Fixed manufacturing costs: $15 x 800 units 12,000 $ 12,000
Cost of goods available for sale 28,000 $ 28,000
Deduct ending inventory: ($20 + $15) x 200 units (7,000) $ 7,000
Adjustment for manufacturing variances 0 0
Cost of goods sold $ 21,000
Gross margin 39,000 $ 39,000
Operating costs
Variable marketing costs: $19 x 600 units 11,400 $ 11,400
Fixed marketing costs 10,800 $ 10,800
Adjustment for operating cost variances 0 0
Total operating costs $ 22,200
Operating income $ 16,800

underabsorption costing regards each finished unit as absorbing $15 of fixed manufacturingcost. Under absorption costing, th
($15 per unit x 800 units) is initially treatedas an inventoriable cost in 2003. Given the preceding data for Stassen, $9,000 ($15
600 units) subsequently becomes a part of cost of goods sold in 2003, and $3,000 ($]5 per unit x 200 units) remains an asset-
ending finished goods inventory on December31, 2003. Operating income is $3,000 higher under absorption costing compare
variable costing, because only $9,000 of ftxed manufacturing costs are expensed underabsorption costing, whereas all $12,00
manufacturing costs are expensed undervariable costing. The variable manufacturing cost of $20 per unit is accounted for the
in bothincome statements in Exhibit 9-l.

Thesepoints can be summarized as follows:

Variable Costing Absorption Costing


Variable manufacturing costs:
$ 20 per telescope produced Inventoriable Inventoriable
Fixedmanufacturing costs:
$ 12,000 per year Deducted as an expense Inventoriable at $15 per
of the period telescope produced using
budgeted denominator level of
800 units produced per year

Besure that other issues don't distract you from seeing clearly that the difference betweenvariable costing and
absorption costing is how fixed manufacturing costs are accounted for. If inventory levels change, operating income will
differ between the two methods because of the difference in accounting for fixed manufacturing costs. To see
this,let's compare telescope sales of 600, 700, and 800 units by Stassen in 2003, when 800 units were produced. Of the
$12,000 total fixed manufacturing costs, the amount expensedin the 2003 income statement would be

Under variable costing, whether


sales are 600, or 700, or 800 units $ 12,000 expensed

Under absorption costing, when


sales are 600 units, inventory is 200 units, $ 9,000 expensed
and $3,000 ($15 x 200) is included in inventory ($12,000 - $3,000)

sales are 700 units, inventory is 100 units, 700 $ 10,500 expensed
and $1,500 ($15 x 100) is included in inventory ($12,000 - $1,500)

sales are 800 units, inventory is 0 units, 800 $ 12,000 expensed


and $0 ($15 x 0) is included in inventory ($12,000 - $0)

EXPLAINING DIFFERENCES IN OPERATING INCOME


Data for Three-Year Example
In both 2004 and 2005, Stassen has a production-volume variance because telescope production d
800 units per year used to calculate the budgeted fIxed manufacturing cost per unit.

The actual quantities sold for 2004 and 2005 are the same as the sales quantities budgeted for these respective years, given

2003 2004 2005


Beginning inventory 0 200 50
Production 800 500 1000
Sales 600 650 750
Ending inventory 200 50 300

Comparing Income Statements


Exhibit 9-2 presents the income statement under variable costing, Panel A, and the
income statement under absorption costing, Panel B, for 2003, 2004, and 2005. Keepin
mind the following points about absorption costing as you study Panel B of both Exhibi~
• 9-1 and 9-2:

1. The $15 fIxed manufacturing cost rate is based on a budgeted denominator level of 800 units produced per year ($12,000 +
Whenever production - that's the quantity produced not the quantity sold - deviates from the denominator level, there will be
The amount of the variance here is $15 per unit multiplied by the difference between the actual level of production and the de

In 2004, production was 500 units, 300 lower than the denominator level of 800 2004
units. The result is an unfavorable production-volume variance of $4,500 ($15 per unitx -300
300 units). The year 2005 has a favorable production-volume variance of $3,000 ($15per -4500
unit x 200 units), due to production of 1,000 units exceeding the denominator levelaf
800 units.

Recall how standard costing works. Each time a unit is manufactured, $15 of fixed manufacturing costs is included in
In 2004, when 500 units are manufactured, $7,500 ($15 per unit x 500 units) of fixed costs are included in the cost o
Total fIxed manufacturing costs for 2004 are $12,000. The production-volume variance of $4,500 U equals the differe

In Panel B, note how,for each year, the fixed manufacturing costs included in the cost of goods available for sale plus

2. The production-volume variance, which relates to fIxed manufacturing overhead,exists under absorption costing but not und
Because under variable costing, fIxed manufacturing costs of $12,000 are always treated as an expense of the period, regardle

The PVV is the difference between the lump-sum budgeted FMOH and FMOH allocated

Because FMOH costs aren't allocated to output produced under VC (FMOH costs are ex

9-2 Comparison of Variable Costing and Absorption Costing


Stassen Company: Telescope Product-Line Income Statements for 2003, 2004. and 2005

PANELA: VARIABLE COSTING


2003 2004
Revenues :$100 x600;650;750 units $ 60,000 $ 65,000
Variable costs
Beginning Inventory:$20x0;200;50 units $ - $ 4,000
Variable manufacturing costs:
$20x800;500;1000 units $ 16,000 $ 10,000
Cost of goods available for sale $ 16,000 $ 14,000
Deduct ending inventory: $20x200;50;300 units $ (4,000) $ (1,000)
Variable cost of goods sold $ 12,000 $ 13,000
Variable marketing costs
$19x600;650;750 units $ 11,400 $ 12,350
Adjustment for variable cost variances 0 0
Total Variable costs $ 23,400 $ 25,350
Contribution margin $ 36,600 $ 39,650
Fixedcosts
Fixed manufacturing cost $ 12,000 $ 12,000
Fixed marketing costs $ 10,800 $ 10,800
Adjustment for fixed cost variances 0 0
Total fixed costs $ 22,800 $ 22,800
Operating income $ 13,800 $ 16,850

PANELB: ABSORPTION COSTING 800 500


2003 2004
Revenues :$100 x600;650;750 units $ 60,000 $ 65,000
cost of goods sold
Beginning Inventory:$35x0;200;50 units $ - $ 7,000
Variable manufacturing costs:
$20x800;500;1000 units $ 16,000 $ 10,000
Fixed manufacturing costs:
$15x800;500;1000 units $ 12,000 $ 7,500
Cost of goods available for sale $ 28,000 $ 24,500
Deduct ending inventory: $35x200;50;300 units $ (7,000) $ (1,750)
Adjustment for manufacturing cost variances $ - $ 4,500
cost of goods sold $ 21,000 $ 27,250
Gross margin $ 39,000 $ 37,750
Operating costs
Variable marketing costs:
$19x600;650;750 units $ 11,400 $ 12,350
Fixed marketing costs $ 10,800 $ 10,800
Adjustment for operating cost variances 0 0
Total Operating costs $ 22,200 $ 23,150
Operating income $ 16,800 $ 14,600

Here's a summary of the operating income differences for Stassen Company during the 2003 - 2005 period:

1. Absorption-costing operating income $ 16,800 $ 14,600


2. Variable-costing operating income $ 13,800 $ 16,850
3. Difference: (1)-(2) $ 3,000 $ (2,250)
4. Difference as a % of absorption operating income 17.9% -15.4%

These percentage differences illustrate why managers whose performance is measured by reported income are concerned ab
choice between variable costing and absorption costing.
Why do variable costing and absorption costing usually report different income numbers? In general, if the unit level of inven
increases during an accounting period, less operating income will be reported under variable costing than absorption costing,
Conversely, if the inventory level decreases, more operating income will be reported under variable costing than absorption c
difference in reported operating
ting system in which:

puts produced.
wed for the actual outputs produced.

e for all indirect marketing costs is budgeted units sold.

roduct line. The operating information for the year is

ed-manufacturing cost per unit. 800


800
600
orption costing.

Absorption

$ 20
$ 15
$ 35
The difference between VC and
AC operating incomes is a
matter of timing. Under VC,
der absorption costing, the $12,000 FMOH costs are expensed in
a for Stassen, $9,000 ($15 per unit x the period incurred. Under AC,
0 units) remains an asset-part of FMOH costs are allocated to
sorption costing comparedwith output produced and are not
osting, whereas all $12,000 of fixed expensed until those units are
r unit is accounted for the same way sold

oduced using
nominator level of
oduced per year

osting and
erating income will
ts. To see
e produced. Of the
be
escope production differs from the budgeted level of production of

e respective years, given in units in the following table:

uced per year ($12,000 + 800 units = $15 per unit).


minator level, there will be a production-volume variance.
of production and the denominator level.

2005
200
$ 3,000

ring costs is included in the cost of goods manufactured and available for sale.
e included in the cost of goods available for sale (see Exhibit 9-2, Panel B, line4).
500 U equals the difference between $12,000 and $7,500.

s available for sale plus the production-volume variance always equals $12,000.

rption costing but not under variable costing. Why?


se of the period, regardless of the level of production (and sales).

d FMOH allocated to output produced.

MOH costs are expensed as incurred), there is no PVV under VC.

2005
$ 75,000

$ 1,000

$ 20,000
$ 21,000
$ (6,000)
$ 15,000

$ 14,250
0
$ 29,250
$ 45,750

$ 12,000
$ 10,800
0
$ 22,800
$ 22,950

1000
2005
$ 75,000
$ 1,750
'Production-volume variance:
$ 20,000 Fixedmanufacturing costs per unit x (Denominator level - Actual output
2003:$15 x (800 - 800) units = $15 x 0 = $0
$ 15,000 2004:$15x (800 - 500) units = $15 x 300 = $4,500 U
$ 36,750 2005:$15 x (800 - 1,000)units = $15 x (200) = $3,000 F
$ (10,500)
$ (3,000) $ 12,000 $ 12,000 $ 12,000
$ 23,250
$ 51,750

$ 14,250
$ 10,800
0
$ 25,050
$ 26,700

$ 26,700
$ 22,950
$ 3,750
14.0%

income are concerned about the

, if the unit level of inventory


than absorption costing,
costing than absorption costing. The
ominator level - Actual output units produced)

0) = $3,000 F
Accounting System at Webb
Webbmanufactures and sells a designer jacket that requires tailoring and many hand operations

Webb's only costs are manufacturing costs; it incurs no costs in other value chain
functionssuch as marketing and distribution. Weassume that all units manufactured in April
2003are sold in April 2003. There are no beginning inventories or ending inventories.

Webb has three variable-cost categories.

Variable Cost
Cost Category per Jacket
Directmaterialscosts $ 60
Directmanufacturinglaborcosts $ 16
Variablemanufacturingoverheadcosts $ 12
Total variable costs $ 88
The budgeted fixed manufacturing costs are $276,000 for production
between 0 and 12,000 jackets. $ 276,000
0 12,000.00

The budgeted selling price is $120 per jacket $ 120


Actual selling price is $120 per jacket $ 125

This selling price is the same for all distributors. The static budget for April 2003 is based
on selling 12,000 jackets. Actual sales for April 2003 were 10,000 jackets. Exhibit 7-1,
column 3, presents the static budget for Webb Company for April 2003.

STATIC-BUDGET VARIANCES
Astatic-budget variance is the difference between an actual result and the corresponding
budgeted amount in the static budget. Exhibit 7-1 shows the Level 0 and Level 1 variance
analyses for April 2003. Level 0 gives the least detailed comparison of the actual and
budgeted operating income.

Afavorable variance-denoted F in this book - has the effect of increasing operating


income relative to the budgeted amount. For revenue items, F means actual revenues
exceed budgeted revenues. For cost items, F means actual costs are less than budgeted
costs. An unfavorable variance-denoted U in this book-has the effect of decreasing
operating income relative to the budgeted amount. Unfavorable variances are also called
adverse variances in some countries, for example, the United Kingdom.

Static-Budget-Based Exhibit 7-1


Variance Analysis for
Webb Company for
April2003a

LEVEL 0 ANALYSIS
Actual operating income
Budgeted operating income
Static-budget variance for operating income
LEVEL 1 ANALYSIS

Units sold
Revenues
Variable costs
Direct materials
Direct manufacturing labor
Variable manufacturing overhead
Total variable costs
Contribution margin
Fixed costs
Operating income

Although Level 1 analysis provides more information than Level 0 analysis, managers often desire still more detail

1. The budgeted selling price is the same $120 per jacket used in preparing the static budget.
2. The budgeted variable costs are the same $88 per jacket used in the static budget.
3. The budgeted fixed costs are the same static budget amount of $276,000 (because the
10,000 jackets produced falls within the relevant range of 0 to 12,000 jackets for which
fIxed costs are $276,000).

EXHIBIT 7 - 2 level 2 Flexible-Budget-Based Variance Analysis for Webb Company for April 2003a
LEVEL2 ANALYSIS
Actual
Results
(1)

Units sold 10,000


Revenues $ 1,250,000
Variable costs
Direct materials $ 621,600
Direct manufacturing labor $ 198,000
Variable manufacturing overhead $ 130,500
Total variable costs $ 950,100
Contribution margin $ 299,900
Fixed costs $ 285,000
Operating income $ 14,900
Flexible-Budget
Variances
The sales-volume variance is the difference between a flexible-budget amount and tl
corresponding static-budget amount. The flexible-budget variance is the differen,
between an actual result and the corresponding flexible-budget amount based on tl
actual output level in the budget period.
What useful information comes from subdividing the static-budget variance into i
two components? Remember, Webb actually produced and sold 10,000 jackets, althoug
the static budget had anticipated an output of 12,000 jackets. Managers would like to kno,
how much of the static-budget variance is due to inaccurate forecasting of output units sold an
how much of the static-budget variance is due to Webb's performance for 2003. Creating a flexJ
ble budget makes it possible for managers to learn these two amounts.

Sales-Volume Variances
Keep in mind the flexible-budget amounts in column 3 of Exhibit 7-2 and the static
budget amounts in column 5 are both computed using budgeted selling prices, budgetec
variable cost per jacket, and budgeted fIxed costs. The only distinction is that the flexible.
budget amount is calculated using the actual output level, whereas the static-budget
amount is calculated using the budgeted output level. The difference between these two
amounts is called the sales-volume variance because it represents the difference caused
solely by the difference in the 10,000 actual quantity (or volume) of jackets sold and the
12,000 quantity of jackets expected to be sold in the static budget.

Sales-Volume Variances
BUdgeted selling price Budgeted variable cost per unit
$ 120 $ 88 $ 32

Webb's unfavorable sales-volume variance could be due to one or more of the


following:
1. The overall demand for jackets is not growing at the rates that were anticipated.
2. Competitors are taking away market share from Webb.
3. Webb did not adapt quickly to changes in customer preferences and tastes.
4. Quality problems developed that led to customer dissatisfaction with Webb's jackets.
5. Budgeted sales targets were set without careful analysis of market conditions.

How Webb responds to the unfavorable sales-volume variance will be influenced by


what is presumed to be the cause of the variance. For example, if Webb believes the variance
was caused by market-related reasons (reasons 1 or 2), the sales manager would be
in the best position to explain what happened and to suggest corrective actions, such as
sales promotions, that may be needed. If however, the unfavorable variance was caused by
quality problems, the manufacturing manager would be in the best position to analyze
the causes and to suggest strategies for improvement, such as changes in the manufacturing
process or investments in new machines.
Exhibit 7-2, column 4, shows a sales-volume variance for each of the line items in
the income statement. Taking out the effects of inaccurate forecasting of output units
sold - the sales-volume variance - from the static-'budget variance enables managers to
compare actual revenues and costs incurred for April 2003 against revenues and costs
Webb would have budgeted for the 10,000 jackets actually produced and sold _ the
flexiblebudget. These flexible-budget variances are a better measure of operating performance
becausethey compare actual revenues to budgeted revenues and actual costs to budgeted costs
for the same 10,000 jackets of output. In contrast, the static-budget variance compares
actualrevenues and costs for 10,000 jackets against budgeted revenues and costs for
12,000 jackets.

Flexible-Budget Variances
Thefirst three columns of Exhibit 7-2 compare actual results with flexible-budget
amounts. Flexible-budget variances are in column 2 for each line item in the income
statement:

Flexible- budget = Actual _


variance results
0

The operating income line in Exhibit 7-2 shows the flexible-budget variance is $29,100 U
($14,900 - $44,000). The $29,100 U arises because actual selling price, variable cost per
unit, and fixed costs differ from their budgeted amounts. The actual and budgeted
amountsfor the selling price and variable cost per unit are

Actual Amount
Selling price 125
Variable cost per jacket 95.01

The flexible-budget variance for revenues is called the selling-price variance because
itarises solely from differences between the actual selling price and the budgeted selling
pnce:
Actual Selling price 125 Budgeted Selling price

Webbhas a favorable selling-price variance because the $125 actual selling price exceeds
the $120 budgeted amount, which increases operating income. Marketing managers are
generallyin the best position to understand and explain the reason for this selling price
difference,for example, due to better workmanship or because of an overall increase in
marketprices.

The flexible-budget variance for variable costs is unfavorable for the actual output of
10,000 jackets. It's unfavorable because either (a) Webb used more quantities of inputs
(suchas direct manufacturing labor-hours) relative to the budgeted quantities of inputs,
or(b)Webb incurred higher prices per unit for the inputs (such as the wage rate per direct
manufacturing labor-hour) relative to the budgeted prices per unit for the inputs, or
(c)both (a) and (b). Higher input quantities relative to the budget and/or higher input
pricesrelative to the budget could be the result of Webb deciding to produce a superior
productto what was planned in the budget, or the result of inefficiencies in Webb's manufacturingand
purchasing, or both. You should always think of variance analysis as providing
suggestionsfor further investigation rather than as establishing conclusive evidence of good or
badperformance

The actual fixed costs of $285,000 are $9,000 more than the budgeted amount of
$276,000. This higher fixed cost decreases operating income, making this flexible-budget
variance unfavorable.

PRICE VARIANCES AND EFFICIENCY VARIANCES FOR


DIRECT-COST INPUTS

Wenow illustrate how the Level 2 flexible-budget variance for direct-cost inputs can be
furthersubdivided into two more detailed variances, which are Level 3 variances: '
1.Apricevariance that reflects the difference between an actual input price and a budgeted
input price
2. An efficiency variance that reflects the difference between an actual input quantity and a
budgeted input quantity
The information available from these Level 3 variances helps managers better understand
past performance and better plan for future performance.

Obtaining Budgeted Input Price and Budgeted Input Quantitie


To calculate price and efficiency variances, Webb needs to obtain budgeted input prices
and budgeted input quantities. Webb's three main sources of information are
1. Actual input data from past periods. Most companies have past data on actual input
prices and actual input quantities. These past prices and quantities could be used as the budgeted
prices and quantities in a flexible budget. Past data are typically available at low cost.
Nevertheless, there are limitations to using this source of data: (i) past data can include inefficiencies,
and (ii) past data do not incorporate any expected changes for the budget period.
2. Data from other companies that have similar processes. The main limitation of
using this source is that input price and input quantity data from other companies may
not be available.
3. Standards developed by Webb. A standard is a carefully determined price, cost, or
quantity. A standard is usually expressed on a per unit basis. Consider how Webb determines
its standards. Using engineering studies, Webb conducts a detailed breakdown of
the steps required to make a jacket. Each step is assigned a standard time based on work
performed by a skilled operator using equipment operating in an efficient manner. There
are two advantages of using standard times: (i) they aim to exclude past inefficiencies, and
(ii) they aim to take into account changes expected to occur in the budget period. An
example of (ii) is the leasing of new sewing machines that operate at a faster speed and
enable output to be produced with lower defect rates. The standard manufacturing labor
cost for each jacket is computed by multiplying the standard time allowed to produce a
jacket by the standard wage rate that Webb expects to pay its operators. Similarly, Webb
determines the standard quantity of square yards of cloth required by a skilled operator to
make each jacket, the standard price per square yard of cloth, and (by multiplying them
together) the standard direct material cost of a jacket.
The term "standard" refers to many different things. Always clarify its meaning and how
it is being used. Astandard input is a carefully determined quantity of input - such as square
yards of cloth or direct manufacturing labor-hours - required for one unit of output, such as
a jacket. A standard price is a carefully determined price that a company expects to pay for a
unit of input. In the Webb example, the standard wage rate is an example of a standard price
of a direct manufacturing labor-hour. Astandard cost is a carefully determined cost of a unit
of output - for example the standard direct manufacturing labor cost of a jacket at Webb.

Standard cost per jacket for each variable direct-cost input


=.
Standard input allowed for one output unIt
x
Standard price per input unit

Standard direct material cost per jacket: 2 square yards of cloth input allowed per output unit
(jacket) manufactured, at $30 standard price per square yard
Standard direct material cost per jacket = 2 square yards x $30 per square yard = $60

Standard direct manufacturing labor cost per jacket: 0.8 manufacturing labor-hour of input
allowed per output unit manufactured, at $20 standard price per hour.
Standard direct manufacturing labor cost per jacket = 0.8 hour x $20 per hour = $16

How are the words "budget" and "standard" related? Budget is the broader term. As
the description above indicates, budgeted input prices, budgeted input quantities, and
budgeted costs need not be based on standards.
However, when standards are used to obtain budgeted input quantities and budgeted input prices, the terms "standard" an
The standard quantity of each input per unit of output and the standard price of each input determine the standard,or bud

See how the standard cost computations for direct materials and direct manufacturing labor equal the budgeted direct
materialcost per jacket of $60 and the budgeted direct manufacturing labor cost of $16 referredto earlier in this chapter (p

In its standard costing system, Webb uses standards that are attainable through efficient operations but allow for normal d
Some companies use ideal standards
or theoretical standards that assume peak operating conditions with no machine breakdownsand
no defective production. Obviously, these kinds of standards are difficult to
achieve.Aswe discussed in Chapter 6, setting difficult standards increases worker frustration
and hurts motivation and performance. The Surveys of Company Practice above
describesthe widespread use of standard costs.

Data for Calculating Webb's Price Variances and Efficiency Variances


Consider Webb's two direct-cost categories. The actual cost for each of these categories for
the 10,000 jackets manufactured and sold in April 2003 is

Direct material purchased and used


1. Square yards of cloth input purchased and used
2. Actual price incurred per square yard
3. Direct material costs (1 x 2) IExhibit 7-2, column 1]

Direct manufacturing labor


1. Direct manufacturing labor-hours
2. Actual price incurred per direct manufacturing labor-hour
3. Direct manufacturing labor costs (1 x 2) [Exhibit 7-2, column 1]

Columnar Presentation of Variance Analysis: Direct Costs for Webb Company for April2003a
EXHIBIT 7-3
LEVEl 3 ANALYSIS
Actual Costs Incurred
(Actual Input Quantity x
Actual Price)
(1)
Direct (22,200 sq. yds. x $28/sq. yd.)
Materials $ 621,600

input-price variance or rate variance

Direct
Manufacturing 9,000 hours x $22/hr.
Labor $ 198,000

Price variances for Webb's two direct-cost categories are

Direct-Cost (Actual price - Budgeted price) X


Category of input of input

Direct material $ 28 $ 30.00


Direct manufacturing $ 22 $ 20.00
labor
Always consider a broad range of possible causes for a price variance. For example,
Webb'sfavorable direct material price variance could be due to one or more of the following:
• Webb's purchasing manager negotiated the direct material price more skillfully than
was planned for in the budget.
• The purchasing manager changed to a lower-price supplier.
• Webb's purchasing manager bought in larger quantities than the quantities budgeted,
thereby obtaining quantity discounts.
• Direct material prices decreased unexpectedly due to, say, industry oversupply.
• Budgeted purchase prices for direct material were set without careful analysis of market
conditions.
• The purchasing manager received unfavorable terms on factors other than price (such
as lower quality material).
How Webb responds to a material price variance will be vitally affected by what is
presumed to be the cause of the variance. Assume Webb's managers attribute the favorable
varianceto the purchasing manager ordering in larger quantities than budgeted, thereby
receivingquantity discounts. Webb could examine if purchasing in these larger quantities
resulted in higher storage costs. If the increase in storage and inventory holding costs
exceedsthe quantity discounts, purchasing in larger quantities is not beneficial. For this
reason,some companies have reduced their materials storage to prevent their purchasing
managersfrom ordering in larger quantities.

Efficiency Variance
For any actual level of output, the efficiency variance is the difference between the input
that was actually used and the input that should have been used to produce the actual
output, holding input price constant at the budgeted price:

Efficiency Actual
vanance = quantIty of -
input

Theidea here is that a company is inefficient if it uses a larger quantity of input than the
budgeted quantity for the actual output units produced; the company is efficient if it uses
fewer inputs than budgeted for the actual output units produced.
The efficiency variances for each of Webb's direct-cost categories are

Direct-Cost (Actual quantity - Budgeted quantity) X


Category of input of input allowed for actual output

Direct material $ 22,200 $ 20,000.00


Direct manufacturing $ 9,000 $ 8,000.00
labor
Thetwo manufacturing efficiency variances - direct material efficiency variance and
directmanufacturing labor efficiency variance - are unfavorable because more input was
usedthan was budgeted, resulting in a decrease in operating income.
Aswith price variances, there is a broad range of possible causes for these efficiency
variances(see also Concepts in Action, p. 226). For example, Webb's unfavorable efficiency
variancefor direct manufacturing labor could be due to one or more of the following:
Webb's personnel manager hired underskilled workers.
• Webb's production scheduler inefficiently scheduled work, resulting in more manufacturing
labor time than budgeted being used per jacket.
• Webb's Maintenance Department did not properly maintain machines, resulting in
more manufacturing labor time than budgeted being used per jacket.
• Budgeted time standards were set too tight without careful analysis of the operating
conditions and the employees' skills.
Suppose Webb's managers determine that the unfavorable variance is due to poor
machine maintenance. Webb may then have a team consisting of plant engineers and
machine operators develop a future maint.enance schedule so that there will be fewer
breakdowns adversely affecting labor time and product quality.

Summary of Variances
Exhibit 7-4 is a summary of the LevelL 2, and 3 variances. Note how the variances in
Level 3 aggregate to the variances in Level 2 and how the variances in Level 2 aggregate to
the variances in Level 1.
The variances show why actual operating income is $14,900 when the static budget
operating income is $108,000. Recall, a favorable variance has the effect of increasing
operatingincome relative to the static budget, and an unfavorable variance has the effect
ofdecreasingoperating income relative to the static budget.

EXHIBIT .7-4 Summary of levels 1, 2, and 3 Variance Analysis

Static-budget variance
Level 1 for operating income
(93,100)
Flexible-budget variance
for operating income
(29,100) U

Individual Sales price Direct Direct manuf


line items variance materials labor
of Level 2 variance variance
flexiblebudget $ 50,000 $ 21,600 $ 38,000
variance F U U

Level 3 Direct materials Direct materials


price variance efficiency varia
$ (44,400) $ 66,000
F U

Static budget operating income


Unfavorable sales-volume variance for operating income
Flexible-budget operating income
Flexible-budget variance for operating income:
Favorable sales-price variance
Direct materials variance:
Favorable direct materials price variance $ 44,400
Unfavorable direct materials efficiency variance $ (66,000)
Unfavorable direct materials variance
Direct manufacturing labor variance:
Unfavorable direct manufacturing labor price variance $ (18,000)
Unfavorable direct manufacturing labor efficiency variance $ (20,000)
Unfavorable direct manufacturing labor variance
Variable manufacturing overhead variance
Fixed manufacturing overhead variance
Unfavorable flexible-budget variance for operating income
Actual operating income

MANAGEMENT USES OF VARIANCES


Performance Measurement Using Variances
Variance analysis is often used for performance evaluation. Two attributes of performance
are commonly evaluated:
• Effectiveness: the degree to which a predetermined objective or target is met .
• Efficiency: the relative amount of inputs used to achieve a given output level. The
fewer the inputs used for a given level of output or the greater the output for a given
level of input, the greater the efficiency.
Be careful to understand the causes of a variance before using it for performance evaluation.
Suppose a Webb purchasing manager has just negotiated a deal that results in a
favorable price variance for direct material. The deal could have achieved a favorable variance
for any or all of three reasons:
1. The purchasing manager bargained effectivelywith suppliers.
2. The purchasing manager secured a discount for buying in bulk with fewer purchase
orders. Alas, buying larger quantities than necessary for the short run resulted in excessive
inventory.
3. The purchasing manager accepted a bid from the lowest-priced supplier after only minimal
effort to check that the supplier monitored the quality of the material before
shipping it.
If the purchasing manager's performance is evaluated solely on price variances, then the
evaluation will be positive. Reason 1 would support this favorable conclusion - the purchasing
manager bargained effectively. Reasons 2 and 3 have short-run gains, buying in
bulk or making less effort to check the supplier's quality-monitoring procedures.
However, these short-run gains could be offset by higher inventory storage costs or higher
inspection costs and defect rates on Webb's production line, leading to unfavorable direct
manufacturing labor and direct materials efficiency variances.
Companies are increasingly evaluating performance based on the effect a manager's
action has had on the total costs of the company as a whole. In the purchasing manager
example, Webb may ultimately lose more money because of reasons 2 and 3 than it gains
from the favorable price variance. Do not automatically interpret a favorable variance as
good news.
A benefit of variance analysis is that it highlights individual aspects of performance.
However, if any single performance measure (for example, a labor efficiency variance or a
consumer rating report) receives excessive emphasis, managers will tend to make decisions
that make that particular performance measure look good. These actions may conflict
with the company's overall goals, inhibiting them from being achieved. This faulty
perspective on performance usually arises when top management designs a performance
evaluation and reward system that does not emphasize total company objectives.

Multiple Causes of Variances and Organization Learning


Variances often affect one another. For example, an unfavorable direct material efficiency
variance may be related to a favorable direct material price variance due to a purchasing
manager buying lower-priced, lower-quality materials. Do not interpret variances in isolation
of each other. The causes of variances in one part of the value chain can be the result of
decisions made in another part of the value chain of the company or even in another
company. Consider an unfavorable direct material efficiency variance on Webb's production
line. Possible operational causes of this variance across the value chain of the
company are
1. Poor design of products or processes,
2. Poor work on the production line,
3. Underskilled labor force, to
4. Inappropriate assignment of labor or machines to specific jobs, and
• Congestion due to scheduling a large number of rush orders from Webb's sales
representatives.

6. Webb'ssuppliers do not manufacture cloth materials of uniformly high quality.


Item6 describes an even broader perspective on the cause of the unfavorable direct material
efficiency variance by considering actions taken in the supply chain of companies. A supply
chain describesthe flow of goods, services,and information from the purchase of materials to
thedeliveryof products to consumers, regardless of whether those activities occur in the same
organizationor other organizations. Webb's supply chain consists of:

Actionsby Webb's suppliers could cause an unfavorable direct material efficiency variance
atWebb.
Thelist of six possible causes of Webb's unfavorable direct material efficiency variance
isfarfrom exhaustive. However, it does indicate that the cause of a variance in one part of
thevalue chain (production in our example) can result from actions taken in other parts
ofthe value chain (for example, product design or marketing) and in the supply chain.
Improvementsin early stages of the value chain and the supply chain can have a large
effecton reducing the magnitude of variances in subsequent stages.
The focus of variance analysis is to understand why variances arise and how to use that
understanding to learn and to improve performance. For instance, to reduce the unfavorable
directmaterial efficiency variance, a company may seek improvements in product design,
inthe quality of supplied materials, and in the commitment of the manufacturing labor
forceto do the job right the first time, among other improvements. Sometimes an unfavorabledirect
material efficiency variance may signal a need to change product strategy,
perhapsbecause the product cannot be made at a low enough cost. Variance analysis
should not be a tool to "play the blame game" (that is, seeking a person to blame for
everyunfavorable variance). Rather, it should help the company learn about what happenedand
how to perform better.
Adelicate balance needs to be struck between the two uses of variances we have discussed:
performance evaluation and organization learning. Variance analysis is helpful for
performance evaluation, but an overemphasis on performance evaluation and meeting
individualvariance targets can undermine learning and continuous improvement. Why?
Becauseachieving the standard becomes an end in and of itself. As a result, managers will
seektargets that are easy to attain rather than targets that are challenging and that require
creativityand resourcefulness. For example, if performance evaluation is overemphasized,
Webb'smanufacturing manager will prefer a standard that allows workers ample time to
manufacture a jacket; he will have little incentive to improve processes and methods to
reducemanufacturing time and cost.

Anoveremphasis on performance evaluation may also cause managers to take actions


to achievethe budget and avoid an unfavorable variance, even if such actions could hurt
thecompany in the long run. For example, the manufacturing manager may push workers
toproduce jackets within the time allowed, even if this action could lead to poorer qualityjackets
being produced. Such negative impacts are less likely to occur if variance analysisis
seen as a way of promoting organization learning. Managers will then be more willingto
experiment with ways to reduce manufacturing costs and will also be less likely to
compromise quality to avoid unfavorable variances.
many hand operations Sales are made to distributo

ufactured in April

The number of units manufactured is the cost driver for direct materials, direct manufacturing
labor, and variable manufacturing overhead.
The relevant range for the cost driveris from 0 to 12,000 jackets.

2003 is based 12000


10000

corresponding
evel 1 variance

$ 14,900
$ 108,000
$ (93,100) The unfavorable variance of $93,100 in Exhibit 7-1 for Level 0 is simply the result of
subtracting the static-budget operating income ofh $108,000 from the actual operating
income of $14,900:
Actual Static-Budget
Results Variances
(1) (2)=(1)-(3)

10,000 (2,000)
$ 1,250,000 $ (190,000)

$ 621,600 $ (98,400)
$ 198,000 $ 6,000
$ 130,500 $ (13,500)
$ 950,100 $ (105,900)
$ 299,900 24.0% $ (84,100)
$ 285,000 $ 9,000
$ 14,900 $ (93,100)

analysis, managers often desire still more detail about the causes of variances. That's when a flexible budget helps.

g the static budget.

0 (because the
ets for which

ompany for April 2003a

Flexible-Budget Sales-Volume
Variances Flexible Budget Variances
(2)=(1)-(3) (3) (4)=(3)-(5)

- 10,000 (2,000)
50,000 $ 1,200,000 (240,000)

21,600 $ 600,000 (120,000)


38,000 $ 160,000 (32,000)
10,500 $ 120,000 (24,000)
70,100 $ 880,000 (176,000)
(20,100) $ 320,000 (64,000)
9,000 $ 276,000 -
(29,100) $ 44,000 (64,000)
ible-Budget Sales-Volume
Variances
(93,100)

ckets, althoug
ould like to kno,
output units sold an
2003. Creating a flexJ

ices, budgetec
hat the flexible.

een these two


erence caused
s sold and the

Actual units sold Static budget unit sold


10,000 12,000 (2,000) $ (64,000)

ebb's jackets.

elieves the variance

tions, such as
ce was caused by
n to analyze
he manufacturing
managers to
es and costs

erating performance
osts to budgeted costs
ce compares

Flexible- budget
amount

e is $29,100 U
iable cost per

Budgeted Amount Diff


120 5
88

dgeted selling

geted Selling price 120 X Actual unit sol 10,000 50,000

g price exceeds
managers are
selling price
ll increase in

ties of inputs
ties of inputs,
rate per direct
ce a superior
n Webb's manufacturingand

evidence of good or

flexible-budget

nputs can be Managers generally have more


control over efficiency variances
and a budgeted than price variances.
That's because the quantity of
ut quantity and a inputs used is primarily affected
by factors inside the company,
tter understand whereas price changes are
primarily due to market forces
outside the company.

d input prices

actual input
be used as the budgeted

a can include inefficiencies,

mpanies may

Webb determines

based on work
manner. There
efficiencies, and

er speed and
facturing labor
to produce a
imilarly, Webb
illed operator to
tiplying them
ning and how
t - such as square
of output, such as
xpects to pay for a
of a standard price
ned cost of a unit
acket at Webb.

owed per output unit 2.00


$ 30.00
e yard = $60 $ 60.00

abor-hour of input 0.80


$ 20.00
$ 16.00

antities, and

d budgeted input prices, the terms "standard" and budget mean the same thing and are used intercqangeably.
rice of each input determine the standard,or budgeted, cost of each input per unit of output.

anufacturing labor equal the budgeted direct


or cost of $16 referredto earlier in this chapter (p. 217).

ough efficient operations but allow for normal disruptions.

achine breakdownsand

worker frustration

ese categories for

22,200 For simplicity, we assume the quantity of direct material used equals the quan
$ 28 material purchased.
$ 621,600

9,000
$ 22
$ 198,000

any for April2003a

Flexible Budget
Actual Input Quantity x (Budgeted Input Quantity Allowed
Budgeted Price for Actual Output x Budgeted Price)
(2) (3)
(22,200 sq. yds. x $30/sq. yd.) (10,000 units x 2 sq. yds./unit x $30/sq. yd.)
$ 666,000 $ 600,000

$ (44,400) F $ 66,000 U
Price variance Efficiency variance
input-price variance or rate variance usage variance.
$ 21,600 U
Flexible-budget variance
$ 21,600 U

9,000 hours x $20/hr. 10,000 units x 0.8 hrs./unit x $20/hr.


$ 180,000 $ 160,000

$ 18,000 F $ 20,000 U
Price variance Efficiency variance

$ 38,000
Flexible-budget variance
$ 38,000

geted price) X Actual quantity = Price


of input Variance

22,200 (44,400) F
9,000 18,000 U
(26,400) F
e of the following:
killfully than

ties budgeted,

alysis of market

an price (such

bute the favorable


geted, thereby
rger quantities

ficial. For this


heir purchasing

ween the input


ce the actual

Budgeted quantity Budget.ed price


of input aIIowed X of input
for actual output

nput than the


fficient if it uses

geted quantity) X Budgeted price = Efficiency Theflexiblebu for inputs is


nput allowed for actual output of input Variance based on the bugted quantit
of inputs allowed for actual
30 per sq. yard 66,000 U output level (BOlA). Tounder
20 per hour 20,000 U BQIA in the Webbexample,
86,000 U distinguish inputs (square
yards of cloth, direct manufa
labor-hours)fromoutput
(jackets).BQIA is computedb
ore input was multiplyingthe actual quanti
of output produced times ho
muchofeach inputshouldhav
orable efficiency been used per outputunit.

more manufacturing

he operating

gineers and
will be fewer

l 2 aggregate to

has the effect

Sales-volume variance
for operating income
(64,000) U

Variable manuf. Fixed manuf


overhead verhead
variance variance
10500 9000
U U

Direct manuf. Direct manuf.


labor price labor efficiency
$ 18,000 $ 20,000
U U

$ 108,000
$ (64,000)
$ 44,000 $ 44,000
$ 50,000

$ (21,600)

$ (38,000)
-10500
$ (9,000)
$ (29,100)
$ 14,900

of performance

mance evaluation.

a favorable variance

lted in excessive

after only minimal

nces, then the


on - the purchasing

costs or higher
favorable direct

sing manager
d 3 than it gains
le variance as

cy variance or a
o make decisions
may conflict

a performance

terial efficiency
o a purchasing
riances in isolation
e the result of

Webb's production

ble direct material


panies. A supply
ase of materials to
s occur in the same

ciency variance

ncy variance
e in one part of
n other parts
w to use that
ce the unfavorable
roduct design,
cturing labor
mes an unfavorabledirect

ut what happenedand

e have discussed:

and meeting
ement. Why?
, managers will
and that require
veremphasized,
ample time to
d methods to

ake actions
ons could hurt
y push workers
o poorer qualityjackets

more willingto
es are made to distributors who sell to independent clothing stores and retail chains.

ply the result of


he actual operating
Static Budget
(3)

U 12,000
U $ 1,440,000

F $ 720,000
U $ 192,000
F $ 144,000
F $ 1,056,000
F $ 384,000 26.7% The budgeted contribution margin percentage of 26.7% decreas
U $ 276,000
U $ 108,000

helps.

Sales-Volume
Static Budget Variances
(5)

12,000 (2,000)
$ 1,440,000

$ 720,000
$ 192,000
$ 144,000
$ 1,056,000
$ 384,000
$ 276,000
$ 108,000
F
erial used equals the quantity of direct
Price Variances
The formula for computing the price variance is
Price = (Actual price _ Budgeted price] x Actual quantity
variance of input of input of input

./unit x $20/hr.
eflexiblebu for inputs is
sed on the bugted quantity
inputs allowed for actual
tput level (BOlA). Tounderstand
IA in the Webbexample,
tinguish inputs (square
rds of cloth, direct manufacturing
or-hours)fromoutput
ckets).BQIA is computedby
ultiplyingthe actual quantity
output produced times how
uchofeach inputshouldhave
en used per outputunit.
percentage of 26.7% decreases to 24.0% for the actual results.
For simplicity, we assume the quantity of direct material used equals the quantity of direct
material purchased. Let's use this Webb Company data to illustrate the price variance and
the efficiency variance.
A price variance is the difference between the actual price and the budgeted price multi.
plied by the actual quantity of input, such as direct material purchased or used. A price vari.
ances is sometimes called an input-price variance or rate variance, especially when referring
to a price variance for direct labor. An efficiency variance is the difference between the actual
quantity of input used - such as square yards of cloth of direct materials - and the budgeted
quantity of input that should have been used to produce the actual output, multiplied by the
budgeted price. An efficiency variance is sometimes called a usage variance.
Exhibit 7-3 shows how the price variance and the efficiency variance subdivide the flexible.
budget variance. Consider the panel for direct materials. The direct material flexible-budget
varia~ce of$21,600 U is the difference between the actual costs incurred (actual input quantity
x actual price) shown in column 1 and the flexible budget (budgeted input quantity allowed
for actual output x budgeted price) shown in column 3. Column 2 (actual input quantity x
budgeted price) is inserted between column 1 and column 3. The difference between columns
1 and 2 is the price variance of $44,400 F because the same actual input quantity is multiplied
by the actual price in column 1 and the budgeted price in column 2. The difference between
columns 2 and 3 is the efficiency variance of $66,000 U because the same budgeted price is
multiplied by the actual input quantity in column 2 and the budgeted input quantity allowed for
actual output in column 3. See how the direct material price variance, $44,400 F, plus the direct
material efficiency variance, $66,000 U, equals the direct material flexible-budget variance,
$21,600 U. We next discuss the price variances and the efficiency variances in greater detail.
It is not unusual for more than This chapter shows how flexible budgets and variance analysis can help
50% of a company's total product plan and control the overhead costs of their companies.
costs throughout the value
chain to be classified as indirect.
Increased automation, In this chapter, we focus on the indirect-cost categories of variable man
more complexity of production fixed manufacturing overhead. And we explain why managers should b
and distribution processes, and interpreting variances based on overhead cost concepts developed prim
product proliferation usually reporting purposes.
increase the proportion of total
product costs that are indirect.
However, the lower cost of
information processing works
against this trend by facilitating
more direct-cost tracing.

How do managers plan variable overhead costs and fixed overhead costs?

Planning of both variable overhead costs and fixed overhead costs involves
undertaking only activities that add value and then being efficient in that undertaking.
The key difference is that for variable-cost planning, ongoing decisions during the
budget period play a larger role; whereas for fixed-cost planning, most key decisions
must be made before the start of the period.

Planning Variable Overhead Costs

In our example, Webb Company should examine how each of the activities in its variable
overhead cost pools is related to delivering a product or service to customers. For
example, customers know sewing to be an essential activity at Webb. Hence, maintenance
activities for sewing machines - included in Webb's variable overhead costs - are also
essential activities. Such maintenance should be done in a cost-effective way. This means,
for example, scheduling equipment maintenance in a systematic way rather than waiting
for sewing machines to break down.

Planning Fixed Overhead Costs

Effective planning of fixed overhead costs is much the same as effective planning for variable
overhead costs - planning to undertake only essential activities and then planning
to be efficient in that undertaking. But with planning fixed overhead costs, there is one
more consideration: choosing the appropriate level of capacity or investment that will
benefithe company over a long time. This third item is a key strategic decision. Consider
Webb's leasing of sewing machines, each having a fIxed cost per year. Leasing insuffIcient
machine capacity - say, because Webb underestimates demand - will result in an inability to
meet demand and lost sales of jackets. Leasing more machines than necessary - if
Webb overestimates demand - will result in additional fIxed leasing costs on machines
not fullyutilized during the year.

At the start of a budget period, management will have made most of the decisions
that determine the level of fIxed overhead costs to be incurred. But, it's the day-to-day,
ongoing operating decisions that mainly determine the level of variable overhead costs
incurred in that period.

STANDARD COSTING AT WEBB COMPANY


Why do companies use standard
costing?

A standard-costing system traces direct costs to a cost object by multiplying the


standard prices or rates times the standard inputs allowed for actual output
produced and allocates indirect costs on the basis of the standard indirect rates
times the standard quantities of the allocation bases allowed for the actual output
produced. The standard costs of products are known at the start of the period. To
manage costs, managers compare actual costs to standard costs.

Webb uses standard costing. The development of standards for Webb's direct-cost categories was described in Chapter 7.

This chapter discusses Webb's indirect-cost categories. Standard costing is a costing method that

(a) traces direct costs to output produced by multiplying the standard prices or rates by the standard quantities of inputs

and (b) allocates indirect costs on the basis of the standard indirect rates times the standard quantities of the allocation

With a standard-costing system, the standard costs of every product or service


plannedto be worked on during the period can be computed at the start of that period.
Thisfeatureof standard costing makes it possible to use a simple recording system. For
calculatingthe cost of products or services, no record need be kept of the actual costs of
itemsused or of the actual quantities of the cost-allocation bases used on individual
productsor services worked on during the period. Once standards have been set, the costs
ofoperating a standard-costing system can be low relative to the costs of operating an
actualor a normal-costing system.

DEVELOPING BUDGETED VARIABLE OVERHEAD COST·ALLOCATION RATES

Variable overhead cost-allocation rates can be developed in four steps.

Step 1: Choose the Period to Be Used for the Budget. Webb uses a 12-month budget
period that includes a full calendar-year cycle that includes different seasons.

Step 2: Select the Cost-Allocation Bases to Use in Allocating Variable Overhead


Costs to Output Produced. Webb's operating managers believe that machine hours
is the cost driver of variable manufacturing overhead. Using the
cause-and-effect criterion, Webb selects standard machine-hours as the cost allocation
base. Webb budgets 57,600 machine-hours for a budgeted output
of 144,000 jackets in 2003.

Step 3: Identify the Variable Overhead Costs Associated with Each Cost-Allocation
Base.Webb groups in a single pool all its variable manufacturing overhead costs,
including costs of energy, machine maintenance, engineering support, indirect
materials, and indirect manufacturing labor. Webb's budgeted variable manufacturing
costs for 2003 are $1,728,000.

Step 4: Compute the Rate per Unit of Each Cost-Allocation Base Used to Allocate
Variable Overhead Costs to Output Produced. Dividing the amount in step 3
($1,728,000) by the amount in step 2 (57,600 machine-hours), Webb estimates
a rate of $30 per standard machine-hour for its variable manufacturing
overhead costs.
In standard costing, the variable overhead rate per unit of the cost-allocation base
(machine-hours for Webb) is generally expressed as a standard rate per output unit. This
standardrate depends on the number of units of the cost-allocation base (that's the input
units) allowed per output unit. On the basis of an engineering study, Webb estimates it
will take 0.40 machine-hours per actual output unit.

Budgeted variable Budgeted inputs Budgeted variable


overhead cost rate = allowed per x Overhead cost rate
per output unit output unit per input unit

0.40 hours per jacket x $30 per hour =

A VARIABLE OVERHEAD COST VARIANCES

We now illustrate how the budgeted variable manufacturing overhead rate is used in computing
Webb's variable manufacturing overhead cost variances. The following data are for
April 2003, when Webb produced and sold 10,000 jackets:

Actual
Cost Item/Allocation Base Result
1. Output units (jackets) 10,000
2. Machine-hours 4,500
3. Machine-hours per output unit (2 -;-1) 0.45
4. Variable manufacturing overhead costs $ 130,500
5. Variable manufacturing overhead costs per
machine-hour (4 -;-2) $ 29
6. Variable manufacturing overhead costs per
output unit (4 -;-1) $ 13.05

The flexible-budget enables Webb to highlight the effect of differences between actual
costs and actual quantities versus budgeted costs and budgeted quantities for the actual
output level of 10,000 jackets.

Flexible-Budget Analysis
As you saw in Chapter 7, the variable overhead flexible-budget variance measures the
difference between actual variable overhead costs and flexible-budget variable overhead
costs. As Exhibit 8-1 shows:

Variable overhead Actual costs -


flexible- budget variance = incurred

= $ 130,500

= $ 10,500

Columnar Presentation of
Variable Manufacturing
Overhead Variance
Analysis: Webb Company Actual Costs Actual Input
for April 2003 a Incurred x Budgeted Rate
(1) (2)
(4,500 hrs. x $29/hr.) (4,500 hrs. x $30/hr.)

level 3 $ 130,500 $ 135,000

$ (4,500) F
Spending variance

level 2 $ 10,500
Flexible-budget variance
$ 10,500

This$10,500 unfavorable flexible-budget variance means Webb's actual variable manufacturingoverhead


exceeded its flexible-budget amount by $10,500 for the 10,000 jackets
actuallyproduced and sold.
Justas we did in Chapter 7 with the flexible-budget variance for direct-cost items, we
willnow obtain additional information by subdividing the Level 2 variable manufacturingoverhead
flexible-budget variance into its Level 3 efficiency variance and spending
variance.

VariableOverhead Efficiency Variance


Thevariable overhead efficiency variance is the difference between the actual quantity of
the cost-allocation base used and the budgeted quantity of the cost-allocation base that
shouldhave been used to produce the actual output, multiplied by budgeted variable
overhead cost per unit of cost-allocation base.

Variable Actual quantity of Budgeted quantity of


overhead variable overhead variable overhead
efficiency = cost-allocation base - cost-allocation base
variance used for actual allowed for
output actual output

4,500 hours 0.40 hrs. / unit x 10,000 units)


4,500 4,000

$ 15,000 U

Columns2 and 3 of Exhibit 8-1 show the variable overhead efficiency variance. The variableoverhead
efficiency variance is computed in the same way as the efficiency variance
fordirect-cost items (Chapter 7, pp. 225-226). But the interpretation of the direct-cost
efficiencyvariances differs from the interpretation of the variable overhead efficiency variance.
In Chapter 7, efficiency variances for direct-cost items are based on differences
betweenactual inputs used and the budgeted inputs allowed for actual output produced.
Forexample, an efficiency variance for direct manufacturing labor for Webb will indicate
whethermore or less direct manufacturing labor is used per jacket than was budgeted for
theactual output produced. In contrast, here in Chapter 8 the efficiency variance for variableoverhead
cost is based on the efficiency with which the cost-allocation base is used.
Webb's unfavorable variable overhead efficiency variance of $15,000 means that
actualmachine-hours (the cost-allocation base) turned out to be higher than the budgetedmachine-
hours allowed to manufacture 10,000 jackets. Possible causes for Webb's
actualmachine-hours used exceeding budgeted machine-hours include:

(i) Workerswere less skillful than expected in using the machines.


(ii) Theproduction scheduler inefficiently scheduled jobs, resulting in more machine-hours
used than budgeted.
(iii) Machines were not maintained in good operating condition.
(iv) Webb promised a distributor a rush delivery, which resulted in more machine-hours
used than budgeted.
(v) Budgeted machine time standards were set too tight.

Management'sresponse to this $15,000 U variance would be guided by which causes best


describethe April 2003 results.
Cause (i) has implications for employee-hiring practices and training procedures.
• Causes (ii) and (iii) relate to plant operations and include the possible use of software
packages for production scheduling and plant maintenance.
• Cause (iv) has implications for coordinating production schedules with distributors
and sharing information with them.
• Cause (v) requires managers to commit more resources to developing reliable standards.
Variable Overhead Spending Variance
Thevariable overhead spending variance is the difference between the actual variable
overheadcost per unit of the cost-allocation base and the budgeted variable overhead cost
per unit of the cost-allocation base, multiplied by the actual quantity of variable overhead
cost-allocation base used for actual output.

Variable Actual variable Budgeted variable


overhead overhead cost per - overhead cost per
spending unit of cost-allocation base unit of cost-allocation base
variance
= ($29 per machine-hour - $30 per machine-hour)
$ 29 $ 30
= (-$1 per machine-hour)

= -$4,500, or $4,500 F (4,500) F

Webb operated in April 2003 with a lower-than-budgeted variable overhead cost per
machine-hour. Hence, there is a favorable variable overhead spending variance. Columns
1 and 2 in Exhibit 8-1 depict this variance.
To understand the variable overhead spending variance, you need to recognize why
the actual variable overhead cost per unit of the cost-allocation base is lower than the
budgeted variable overhead cost per unit of the cost-allocation base. Here's why: Relative to
the flexible budget, the percentage increase in the actual quantity of the cost-allocation
base is more than the percentage increase in actual total costs of individual items in the
indirect-cost pool. In the Webb example, the 4,500 actual machine-hours are 12.5%
greater than the flexible-budget amount of 4,000 machine hours [(4,500 - 4,000) 74,000
= 0.125, or 12.5%]. Actual variable overhead costs of $130,500 are only 8.75% greater
than the flexible-budget amount of $120,000 [($130,500 - $120,000) + $120,000 =
0.0875. or 8.75%]. Because the percentage increase in actual variable overhead costs is
less than the percentage increase in machine-hours, the actual variable overhead cost per
machine-hour is lower than the budgeted amount.

Variable manufacturing overhead costs include costs of energy, machine maintenance,


indirect materials, and indirect manufacturing labor. Two reasons why the percentage
increase in actual variable manufacturing overhead costs is less than the percentage
increase in machine-hours in the Webb example are
1. The actual prices of individual inputs included in variable overhead, such as the price
of energy, indirect materials, or indirect manufacturing labor, are lower than the budgeted
prices of these inputs. For example, the actual price of electricity may only be
$0.09 per kilowatt-hour, compared with a price of $0.10 per kilowatt-hour in the
flexible budget.
2. Relative to the flexible budget, the percentage increase in the actual quantity usage of
individual items in the variable overhead-cost pool is less than the percentage increase in
machine-hours. Suppose actual energy used is 32,400 kilowatt-hours compared with the
flexible-budget amount of30,000 kilowatt-hours. The 8% [(32,400 - 30,000) 730,000]
increase in energy usage compared with the 12.5% [(4,500 - 4,000) 74,000] increase in
machine-hours will lead to a favorable variable overhead spending variance. The spending
variance can be partially or completely traced to the efficient use of energy and other
variable overhead items.

Price effects have implications for Webb's purchasing decisions. Quantity effects have
implications for Webb's production decisions. Distinguishing these two effects for a variable
overhead spending variance requires detailed information about the budgeted prices
and budgeted quantities of the individual items in the variable overhead cost pool.
To clarify the concepts of variable overhead efficiency variance and variable overhead
spending variance, consider the following example, assuming that (a) energy is the only
item of variable overhead and machine-hours is the cost-allocation base, (b) the actual
machine-hours used to produce the actual output equals the budgeted machine-hours,
and (c) the actual price of energy equals the budgeted price. Under those assumptions,
there would be no efficiency variance, but there could be a spending variance. The company
has been efficient with respect to the number of machine-hours used to produce the
actual output. But it could be using too much energy - not because of excessive machinehours
but because of wastage (more energy per ma hine-hour). The cost of this higher
energy usage would be measured by the spending variance.
The variable manufacturing overhead variances computed

The variable manufacturing overhead variances computed in this section can be summarized
as follows:

Flexible-budget variance
Spending variance
(4,500) F

The cause for Webb's unfavorable flexible-budget variance was using a higherthan-
budgeted number of machine-hours. Webb found out later that the machines in
April2003 operated below budgeted efficiency levels due to insufficient maintenance
performedin February and March. Aformer plant manager delayed maintenance in a presumedattempt
to meet monthly budget cost targets. Webb has since strengthened its
internal maintenance procedures so that failure to do monthly maintenance as
completely as needed raises a "red flag" that must be immediately explained to top
management.
Fromvariable overhead costs we now turn our attention to fIXedoverhead costs.

DEVELOPING BUDGETED FIXED OVERHEAD COST-ALLOCATION RATES

Fixed overhead costs are, by definition, a lump sum of costs that remains unchanged in
total for a given period despite wide changes in the level of total activity or volume related
tothose overhead costs. Total fIXed costs are usually included in flexible budgets, but they
remain the same total amount within the relevant range of activity regardless of the output
level chosen to "flex" the variable costs and revenues. The steps in developing the
budgetedfIXedoverhead rate are

Step 1: Choose the Period to Use for the Budget. Aswith variable overhead costs, the
budget period for fIXedcosts is typically 12 months. Chapter 4 (pp. 105-106)
provides several reasons for using annual overhead rates rather than, say,
monthly rates: numerator reasons-such as reducing the influence of seasonality
and denominator reasons - such as reducing the effect of varying output
and number of days in a month. In addition, setting annual overhead rates
once a year saves management time from being tied up 12 times during the
year if budget rates had to be set monthly.

Step 2: Select the Cost-Allocation Base to Use in Allocating Fixed Overhead Costs to
Output Produced. Webb uses standard machine-hours as the cost-allocation
base for fixed manufacturing overhead costs. This is the denominator of the budgeted
fIXed overhead rate computation and is called the denominator level. In
manufacturing settings, the denominator level is called more specifically, the
production-denominator level. Standard machine-hours is also the allocation
base Webb uses for its variable manufacturing overhead costs. For simplicity, we
assume Webb expects to operate at capacity in 2003 - budgeted machine-hours
of 57,600 hours for a budgeted output of 144,000 jackets.1

Step 3: Identify the Fixed Overhead Costs Associated with Each Cost-Allocation Base.
Webb groups all its fIXedmanufacturing overhead costs in a single cost pool.
Costs in this pool include depreciation on plant and equipment, plant and
equipment leasing costs, the plant manager's salary, and some administrative
costs. Webb's fixed manufacturing budget for 2003 is $3,312,000.

Step 4: Compute the Rate per Unit of Each Cost-Allocation Base Used to Allocate
Fixed Overhead Costs to Output Produced. Dividing the $3,312,000 from step
3 by the 57,600 machine-hours from step 2, Webb estimates a fixed manufacturing
overhead cost rate of $57.50 per machine-hour:

Budgeted fixed Budgeted total costs


overhead cost per infixed overhead cost pool
unit of cost· allocation Budgeted total quantity of
base cost -allocation base

$3,312,000
57,600

$57.50 per machine-hour

In standard costing, the $57.50 fixed overhead cost per machine-hour is usually expressed
as a standard cost per output unit:

Budgeted fixed Budgeted quantity of


overhead cost per cost-allocation
output unit = base allowed per X
output unit

0.40 machine-hours per jacket


0.40
$23.00 per jacket
$ 23.00

When preparing monthly budgets for 2003, Webb divides the $3,312,000 annual total
fixed costs into 12 equal monthly amounts of $276,000.

FIXED OVERHEAD COST VARIANCES

The flexible-budget amount for a fixed-cost item is also the amount included in the static
budget prepared at the start of the period. No adjustment is required for differences
between the actual output and the budgeted output for fixed costs. By definition, fixed
costs are unaffected by changes in the output level within the relevant range. At the start of
2003, Webb budgeted fixed manufacturing overhead costs to be $276,000 per month. The
actual amount for April 2003 turns out to be $285,000. As we saw in Chapter 7, the fixed
overhead flexible-budget variance is the difference between actual fixed overhead costs
and the fixed overhead costs in the flexible budget:

Fixed overhead Actual costs


flexible-budget variance incurred

= $ 285,000 -

= $ 9,000 U

As Exhibit 8-2 shows, the variance is unfavorable because $285,000 actual fIxed manufacturing
overhead costs exceed the $276,000 budgeted for April 2003, which decreases that
month's operating income compared to the budget by $9,000.
The variable overhead flexible-budget variance described earlier in this chapter was
subdivided into a spending variance and an efficiency variance. There is not an efficiency
variance for fixed costs. That's because a given lump sum of fIxed costs will be unaffected
by how effIciently machine-hours are used to produce output in a given budget period. As
Exhibit 8-2 shows, the fIxed overhead spending variance, a Level 3 variance, is the same
amount as the Level 2 fixed overhead flexible-budget variance:

Fixed overhead Actual costs


spending variance incurred

= $ 285,000
= $ 9,000 U

Webb investigated this variance and found that there was a $9,000 per month unexpected
increase in its equipment leasing costs. However, management concluded that the new
lease rates were competitive with lease rates available elsewhere

PRODUCTION-VOLUME VARIANCE

Computation of Production-Volume Variance


Webb's budgeted fixed manufacturing overhead costs are allocated to actual output produced
during the period at the budgeted rate of $57.50 per standard machine-hour. We

Flexible Budget:
Same Budgeted
lump Sum
(as in Static Budget)
Actual Costs Regardless of
Incurred Output level
(1) (2)

$ 285,000 $ 276,000

Level 3 $ 9,000 U
Spending variance

Level 2 $ 9,000 U
Flexible-budget variance

nowconsider a variance that arises when the actual level of the cost-allocation base for
allocatingfIxed overhead costs differs from the budgeted level of the cost-allocation base
chosen at the start of the period. This budgeted level for Webb in April 2003 was
4,800 hours (0.40 machine-hours per output unit X 12,000 budgeted output units).
The production-volume variance is the difference between budgeted fIxed overhead
and fIxed overhead allocated on the basis of actual output produced. The productionvolume
variance is also referred to as denominator-level variance, as well as outputleveloverhead
variance.
The formula for calculating the production-volume variance, expressed in terms of
allocation base units (machine-hours for Webb), is

Production volume Budgeted Fixed overhead allocated using


variance = fIxed - budgeted input allowed for
actual output units produced
$ 276,000 $ 230,000
= 276,000 - (0.40 hours per unit x 10,000 x $57.50 per hour)
= $276,000 - $230,000
= $46,000, or $46,000 U $ 46,000

The formula can also be expressed in terms of the budgeted fIxed cost per output unit:

Production volume Budgeted Fixed overhead allocated using


variance = fIxed budgeted cost per output unit
allowed for actual output produced
$ 276,000 - $ 230,000
$276,000 - ($23 per jacket x 10,000 jackets)

$ 46,000 U

As shown in Exhibit 8-2, the amount used for budgeted fIxed overhead will be the
samelump sum shown in the static budget and also in any flexible budget within the relevantrange.
Fixed overhead allocated is the amount of the fIxed overhead costs allocated
to each unit of output multiplied by the number of output units produced during the
budgetperiod

Interpreting the Production-Volume Variance

The production-volume variance arises whenever the actual level of the denominator used
for allocating fIxed overhead costs differs from the level used to calculate the budgeted
ftxed overhead cost rate. We compute this budgeted fIxed overhead rate because inventory
costing and some types of contracts require fIxed overhead costs to be expressed on a unit of-
output basis. The production-volume variance results from "unitizing" fIxed costs. In
our Webb example, each jacket produced is assumed to use $23 of fixed costs.
An unfavorable production-volume variance means we have underallocated the fixed overhead costs to actual output pr
A favorable production-volume variance indicates overallocated fIxed overhead costs to actual output produced.

Lump-sum fixed costs represent costs of acquiring capacity, such as plant and equipment
leases, that cannot be decreased if the resources needed turn out to be less than the
resources acquired. Sometimes, costs are fixed for contractual reasons such as a lease contract;
at other times, costs are fixed because of lumpiness in acquiring and disposing of
capacity (as we discussed in Chapter 2).

Webb leased equipment capacity to produce 12,000 jackets per month. Although it
produced only 10,000 jackets, the lease contract prevented Webb from reducing equipment
lease costs during April 2003.
Unitizing and allocating fixed costs at $23 per jacket helps Webb to measure the amount of fixed-cost resources it used to
$230,000 ($23 per jacket x 10,000 jackets).
The unfavorable production-volume variance of $46,000 (budgeted fixed overhead costs of $276,000 minus $230,000 bud
measures the amount of extra fixed costs that Webb incurred for manufacturing capacity it planned to use but did not us

Webb's management would want to analyze why this over capacity occurred.
Is demand weak? Should Webb reevaluate its product and marketing strategies? ;Is there a quality
problem? Or did Webb make a strategic mistake and acquire too much capacity?

Becareful, when making inferences about a company's decision.Sflbout capacity plan·


ning and how that capacity was used from the sign (that is, favoralbe, F, or unfavorable,
U) or the magnitude associated with a production-volume variance.
To interpret the $46,000 unfavorable variance, Webb should consider why it only sold 10,000 jackets in April.
Suppose a new competitor had gained market share by pricing below Webb's selling
price. To sell the budgeted 12,000 jackets, Webb may have had to reduce its own selling
price on all 12,000 jackets. Suppose it decided that selling 10,000 jackets at a higher price
yielded higher operating income than selling 12,000 jackets at a lower price.
The production-volume variance does not take into account such information. That's why Webb
should not interpret the $46,000 U amount as the total economic cost of selling 2,000
jackets less than the 12,000-jacket denominator level.

Companies plan their plant capacity strategically on the basis of expectations of


how much capacity will be needed over some future time horizon. For 2003, Webb's
budgeted quantity of output is equal to the maximum capacity of the plant for that budget
period. Actual demand (and quantity produced) turned out to be below the budgeted
quantity of output. Webb reports an unfavorable production-volume variance for April 2003.
However, it would be incorrect to infer this means that Webb's management made a bad planning decision regarding the p
Demand for Webb's jackets might be highly uncertain.
Given this uncertainty and the cost of not having suffIcient
capacity to meet sudden demand surges (for example, lost contribution margins and
reduced follow-on business), Webb's management may have made a wise choice in
planning 2003 plant capacity.

Always explore the why of a variance before concluding that the label unfavorable or
favorable necessarily indicates, respectively, poor or good management performance.
Understanding the reasons for a variance also helps managers decide on future courses of
action (see Concepts in Action, p. 261). Should they try to reduce capacity, increase sales,
or do nothing? Chapter 9 and Chapter 13 examine these issues in more detail.

INTEGRATED ANALYSIS OF OVERHEAD COST VARIANCES

As our discussion indicates, the variance calculations for variable manufacturing overhead and fIxed manufacturing overhe
Variable manufacturing overhead has no production-volume variance .
Fixed manufacturing overhead has no effIciency variance.

Exhibit 8-3 presents an integrated summary of the variable overhead variances and
the fixed overhead variances computed using standard costs at the end of April 2003

Exhibit8-3 indicates the columns for which no variances are calculated. Panel A shows
thevariances for variable manufacturing overhead; Panel B shows the variances for fIxed
manufacturingoverhead. As you study Exhibit 8-3, note how the columns in Panels A and
B arealigned to measure the different variances. In both Panels A and B,
1 .The difference between columns 1 and 2 measures the spending variance.
2 .The difference between columns 2 and 3 measures the efficiencyvariance (when applicable).
3.The difference between columns 3 and 4 measures the production-volume variance (when applicable).

Panel A has an effIciency variance; Panel B has no efficiency variance. A lump-sum


amount of fIxed costs will be unaffected by the degree of operating efficiency in a given
budgetperiod.
Panel A does not have a production-volume variance. That's because the amount of
variableoverhead allocated is always the same as the flexible-budget amount. Variable,
costsnever have any unused capacity. When production and sales of jackets decline from
12,000 jackets to 10,000 jackets, budgeted variable overhead costs proportionately
decline.Fixed costs are different. Panel Bhas a production-volume variance because Webb
had to pay for the fIxed manufacturing overhead resources it had committed to when it
planned production of 12,000 jackets, even though it produced only 10,000 jackets and
did not need nor use some of its capacity.

4-, 3-, 2-, and 1-Variance Analysis


When all four overhead variances in Exhibit 8-3 are presented together, it is called a 4-variance analysis

4-Variance Analysis Spending Efficiency


Variance Variance
Variable Manufacturing Overhead (4,500) F $ 15,000
Fixed Manufacturing Overhead $ 9,000 U Never a variance

Note"Never a variance," for production-volume variance in the case of variable manufacturingoverhead.


and "Never a variance," for efficiency variance for fIxed manufacturing overhead.
We can modify this 4-variance analysis by combining the two spending variances. A
3-varianceanalysis is

3-Variance Analysis Spending Efficiency


Variance Variance
Total Manufacturing Overhead 4,500 U $ 15,000

The3-variance analysis simplifIes the accounting for variances relative to 4-variance


analysis,but some information is lost. Because 3-variance analysis combines variable and
fixedoverhead spending variances when reporting overhead cost variances, it is sometimescalled
combined variance analysis. A 2-variance analysis combines the spending and
efficiencyvariances from the 3-variance analysis:

2-Variance Analysis Flexible-Budget


Variance
$ 19,500 U
A I-variance analysis combines the flexible-budget variance and the production-volume
variancefrom 2-variance analysis:

l-Variance Analysis
Total Manufacturing Overhead
Thesingle variance of $65,500 U in the I-variance analysis is called total-overhead variance.
Using fIgures from Exhibit 8-3, the $65,500 U total-overhead variance is the differencebetween
(a) the total actual manufacturing overhead incurred ($130,500 + $285,000
= $415,500) and (b) the manufacturing overhead allocated ($120,000 + $230,000 =
$350.000) to the actual output produced.
As you have seen in the case of other variances, the variances in Webb's 4-variance
analysisare not necessarily independent of each other. For example, Webb may purchase
lower-quality machine fluids (leading to a favorable variable overhead spending variance),
which results in the machines taking longer to operate than budgeted (causing an
unfavorable variable overhead efficiency variance).

DIFFERENT PURPOSES OF MANUFACTURING OVERHEAD COST ANALYSIS


Different types of cost analysis are used for different purposes (see Surveys of Company Practice,p. 265).
We consider two purposes for variable manufacturing overhead and fIXedmanufacturing overhead:
(1) planning and control and (2) inventory costing for financial reporting.

Variable Manufacturing Overhead Costs


In Exhibit 8-4. Panel A, Webb's variable manufacturing overhead is shown as variable with respect to output units (jacke
and also for the inventory costing purpose.

The greater the number of output units manufactured,


the higher the budgeted variable manufacturing overhead costs and
the higher the variable manufacturing overhead costs allocated to output units.

Panel A presents an overall picture of how total variable overhead might behave. Of
course, variable overhead consists of many items, including energy costs, repairs, indirect
labor, and so on. Managers help control variable overhead costs by budgeting each line
item and then investigating possible causes for any significant variances

Fixed Manufacturing Overhead Costs


Exhibit 8-4, Panel B, shows that for the planning and control purpose, fixed overhead costs do not change in the 0- to 12,OO

Consider the monthly leasing costs for building and equipment included in Webb's budgeted fixed manufacturing overhead
Managers control this fixed leasing cost at the time the lease is signed.
For any month during the leasing period, management can do little-most likely can do nothing - to change the lease payme
Contrast this description of fixed overhead with how these costs are depicted for the inventory costing purpose in Panel B.

Under generally accepted accounting principles,


fixed manufacturing costs are allocated as an inventoriable cost based on the level of output units produced.
Every output unit that Webb manufactures will increase the fixed overhead allocated to products by $23.
Because Webb produces 10,000 jackets, only $230,000 ($23 per jacket x 10,000 jackets) will be allocated to products.
As the graph in Panel B shows, the difference between the fixed manufacturing overhead costs budgeted of $276,000 and
is the $46,000 U production-volume variance, which will be written off to the Cost
of Goods Sold account or prorated among Work in Process, Finished Goods and Cost of Goods Sold accounts.
When allocating FMOH for the
inventory costing purpose, we Managers should not use the unitization of fixed manufacturing ov
unitize FMOH (we treat it as if planning and control. However, computing the fixed manufacturin
it were a variable cost). In contrast, of $230,000 allows managers to identify the production-volume va
budgeted FMOH is fixed $46,000 U, representing unused fixed manufacturing overhead ca
over a wide range of output
levels. The production-volume
variance is zero only if FMOH
allocated equals budgeted
FMOH. If so, the actual output
level (expressed in terms of the
FMOH cost-allocation base) is
equal to the denominator level
used to compute the budgeted
FMOH cost rate.
gets and variance analysis can help managers
of their companies.

ect-cost categories of variable manufacturing overhead and


we explain why managers should be careful when
head cost concepts developed primarily for financial

The two ways of managing


variable overhead costs are
(1) eliminate nonvalue-added
costs (for example, consume
‫خياطة‬ less electricity by using more
energy-efficient equipment)
and (2) reduce consumption of
the cost-allocation bases (for
example, redesign products to
require fewer machine-hours of
processing time).

The two ways of managing


fixed overhead costs are
(1) eliminate nonvalue-added
costs (for example, arrange to
have vendors deliver direct
materials to the production floor
just when needed, which
enables a warehouse lease to
be terminated) and (2) plan for
appropriate capacity levels.

tegories was described in Chapter 7.

method that

s by the standard quantities of inputs allowed for actual outputs produced

standard quantities of the allocation bases allowed for the actual outputs produced.
57,600
144,000
The budgeted variable manufacturing
overhead (BVMOH)
cost rate differs from the budgeted
price of direct materials
(DM) or direct manufacturing
$ 1,728,000 labor (DML). The BVMOH cost
rate encompasses costs of
many diverse VMOH items per
unit of the cost-allocation base.
$ 30 per standard machine-hour In the Webb example, the
BVMOH cost rate is $30 per
machine-hour: That is, Webb
expects to spend about $30 on
a "market basket" of VMOH
items. The market basket aspect
of the BVMOH cost rate makes
0.40 VMOH variances more difficult
to interpret than OM and OML
variances.
Budgeted variable
Overhead cost rate
per input unit

$30 per hour = $ 12 per jacket (output unit)


Webb uses $12 per jacket in both its static budget for 2003 and in the monthly performan
reports it prepares during 2003.

computing

Flexible-Budget Be sure to distinguish budgeted


Amount VMOH cost per unit of output
10,000 ($12 per jacket) from budgeted
4,000 VMOH cost per unit of the input
0.40 cost-allocation base ($30 per
$ 120,000 machine-hour). Either cost rate
can be used to compute VMOH
$ 30 allocated: 10,000x (0.40x $30)=
$120,000, or 10,000 x $12 =
$ 12.00 ###

Actual costs - Flexible-budget


amount

- $ 120,000

Flexible Budget:
Budgeted Input
Allowed for
Actual Input Actual Output
x Budgeted Rate x Budgeted Rate
(3)
0 hrs. x $30/hr.) (0.40 hrs/unit x 10,000 units x $30/hr.)
4,000 hrs. x $30/hr.
$ 120,000

$ 15,000 U
Efficiency variance

U
Flexible-budget variance
U

ufacturingoverhead

ingoverhead

In this chapter, we use the general


term cost-allocation base,
instead of cost driver. A costallocation
base is a cost driver
only if the relationship between
the cost-allocation base and
the cost pool is strong. In the
case of overhead costs, the
Budgeted variable cause-and-effect relationship
x overhead cost per unit between the cost-allocation
of cost - allocation base base (such as machine-hoursl
and some overhead costs (such
as plant rent, engineering support,
x 10,000 units) $30 per hour and production schedulingl
$ 30 may not be strong.

ariableoverhead

variableoverhead

getedmachine-

machine-hours

The unfavorable VMOH efficiency


achine-hours variance doesn't mean
workers wasted VMOH items
and used more VMOH per
machine-hour. This variance
arose because Webb used too
much of the cost-allocation
base, machine-hours. Because
e of software the machines ran an extra 500
hours, they used extra VMOH
items, such as electricity, maintenance,
and supplies.
liable standards.
Budgeted variable Actual quantity of
overhead cost per variable overhead
unit of cost-allocation base cost-allocation bas
used for actual output
$30 per machine-hour) x 4,500 machine-hours
4,500
Efficiency variance
$ 15,000 U

presumedattempt
1BecauseWebb plans its capacity over multiple periods, anticipated demand in 2003 could be such that bud
output for 2003 is less than capacity. The analysis presented in this chapter is unchanged if the master budg
57,600.00 levelis used as the denominator level. If capacity is used as the denominator level, some additional issues a
144,000.00 that are beyond the scope of this chapter. Chapter 9 discusses choice of a denominator level in more detail

$ 3,312,000

$ 3,312,000
57,600.00

$ 57.50
Budgeted fixed
overhead cost x
per unit of
cost-allocation base

$57.50 per machine-hour


$ 57.50

$ 276,000

$ 276,000
$ 285,000

Flexible-budget
amount

$ 276,000

The flexible-budget variance for


fixed manufacturing overhead
(FMOH) is not subdivided into
separate price and efficiency
variances. The $9,000 unfavorable
flexible-budget variance
Flexible-budget for FMOH arose because Webb
amount incurred more FMOH costs than
the lump-sum amount budgeted.
$ 276,000 That's why it's called a
spending variance.

EXHIBIT 8-2
Columnar Presentation of
Fixed Manufacturing
Overhead Variance
Analysis: Webb Company
for April200J8
Allocated:
Budgeted Input
Allowed for
Actual Output
x Budgeted Rate
(3)
(0.40 hrs./unit x 10,000 units X $57.50/hr.)
(4,000 hrs. X $57.50/hr.)
$ 230,000

$ 46,000 U
Production-volume variance

allocated using

nits produced
allocated using
er output unit
al output produced

relevantrange.

Question: When the production volume


variance 'is unfavorable,
what is the realation between
FMOH allocated and budgeted
FMOH? .
Answer: FMOH allocated is less
than budgeted FMOH, as in the
Webb example.

ed overhead costs to actual output produced.


sts to actual output produced.

ount of fixed-cost resources it used to produce 10,000 jackets,

osts of $276,000 minus $230,000 budgeted fixed overhead costs allocated)


pacity it planned to use but did not use in April 2003.

capacity occurred.
here a quality

d 10,000 jackets in April.

s why Webb

bad planning decision regarding the plant capacity.

head and fIxed manufacturing overhead differ.


ce (when applicable).
me variance (when applicable).

4-variance analysis

Production-Volume
Variance
U Never a variance
Never a variance $ 46,000 U

ufacturingoverhead.

Production-Volume
Variance
U $ 46,000 U

etimescalled

Production-Volume
Variance
$ 46,000 U

Total Overhead
Variance
$ 65,500
HEAD COST ANALYSIS
any Practice,p. 265).
uring overhead:

le with respect to output units (jackets) for the planning and control purpose

costs do not change in the 0- to 12,OOO-unit relevant range.

udgeted fixed manufacturing overhead costs of $276,000$ 276,000

o nothing - to change the lease payment.


inventory costing purpose in Panel B.

output units produced.


d to products by $23. $ 23.00
ets) will be allocated to products.
head costs budgeted of $276,000 and the $230,000 of costs allocated

of Goods Sold accounts.


e unitization of fixed manufacturing overhead costs for
ver, computing the fixed manufacturing overhead costs allocated
s to identify the production-volume variance of
used fixed manufacturing overhead capacity.
eted variable manufacturing

differs from the budgeted


irect materials
direct manufacturing
ML). The BVMOH cost
mpasses costs of
erse VMOH items per
e cost-allocation base.
ebb example, the
cost rate is $30 per
hour: That is, Webb
o spend about $30 on
t basket" of VMOH
e market basket aspect
MOH cost rate makes
ariances more difficult
ret than OM and OML

03 and in the monthly performance


nd in 2003 could be such that budgeted
r is unchanged if the master budget
or level, some additional issues arise
denominator level in more detail.
Question 7 (Suggested Time – 45 minutes) MASTER BUDGET

Rein Company, a compressor manufacturer, is developing a budgeted income statement for the
calendar year 2006. The president is generally satisfied with the projected net income for 2005 of
$700,000 resulting in an earnings per share figure of $2.80.

However, next year he would like earnings per share to increase to at least $3. Rein Company employs a standard absorptio
system. Inflation necessitates an annual revision in the standards as evidenced by an increase in
production costs expected in 2006. The total standard manufacturing cost for 2005 is $72 per unit
produced.

Rein expects to sell 100,000 compressors at $110 each in the current year (2005). Forecasts from the
sales department are favorable, and Rein Company is projecting an annual increase of 10% in unit
sales in 2006 and 2007.

This increase in sales will occur even though a $15 increase in unit selling price will be implemented in 2006. The selling pr
for the increased production costs and operating expenses. However, management is concerned that any additional sales p

Standard production costs are developed for the two primary metals used in the compressor (brass
and a steel alloy), the direct labor, and manufacturing overhead. The following schedule represents
the 2006 standard quantities and rates for material and labor to produce one compressor

Brass 4 pounds @ $5.35/pound $21.40 4 $5.35 $21.40


Steel alloy 5 pounds @ $3.16/pound 15.80 5 $3.16 $15.80
Direct labor 4 hours @ $7.00/hour 28.00 4 $7.00 $28.00
Total prime costs $65.20 $65.20

The material content of the compressor has been reduced slightly, hopefully without a noticeable
decrease in the quality of the finished product. Improved labor productivity and some increase in
automation have resulted in a decrease in labor hours per unit from 4.4 to 4.0.

However, the significant increases in material prices and hourly labor rates more than offset any savings from
reduced input quantities. The manufacturing overhead cost per unit schedule has yet to be
completed. Preliminary data is as follows:

Activity Level (units)


Overhead items 100,000 110,000 120,000
Supplies $475,000 $522,000 $570,000
Indirect labor $530,000 $583,000 $636,000
Utilities $170,000 $187,000 $204,000
Maintenance $363,000 $378,000 $392,000
Taxes and insurance $87,000 $87,000 $87,000
Depreciation $421,000 $421,000 $421,000
Total overhead $2,046,000 $2,178,000 $2,310,000
$19.80

The standard overhead rate is based upon direct labor hours and is developed by using the total
overhead costs from the above schedule for the activity level closest to planned production. In
developing the standards for the manufacturing costs the following two assumptions were made.

• The cost of brass is currently selling at $5.65/pound. However, this price is historically high
and the purchasing manager expects the price to drop to the predetermined standard early in 2006

• Several new employees will be hired for the production line in 2006. The employees will be
generally unskilled. If basic training programs are not effective and improved labor
productivity is not experienced, then the production time per unit of product will increase by
15 minutes over the 2006 standards.

Rein employs a LIFO inventory system for its finished goods. Rein’s inventory policy for finished
goods is to have 15% of the expected annual unit sales for the coming year in finished goods
inventory at the end of the prior year. The finished goods inventory at December 31, 2005, is
expected to consist of 16,500 units at a total carrying cost of $1,006,500.
BI 16500 $1,006,500

Operating expenses are classified as selling, which are variable, and administrative, which are all
fixed. The budgeted selling expenses are expected to average 12% of sales revenue in 2006 which is
consistent with the performance in 2005.

The administrative expenses in 2006 are expected to be 20% higher than the predicted 2005 amount of $907,850.

Management accepts the cost standards developed by the production and accounting department.
However, they are concerned about the possible effect on net income if the price of brass does not
decrease, and/or the labor efficiency does not improve as expected. Therefore management wants the
budgeted income statement to be prepared using the standards as developed but to consider the worst
possible situation for 2006.

Each resulting manufacturing variance should be separately identified and added to or subtracted from budgeted cost of go
Rein is subject to a 45% income tax rate. 45%

REQUIRED:
A. Prepare the budgeted income statement for 2006 for Rein Company as specified by
management. Round all calculations to the nearest dollar.
B. Review the 2006 budgeted income statement prepared for Rein Company and discuss whether
the president’s objectives can be achieved.

Rein Company
Budgeted Income Statement
For the Year Ended December 31, 2006
Sales [100,000 x 1.1 x ($110 + 15)] $13,750,000 $13,750,000
Cost of goods sold at standard
[110,000 x (65.20 + 19.80)1] 9,350,000 $9,350,000
Gross margin at standard $ 4,400,000 $4,400,000
Variances
Material-brass-unfavorable $133,980.00
[(111,650 compressors2) x
(4 lbs/compressor) x ($.30/lb)] $(133,980)
Labor efficiency-unfavorable
[(111,650 compressors) x
(.25 hours/compressor) x ($7/hr)] (195,388) $195,387.50
Variable overhead efficiency unfavorable
[(111,650 compressors) x $ 92,111.25
(.25 hrs/compressor) x ($3.30/hour3)] ( 92,111)
Fixed overhead volume favorable
[(111,650-110,000 $10,890.00
compressors) x $6.60/compressor4)] 10,890 (410,589)

Gross margin at actual $ 3,989,411 $3,989,411.25


Operating expenses
Selling expense ($13,750,000 x .12) $1,650,000 $1,650,000
Administrative expense ($907,850 x 1.2) 1,089,420 2,739,420 $1,089,420
Income before taxes $ 1,249,991 $1,249,991.25
Income tax expense (45%) 562,496 $562,496.06
Net income $ 687,495 $687,495.19
Earnings per share (250,000 shares) $2.75 $ 2.75

1Standard cost of compressor


Brass 4 lbs. @ $5.35/lb. $21.40
Steel alloy 5 lbs. @ $3.16/lb. 15.80
Direct labor 4 hrs. @ $7.00/hr. 28.00
Overhead (2,178,000 ÷ 110,000) 19.80
Total cost per compressor $85.00

2Production schedule
2006 sales 110,000 110,000
Desired ending inventory 12/31/06 (110,000 x 1.1 x .15) 18,150 18,150
Required inventory 128,150 128,150
Beginning inventory 1/1/06 (110,000 x .15) 16,500 16,500
2006 production 111,650 111,650

3Determination of the variable overhead rate and total fixed overhead


Variable overhead rate per compressor = Change in Overhead
Change in Activity
($2,178,000-$2,046,000) = $132,000
110,000- 100,000 10,000
= $13.20/compressor
Variable overhead rate/direct labor hour = $13.20/compressor
4 hrs./compressor
= $3.30/direct labor hour
Total overhead at 110,000 compressors $2,178,000 $2,178,000
Total variable overhead at 110,000 compressors (110,000 x $13.20) -1,452,000 $1,452,000
Total budgeted fixed overhead $ 726,000 $726,000

4Normal activity level and fixed overhead rate.


Fixed overhead rate = Budgeted fixed overhead (See 3)
Normal production activity level
= $726,000
110,000 $6.60 /compressor
= $6.60/compressor

B. Based upon the results of the 2006 budgeted income statement, the president’s objective cannot
be achieved. A review of the statement highlights the following circumstances.
! A 2006 income statement prepared using the worst situation gives a net income of $687,495 and
earnings per share of $2.75 which is a decrease from the 2005 income of $700,000 and earnings
per share of $2.80. These budgeted figures are also considerably below the president’s objective
of $750,000 net income and $3 earnings per share.
! If the unfavorable variances do not occur, net income will increase by $231,813 after taxes
($421,479 x .55) resulting in an increase in earnings per share of $.927, giving a total earnings
per share of $3.677 which is well above the president’s objective.
! Manufacturing costs are 65.5% of the selling price ($72/$110) in 2005, and 68% of the selling
price in 2006. Administrative expenses increased 20% in 2006. Therefore, the 13.6% sales price
increase in 2006 was not sufficient to cover the increases in manufacturing cost and increases in
administrative expense
oys a standard absorption cost

2005 100,000 $110.00


2006 110,000 $125.00 $13,750,000
2007 121,000

d in 2006. The selling price increase was absolutely essential to compensate


hat any additional sales price increase would curtail the desired growth in volume.

4 $5.65 $22.60 $0.30 446,600 $133,980.00


5 $3.16 $15.80
4.25 $7.00 $29.75 0.25 27,912.50 $195,387.50
$68.15

10,000

$132,000
$13.20 /compressor
$ 3.30

15%

12% $1,650,000

nt of $907,850. $1,089,420

from budgeted cost of goods sold at standard.

$4,400,000
$10,890.00

$4,410,890

$1,650,000
$1,089,420
$1,671,470
$752,161.50
$919,308.50 $231,813.31
$ 3.68
4-33 Proration of overhead.IZ. Iqbal. adapted) The Zaf Radiator Company uses a normal-costing system
with a single manufacturing overhead cost pool and machine-hours as the cost-allocation base. The following data
are for 2007:
Budgeted manufacturing overhead $4.800,000 4,800,000
Overhead allocation base Machine-hours
Budgeted machine-hours 80,000 80,000
Manufacturing overhead incurred $4,900,000 4,900,000
Actual machine-hours 75,000 75,000
Machine-hours data and the ending balances (before proration of under- or overallocated overhead) are
as follows:
Actual 2007 End-aI-Year
Machine·Hours Balance
Cost of Goods Sold 60,000 $8,000.00
Finished Goods Control 11,000 1,250,000
Work in Process Control 4,000 750,000
75,000

Re'lufred
1. Compute the budgeted manufacturing overhead rate for 2007.
2. Compute the under- or overallocated manufacturing overhead of Zaf Radiator in 2007. Dispose of this
amount using
a. Write-off to Cost of Goods Sold
b. Proration based on ending balances (before prorationl in Work·in-Process Control, Finished Goods
Control, and Cost of Goods Sold
c. Proration based on the allocated overhead amount (before proration) in the ending balances of
Work-in-Process Control, Finished Goods Control, and Cost of Goods Sold
3. Which method do you prefer in requirement 2? Explain.

2. Compute the under- or overallocated manufacturing overhead of Zaf Radiator in 2007. Dispose of this
amount using
2. Manufacturing overhead
underallocated = Manufacturing overhead
incurred – Manufacturing overhead
allocated
= $4,900,000 – $4,500,000*
= $400,000 $ 400,000
*$60 × 75,000 actual machine-hours = $4,500,000 $ 4,500,000

a. Write-off to Cost of Goods Sold

a. Write-off to Cost of Goods Sold


Write-off
of $400,000
Account Underallocated
Balance Manufacturing
Account (Before Proration) Overhead
Work in Process $750,000 $0
Finished Goods 1,250,000 0
Cost of Goods Sold 8,000,000 400,000
Total $10,000,000 $400,000

b. Proration based on ending balances (before proration) in Work in Process, Finished


Goods and Cost of Goods Sold.

400,000
Proration of $400,000
Account Underallocated
Balance Manufacturing
Account (Before Proration) Overhead

Work in Process $750,000 7.50% $30,000


Finished Goods 1,250,000 12.50% $50,000
Cost of Goods Sold 8,000,000 80.00% $320,000
Total $10,000,000 $400,000

c. Proration based on the allocated overhead amount (before proration) in the ending
balances of Work in Process, Finished Goods, and Cost of Goods Sold.

Allocated Overhead
Account Component in
Balance the Account Balance
Account (Before Proration) (Before Proration

Work in Process $750,000 240,000 5.333%


Finished Goods 1,250,000 660,000 14.67%
Cost of Goods Sold 8,000,000 3,600,000 80.00%
Total $10,000,000 4,500,000

a
$60 × 4,000 machine-hours; b
$60 × 11,000 machine-hours; c
$60 × 60,000 machine-hours

3. Which method do you prefer in requirement 2? Explain.


3. Alternative (c) is theoretically preferred over (a) and (b). Alternative (c) yields the same
ending balances in work in process, finished goods, and cost of goods sold that would have been
reported had actual indirect cost rates been used.
Chapter 4 also discusses an adjusted allocation rate approach that results in the same
ending balances as does alternative (c). This approach operates via a restatement of the indirect
costs allocated to all the individual jobs worked on during the year using the actual indirect cost
rate.

4-34 Normal costing, overhead allocation, working backward. 1M. Rajan. adaptedl Gibson Manufacturing
usesnormal costing for its job-costing system, which has two direct-cost categories (direct materials and
direct manufacturing labor) and one indirect-cost category (manufacturing overhead). The following information
is obtained for 2007:
• Total manufacturing costs, $8,000,000
• Manufacturing overhead allocated, $3,600,000 (allocated at a rate of 200% of direct manufacturing
labor costs)
• Work-in-process inventory on January 1, 2007, $320,000
• Cost of finished goods manufactured, $7,920,000
1. Use information in the first two bullet points to calculate (a) direct manufacturing labor costs in 2007
and Ibl cost of direct materials used in 2007.
2. Calculate the ending work-in-process inventory on December 31,2007.

4-34 (15 min.) Normal costing, overhead allocation, working backward.


1.a. Manufacturing overhead allocated = 200% × Direct manufacturing labor cost
$3,600,000 = 2 × Direct manufacturing labor cost
Direct manufacturing labor cost =
2
$3,600,000 = $1,800,000
b. Total manufacturing
cost = Direct material
used + Direct manufacturing
labor cost + Manufacturing
overhead allocated
$8,000,000 = Direct material used + $1,800,000 + $3,600,000
Direct material used = $2,600,000
2. Work in Process
1/1/2007 + Total manufacturing cost =
Cost of goods manufactured + Work in Process
1/1/2007
Denote Work in Process on 12/31/2007 by X
$320,000 + $8,000,000 = $7,920,000 + X

4-39 Allocation and proratioo of manufacturing overhead. (SMA, heavily adapted) Nicole Limited is a companythat
produces machinery to customer order. Its job-costing system (using normal costing) has two directcostcategories
(direct materials and direct manufacturing labor) and one indirect-cost pool (manufacturing overhead,
allocated using a budgeted rate based on direct manufacturing labor costs). The budgetfor 2007was:
Direct manufacturing labor $420,000
Manufacturing overhead $252,000
5-17 ABC, cost hierarchy, service. leMA, adaptedl
Plymouth Test Laboratories does heat testing(HT)and stress testing (ST) on materials.

Under its current simple costing system, Plymouth aggregates all operating costs of S1,200,000 into a single overhead cost
Plymouth calculates a rate per test hour of $15 ($1,200,000/ 80,000 total test-hours).
HT uses 50,000 test-hours, and ST uses 30,000 testhours. 50,000 30000

Gary Celeste, Plymouth's controller, believes that there is enough variation in test procedures
andcost structures to establish separate costing and billing rates for HT and ST. The market for test
servicesis becoming competitive. Without this information, any miscosting and mispricing of its servicescould
cause Plymouth to lose business.

Celeste divides Plymouth's costs into four activity-cost categories.


a. Direct·labor costs, $240,000. These costs can be directly traced to HT,S I80,000, and ST, $60,000.
b. Equipment-related costs (rent, maintenance, energy, and so on), $400,000. These costs are allocated to
HTand ST on the basis of test-hours.

c. Setup costs, $350,000. These costs are allocated to HT and ST on the basis of the number of setup hours
required. HT requires 13,500 setup-hours, and ST requires 4,000 setup-hours. 13500

d. Costsof designing tests, $210,000. These costs are allocated to HT and ST on the basis of the time
required to design the tests. HT requires 2,800 hours, and ST requires 1,400 hours.

1. Classifyeach activity cost as output unit-level, batch-level, product- or service-sustaining, or facilitysustaining.


Explain each answer.
2. Calculate the cost per test-hour for HT and ST. Explain briefly the reasons why these numbers differ
from the 515 per test-hour that Plymouth calculated using its simple costing system.
J. Explainthe accuracy of the product costs calculated using the simple costing system and the ABC system.
How might Plymouth's management use the cost hierarchy and ABC information to better manage
itsbusiness?

1. Classifyeach activity cost as output unit-level, batch-level, product- or service-sustaining, or facilitysustaining.


Explain each answer.

5-17 (25 min.) ABC, cost hierarchy, service


1. Output unit-level costs
a. Direct-labor costs, $240,000
b. Equipment-related costs (rent, maintenance, energy, and so on), $400,000
These costs are output unit-level costs because they are incurred on each unit of materials
tested, that is, for every hour of testing.
Batch-level costs
c. Setup costs, $350,000
These costs are batch-level costs because they are incurred each time a batch of materials
is set up for either HT or ST, regardless of the number of hours for which the tests are
subsequently run.
Service-sustaining costs
d. Costs of designing tests, $210,000.
These costs are service-sustaining costs because they are incurred to design the HT and
ST tests, regardless of the number of batches tested or the number of hours of test time.

2 Heat Testing (HT) Stress Testing (ST)


Per Hour
Total (2) = Total
-1 (1) ÷ 50,000 50000 -3
Direct labor costs (given) 180000 3.60
Equipment-related costs
$5 per hour* × 50,000 hours 250,000 5.00
$5 per hour* × 30,000 hours 150,000 3.00

Setup costs
$20 per setup-hour† × 13,500 setup-hours
$20 per setup-hour† × 4,000 setup-hours

Costs of designing tests


$50 per hour** × 2,800 hours
$50 per hour** × 1,400 hours

2.33
Total costs $840,000 $16.80 $360,000 $12.00
*$400,000 ÷ (50,000 + 30,000) hours = $5 per test-hour
†$350,000 ÷ (13,500 + 4,000) setup hours = $20 per setup-hour
**$210,000 ÷ (2,800 + 1,400) hours = $50 per hour
6-30 Revenue and production budgets. (CPA, adaptedl The Scarborough Corporation manufactures and
sells two products: Thingone and Thingtwo. In July 2006, Scarborough's budget department gathered the
following data to prepare budgets for 2007:
2007 Projected Sales
Product Units
Thingone 60,000
Thingtwo 40,000

2007 inventories in Units Expected Target


Product Jan 1 2007 Dec 31 2007
Thingone 20,000 25,000
Thingtwo 8,000 9,000

Thefollowing direct materials are used in the two products


Amount Used per Unit
Direct Material Unit Thingone Thingtwo
A pound 4 5
B pound 2 3
C pound 0 1

Projected data for 2007 with respectto direct materials are as follows:
Anticipated EXPECTED TARGET INV.DEC 2007
Direct Material PURCHASE INVETORIES
PRICE JAN 1 2007
A 12 32000 36000
B 5 29000 32000
C 3 6000 7000

Projected direct manufacturing labor requirements and rates for 2007 are as follows:

Product Hours per Unit Rate per Hour


Thingone 2 12
Thingtwo 3 16

Manufacturing overhead is allocated at the rate of $20 per direct manufacturing labor-hour.
Based on the preceding projections and budget requirements for Thingone and Thingtwo, prepare the
following budgets for 2007:

1. Revenues budget lin dollars)


2, Production budget (in units)
3. Direct material purchases budget (in quantities)
4. Direct material purchases budget (in dollars)
5. Oirect manufacturing labor budget (in dollars)
6. Budgeted finished goods inventory at December 31, 2007 (in dollars
2. Production budget (in units)
Scarborough Corporation
Production Budget (in units) for 2007

Thingone Thingtwo
65,000 41,000

3.4. Scarborough Corporation


Direct Materials Purchases Budget (in quantities) for 2007

A B C
Thingone 260,000 130,000 -
Thingtwo 205,000 123,000 41,000
PURCHASE 469,000 256,000 42,000
PURCHASE$ $ 5,628,000 $ 1,280,000 $ 126,000

5. Scarborough Corporation
Direct Manufacturing Labor Budget (in dollars) for 2007
Thingone 130,000 12
Thingtwo 123,000 16

Scarborough Corporation
6 Budgeted Finished Goods Inventory
at December 31, 2007
Thingone
DM A 48
B 10
C
DL 24

MOH 40
122 3,050,000

Thingtwo A 60
B 15
C 3

DL 48

MOH 60
186 1,674,000
Budgeted finished goods inventory, December 31, 4,724,000
6-40 Comprehensive Review 01 Budgeting, Cash Budgeting, Chapter Appendix

Wilson Beverages bottles two soft drinks under license to Cadbury Schweppes at its Manchester plant.
All inventory is in direct materials and finished goods at the end of each working day. There is no work-in-process inventor

The two soft drinks bottled by Wilson Beverages are lemonade and diet lemonade.

The syrup for both soft drinks is purchased from Cad bury Schweppes.

Wilson Beverages uses a lot size of 1,000 cases as the unit of analysis in its budgeting. (Each case contains 24 bottles.)
1000
Direct materials are expressed in terms of lots, in which one lot of direct materials is the input necessary to yield one lot
1
The following purchase prices are forecast for direct materials in 2005:

lemonade Diet lemonade


Syrup $ 1,200 per lot $ 1,100
Containers (bottles, caps, etc.) $ 1,000 per lot $ 1,000
Packaging $ 800 per lot $ 800

All direct material purchases are on account.


The two soft drinks are bottled using the same equipment. The only difference in the bottling process for
the two soft drinks is the syrup.

Summary data used in developing budgets for 2005 are

1. Sales
• Lemonade, 1,080 lots at $9,000 selling price per lot 1080 $ 9,000
All sales are on account.

2. Beginning (January I, 2005) inventory of direct materials


• Syrup for lemonade, 80 lots at $1,100 purchase price per lot 80 $ 1,100
• Syrup for diet lemonade, 70 lots at $1,000 purchase price per lot 70 $ 1,000

3. Beginning IJanuary I, 2005) inventory of finished goods


• Lemonade, 100 lots at $5,300 per lot 100 $ 5,300
$ 530,000
4. Target ending IDecember 31, 2005) inventory of direct materials
• Syrup for lemonade, 30 lots 30
• Syrup for diet lemonade, 20 lots 20

5. Target ending (December 31, 20051 inventory of finished goods


• Lemonade, 20 lots 20
6. Each lot requires 20 direct manufacturing labor-hours at the 2005 budgeted rate of $25 per hour. Direct
manufacturing labor costs are paid atthe end of each month.

7. Variable manufacturing overhead is forecast to be $600 per hour of bottling time; bottling time is the
time the filling equipment is in operation. Ittakes two hours to bottle one lot of lemonade and two hours
to bottle one lot of diet lemonade. Assume all variable manufacturing overhead costs are paid during
the same month when incurred.

VOH 600 PER HOUR lemonade


2 H/LOT

Fixed manufacturing overhead is forecast to be $1,200,000for 2005. Included in the fixed manufacturing
overhead forecast is $400,000 for depreciation. All manufacturing overhead costs are paid as incurred.

FOH $ 1,200,000 PER HOUR lemonade


DPR EXP $ 400,000 1 H/LOT

8. Hours of budgeted bottling time is the sole cost allocation base for all fixed manufacturing overhead.

9. Administration costs are forecast to be 10% of the cost of goods manufactured for 2005.
Marketing costs are forecast to be 12% of revenues for 2005.
Distribution costs are forecast to be 8% of revenues for 2005.
All these costs are paid during the month when incurred. Assume there are no depreciation or amortization expenses.

10. Budgeted beginning balances on January I, 2005:


Accounts receivable (from sales) $ 550,000
Accounts payable Ifor direct materials) $ 300,000
Cash $ 100,000

11. Budgeted ending balances on December 31, 2005:


Accounts receivable (from sales) $600,000 $ 600,000
Accounts payable (for direct materials) $ 400,000

12. Budgeted equipment purchase in May $1,350,000 $ 1,350,000


13. Estimated income tax expense for 2005 $ 625,000 $ 625,000

REQURED
Assume Wilson Beverages uses the first-in, first-out method for costing all inventories. On the basis of the
preceding data, prepare the following budgets for 2005:
A. Revenues budget (in dollars) g. Ending finished goods inventory budget
b, Production budget (in unitsl h, Cost of goods sold budget
c. Direct materials usage budget (in units and dollars) i. Marketing costs budget
d. Direct materials purchases budget j. Distribution costs budget
k. Administration costs budget (in units and dollars) I. Budgeted income statement
e. Direct manufacturing labor budget m, Cash budget
f. Manufacturing overhead costs budget
6-40 (60 min.) Comprehensive Review of Budgeting, Cash Budgeting,Chapter Appendix.

a. Schedule 1: Revenues Budget for the Year Ended December 31, 2005
Units (Lots) Selling Price Total Sales
Lemonade 1,080 $9,000 $ 9,720,000 1080 $ 9,000 $ 9,720,000
Diet Lemonade 540 8,500 4,590,000 540 $ 8,500 $ 4,590,000
Total $14,310,000 $ 14,310,000

b. Schedule 2: Production Budget in Units for the Year Ended December 31, 2005
Products
Lemonade Diet Lemonade
Budgeted unit sales (Schedule 1) 1,080 540 1080 540
Add target ending finished goods inventory 20 10 20 10
Total requirements 1,100 550 1100 550
Deduct beginning finished goods inventory 100 50 100 50
Units to be produced 1,000 500 1000 500

c. Schedule 3A: Direct Materials Usage Budget in Units and Dollars for the Year Ended December 31, 2005
Syrup-
Lemon.
Units of direct materials to be used for production of
Lemonade (1,000 lots × 1) 1,000 1,000 1,000 1000 1 1000
Units of direct materials to be used for production of
Diet Lemonade (500 lots × 1) 500 500 500 500 1
Total direct materials to be used (in units) 1000

Units of direct materials to be used from beginning


inventory (under FIFO) 80 70 200 400 80
Multiply by cost per unit of beginning inventory $ 1,100 $ 1,000 $ 950 $ 900 $ 1,100
Cost of direct materials to be used from beginning inventory (a) $ 88,000

Units of direct materials to be used from purchases 920


Multiply by cost per unit of purchased materials $ 1,200 $ 1,100 $ 1,000 $ 800 $ 1,200

Cost of direct materials to be used from purchases (b) $ 1,104,000


Total cost of direct materials to be used (a + b) $ 1,192,000

d. Schedule 3B: Direct Materials Purchases Budget in Units and Dollars for the Year Ended December 31, 2005
Syrup-
Lemon.
Direct materials to be used in production (in units) from Schedule 3A 1000
Add target ending direct materials inventory in units 30
Total requirements in units 1030
Deduct beginning direct materials inventory in units 80
Units of direct materials to be purchased 950
Multiply by cost/unit of purchased materials $ 1,200
Direct materials purchase costs $ 1,140,000

e. Schedule 4: Direct Manufacturing Labor Budget for the Year Ended December 31, 2005
Output Direct
Units Manufacturing
Produced Labor HourTotal Hourly
(Schedule 2) per Unit Hours Rate Total
Lemonade 1000 20 20000 $ 25 $ 500,000
Diet Lemonade 500 20 10000 $ 25 $ 250,000
Total 30000 $ 750,000

f. Schedule 5: Manufacturing Overhead Costs Budget for the Year Ended December 31, 2005
Variable manufacturing overhead costs:
Lemonade [$600 × 2 hours per lot × 1,000 lots (Schedule 2)] $1,200,000 $ 1,200,000
Diet Lemonade [$600 × 2 hours per lot × 500 lots (Schedule 2)] 600,000 $ 600,000
Variable manufacturing overhead costs 1,800,000 $ 1,800,000
Fixed manufacturing overhead costs 1,200,000 $ 1,200,000
Total manufacturing overhead costs $3,000,000 $ 3,000,000
Fixed manufacturing overhead per bottling hour = $1,200,000 ÷ 3,000 = $400.
$ 1,200,000 3000 $ 400
Note that the total number of bottling hours is 3,000 hours: 2,000 hours for Lemonade (2 hours per lot ×1,000 lots) plus 1,0
2
g. Schedule 6A: Ending Finished Goods Inventory Budget as of December 31, 2005
Cost per
Units Unit
(Lots) (Lot) Total
Direct materials
Syrup for lemonade 30 $1,200 $ 36,000 30 $ 1,200 $ 36,000
Syrup for diet lemonade 20 1,100 22,000 20 $ 1,100 $ 22,000
Containers 100 1,000 100,000 100 $ 1,000 $ 100,000
Packaging 200 800 160,000 $318,000 200 $ 800 $ 160,000
Finished goods
Lemonade 20 $5,500* $110,000 20 $ 5,500 $ 110,000
Diet lemonade 10 5,400* 54,000 164,000 10 $ 5,400 $ 54,000
Total ending inventory $482,000
*See Schedule 6B

Schedule 6B: Computation of Unit Costs of Manufacturing Finished Goods For the Year Ended December 31, 2005

Cost per Lemonade Diet Lemonade


Unit (Lot) Inputs in Inputs in
or Hour Units (Lots) Units (Lots)
of Input or Hours Amount or Hours Amount
Syrup $1,200 $1,100 $ 1,200 $ 1,100
Containers 1,000 1,000 $ 1,000 $ 1,000
Packaging 800 800 $ 800 $ 800
Direct manufacturing la $ 25 20 $ 500 20 $ 500
Variable manufacturing
overhead* 600 2 1,200 $ 600 2 1200 2 1200
Fixed manufacturing ov $ 400 2 $ 800 2 $ 800
Total $ 5,500 $ 5,400

*Variable manufacturing overhead varies with bottling hours (2 hours per lot for both Lemonade and Diet
Lemonade). Fixed manufacturing overhead is allocated on the basis of bottling hours at the rate of $400 per bottling
hour calculated in Schedule 5.

h. Schedule 7: Cost of Goods Sold Budget for the Year Ended December 31, 2005
From
Schedule
Beginning finished goods inventory,
January 1, 2005. Given*
Direct materials used 3A $ 4,465,000
Direct manufacturing labor 4 $ 750,000
Manufacturing overhead 5 $ 3,000,000
Cost of goods manufactured
Cost of goods available for sale
Deduct ending finished goods inventory,
December 31, 2005. 6A
Cost of goods sold
*Given in description of basic data and requirements (Lemonade, $5,300 × 100; diet Lemonade, $5,200 × 50)

i. Schedule 8: Marketing Costs Budget for the Year Ended December 31, 2005
Marketing costs, 12% × Revenues, $14,310,000 $1,717,200 $ 1,717,200

j. Schedule 9: Distribution Costs Budget for the Year Ended December 31, 2005
Distribution costs, 8% × Revenues, $14,310,000 $1,144,800 $ 1,144,800

k. Schedule 10: Administration Costs Budget for the Year Ended December 31, 2005
Administration costs, 10% × Cost of goods $ 821,500
manufactured, $8,215,000 $ 821,500

l. Budgeted Income Statement for the Year Ended December 31, 2005
Sales Schedule 1 $ 14,310,000
Cost of goods sold Schedule 7 $ 8,841,000
Gross margin $ 5,469,000
Operating costs:
Marketing costs Schedule 8 $ 1,717,200
Distribution costs Schedule 9 $ 1,144,800
Administration costs Schedule 10 $ 821,500
Total operating costs 3,683,500 $ 3,683,500
Operating income $ 1,785,500
Income tax expense Given 625,000 $ 625,000
Net income $ 1,160,500

m. Schedule 11: Collections from Customers


Budgeted Revenue for 2005 Schedule 1 $14,310,000 $ 14,310,000
Add collections from beginning
accounts receivable balance Given $ 550,000
$ 14,860,000
Deduct ending accounts receivable balance Given $ 600,000
Collections from customers $14,260,000 $ 14,260,000

Schedule 12: Direct Materials Disbursements


Budgeted direct material purchase
costs for 2005 Schedule 3B $ 4,075,000 $ 4,075,000
Add payment for beginning accounts payable balanGiven $ 300,000
$ 4,375,000
Deduct ending accounts payable balance Given $ 400,000
Disbursements for direct materials $ 3,975,000 $ 3,975,000

Schedule 13: Variable Manufacturing Overhead Disbursements


Variable Manufacturing Overhead:
Lemonade (1,000 × $600 × 2) Schedule 5 $ 1,200,000
Diet Lemonade (500 × $600 × 2) Schedule 5 $ 600,000
Total $ 1,800,000

Schedule 14: Fixed Manufacturing Overhead Disbursements


Budgeted fixed manufacturing overhead Schedule 5 $ 1,200,000
Deduct depreciation Given $ 400,000
Cash disbursements for fixed overhead $ 800,000

Cash Budget
31-Dec-05
Cash balance, beginning Given $ 100,000 Given $ 100,000

Add receipts
Collections from customers Schedule 11 14,260,000 $ 14,260,000
Total cash available for needs $14,360,000 $ 14,360,000

Deduct disbursements
Direct materials Schedule 12 $ 3,975,000 $ 3,975,000
Direct manufacturing labor Schedule 4 750,000 $ 750,000
Variable manufacturing overhead Schedule 13 1,800,000 $ 1,800,000
Fixed manufacturing overhead Schedule 14 800,000 $ 800,000
Equipment purchase Given 1,350,000 $ 1,350,000
Marketing costs Schedule 8 1,717,200 $ 1,717,200
Distribution costs Schedule 9 1,144,800 $ 1,144,800
Administration costs Schedule 10 821,500 $ 821,500
Income tax expense Given 625,000 $ 625,000
Total disbursements $12,983,500 $ 12,983,500
Cash excess (deficiency) $ 1,376,500 $ 1,376,500
Financing
Borrowing 0 0
Repayment 0 0
Interest 0 0
Total effects of financing 0 0
Cash balance ending $ 1,376,500 $ 1,376,500
4-33
se. The following data 4-34
5-17
6-30

erhead) are

$ 60 per machine-hour.
pose of this

pose of this

Account
Balance
(After Proration)
750,000
1,250,000
8,400,000
$10,400,000

Account
Balance
(After Proration)

780,000
1,300,000
8,320,000
$10,400,000

Account
Proration of $400,000 Balance
Underallocated (After
Manufacturing Overhead Proration)

$ 21,333.33 21320 771,320


$ 58,666.67 58680 1,308,680
$ 320,000.00 320000 8,320,000
$ 400,000.00 $ 400,000 10,400,000
anufacturing

llowing information

$ 8,000,000
$ 3,600,000 200%

320000
7920000

ited is a companythat
directcostcategories
00 into a single overhead cost pool. $ 1,200,000
80000 $ 15.00

its servicescould

$ 180,000 $ 60,000
e allocated to $ 400,000

of setup hours $ 350,000


4000

$ 210,000
2800 1400

r facilitysustaining.

e ABC system.

r facilitysustaining.
Testing (ST)
Per Hour
(4) =
(3) ÷ 30,000
gathered the

Price
$ 165 9,900,000
$ 250 10,000,000
19,900,000

ET INV.DEC 2007

20
$ 7,034,000

llars) for 2007


1,560,000
1,968,000
3,528,000
s no work-in-process inventory..

case contains 24 bottles.)


24
put necessary to yield one lot (1,000 cases) of beverage.

per lot
per lot
per lot

g process for

Diet lemonade, 540 lots atSS,500 selling price per lot 540 $ 8,500

• Containers, 200 lots at S950 purchase price per lot 200 $ 950
•Packaging, 400 lots at $900 purchase price per lot 400 $ 900

•Diet lemonade, 50 lots at $5,200 per lot 50 $ 5,200


### ###

Containers, 100 lots 100


• Packaging, 200 lots 200

• Diet lemonade, 10 lots 10


r hour. Direct 20 $ 25

d two hours

Diet lemonade
2 H/LOT

nufacturing

Diet lemonade
2 H/LOT

0.1
0.12
0.08
or amortization expenses.

e basis of the

inventory budget
mber 31, 2005
Syrup-
Diet Lem. Containers Packaging Total

1000 1000

500 500 500


500 1500 1500

70 200 400
$ 1,000 $ 950 $ 900
$ 70,000 $ 190,000 $ 360,000 $ 708,000

430 1300 1100


$ 1,100 $ 1,000 $ 800

$ 473,000 $ 1,300,000 $ 880,000 $ 3,757,000


$ 543,000 $ 1,490,000 $ 1,240,000 $ 4,465,000

December 31, 2005


Syrup-
Diet Lem. Containers Packaging Total
500 1500 1500
20 100 200
520 1600 1700
70 200 400
450 1400 1300
$ 1,100 $ 1,000 $ 800
$ 495,000 $ 1,400,000 $ 1,040,000 $ 4,075,000

urs per lot ×1,000 lots) plus 1,000 hours for Diet Lemonade (2 hours per lot × 500 lots).
1000 2000 2 500 1000

$ 318,000

$ 164,000
$ 482,000

ed December 31, 2005


ade and Diet
ate of $400 per bottling

Total

$ 790,000

$ 8,215,000
$ 9,005,000

$ 164,000
$ 8,841,000
Q Desc Type N/A
204 balanced SC T
205 T measuring a division manager’s efficiency
206 T motivation and performance
207 DM DL VAR T
208 T goal congruence,
209 balanced SC T
210 T goal congruence,
211 T goal congruence between the economic order quantity (EOQ) model and the manufacturin
212 VOH VAR T cause of variance
213 T performance is measured by how well she performs to budget.
214 T evaluating the performance of the Repair and Maintenance Department
215 T CONTROLABILITY
216 DM VAR T
217 SERVICE DPT T goal congruence USING Budgeted rate times actual hours of computer usage
218 T unfavorable operating income variance
219 DM DL VAR T
220 DM VAR T
221 DL VAR T
222 flexible budge T advantage
223 static budget T disadvantage
224 static budget T var
225 flexible budge T
226 flexible budge T
227 sales quantity T
228 sales-volume vT
229 management bT
230 standard cost T
231 standard cost T
232 standard cost T
233 DL VAR p
234 DM VAR p
235 DM VAR p
236 DM VAR p
237 DL VAR p
238 DM VAR p
239 DL VAR T
240 DL VAR T
241 DM VAR T
242 DL VAR p
243 VOH VAR p
244 PVV p
245 DL VAR T
246 overhead spenp
247 overhead spenp
248 overhead spenT
249 overhead spenT
250 Ethical T
251 DM VAR T
252 DL VAR T
253 DM VAR T
254 PVV T
255
256
257
258
259
260
261
262
263
264
265
266
267
268
269
270
271
272
model and the manufacturing manager’s own preference

omputer usage
Operating Data for Horizon
Petroleum
External market: Transportation Division
Contract price Variable cost per barrel of crude oil $ 1.00
for supplying crude Fixed cost per barrel of crude oil $ 3.00
oil = $12 per barrel Full cost per barrel of Clude oil $4 $ 4.00
$ 12.00

External market: Refining Division


Market price of Variable cost per barrel of gasoline $ 8.00
outside suppliers Fixed cost per barrel of gasoline $ 6.00
to Houston refinery Full cost per barrel of gasoline $ 14.00
= $21 per barrel
of crude oil
$ 21.00

Division Operating In-come of Horizon Petroleum for 100 Barrels of Crude Oil Under Alternative
Transfer-Pricing Methods
Method A:
Internal Transfers Internal Transfers
at Market
Prices
Transportation Division
Revenues $ 21.00
100
$ 2,100.00
$ 340.00
Deduct:
Crude oil purchase costs, 100
$ 1,200.00

Division variable costs,


$1 X 100 barrels of crude oil $ 100.00

Division fixed costs,


$3 X 100 barrels of crude oil $ 300.00

Division operating income $ 500.00


Refining Division
Revenues, 58
$58 X 50 barrels of gasoline 50
$ 2,900

Deduct:
Transferred-in costs, $21, $17.60, $19.25 $ 2,100
X 100 barrels of crude oil

Division variable costs,


$8 X 50 barrels of gasoline $ 400

Division fixed costs,


$6 X 50 barrels of gasoline $ 300

Division operating income $ 100


$ (340.00)
Operating income of both
divisions together $ 600
External market:
Market price to
outside parties
= $58 per barrel
of gasoline $ 58.00

Method B: Method C:
Internal Transfers Internal Transfers
at 110% of at Negotiated
Full Costs Prices
110%
$ 17.60 $ 19.25
100 100
$ 1,760.00 $ 1,925.00

100 100
$ 1,200.00 $ 1,200.00

$ 100.00 $ 100.00

$ 300.00 $ 300.00

$ 160.00 $ 325.00
58 58
50 50
$ 2,900 $ 2,900

$ 1,760 $ 1,925

$ 400 $ 400

$ 300 $ 300

$ 440 $ 275

$ 600 $ 600
Allocated
MOH
Included in each
Account Bal. Account Bal.
Account Control (Before proration) (Before proration)
WIP Control $50,000 $16,200
FG Control $75,000 $31,320
COGS Control $2,375,000 $1,032,480
$2,500,000 $1,080,000

Allocated Allocated
MOH MOH
Included in each Included in each Proration of $135,000
Account Bal. Account Bal. Account Bal.
Account Control (Before proration) (Before proration) as a % of total
WIP Control $50,000 $16,200 1.50% $135,000
FG Control $75,000 $31,320 2.90% $135,000
COGS Control $2,375,000 $1,032,480 95.60% $135,000
$2,500,000 $1,080,000 100.0%

Account Bal. Proration of $135,000 of


Account Bal. as a % of total Underallocated MOH
Account Control (Before proration) (2)=(1)X$ 2,500,000 (3)=(2)X $ 135,000
WIP Control $50,000 2.00% $135,000 $2,700
FG Control $75,000 3.00% $135,000 $4,050
COGS Control $2,375,000 95.00% $135,000 $128,250
$2,500,000 100.0% $135,000

Manufacturing Jab-Casting Syslem Using Normal Cosling: Diagram of General Ledger


Relalionships for February 2006

1 Purchase of materials, $ 89,000


2 Usage of direct materia $ 81,000 Usage of indirect materials
3 Incurrence of liability $ 39,000 Incurrence of liability for ind
4 Payment of wages, $ 54,000
5 Incurrence of other Ma $ 75,000
6 Allocation of manufactu $ 80,000
7 Completion and transfer$ 188,800
8 Cost of goods sold, $ 180,000
9 Incurrence of marketing $ 60,000
10 sales $ 270,000
$60,000
MATERIALS CONTROL
$ 89,000 $ 81,000
$ 4,000

WAGES PAYABLE CONTROL


$ 39,000 $ 39,000
$ 15,000 $ 15,000

MANUFACTURJNC OVERHEAD CONTROL


$ 4,000
$ 15,000
$ 75,000

MANUFACrURJNC OVERHEAD ALLOCATED


$ 80,000

WIP
$ 81,000 $ 188,800
$ 39,000
$ 80,000
$ 11,200
Job 298
Actual DM
Actual DL

Actual DL hours for Job 298


Actual Allocated OH 88 X $45

Total Manf. Cost


Actual Price
Gross Margin
Gross Margin %

Proration of $135,000 of
Underallocated Account Bal. Job 298Applied MOH 88X$40
MOH (After proration) Total Manf. Cost(normal costing)
$2,025 $52,025
$3,915 $78,915 Diff
$129,060 $2,504,060
$135,000 $2,635,000

Account Bal.
(After proration)
(4) =(1)+(3)
$52,700
$79,050
$2,503,250
$2,635,000

$ 4,000
$ 15,000
AP

FINISHED GOODS CONTROL


$ 188,800 $ 180,000
$ 8,800

COGS
$ 180,000
4606 2003 Actual DL hours 27,000.00
1579 Actual MOH 1,215,000.00
6185 Actual rate $ 45.00
88
$ 3,960.00 2003 Budgeted DL hours 28,000.00
Budgeted MOH 1,120,000.00
$ 10,145.00 Budgeted rate $ 40.00
$ 15,000.00
$ 4,855.00
32%
Applied MOH 1,080,000.00
Net (Dr) Balance 135,000.00 12.05%
$ 3,520.00
$ 9,705.00 Dr CoGS 135,000.00
DR Applied 1,080,000.00
$ 440.00 CR MOH control 1,215,000.00

DR Cogs 2,025.00 2%
DR WIP 3,915.00 3%
2. The table below shows the estimated probabilities of the percent of defective units resulting from a production run.

Percent Defective Probability


2% 30% 1%
3% 50% 2%
4% 20% 1%
2.90%
The expected percent defective for a production run would be

3. Reeves Inc. has developed a new production process to manufacture its product. The new
process is complex and requires a high degree of technical skill. However, management
believes there is a good opportunity for the employees to improve as they become more
familiar with the production process.
The production of the first unit requires 100 direct labor hours.
If a 70% learning curve is used,
the cumulative direct labor hours required to produce a total of eight units would be

1 100 2
2 140
4 196
8 274.4

or we have 3 doubling
34.3 x 274.4

6. Carson Products sells sweatshirts and is preparing for a World Cup Soccer match. The cost per sweatshirt varies with the

Quantity Unit cost


4,000 $ 14.00 $ 56,000
5,000 $ 13.50 $ 67,500
6,000 $ 13.00 $ 78,000
7,000 $ 12.50 $ 87,500

Carson must purchase the shirts one month before the game and has analyzed the market and estimated sales levels as fol
Unit sales Probability
4,000 15% 600 25
5,000 20% 1000 25
6,000 35% 2100 25
7,000 30% 2100 25
5800

The estimated selling price is $25 for sales made before and during the day of the game.
Any shirts remaining after game day can be sold at wholesale to a local discount store for $10.
The expected profit if Carson purchased 6,000 shirts is

Revenue
Demand
4000 5000 6000 7000*
Revenue (orig. sales) 100000 125000 150000 150000
Wholesale revenue $ 20,000 $ 10,000
Cost of shirts $ 78,000 $ 78,000 $ 78,000 $ 78,000
Profit $ 42,000 $ 57,000 $ 72,000 $ 72,000
Probability 15% 20% 35% 30%
Profit x probability $ 6,300 $ 11,400 $ 25,200 $ 21,600
Expected profit (sum) $ 64,500

* Sales limited to 6000, since purchased only 6000

15. Werner Company buys raw materials from several suppliers, and makes payments according to the following schedule.
In the month of purchase 25% 25%
In the month after purchase 60% 60%
In the second month after purchase 15% 15%
In preparing the master budget for the fourth quarter of the year, Werner assumed that total purchases for the quarter wo
In its pro forma balance sheet, Werner anticipated a December 31 account payable balance of $207,000. What amount o

10 11 12

60%
15% 15% 90%

19. The Profit and Loss Statement of Madengrad Mining Inc. includes the following information for the current fiscal year.
Sales $160,000 $ 160,000
Gross profit 48,000 $ 48,000
Year-end finished goods inventory 58,300 $ 58,300
Opening finished goods inventory 60,190 $ 60,190
The cost of goods manufactured by Madengrad for the current fiscal year is
COGS $ 112,000
$ 110,110

21. Garland Company uses a standard cost system.


The standard for each finished unit of product allows for 3 pounds of plastic at $0.72 per pound.
During December, Garland bought 4,500 pounds of plastic at $0.75 per pound, and used 4,100 pounds in the production o
4500 $ 0.75
What is the material purchase price variance for the month of December?

29. Pane Company uses a job costing system and applies overhead to products on the basis of direct labor cost.
Job No. 75, the only job in process on January 1, had the following costs assigned as of that date:
direct materials, $40,000; $ 40,000
direct labor, $80,000; and $ 80,000
factory overhead, $120,000. $ 120,000

The following selected costs were incurred during the year.


Traceable to jobs:
Direct materials $178,000 $ 178,000
Direct labor 345,000 $523,000 $ 345,000 $ 523,000
Not traceable to jobs:
Factory materials and supplies 46,000 $ 46,000
Indirect labor 235,000 $ 235,000
Plant maintenance 73,000 $ 73,000
Depreciation on factory equipment 29,000 $ 29,000
Other factory costs 76,000 459,000 $ 76,000 $ 459,000

Pane’s profit plan for the year included budgeted direct labor of $320,000 and factory overhead of $448,000. There was no
320000
BOH RATE $ 1.40
OH APPL. $ 345,000 $ 483,000.00 $ 24,000 OVER

34. Juniper Manufacturing uses a weighted-average process costing system at its satellite plant.
Goods pass from the Major Assembly Department to the Finishing Department to finished goods inventory. The goods are
The first inspection occurs when the goods are 30% complete, and the second inspection occurs at the end of production.
The following data pertain to the Finishing Department for the month of July.
Units
Good units started and completed during July 65,000
Normal spoilage - first inspection 2,000
Abnormal spoilage - second inspection 150
Ending work-in-process inventory, 60% complete 15,000

There was no beginning work-in-process inventory in July.


Juniper recognizes spoiled units to make the cost of all spoilage visible in their management reporting.
Equivalent units for assigning costs for July would total
34. Key = d
Physical Units Equivalent Units
Started and completed during month (1 100% 65000 65000
Normal spoilage (30%) 2,000 600 30% 2000 600
Abnormal spoilage (100%) 150 150 100% 150 150
Ending work-in-process inventory (60%) 60% 15000 9000
82,150 74,750
ng from a production run. 2. Key = c
Sum of the percent defective multiplied by its probability o
(2% x .30) + (3% x .50) + (4% x .20) = .60 + 1.50 + .80 = 2.90

3. Key = b
100 x .7 x .7 x.7 = 34.3 average hours for 8 units; total hours = 8 x 34.3

100
70%

per sweatshirt varies with the quantity purchased as follows.

nd estimated sales levels as follows.


25
$ 10.00
6000

15. Key = d
ing to the following schedule. Accounts payable at 12/31 = 75% of December and 15% of
$207,000/.90 = $230,000 for month; $690,000 for the qua

purchases for the quarter would be spread evenly over the three months.
of $207,000. What amount of purchase did Werner anticipate for the fourth quarter of the year?
$ 207,000

$ 230,000 $ 690,000

19. Key = c
on for the current fiscal year. Cost of goods sold = Sales – Gross profit = $160,000 - $48,000 = $112
Available for sale finished goods = Cost of goods sold + Ending finishe
Cost of goods manufactured = Available for Sale finished goods - Ope
21. Key = c
(Standard price-actual price) x number of pound
3 $ 0.72 ($0.72 - $0.75) x 4,500 pounds = $135 unfavorab
00 pounds in the production of 1,300 finished units of product.
4,100 1,300
$ (135.00)

f direct labor cost.

29. Key = b
Applied overhead - actual = amount over/underapplied
$448,000/$320,000 = budgeted application rate of 1.4
$345,000 direct labor actual x 1.4 = $483,000 applied
$483,000 applied - $459,000 total not traceable = $24,000 overapplied

ad of $448,000. There was no work-in-process on December 31. Pane’s overhead for the year was
448000

oods inventory. The goods are inspected twice in the Finishing Department.
urs at the end of production.
multiplied by its probability of occurrence
.20) = .60 + 1.50 + .80 = 2.90%

8 units; total hours = 8 x 34.3 = 274.4 hours


5% of December and 15% of November
month; $690,000 for the quarter

= $160,000 - $48,000 = $112,000.


goods sold + Ending finished goods inventory = $112,000 + $58,300 = $170,300.
or Sale finished goods - Opening finished goods inventory = $170,300 – $60,190 = $110,110.
ual price) x number of pounds purchased
00 pounds = $135 unfavorable

Você também pode gostar